tele

Pataasin ang iyong marka sa homework at exams ngayon gamit ang Quizwiz!

Block Time Remaining: 00:39:49 TUTOR Test Id: 72490738 QId: 31364 (921666) 58 of 75 A A A The clinic nurse is preparing to administer an allergy immunotherapy injection to a client recently initiated on the therapy. Which statement by the client indicates a need for further teaching? 1. "I can leave right after the shot as I didn't have a reaction last time." [79%] 2. "I will be back in a week for my next allergy shot." [6%] 3. "I will let the doctor know if I get any itchy hives tonight." [3%] 4. "It is okay if I have some redness at the injection site tonight." [11%] Omitted Correct answer 1 Answered correctly 79% Time: 1 seconds Updated: 12/20/2016 Explanation: Allergy immunotherapy injections (allergy shots) trigger an increase in the body's production of specific immunoglobulins to reduce the client's allergy symptoms when exposed to specific allergens (eg, pollen, cat dander, dust mite). Small doses of the allergen(s) are injected subcutaneously on a client-specific schedule. Rarely, allergy shots may induce an immediate and potentially fatal anaphylactic reaction. The client must remain at the facility for 30 minutes after an injection so the nurse can monitor for severe systemic reactions (eg, respiratory failure, tongue and throat swelling) (Option 1). (Option 2) For the first few months, allergy shots are typically given every week, with a dose increase at every injection until the target maintenance dose is reached. The maintenance dose is then given every few weeks for 3-5 years. (Option 3) Although rare, the client may have a mild, systemic allergic reaction (eg, hives, itching, facial swelling, mild asthma) up to 24 hours after an allergy shot. The occurrence of any systemic reaction should be reported to the health care provider as the next dose increase may need to be delayed. (Option 4) It is common to have a localized reaction to an allergy shot. The nurse should reinforce teaching that some redness and swelling at the injection site is expected and not life-threatening. Educational objective: A client receiving an allergy shot is at risk for anaphylaxis immediately after the injection, so the client must remain at the facility and be monitored for 30 minutes after the injection. Copyright © UWorld. All rights reserved.

Block Time Remaining: 00:39:56 TUTOR Test Id: 72490738 QId: 30377 (921666) 59 of 75 A A A An elderly client with a history of stable chronic obstructive pulmonary disease, alcohol abuse, and cirrhosis has a serum theophylline level of 25.8 mcg/mL (143 µmol/L). Which clinical manifestation associated with theophylline toxicity should worry the nurse most? 1. Alterations in color vision [9%] 2. Gum (gingival) hypertrophy [8%] 3. Hyperthermia [6%] 4. Seizure activity [75%] Omitted Correct answer 4 Answered correctly 75% Time: 7 seconds Updated: 04/26/2017 Explanation: Theophylline has narrow therapeutic index and plasma concentrations >20 mcg/mL (111 µmol/L) are associated with theophylline drug toxicity. Toxicity can be acute or chronic. Conditions associated with chronic toxicity include advanced age (>60), drug interactions (eg, alcohol, macrolide and quinolone antibiotics), and liver disease. Acute toxicity is associated with intentional or accidental overdose. Symptoms of toxicity usually manifest as central nervous system stimulation (eg, headache, insomnia, seizures), gastrointestinal disturbances (eg, nausea, vomiting), and cardiac toxicity (eg, arrhythmia). (Option 1) Alteration in color perception and visual changes are commonly seen with digoxin toxicity. (Option 2) Gum hypertrophy is seen with phenytoin toxicity. (Option 3) Hyperthermia and tinnitus are often seen with aspirin overdose. Educational objective: Theophylline plasma concentrations >20 mcg/mL (111 µmol/L) are associated with theophylline drug toxicity. Seizures (central nervous system stimulation) and cardiac arrhythmias are the most serious and lethal consequences. Copyright © UWorld. All rights reserved.

Block Time Remaining: 00:00:05 TUTOR Test Id: 72419393 QId: 31755 (921666) 10 of 75 A A A The nurse is providing discharge teaching to several clients with new prescriptions. Which instructions by the nurse are correct in regard to medication administration? Select all that apply. 1. Avoid salt substitutes when taking valsartan for hypertension 2. Take levofloxacin with an aluminum antacid to avoid gastric irritation 3. Take sucralfate after meals to minimize gastric irritation associated with a gastric ulcer 4. When taking ethambutol, notify the health care provider (HCP) of any changes in vision 5. When taking rifampin, notify the HCP if the urine turns red-orange Incorrect Correct answer 1,4 Answered correctly 32% Time: 79 seconds Updated: 03/19/2017 Explanation: Both ACE inhibitors ("prils" - captopril, enalapril, lisinopril, ramipril) and angiotensin receptor blockers ("sartans" - valsartan, losartan, telmisartan) cause hyperkalemia. Salt substitutes contain high potassium and must not be consumed unless approved by the health care provider (HCP) (Option 1). Ethambutol (Myambutol) is used to treat tuberculosis but can cause ocular toxicity, resulting in vision loss and loss of red-green color discrimination. Vision acuity and color discrimination must be monitored regularly (Option 4). (Option 2) Levofloxacin (Levaquin) is a quinolone antibiotic. For this class of antibiotics, 2 hours should pass between drug ingestion and consumption of aluminum/magnesium antacids, iron supplements, multivitamins with zinc, or sucralfate. These substances can bind up to 98% of the drug and make it ineffective. (Option 3) Sucralfate (Carafate, Sulcrate), prescribed to treat gastric ulcers, should be administered before meals to coat the mucosa and prevent irritation of the ulcer during meals. It should also be given at least 2 hours before or after other medications to prevent interactions that reduce drug efficacy. (Option 5) Rifampin (Rifadin), used to treat tuberculosis, normally causes red-orange discoloration of all body fluids. The client should be alerted to expect this change but does not need to notify the HCP. Educational objective: The nurse should watch for vision changes with ethambutol. Potassium supplements or salt substitutes should not be given to a client taking an ACE inhibitor or angiotensin receptor blocker. Sucralfate must be given before meals to prevent irritation of the ulcer. Quinolone antibiotics should not be given with antacids or supplements that reduce drug efficacy. Rifampin commonly causes red-orange discoloration of body fluids. Copyright © UWorld. All rights reserved.

Block Time Remaining: 00:00:05 TUTOR Test Id: 72419393 QId: 34480 (921666) 11 of 75 A A A A client in the postoperative period after an open reduction and internal fixation of a left wrist fracture reports constant, severe arm pain that is unrelieved by prescribed morphine administered 30 minutes ago. The client's nail beds appear dusky. What are the nurse's appropriate actions? Select all that apply. 1. Apply a heating pad and encourage range-of-motion exercises 2. Assess the temperature and movement of the fingers 3. Elevate the arm on pillows above the heart level 4. Notify the health care provider 5. Reassure client, document findings, and reassess in 1 hour Incorrect Correct answer 2,4 Answered correctly 24% Time: 67 seconds Updated: 03/15/2017 Explanation: Compartment syndrome, a serious postoperative complication, is caused by decreased blood flow to the tissue distal to the injury. This results from either decreased compartment size (restrictive dressings/splints/casts) or increased pressure within the compartment (bleeding, inflammation, and edema). Earliest symptoms may include pain or numbness that is unrelieved by medication. Subsequent findings include diminished/lost pulses, pallor, coolness, swelling, decreased movement, and cyanosis. Failure to treat this condition can lead to loss of limb function, paralysis, and tissue necrosis. The nurse should assess neurovascular status and report to the health care provider immediately (Options 2 and 4). Removing tight bandages/casts and fasciotomy (surgery) are required to relieve the pressure. (Option 1) Heat should not be applied to a client experiencing altered sensation, as it may burn the client. Active range of motion will not resolve compartment syndrome and delays needed care. (Option 3) Elevating the arm on pillows and providing additional analgesia may help reduce symptoms but may also reduce perfusion of the extremity. Do not elevate the extremity for long periods; instead, just keep at the level of the heart. (Option 5) Documenting findings is important. However, reassurance and reassessment 1 hour later without immediate intervention will delay the care. Educational objective: Compartment syndrome is caused by decreased blood flow to the tissue distal to the injury and can cause ischemic necrosis. Acute compartment syndrome following surgery or casting is potentially limb-threatening and requires emergent evaluation by a health care provider. Copyright © UWorld. All rights reserved.

Block Time Remaining: 00:01:16 TUTOR Test Id: 72797956 QId: 31367 (921666) 38 of 75 A A A The nurse is caring for a client who is having a thoracentesis. Following the procedure, the nurse monitors for complications. The initial postprocedure monitoring plan should include what? Select all that apply. 1. Level of alertness 2. Lung sounds 3. Oxygen saturation 4. Respiratory pattern 5. Temperature 6. Urine output Omitted Correct answer 1,2,3,4 Answered correctly 37% Time: 1 seconds Updated: 05/30/2017 Explanation: Thoracentesis is commonly used to treat pleural effusion. The health care provider (HCP) will prepare the skin, inject a local anesthetic, and then insert a needle between the ribs into the pleural space where the fluid is located. A complication of thoracentesis is pneumothorax, which occurs when the needle goes into the lung and causes the lung to slowly deflate, like a balloon with a small hole in it. Bleeding is another, yet less common, complication of the procedure. Signs of pneumothorax include increased respiratory rate, increased respiratory effort, respiratory distress, low oxygen saturation, and absent breath sounds on the side where the procedure was done (where the lung is collapsed) (Options 2, 3, and 4). Tension pneumothorax may also develop, with tracheal shift to the unaffected side, severe respiratory distress, and cardiovascular compromise. Altered level of consciousness may occur due to decreased oxygenation and blood flow to the brain (Option 1). A tension pneumothorax may be prevented by early detection of pneumothorax through appropriate monitoring. (Option 5) Infection would be a later complication (occurring a few days after the procedure), so monitoring temperature is not required during the initial postprocedure period. (Option 6) Urine output should not be affected by thoracentesis or the drugs administered for this procedure. Educational objective: Following thoracentesis, the nurse should monitor for signs of pneumothorax, including level of alertness, respiratory rate, respiratory effort, oxygen saturation, and lung sounds. Copyright © UWorld. All rights reserved.

Block Time Remaining: 00:01:17 TUTOR Test Id: 72797956 QId: 31023 (921666) 39 of 75 A A A The health care provider writes a prescription for hydromorphone 10 mg intravenous push every 2 hours prn for the post-operative client. The usual recommended dose is 0.2-1 mg every 2-3 hours prn. What action should the nurse initially take? 1. Administer the medication and monitor client frequently [1%] 2. Ask a nursing colleague if this drug amount is used [0%] 3. Check hydromorphone dose that the client had previously [14%] 4. Question the prescription with the prescriber [82%] Omitted Correct answer 4 Answered correctly 82% Time: 1 seconds Updated: 12/22/2016 Explanation: The nurse needs to have appropriate knowledge about a medication prior to administering it. Hydromorphone (Dilaudid) is a potent narcotic that has 5-10 times the strength of morphine. This client was prescribed a hydromorphone dose that is too high given that the typical maximum dose is 2 mg. As the drug prescription is outside a safe range, it must be questioned and cannot be administered automatically. (Option 1) A prescription that greatly exceeds the safety range should not be given without questioning/clarification. However, anytime the outer limit of drug dosing of a potent narcotic is administered, the client should be monitored frequently for adverse effects. This includes the sedation scale and arousability as sedation precedes respiratory depression for narcotics. (Option 2) When there is a medication dosing question, authoritative resources (eg, the pharmacist, current drug literature) should be consulted rather than relying on a nursing colleague who could be mistaken. (Option 3) Even if the client is opiate-tolerant, the dosage is significantly outside the safety range and the prescription should be questioned or clarified. Educational objective: When a medication prescription is outside the safety range, the nurse must question/clarify the prescription with the prescriber and not administer the drug automatically. Copyright © UWorld. All rights reserved.

Block Time Remaining: 00:02:12 TUTOR Test Id: 81040578 QId: 30243 (921666) 5 of 75 A A A The registered nurse (RN) is caring for an elderly client with chronic obstructive pulmonary disease (COPD) whose pulse oximeter shows 91% on room air. After physical therapy, the client reports feeling "short of breath and exhausted" to the student nurse and says he just wants to sleep. To provide comfort, the student nurse initiates the prn nasal oxygen to maintain a saturation ≥92%, as prescribed. When the RN conducts end-of-shift rounds 3 hours later, the client is still sleeping soundly and the pulse oximeter shows 91%. Which nursing action is most appropriate at this time? 1. Check a full set of vital signs [40%] 2. Continue to monitor [23%] 3. Increase the oxygen flow by 1 L/min [10%] 4. Remove the nasal oxygen and measure saturation [25%] Incorrect Correct answer 4 Answered correctly 25% Time: 31 seconds Updated: 04/26/2017 Explanation: When oxygen is delivered via a nasal cannula, the concentration of inspired oxygen will vary with changes in ventilation and respiratory pattern. In a deep sleep, with smaller tidal volumes and decreased respiratory rate, the concentration of inspired oxygen increases. In some clients with COPD, too high a level of inspired oxygen can depress the respiratory drive to breathe, causing alveolar hypoventilation. Because the saturation has remained constant at 91% while the client sleeps soundly, the nurse should remove the nasal oxygen, recheck the saturation, and re-evaluate. Removing the supplemental oxygen may actually increase the reading. (Option 1) The findings from a full set of vital signs would not provide information about the presence of alveolar hypoventilation. (Option 2) The client continues to sleep soundly after the initiation of supplemental oxygen. Continuing to monitor, without assessing and re-evaluating oxygenation status, is not an appropriate intervention. (Option 3) If sleeping soundly is related to depressed respiratory drive and hypoventilation, increasing oxygen concentration will worsen the problem. Educational objective: Some clients with COPD breathe in response to low arterial oxygen levels (hypoxemia). If they receive more oxygen than they need to maintain an arterial saturation, the increased level can decrease the drive to breathe. Therefore, supplemental oxygen should be administered in the lowest concentration possible to maintain a pulse oxygen saturation of 90%-93% or a PaO2 of 60-70 mm Hg. Copyright © UWorld. All rights reserved.

Block Time Remaining: 00:03:30 TUTOR Test Id: 81040578 QId: 30761 (921666) 7 of 75 A A A The hospice nurse is caring for an actively dying client who is unresponsive and has developed a loud rattling sound with breathing ("death rattle") that distresses family members. Which prescription would be most appropriate to treat this symptom? 1. Atropine sublingual drops [33%] 2. Lorazepam sublingual tablet [13%] 3. Morphine sublingual liquid [48%] 4. Ondansetron sublingual tablet [4%] Incorrect Correct answer 1 Answered correctly 33% Time: 15 seconds Updated: 01/16/2017 Explanation: The "death rattle" is a loud rattling sound with breathing that occurs in a client who is actively dying. When the client cannot manage airway secretions, the movement of these secretions during breathing causes a noisy rattling sound. This can distress family and friends at the bedside of the dying client. The "death rattle" can be treated using anticholinergic medications to dry the client's secretions. Medications include atropine drops administered sublingually or a transdermal scopolamine patch. (Option 2) Lorazepam is a benzodiazepine that is used to treat anxiety and restlessness in terminally ill clients. It can be effective for alleviating dyspnea exacerbated by anxiety, but it is ineffective for controlling secretions (the cause of the "death rattle"). (Option 3) Morphine is an opioid analgesic that is effective for pain treatment as well as terminal dyspnea. The client is not exhibiting these symptoms, so morphine would be inappropriate. (Option 4) Ondansetron will help the nausea and vomiting but is not very effective for treating the "death rattle." Educational objective: The "death rattle" is a noisy rattling sound with breathing commonly seen in a dying client who is unresponsive and no longer able to manage airway secretions. Anticholinergic medications such as transdermal scopolamine or atropine sublingual drops effectively treat this symptom by drying up the excess secretions. Copyright © UWorld. All rights reserved.

Block Time Remaining: 00:03:24 TUTOR Test Id: 72419393 QId: 30138 (921666) 53 of 75 A A A The emergency department nurse would administer a prescribed isotonic crystalloid solution to which client? 1. 25-year-old with a closed-head injury and signs of increasing intracranial pressure (ICP) [13%] 2. 45-year-old with acute gastroenteritis and dehydration [61%] 3. 68-year-old with chronic renal failure and hypertensive crisis [8%] 4. 60-year-old with seizures and serum sodium of 112 mEq/L [17%] Correct Answered correctly 61% Time: 10 seconds Updated: 04/26/2017 Explanation: Acute gastroenteritis is associated with nausea, vomiting, diarrhea, and dehydration. An isotonic crystalloid intravenous (IV) solution (eg, 0.9% normal saline, lactated Ringer's) has the same tonicity as plasma and when infused remains in the vascular compartment, quickly increasing circulating volume. It is appropriate to correct the extracellular fluid volume deficit (dehydration) in this client. (Option 1) A hypertonic, rather than isotonic, solution would be infused in clients with ICP. Increasing circulating volume would only further increase ICP. (Option 3) Isotonic solutions can exacerbate fluid overload in chronic renal failure and increase blood pressure. (Option 4) Clients with severe hyponatremia and neurologic manifestations need rapid correction of hyponatremia with hypertonic saline (3% saline). Educational objective: Depending on the type/tonicity of intravenous (IV) solution infused, fluids can remain in the vascular compartment or can shift from the extracellular to intracellular compartments, and vice versa. The nurse must be able to assess which type of IV fluid is appropriate in relation to a client's diagnosis and condition. Copyright © UWorld. All rights reserved.

Block Time Remaining: 00:03:42 TUTOR Test Id: 72419393 QId: 34143 (921666) 55 of 75 A A A A nurse is caring for a client who is meeting with the palliative care team. After the meeting, the client's family asks for clarification about palliative care. Which statements about palliative care are accurate? Select all that apply. 1. Palliative care focuses on quality of life and can be provided at any time 2. Palliative care is only possible with a terminal diagnosis of ≤6 months 3. Palliative care is provided by a multidisciplinary team 4. Palliative care is another term for hospice care 5. Palliative care provides relief from symptoms associated with chronic illnesses Correct Answered correctly 38% Time: 16 seconds Updated: 05/03/2017 Explanation: Palliative care is a model of treatment that involves managing symptoms, providing psychosocial support, coordinating care, and assisting with decision making to relieve suffering and improve quality of life for clients and families facing serious illnesses. An interdisciplinary palliative assessment team often includes nursing staff, chaplains, social workers, therapists, and nutritionists who work together on a comprehensive treatment plan. This model of care has been found to decrease unnecessary medical interventions and reduce depressive symptoms. Families of clients who receive palliative care interventions also experience lower rates of prolonged grief and post-traumatic stress disorder. (Option 2) Palliative care is not limited to the last 6 months of life and can begin immediately after diagnosis of terminal disease (eg, advanced heart failure or cancer). (Option 4) The main difference between palliative care and hospice is that clients receiving palliative care can receive concurrent curative treatment. Hospice care is only started once the client decides to forego curative treatment. Educational objective: Palliative care focuses on quality of life and symptom management (eg, pain, dyspnea, fatigue, constipation, nausea, loss of appetite, difficulty sleeping, depression). It can be given concurrently with life-prolonging treatment in the setting of terminal disease. Palliative care is provided by a multidisciplinary care team with a focus on the clients and their families. Copyright © UWorld. All rights reserved.

A couple is excited about finding out the sex of their baby during ultrasound at 14 weeks gestation. What is the nurse's best response? 1. "Basic structures of major organs are not yet formed." [7%] 2. "External genitalia are not usually visualized until 21-24 weeks." [47%] 3. "If the baby is in the right position, the genitalia may be visualized." [42%] 4. "Sex cannot be determined until fetal movement is felt." [1%] Explanation: By the end of 12 weeks gestation, fetal sex can often be determined by the appearance of the external genitalia on ultrasound, depending on the quality of the image. (Option 1) By the end of 8 weeks gestation, all major organ systems are in place, and many are functioning in a simple way. By 7 weeks gestation, fetal heart tones can be detected. (Options 2 and 4) Clients typically begin feeling fetal movements in the second trimester at around 16-20 weeks gestation. Parous (have been pregnant before) clients can notice this earlier than the nulliparous (first pregnancy). Fetal sex can be determined as early as the end of 12 weeks gestation. Educational objective: Fetal heart tones can be detected by 7 weeks gestation. Fetal sex may be determined on ultrasound as early as the end of 12 weeks gestation. Fetal movements are typically felt at around 16-20 weeks gestation.

The initial prenatal laboratory screening results of a client at 12 weeks gestation indicate a rubella titer status of nonimmune. What will the nurse anticipate as the plan of care for this client? 1. Administer measles-mumps-rubella (MMR) vaccine now [15%] 2. Administer MMR vaccine immediately postpartum [57%] 3. Administer MMR vaccine in the third trimester [8%] 4. An MMR vaccine is not indicated for this client [18%] Incorrect Correct answer 2 Explanation: In a pregnant client, a serum sample is collected at the first prenatal visit to determine immunity to the rubella virus. A positive immune response indicates immunity to the rubella virus, attributed to either past infection or vaccination. A negative, or nonimmune, response indicates that the client is susceptible to rubella disease and requires vaccination. An equivocal response indicates partial immunity to rubella and is treated clinically the same as nonimmune status. Measles-mumps-rubella (MMR) is a live attenuated vaccine. Live vaccines are contraindicated in pregnancy due to the theoretical risk of contracting the disease from the vaccine. Maternal rubella infection can be teratogenic for the fetus. The fetal effects of congenital rubella syndrome include congenital cataracts, deafness, heart defects (patent ductus arteriosus), and cerebral palsy. The best time to administer an MMR vaccine to a nonimmune client is in the postpartum period just prior to discharge (Option 2). The MMR vaccine can safely be administered to breastfeeding clients. (Options 1 and 3) MMR vaccine is contraindicated in pregnancy. Also, pregnancy should be avoided for at least 1-3 months after the immunization is given. (Option 4) This client is rubella nonimmune and is susceptible to rubella if exposed. The vaccine should be offered in the postpartum period. Educational objective: The measles-mumps-rubella vaccine is a live attenuated vaccine and is contraindicated in pregnancy due to the risk of teratogenic effects to the fetus. Clients who are nonimmune to rubella should receive the vaccine in the postpartum period. Pregnancy should be avoided for at least 1-3 months after immunization.

Block Time Remaining: 00:01:38 TUTOR Test Id: 72797956 QId: 32146 (921666) 46 of 75 A A A A 55-year-old male client has a 16-Fr indwelling urethral catheter with a 5-mL balloon inserted to relieve postoperative urinary retention. The nurse observes urine leaking from the insertion site, past the catheter. What is the nurse's first action? 1. Check the urethral catheter and drainage tubing [82%] 2. Irrigate the catheter with 30 mL of sterile normal saline [3%] 3. Notify the health care provider [1%] 4. Remove and reinsert the next-larger-size catheter [12%] Omitted Correct answer 1 Answered correctly 82% Time: 2 seconds Updated: 02/08/2017 Explanation: Obstruction (eg, clots, sediment), kinking/compression of catheter tubing, bladder spasms, and improper catheter size can cause leakage of urine from the insertion site of an indwelling urinary catheter. The nurse's first action should be to assess for a mechanical obstruction by inspecting the catheter tubing (Option 1). These interventions may alleviate obstruction: Remove kinking or compression of the catheter or tubing. Attempt to dislodge a visible obstruction by milking the tubing. This involves squeezing and releasing the full length of the tubing, starting from a point close to the client and ending at the drainage bag. If these interventions fail, the nurse should then notify the health care provider (HCP) (Option 3). (Option 2) Irrigation is usually avoided as pus or sediment can be washed back into the bladder; however, it is sometimes prescribed to relieve an obstruction to urine flow. If there is a discrepancy in expected urine output compared with fluid intake, a blockage is suspected and a bladder scan is then performed to confirm the presence of urine in the bladder. (Option 4) The client has the recommended size of catheter and balloon for an adult male. The HCP may prescribe removal and reinsertion of a different-size catheter if other measures fail to relieve obstruction. Educational objective: If leakage of urine is observed from the insertion site of an indwelling urinary catheter, the nurse should assess for obstruction, kinking, or compression of the catheter or drainage tubing; bladder spasms; and improper catheter size. Copyright © UWorld. All rights reserved.

Block Time Remaining: 00:01:39 TUTOR Test Id: 72797956 QId: 31314 (921666) 47 of 75 A A A The male client had a hemicolectomy. The client is refusing to wear the prescribed sequential compression devices (SCDs). What is most important for the nurse to communicate to the client? 1. An appropriate form must be signed, verifying refusal [13%] 2. Complications, including death, could result [73%] 3. The client will be billed for the equipment regardless [0%] 4. The surgeon will be informed of the refusal [12%] Omitted Correct answer 2 Answered correctly 73% Time: 1 seconds Updated: 01/09/2017 Explanation: Just as there is informed consent, there is informed refusal. The client should be made aware of all the possible complications (including the possible worst-case scenario, which is usually death) when making a decision, and this should be documented. The nurse should try to work with the client to get at least partial compliance when it is in the client's best interest (eg, wear the SCDs for a limited time). (Option 1) This would occur, but it is more important to make the client aware of the potential implications of this refusal so that the client can make an informed refusal. (Option 3) Safe, quality care is the priority, not financial concerns. The nurse should avoid discussing financial implications when a client is making care decisions. (Option 4) Depending on the hospital policy, a refusal to wear the SCDs could result in an additional form being completed and the refusal documented in the medical record. Documentation should include the information given to the client and the client's understanding of that information. Even if the client refuses to sign the form, the nurse should obtain other witnesses and document the refusal in detail in the medical record. Educational objective: The most important aspect of a client's refusal for treatment is to make sure that the client is informed of the potential results of the refusal. Copyright © UWorld. All rights reserved.

Block Time Remaining: 00:39:10 TUTOR Test Id: 81075711 QId: 30782 (921666) 72 of 75 A A A The nurse will implement which nursing actions when caring for a client recently diagnosed with a hiatal hernia? Select all that apply. 1. Elevate the head of the hospital bed 2. Instruct the client to avoid tobacco and caffeine 3. Offer small, frequent, low-fat meals 4. Provide a girdle to reduce the hernia 5. Teach the client to avoid lifting or straining Correct Answered correctly 33% Time: 94 seconds Updated: 12/11/2016 Explanation: Conditions that increase intraabdominal pressure (eg, pregnancy, obesity, ascites, tumors, heavy lifting) and weaken the muscles of the diaphragm may allow a portion of the stomach to herniate through an opening in the diaphragm, causing a hiatal hernia. A sliding hernia occurs when a portion of the upper stomach squeezes through the hiatal opening in the diaphragm. A paraesophageal hernia (rolling hernia) occurs when the gastroesophageal junction remains in place but a portion of upper stomach folds up along the esophagus and forms a pocket. Paraesophageal hernias are a medical emergency. Although hiatal hernias may be asymptomatic, many clients experience signs and symptoms commonly associated with gastroesophageal reflux disease (GERD), including heartburn, dysphagia, and pain caused by increased intraabdominal pressure or supine positioning. Interventions to reduce herniation include the following: Diet modification—avoid high-fat foods and those that decrease lower esophageal sphincter pressure (eg, chocolate, peppermint, tomatoes, caffeine). Eat small, frequent meals, and decrease fluid intake during meals to prevent gastric distension. Avoid consumption of meals close to bedtime and nocturnal eating (Option 3). Lifestyle changes—smoking cessation, weight loss (Option 2). Avoid lifting or straining (Option 5). Elevate the head of the bed to approximately 30 degrees—this can be done at home using pillows or 4 - 6 inch blocks under the bed (Option 1). (Option 4) Wearing a girdle or tight clothes increases intraabdominal pressure and should be avoided. Educational objective: Hiatal hernias occur due to a weakening diaphragm and increased intraabdominal pressure. Nursing interventions to prevent hiatal hernias are similar to those used for gastroesophageal reflux disease (GERD), and they focus on decreasing intraabdominal pressure. Copyright © UWorld. All rights reserved.

Block Time Remaining: 00:02:04 TUTOR Test Id: 81114149 QId: 30865 (921666) 4 of 73 A A A A client with stroke symptoms has a blood pressure of 240/124 mm Hg. The nurse prepares the prescribed nicardipine intravenous (IV) infusion solution correctly to yield 0.1 mg/mL. The nurse then administers the initial prescription to infuse at 5 mg/hr by setting the infusion pump at 50 mL/hr. What is the nurse's priority action at this time? 1. Assess hourly urinary output [19%] 2. Increase pump setting to correct administration rate to 100 mL/hr [8%] 3. Keep systolic blood pressure above 170 mm Hg [35%] 4. Monitor for a widening QT interval [36%] Incorrect Correct answer 3 Answered correctly 35% Time: 33 seconds Updated: 06/07/2017 Explanation: A client with an acute stroke presentation (brain attack) requires "permissive hypertension" during the first 24-48 hours to allow for adequate perfusion through the damaged cerebral tissues. However, the blood-brain barrier is no longer intact once the blood pressure is >220/120 mm Hg. Therefore, "mild" lowering is required, usually to a systolic pressure that is not below 170 mm Hg. Nicardipine (Cardene) is a prototype of nifedipine and is a potent calcium channel blocking vasodilator. It takes effect within 1 minute of IV administration. It is essential to monitor that the blood pressure is not being lowered too quickly or too slowly as this would extend the stroke. Hypotension can occur with or without reflex tachycardia. The drug must be discontinued if hypotension or reflex tachycardia occurs. (Option 1) It is important to watch for signs of adequate perfusion to the kidneys as indicated by adequate urine output. Approximately 25% of the cardiac output goes to the kidney. However, the priority is to monitor for the rapid onset of effect that this potent antihypertensive agent has after initiation. The nurse should not wait until the urine output drops. (Option 2) The initial rate set by the nurse is correct as desired dose/dose on hand is 5 mg/hr divided by 0.1 mg/mL and equals 50 mL/hr. (Option 4) Widening of the QT interval can increase the risk of life-threatening torsades de pointes. It is most commonly seen with haloperidol (Haldol), methadone, ziprasidone (Geodon), and erythromycin. However, this is not an expected complication of nicardipine. Educational objective: Nicardipine (Cardene) is a potent calcium channel blocking vasodilator. The priority nursing intervention with IV administration is to monitor the hypotensive effects of this drug. Copyright © UWorld. All rights reserved.

Block Time Remaining: 00:40:13 TUTOR Test Id: 72490738 QId: 32250 (921666) 74 of 75 A A A A nursing diagnosis of "ineffective airway clearance related to pain" is identified for a client who had open abdominal surgery 2 days ago. Which intervention should the nurse implement first? 1. Administer prescribed analgesic medication for incisional pain [55%] 2. Encourage use of incentive spirometer every 2 hours while awake [13%] 3. Offer an additional pillow to splint the incision while coughing [27%] 4. Promote increased oral fluid intake [2%] Omitted Correct answer 1 Answered correctly 55% Time: 1 seconds Updated: 01/18/2017 Explanation: Postoperative clients are at risk for atelectasis and possibly for pneumonia following surgery as a result of retained secretions. Effective coughing is essential to prevent these complications. The nurse can promote many client actions that will facilitate effective coughing. These include splinting the incision while coughing, changing position every 1-2 hours, ambulating early, using an incentive spirometer, and hydrating adequately to thin the secretions. However, all of these interventions are less effective if the client is in pain. The nurse should instruct the client to request pain medication before the pain becomes intense. Pain relief should be addressed prior to implementing coughing exercises and ambulation. (Options 2, 3, and 4) These are appropriate interventions but will be more effective if pain is managed first. Educational objective: The nurse should ensure that the postoperative client has effective pain relief before performing coughing exercises. Copyright © UWorld. All rights reserved.

Block Time Remaining: 00:40:13 TUTOR Test Id: 72490738 QId: 32250 (921666) 74 of 75 A A A A nursing diagnosis of "ineffective airway clearance related to pain" is identified for a client who had open abdominal surgery 2 days ago. Which intervention should the nurse implement first? 1. Administer prescribed analgesic medication for incisional pain [55%] 2. Encourage use of incentive spirometer every 2 hours while awake [13%] 3. Offer an additional pillow to splint the incision while coughing [27%] 4. Promote increased oral fluid intake [2%] Omitted Correct answer 1 Answered correctly 55% Time: 1 seconds Updated: 01/18/2017 Explanation: Postoperative clients are at risk for atelectasis and possibly for pneumonia following surgery as a result of retained secretions. Effective coughing is essential to prevent these complications. The nurse can promote many client actions that will facilitate effective coughing. These include splinting the incision while coughing, changing position every 1-2 hours, ambulating early, using an incentive spirometer, and hydrating adequately to thin the secretions. However, all of these interventions are less effective if the client is in pain. The nurse should instruct the client to request pain medication before the pain becomes intense. Pain relief should be addressed prior to implementing coughing exercises and ambulation. (Options 2, 3, and 4) These are appropriate interventions but will be more effective if pain is managed first. Educational objective: The nurse should ensure that the postoperative client has effective pain relief before performing coughing exercises. Copyright © UWorld. All rights reserved.

Block Time Remaining: 00:01:13 TUTOR Test Id: 72797956 QId: 30103 (921666) 34 of 75 A A A The nurse is caring for a client with active pulmonary tuberculosis. Which elements of infectious disease precautions are mandatory for the nurse when providing routine care? Select all that apply. 1. Gown 2. Goggles or face shield 3. Hand washing 4. N95 particulate respirator 5. Surgical mask Omitted Correct answer 3,4 Answered correctly 33% Time: 1 seconds Updated: 03/13/2017 Explanation: Common applications of airborne precautions Equipment used for airborne precautions Tuberculosis Varicella (chickenpox)* Rubeola (measles) Personal N95 particle respirator As needed for procedures with risk of splash or body fluid contact: Nitrile gloves, disposable gown, goggles/shield Rooms Negative air pressurization High-efficiency particulate air filters *Contact precautions also required. Isolation is mandatory for clients with conditions that involve airborne transmission, and rooms must use both negative air pressurization and high-efficiency particulate air (HEPA) filters to avoid contamination. A class N95 or higher particulate respirator must be worn during client care. All clients with symptoms consistent with a suspected airborne illness should be given a surgical mask to wear as soon as they are assessed during triage. Good hand hygiene is always the first and last element of infection control in any client care setting. (Options 1 and 2) Wearing a gown and face shield would be necessary only if the nurse suspected splash of body fluids from procedural client care, not from routine care such as assessment or medication administration. Contact precautions may also be necessary if the tuberculosis is extrapulmonary with draining lesions (eg, cutaneous tuberculosis). (Option 5) For client care involving airborne precautions, a class N95 or higher respirator must be used in lieu of a surgical mask to avoid potential exposure to aerosolized particles. Surgical masks are rated for barrier protection for droplet splashing and filtration of large respiratory particles only. Clients should be given surgical masks during their transportation. Educational objective: Tuberculosis requires airborne precautions. Clients suspected of having tuberculosis should be given a surgical mask to wear on entering any health care setting. Clients are placed in negative-pressure isolation rooms. Nurses must use a class N95 or higher particulate respirator. Copyright © UWorld. All rights reserved.

Block Time Remaining: 00:01:14 TUTOR Test Id: 72797956 QId: 31615 (921666) 35 of 75 A A A The nurse is caring for a client on droplet precautions who has a prescription for a CT scan. When transporting the client to radiology, the nurse should ensure that the transporter uses protective equipment correctly to reduce the environmental spread of infection when the client is outside the room. Which instruction should the nurse give the transporter? 1. Have the client wear a mask [88%] 2. Have the client wear gloves [1%] 3. Wear a mask [8%] 4. Wear an isolation gown [1%] Omitted Correct answer 1 Answered correctly 88% Time: 1 seconds Updated: 01/31/2017 Explanation: Droplet precautions are used to prevent transmission of respiratory infection. These precautions include the use of a mask and a private room. When the client is in the room, staff should wear masks and follow standard precautions. The client on droplet precautions should wear a mask at all times when outside the hospital room. (Option 2) Gloves are not required as part of droplet precautions. Standard precautions should guide the use of gloves in clients on droplet precautions. (Option 3) The transporter does not need to wear a mask outside of the client's room as long as the client keeps a mask on to prevent transmission of infection. (Option 4) An isolation gown is not required for droplet precautions. Educational objective: Droplet precautions require the use of regular masks to prevent the transmission of infection. A mask should be worn by the client when outside the hospital room and by staff when in the client's room. Copyright © UWorld. All rights reserved.

Block Time Remaining: 00:02:33 TUTOR Test Id: 72797956 QId: 32539 (921666) 67 of 75 A A A The emergency department nurse receives report on 4 clients. Which client will the nurse prioritize for placement in an isolation room? 1. 4-year-old diagnosed with scabies who has red burrows and bumps along the neckline and inner elbows [6%] 2. 7-year-old diagnosed with measles who has a fever, conjunctivitis, cough, and maculopapular rash [60%] 3. 12-year-old with a positive rapid influenza test who has a fever, cough, and runny nose [9%] 4. 14-year-old with 4-inch wound on inner aspect of thigh with a positive culture for methicillin-resistant Staphylococcus aureus [24%] Omitted Correct answer 2 Answered correctly 60% Time: 3 seconds Updated: 03/06/2017 Explanation: Infectious agents that are spread by air currents are among the most contagious of pathogens. Therefore, clients with airborne infections (measles, tuberculosis, varicella, severe acute respiratory syndrome) should be isolated first using airborne precautions. These infections are spread via very small particles that circulate in the air. Clients with airborne infections are placed in an isolation room with negative pressure that provides air exchange or with a high-efficiency particulate air filtration system. (Option 1) Clients with scabies will be placed in contact isolation. The 4-year-old is contagious, but only if direct contact is made. Therefore, isolating the client with airborne precautions is the priority. (Option 3) Clients with influenza are placed on droplet precautions. The 12-year-old can spread pathogens via large droplets released into the air when coughing, sneezing, or talking. The client would be the second priority for isolation. (Option 4) Clients with methicillin-resistant Staphylococcus aureus infection are placed on contact precautions. The 14-year-old is contagious, but only if direct contact is made. The client requires isolation but is not a priority over the client whose pathogens are airborne. Educational objective: Airborne infections (eg, measles, tuberculosis, varicella) are spread by air currents and are among the most contagious of diseases. Clients with these infections require airborne precautions and should be isolated first. Copyright © UWorld. All rights reserved.

Block Time Remaining: 00:02:36 TUTOR Test Id: 72797956 QId: 32831 (921666) 70 of 75 A A A The charge nurse on a pediatric unit recognizes that it is acceptable for which pair of clients to be assigned to a semi-private room? 1. 4-year-old girl in Buck traction and 5-year-old boy post laparoscopic appendectomy [61%] 2. 6-year-old girl with varicella and 7-year-old girl with measles [18%] 3. 9-month-old boy with rotavirus infection and 8-month-old boy with salmonella infection [7%] 4. 14-year-old girl with sickle cell anemia and 13-year-old girl with periorbital cellulitis [12%] Omitted Correct answer 1 Answered correctly 61% Time: 1 seconds Updated: 03/07/2017 Explanation: Although placing pediatric clients of different sexes in a semi-private room is not ideal, the charge nurse must prioritize client room assignments based on client safety. At ages 4 and 5, the male-female pair can room together. The client in Buck traction does not have a transmittable illness. The client post laparoscopic appendectomy is also not infectious. Given the options above, this is the safest room assignment. (Option 2) Children with infections requiring airborne precautions (eg, varicella, tuberculosis, measles) should be placed in a private, airborne infection isolation room (eg, negative airflow room). If required, clients infected with the same organism can be roomed together, but a private room is preferred. (Option 3) Rotavirus is a viral gastroenteritis, and salmonella is a bacterial gastroenteritis. The risk for cross contamination is high, especially with caregivers sharing the facilities. (Option 4) A client with sickle cell anemia is at risk for infection due to spleen dysfunction (repeated infarctions), and a client with periorbital cellulitis has an infection. Although compatible in age and sex, these clients should not share a room. Educational objective: Pediatric room placement should be based on disease process, sex, and developmental stage. When assigning children to semi-private rooms, the charge nurse must consider client safety first. Copyright © UWorld. All rights reserved.

Block Time Remaining: 00:03:16 TUTOR Test Id: 81012973 QId: 33433 (921666) 4 of 75 A A A The clinic nurse speaks with the spouse of a client being treated for alcohol use disorder. Which statements by the spouse indicate codependence? Select all that apply. 1. "I am focusing on my new hobby and my friends in the book club." 2. "I left and didn't awaken my spouse, who went back to sleep after turning off the alarm clock." 3. "I try to get up early and keep the children from being too loud in the mornings." 4. "If I didn't get so stressed about my job, my spouse wouldn't drink so much." 5. "When my spouse was sick, I called and rescheduled clients so my spouse could rest." Incorrect Correct answer 3,4,5 Answered correctly 27% Time: 100 seconds Updated: 05/05/2017 Explanation: Codependent behaviors are those that allow the codependent person to maintain control by fulfilling the needs of the addict first. Behaviors such as keeping the addiction secret, suffering physical or psychological abuse from the addict, not allowing the addict to suffer the consequences of actions, and making excuses for the addict's habit are hallmarks of codependency. If the addict isn't happy, the codependent person will try to make the addict happy. Codependent persons will focus all their attention on others at the expense of their own sense of self. Codependent spouses, friends, and family members keep the client from focusing on treatment; this behavior is counterproductive to both themselves and the client. (Option 1) This statement does not represent codependency but rather indicates that the spouse is focusing on the spouse's own growth and needs rather than the needs of others. (Option 2) This statement does not represent codependency; it indicates that the spouse is allowing the client to suffer the consequence of actions. Educational objective: Codependent spouses, friends, and family members can impede treatment progress of clients with substance use disorders. Codependent behaviors include making excuses for a client's drug/alcohol use, putting a client's needs before one's own, and not allowing a client to suffer the consequences of actions. Copyright © UWorld. All rights reserved.

Block Time Remaining: 00:04:02 TUTOR Test Id: 81012973 QId: 30657 (921666) 6 of 75 A A A The nurse has just completed discharge teaching for a client who had aortic valve replacement with a mechanical heart valve. Which statement by the client indicates that teaching has been effective? 1. "I'm glad that I can continue taking my Ginkgo biloba." [5%] 2. "I will increase my intake of leafy green vegetables." [17%] 3. "I will start applying vitamin E to my chest incision after showering." [9%] 4. "I will shave with an electric razor from now on." [67%] Incorrect Correct answer 4 Answered correctly 67% Time: 31 seconds Updated: 01/22/2017 Explanation: Mechanical prosthetic valves are more durable than biological valves but require long-term anticoagulation therapy due to the increased risk of thromboembolism. The client should be taught ways to reduce the risk of bleeding. Teaching topics for clients on anticoagulants: Take medication at the same time daily Depending on medication, report for periodic blood tests to assess therapeutic effect Avoid any action that may cause trauma/injury and lead to bleeding (eg, contact sports, vigorous teeth brushing, use of a razor blade) (Option 4) Avoid aspirin and nonsteroidal anti-inflammatory drugs (NSAIDs) Limit alcohol consumption Avoid changing eating habits frequently (eg, dramatically increasing intake of foods high in vitamin K such as kale, spinach, broccoli, greens) (Option 2) and do not take vitamin K supplements Consult with health care provider before beginning or discontinuing any medication or dietary/herbal supplement (eg, Ginkgo biloba and ginseng affect blood clotting and may increase bleeding risk) (Option 1) Wear a medical alert bracelet indicating what anticoagulant is being taken (Option 3) Early in the recovery period, care of the incision site typically includes washing with soap and water and patting it dry. Ointments (eg, vitamin E) may be applied after the incision has healed. Educational objective: Clients who are on anticoagulants should avoid aspirin, NSAIDS, and other over-the-counter or herbal products (eg, Ginkgo biloba) that can increase bleeding risk. They should also avoid behaviors that increase the risk of clotting (eg, eating excess green leafy vegetables). Copyright © UWorld. All rights reserved.

Block Time Remaining: 00:03:05 TUTOR Test Id: 81114149 QId: 30559 (921666) 6 of 73 A A A The nurse is caring for a client with an acute ischemic stroke who has a blood pressure of 178/95 mm Hg. The health care provider prescribes as-needed antihypertensives to be given if the systolic pressure is >200 mm Hg. Which action by the nurse is most appropriate? 1. Give the antihypertensive medication [4%] 2. Monitor the blood pressure [61%] 3. Notify the health care provider [6%] 4. Question the prescription [27%] Incorrect Correct answer 2 Answered correctly 61% Time: 35 seconds Updated: 01/30/2017 Explanation: An ischemic stroke is a loss of brain tissue perfusion due to blockage in blood flow. Elevated blood pressure is common and permitted after a stroke and may be a compensatory mechanism to maintain cerebral perfusion distal to the area of blockage. This permissive hypertension usually autocorrects within 24-48 hours and does not require treatment unless the hypertension is extreme (systolic blood pressure >220 mm Hg or diastolic blood pressure >120 mm Hg) or contraindicated due the presence of another illness requiring strict blood pressure control (eg, active ischemic coronary disease, heart failure, aortic dissection). A blood pressure of 178/95 mm Hg should be monitored, along with the client's other vital signs and status (Option 2). (Option 1) The elevated blood pressure may be a protective measure to ensure tissue perfusion; therefore, the antihypertensive drug should not be given unless indicated by the prescription parameters. (Options 3 and 4) The as-needed prescription is appropriate in this case, so there is no reason to question it and call the health care provider. Educational objective: Elevated blood pressure in the presence of an acute ischemic stroke is an expected finding and may be a protective measure to maintain tissue perfusion. Copyright © UWorld. All rights reserved.

Block Time Remaining: 00:07:02 TUTOR Test Id: 81114149 QId: 31012 (921666) 15 of 73 A A A The nurse completes a neurological examination on a client who has suffered a stroke to determine if damage has occurred to any of the cranial nerves. The nurse understands that damage has occurred to cranial nerve IX based on which assessment finding? 1. A tongue blade is used to touch the client's pharynx; gag reflex is absent [65%] 2. Only one side of the mouth moves when the client is asked to smile and frown [12%] 3. The absence of light touch and pain sensation on the left side of the client's face [8%] 4. When the client shrugs against resistance, the left shoulder is weaker than the right [13%] Correct Answered correctly 65% Time: 29 seconds Updated: 12/19/2016 Explanation: Cranial nerves IX (glossopharyngeal) and X (vagus) are related to the movement of the pharynx and tongue. To evaluate cranial nerves IX and X, the nurse assesses for the presence of a gag reflex and symmetrical movements of the uvula and soft palate, and listens to voice quality. A tongue blade can be used to touch the posterior pharyngeal wall to assess for a gag reflex. Asking the client to say "ah" will allow assessment of the uvula and soft palate. Harsh or brassy voice quality indicates dysfunction with the vagus nerve (X) (Option 1). (Option 2) The facial nerve (VII) is assessed by observing for symmetrical movements during facial expressions (eg, smile, frown, close eyes). (Option 3) The trigeminal nerve (V) has both sensory and motor functions. The nurse assesses for equal jaw strength by palpating the masseter muscle while the client clenches the jaw. To assess sensory function, the nurse touches the client's face with the client's eyes closed to determine if sensations are equal. (Option 4) The spinal accessory nerve innervates the sternocleidomastoid and part of the trapezius muscles. The nurse applies resistance during shrugging and head turning and assesses for equal strength. Educational objective: Cranial nerves IX and X work together to create movement of the pharynx and tongue. An absent gag reflex, asymmetrical uvular and palate movement, or a change in voice quality indicates damage. Copyright © UWorld. All rights reserved.

Block Time Remaining: 00:39:11 TUTOR Test Id: 81012973 QId: 30379 (921666) 54 of 75 A A A The nurse is teaching general skin care guidelines to a client receiving teletherapy (external beam radiation therapy). Which statements does the client make that indicate proper understanding of the teaching? Select all that apply. 1. "I may apply an ice pack to the treatment site if it begins to burn." 2. "I will rub baby oil after each treatment to prevent dry skin." 3. "I will use extra measures to protect my skin from sun exposure." 4. "I will wash the treatment site with lukewarm water and mild soap." 5. "I will wear soft, loose-fitting clothing." Correct Answered correctly 56% Time: 53 seconds Updated: 02/02/2017 Explanation: Clients receiving teletherapy (external beam radiation therapy) often experience significant effects to the skin of the treatment area. Teaching essential skin care standards to these clients is focused on preventing infection and promoting healing of the affected skin. Key measures of skin care that clients receiving teletherapy should take include: Protect the skin from infection by not rubbing, scratching, or scrubbing (Option 2) Wear soft, loose-fitting clothing Use soft, cotton bed sheets and towels Pat skin dry after bathing Avoid applying bandages or tape to the treatment area Cleanse the skin daily by taking a lukewarm shower Use mild soap without fragrance or deodorant Do not wash off any radiation ink markings Use only creams or lotions approved by the health care provider (HCP) Avoid over-the-counter creams, oils, ointments, or powders unless specifically recommended by the HCP as they can worsen any irritation Shield the skin from the effects of the sun during and after treatment Avoid tanning beds and sunbathing Wear a broad-brimmed hat, long sleeves, and long pants when outside Use a sunscreen that is SPF 30 or higher Avoid extremes in skin temperature Avoid heating pads and ice packs (Option 1) Maintain a cool, humid environment for comfort Educational objective: The client receiving teletherapy is taught measures to implement to protect the skin from infection and promote healing. Recommended skin care measures include taking a lukewarm shower daily, avoiding rubbing or scratching the skin, using only approved lotions, shielding the skin from the effects of the sun, and avoiding extremes in temperature. Copyright © UWorld. All rights reserved.

Block Time Remaining: 00:39:29 TUTOR Test Id: 81012973 QId: 30476 (921666) 55 of 75 A A A During the first prenatal assessment, the client reports the last normal menstrual period starting on March 1 and ending on March 5, but also slight spotting on March 23. The client had unprotected intercourse on March 15. Using Naegele's rule, what is the estimated date of birth? 1. December 8 [57%] 2. December 12 [22%] 3. December 22 [13%] 4. December 30 [6%] Correct Answered correctly 57% Time: 18 seconds Updated: 12/27/2016 Explanation: Various methods to determine the estimated date of birth (EDB) include use of Naegele's rule, ultrasound, uterine height measurement (McDonald's measurement), and auscultation of fetal heart rate with a Doppler device. The most accurate dating of pregnancy involves use of ultrasound around the 16th-18th week of pregnancy. However, Naegele's rule can be used to quickly determine an EDB early in the pregnancy. This calculation uses the date of the first day of the last normal menstrual period (LMP) for determination of EDB. EDB = (LMP minus 3 months) + 7 days This client's LMP is March 1, minus 3 months = December 1. Add 7 days to obtain EDB = December 8. Clients who conceive in January, February, and most of March will deliver in the current year. Those who conceive after March will deliver in the following year; as a result, a third step is adding 1 to the current year to determine the estimated date of birth. For example, LMP of May 10, 2014, would have an EDB of February 17, 2015. It is important to note that Naegele's rule is based on a client having a menstrual cycle of 28 days. It therefore may not be as accurate if the client has a shorter or longer menstrual cycle. (Option 2) Using the last day of the LMP to calculate EDB provides an inaccurate due date as clients may have varying lengths of menstrual bleeding. (Option 3) Conception occurs around the time of ovulation and is about 14 days from the beginning of the LMP. Eggs are fertile for about 12-24 hours after ovulation with sperm able to remain fertile for 24-72 hours. Implantation of the trophoblast occurs about 7-10 days after fertilization. Using the conception date calculates the gestational age of the embryo approximately 2 weeks later than the true gestational age. (Option 4) Spotting around the time the next menstrual period is due may be considered normal and is probably caused by implantation of the trophoblast into the uterine endometrial lining. This is not considered a problem, but using this occurrence to date the pregnancy erroneously delays the EDB by 4 weeks. It is important to calculate EDB from the beginning of the last normal menstrual period. Educational objective: Naegele's rule provides a quick determination of the estimated date of birth (EDB). EDB = (LMP minus 3 months) + 7 days. If the LMP occurs in January, February, or March, the EDB will be in the current year. If the LMP occurs after March, the EDB will be in the next year. Copyright © UWorld. All rights reserved.

Block Time Remaining: 00:04:40 TUTOR Test Id: 81057403 QId: 31967 (921666) 8 of 75 A A A The student nurse is performing an assessment of a 10-year-old diagnosed with attention-deficit hyperactivity disorder (ADHD). In addition to the 3 core symptoms of ADHD (hyperactivity, impulsiveness, and inattention), which of the following would the student nurse expect to find during the assessment? 1. Confusion and a learning disability [24%] 2. Delayed physical and emotional development [12%] 3. Disorientation and cognitive impairment [14%] 4. Low self-esteem and impaired social skills [49%] Omitted Correct answer 4 Answered correctly 49% Time: 42 seconds Updated: 12/16/2016 Explanation: The core symptoms of ADHD include hyperactivity, impulsiveness, and inattention. Hyperactive children are restless; have difficulty remaining seated when required; and exhibit excessive talking, blurting out answers prematurely, and interrupting others. Inattention is characterized by reduced ability to focus and attention to detail, easy distractibility, and failure to follow through (eg, homework, chores). The primary symptoms of ADHD have a negative impact and can make life difficult for children in school, at work, and in social situations. Symptoms interfere with opportunities to acquire social skills and may also result in rejection and critical judgment by peers. The negative consequences of ADHD include: Poor self-esteem Increased risk for depression and anxiety Increased risk for substance abuse Academic or work failure Trouble interacting with peers and adults (Option 1) Children with ADHD are more likely to have a learning disability. Confusion is not a typical clinical finding. (Option 2) Although children with ADHD may appear to be emotionally immature for their age, ADHD is not associated with delayed physical growth. (Option 3) Children with ADHD are not disoriented. ADHD is associated with a range of cognitive impairments, but no single cognitive dysfunction typifies all children with the disorder. Some children have no impairment at all. Educational objective: The diagnosis of ADHD includes the presence of hyperactivity, impulsiveness, and inattention. The negative consequences of the core manifestations include impaired social skills, poor self-esteem, academic or work failure, increased risk for depression and anxiety, and increased risk for substance abuse. Copyright © UWorld. All rights reserved.

Block Time Remaining: 00:41:21 TUTOR Test Id: 81057403 QId: 30572 (921666) 73 of 75 A A A A client with schizophrenia says to the nurse, "The world turns as the world turns on a ball at the beach. But all the world's a stagecoach and I took the bus home." The nurse recognizes this statement as an example of which of the following? 1. Concrete thinking [1%] 2. Loose associations [57%] 3. Tangentiality [6%] 4. Word salad [33%] Correct Answered correctly 57% Time: 71 seconds Updated: 05/04/2017 Explanation: Disturbance in logical form of thought is characteristic and one of the positive symptoms of schizophrenia. The client will often have trouble concentrating and maintaining a train of thought. Thought disturbances are often accompanied by a high level of functional impairment, and the client may also be agitated and behave aggressively. Types of impaired thought processes seen in individuals with schizophrenia include the following: Neologisms - made-up words or phrases usually of a bizarre nature; the words have meaning to the client only. Example: "I would like to have a phjinox." Concrete thinking - literal interpretation of an idea; the client has difficulty with abstract thinking. Example: The phrase, "The grass is always greener on the other side," would be interpreted to mean that the grass somewhere else is literally greener (Option 1). Loose associations - rapid shifting from one idea to another, with little or no connection to logic or rationality (Option 2) Echolalia - repetition of words, usually uttered by someone else Tangentiality - going from one topic to the next without getting to the point of the original idea or topic (Option 3) Word salad - a mix of words and/or phrases having no meaning except to the client. Example: "Here what comes table, sky, apple." (Option 4) Clang associations - rhyming words in a meaningless, illogical manner. Example: "The pike likes to hike and Mike fed the bike near the tyke." Perseveration - repeating the same words or phrases in response to different questions Educational objective: Disturbance in thought process (form of thought) is one of the positive symptoms of schizophrenia. The nurse needs to be able to recognize and identify the various types of thought disturbances experienced by clients with schizophrenia. These include loose associations, neologisms, word salad, echolalia, tangentiality, clang association, and perseveration. Copyright © UWorld. All rights reserved.

Block Time Remaining: 00:47:35 TUTOR Test Id: 81012973 QId: 33394 (921666) 66 of 75 A A A A nurse is caring for a 6-year-old client with tonsillitis. Which further assessment finding requires immediate intervention? 1. Dry mucous membranes [13%] 2. Presence of trismus [39%] 3. Pulling at the ears [26%] 4. Sandpaper-like skin rash [20%] Correct Answered correctly 39% Time: 37 seconds Updated: 04/21/2017 Explanation: Trismus (inability to open the mouth due to a tonic contraction of the muscles used for chewing) may indicate a more serious complication of tonsillitis, a peritonsillar or retropharyngeal abscess (collection of pus). Other features include a "hot potato" or muffled voice, pooling of saliva, and deviation of the uvula to one side. This abscess can occlude the airway, making it a medical emergency. Surgical intervention (tonsillectomy or incision and drainage) is often required. In the meantime, maintaining an adequate airway is essential. (Option 1) Dry mucous membranes indicate dehydration. This is to be expected in a child with tonsillitis due to the refusal to swallow. Hydration with IV fluids may be indicated in severe cases. Dehydration requires intervention but is not the priority. (Option 3) Pulling at the ears is a common symptom in children with otitis media. An ear infection frequently accompanies tonsillitis in children and is easily treated with antibiotics. This is not a serious finding. (Option 4) The most common bacterial cause of tonsillitis is group A streptococcal infection. The same organism can also result in scarlet fever, which manifests as fine sandpaper-like skin rash and fever. Antibiotics will effectively treat this condition, and it is not life-threatening. Educational objective: Peritonsillar or retropharyngeal abscess is a serious complication that can result from tonsillitis or pharyngitis. A "hot potato" or muffled voice, trismus (inability to open the mouth), pooling of saliva, and deviation of the uvula to one side are the presenting features. Maintaining an adequate airway is essential. Copyright © UWorld. All rights reserved.

Block Time Remaining: 00:49:39 TUTOR Test Id: 81012973 QId: 31103 (921666) 72 of 75 A A A The nurse is assessing a group of clients in the community health clinic for metabolic syndrome. Which clients exhibit features of the syndrome? Select all that apply. 1. Female with a low-density lipoprotein (LDL) level of 96 mg/dL (2.5 mmol/L) 2. Female with a waist circumference of 38 inches (96.5 cm) 3. Female with blood pressure of 148/90 mm Hg 4. Male with a fasting blood glucose of 99 mg/dL (5.5 mmol/L) 5. Male with a triglyceride level of 201 mg/dL (2.3 mmol/L) Correct Answered correctly 45% Time: 20 seconds Updated: 05/21/2017 Explanation: Individuals with metabolic syndrome (insulin resistance syndrome) have an increased risk of diabetes and coronary artery disease. The presence of abdominal obesity causes increased insulin production (hyperinsulinemia); this excess insulin leads to insulin resistance, the primary feature of metabolic syndrome. Metabolic syndrome is characterized by the presence of 3 or more of the following criteria: Increased waist circumference: ≥40 in (102 cm) in men, ≥35 in (89 cm) in women (Option 2) Blood pressure: ≥130 mm Hg systolic or ≥85 mm Hg diastolic or drug treatment for hypertension (Option 3) Triglyceride level: >150 mg/dL (1.7 mmol/L) or drug treatment for elevated triglycerides (Option 5) High-density lipoprotein (HDL) levels: <40 mg/dL (1.04 mmol/L) in men and <50 mg/dL (1.3 mmol/L) in women or drug treatment for low HDL-C Fasting glucose levels: ≥100 mg/dL (5.6 mmol/L) or drug treatment for elevated blood glucose (Option 4) The mnemonic for metabolic syndrome is "We Better Think High Glucose" (Waist circumference, Blood pressure, Triglyceride, HDL, Glucose). (Option 1) The normal LDL level is <100 mg/dL (2.6 mmol/L); therefore, this client's LDL level is within normal limits. LDL level is not a criterion for diagnosing metabolic syndrome, although a normal level is important for cardiovascular health. Educational objective: Features of metabolic syndrome include increased waist circumference, elevated blood pressure, increased triglycerides, decreased HDL, and increased fasting blood glucose. The mnemonic is "We Better Think High Glucose" (Waist circumference, Blood pressure, Triglyceride, HDL, Glucose). Copyright © UWorld. All rights reserved.

Basic Care & Comfort/Pain Management Postpartum Labor/Delivery Culture / Spirituality Test Id: 72315460 QId: 31945 (921666) 9 of 10 A A A The community health nurse is preparing to teach a group of African American women about prevention of diseases common to their ethnic group. Based on the incidence of disease within this group, which disorders should the nurse plan to discuss? Select all that apply. 1. Cervical cancer 2. Hypertension 3. Ischemic stroke 4. Osteoporosis 5. Skin melanoma Incorrect Correct answer 1,2,3 Answered correctly Explanation: The incidence of cervical cancer is higher among Hispanics, American Indians, and African Americans. The mortality rate for cervical cancer among African American women is twice as high as that for white American women (Option 1). African Americans have the highest incidence of hypertension in the world, and this condition is more prevalent among the women than men in this ethnic group. The mortality rate for hypertension among African American women is higher than that for white American women (Option 2). African Americans have a higher incidence of ischemic stroke than whites or Hispanics. Risk factors for stroke are related to an increased rate of hypertension, diabetes mellitus, and sickle cell anemia (Option 3). (Option 4) White and Asian women have a higher incidence of osteoporosis than African Americans, but the disease affects all ethnic groups. (Option 5) Melanoma of the skin is more common in people who are of white ancestry, light-skinned, and over age 60 with frequent sun exposure. The incidence of melanoma is 10 times higher in white Americans than African Americans. Educational objective: African Americans have the highest incidence of hypertension in the world as well as increased incidence of stroke and cervical cancer. Whites have a high incidence of osteoporosis and skin cancer (melanoma).

Newborn Postpartum Labor/Delivery Developmental stage Test Id: 72315460 QId: 31458 (921666) 10 of 10 A A A Which assessment findings would the nurse most likely expect to find in a male infant born at 28 weeks gestation? Select all that apply. 1. Abundant lanugo on shoulders and back 2. Deep creases and peeling skin on soles of feet 3. Flat areolae without palpable breast buds 4. Smooth, pink skin with visible veins 5. Testes completely descended into the scrotum Incorrect Correct answer 1,3,4 Answered correctly Explanation: A gestational age assessment assists the nurse in providing developmentally appropriate care to preterm newborns. This assessment uses indicators of neuromuscular and physical maturity that are assessed, scored, and added, which correlates to an estimation of gestational age. Lanugo, a fine, downy hair found mostly on the backs and shoulders of preterm newborns, begins disappearing around 36 weeks gestation. At 28 weeks, the newborn has abundant lanugo over most of the body (Option 1). The 28-week newborn also has smooth, pink skin with visible veins as skin is thin and transparent with lack of subcutaneous fat (Option 4). The areolae of extremely premature infants may be barely visible, with no raised breast buds (Option 3). Palpable, raised breast buds measuring 5-10 mm would be expected in newborns closer to term gestation. (Option 2) At 28 weeks gestation, a newborn's feet have very smooth soles with only faint red marks or possibly a single anterior transverse crease. Creases over the entire sole and/or peeling skin would be expected in a full- or post-term (ie, 40+ weeks) newborn. (Option 5) The testes of a male infant born at 28 weeks gestation would not yet have descended into the scrotal sac and may be palpable in the upper inguinal canal. Educational objective: A gestational age assessment assists the nurse in providing developmentally appropriate care for preterm newborns. The newborn at 28 weeks gestation has abundant lanugo; flat areolae without palpable breast buds; and smooth, pink skin with visible veins.

Block Time Remaining: 00:01:18 TUTOR Test Id: 72797956 QId: 30426 (921666) 40 of 75 A A A It is 0700 and the nurse is caring for an 84-year-old client with dementia and a fractured hip. The client has been disoriented to time, place, and person since admission. The client moans frequently and grimaces when moving. He is prescribed morphine IV every 2 hours as needed for pain and was last medicated at 0530. He is scheduled for surgery at 1000 to repair the hip fracture, but the consent has not yet been signed. The client's spouse and child are to arrive at 0900. Which intervention should the nurse carry out first? 1. Administer pain medication [20%] 2. Call the health care provider to meet with the family to obtain informed consent [23%] 3. Complete the preoperative checklist [4%] 4. Perform the morning assessment [51%] Omitted Correct answer 4 Answered correctly 51% Time: 1 seconds Updated: 04/26/2017 Explanation: The morning shift assessment should be completed first to collect baseline assessment data (eg, vital signs, lung sounds, level of consciousness), assess pain, and collect necessary information for the preoperative checklist. (Option 1) Pain medicine is not due until 0730 and can be administered after the initial assessment if necessary. (Option 2) The nurse should call the health care provider after the initial assessment (by 0730) and arrange for a meeting with family members at 0900 to obtain informed consent as the client is not capable of giving it. (Option 3) The preoperative checklist can be completed after consent is obtained. Educational objective: Before surgery, the nurse makes sure informed consent is obtained, performs a complete physical assessment to collect baseline data and determine the client's physiologic and psychologic status, and completes the preoperative checklist. Copyright © UWorld. All rights reserved.

Block Time Remaining: 00:01:19 TUTOR Test Id: 72797956 QId: 32357 (921666) 41 of 75 A A A A fire has started in the respiratory supply cabinet on the medical/surgical floor. Place the nurse's actions in the appropriate order. All options must be used. Your Response/ Incorrect Response Correct Response Rescue/remove any clients in immediate danger Pull the fire alarm Close the door to the supply room, confining the fire Attempt to put out the fire with a fire extinguisher Discourage visitors from using the elevators Omitted Correct answer 5,4,2,1,3 Answered correctly 31% Time: 1 seconds Updated: 02/06/2017 Explanation: Fires can be extremely dangerous in health care facilities, where clients may be incapacitated. Nurses must be aware of the agency's fire safety plan. Most agencies use the mnemonic RACE in their protocols to ensure that all employees perform the priority actions consistently. These actions are: R - Rescue any clients in immediate danger and move them to safety (Option 5) A - Alarm - sound the alarm and activate the agency's fire response (Option 4) C - Confine the fire by closing all doors to all rooms and fire doors to the entrance of the unit (Option 2) E - Extinguish the fire, if possible, with a fire extinguisher (Option 1) Visitors may be discouraged from using the elevators after the other actions have taken place (Option 3). Educational objective: When a fire occurs in a health care agency, the nurse should use the mnemonic RACE to remember the priority steps: R - Rescue any clients in immediate danger; A - Alarm, sound the fire alarm; C - Confine the fire by closing doors; E- Extinguish the fire, if possible, with a fire extinguisher. Copyright © UWorld. All rights reserved.

Block Time Remaining: 00:01:47 TUTOR Test Id: 72797956 QId: 32538 (921666) 51 of 75 A A A The nurse in the oral surgery clinic reviews a client's medical record prior to surgery. Which will the nurse immediately report to the oral surgeon? Select all that apply. 1. Client is on a calorie-restricted diet for obesity 2. Creatinine is 1.3 mg/dL (115 µmol/L) 3. History of congenital heart disease 4. International Normalized Ratio of 2.5 5. Presence of prosthetic valve Omitted Correct answer 3,4,5 Answered correctly 36% Time: 1 seconds Updated: 03/10/2017 Explanation: Clients with a history of congenital heart disease and those with prosthetic valves are at risk for developing infective endocarditis, an infection of the endothelial lining of the heart, with oral surgery and certain procedures (eg, dental work). These clients should receive prophylactic antibiotic therapy prior to any such procedure or surgery. Clients on warfarin therapy due to the presence of prosthetic valves or for other reasons will have a therapeutically elevated International Normalized Ratio (2.0-3.0) to inhibit blood clot formation. However, this will place these clients at risk for excessive bleeding during surgical procedures (Options 3, 4, and 5). (Option 1) A history of obesity and a calorie-restricted diet are not significant for oral surgery. (Option 2) This creatinine level is within normal limits (0.6-1.3 mg/dL [53-115 µmol/L]) and would not require reporting. Educational objective: Prior to oral surgery, it is necessary to report findings that will place a client at risk for the development of endocarditis (eg, presence of prosthetic valves, history of congenital heart disease) and bleeding (eg, elevated International Normalized Ratio). Copyright © UWorld. All rights reserved.

Block Time Remaining: 00:01:48 TUTOR Test Id: 72797956 QId: 30364 (921666) 52 of 75 A A A An experienced nurse precepts a graduate nurse caring for a hospitalized client who has a prescription for a transfusion of packed red blood cells (RBCs) to be hung over 3 hours. Which statement by the graduate nurse indicates the correct rationale for asking the client to void prior to starting the transfusion? 1. "A drop in blood pressure is expected during the transfusion and getting up to void may cause a fall." [24%] 2. "Bedrest is required; therefore, voiding will prevent intermittent catheterization during the procedure." [15%] 3. "If a transfusion reaction occurs, it will be important to collect a fresh urine specimen to check for hemolyzed RBCs." [41%] 4. "The urine is collected and analyzed prior to starting the transfusion to assess the client's baseline results." [17%] Omitted Correct answer 3 Answered correctly 41% Time: 1 seconds Updated: 06/02/2017 Explanation: The nurse should ask the client to void or empty the urinary catheter and discard urine prior to starting a blood transfusion. In the event of an acute hemolytic transfusion reaction, a fresh urine specimen should be collected and sent to the laboratory to analyze for hemolyzed RBCs. An acute hemolytic transfusion reaction is a life-threatening reaction in which the host's antibodies rapidly destroy the transfused RBCs and is generally related to incompatibility. Early signs of a hemolytic reaction include red urine, fever, and hypotension; late signs include disseminated intravascular coagulation and hypovolemic shock. The transfusion should be stopped immediately if any sign of transfusion reaction occurs. Starting the transfusion with an empty bladder will help ensure that any urine specimen collected after a reaction is reflective of the body's physiological processes after the blood transfusion has started (Option 4). (Options 1 and 2) Hypotension is a sign of a transfusion reaction and is not expected. Bedrest is not required, but the client should be assisted if out of bed during a transfusion to prevent falls. Educational objective: An acute hemolytic transfusion reaction is a life-threatening reaction caused primarily by blood incompatibility. If it occurs, the transfusion should be stopped and a fresh urine specimen should be collected and sent to the laboratory to analyze for hemolyzed RBCs. Asking the client to void prior to starting the transfusion helps ensure that any urine specimen collected after a reaction is reflective of the body's physiological processes after the blood transfusion. Copyright © UWorld. All rights reserved.

Block Time Remaining: 00:02:01 TUTOR Test Id: 72797956 QId: 31263 (921666) 58 of 75 A A A Which measures will help prevent falls in the elderly clients of a long-term care facility? Select all that apply. 1. Exercise programs 2. Good room lighting 3. Handrails in stairwell 4. Smooth-soled shoes 5. Staff hourly rounds Omitted Correct answer 1,2,3,5 Answered correctly 36% Time: 6 seconds Updated: 01/30/2017 Explanation: Falls are a leading predictor of mortality and morbidity in older adults. General exercise programs, especially those including gait, balance, and strength training, not only reduce the risk of falls but also prevent injuries from falls (Option 1). Vision impairment can contribute to fall risks; most adults need additional light by age 50. The nurse should ensure that clients are wearing needed prescription glasses (Option 2). Handrails, particularly in stairwells, hallways, and bathrooms, have been shown to reduce falls (Option 3). Studies show that staff rounds at regular intervals (hourly or every other hour) decrease falls and call light use. The practice allows staff to intervene early in needs. Typically staff checks on the "Ps": potty, position, pain, and placement/proximity of personal items (eg, bed height, call light, water, tissues, urinal). A common reason clients get out of bed unassisted is to use the bathroom (Option 5). (Option 4) Non-slip rubber-soled shoes are recommended to prevent falls. Educational objective: Client falls can be prevented with exercise programs, good lighting, handrails, and hourly staff rounds. Copyright © UWorld. All rights reserved.

Block Time Remaining: 00:02:21 TUTOR Test Id: 72797956 QId: 32091 (921666) 64 of 75 A A A The nurse has unlicensed assistive personnel (UAP) caring for a client with an acute attack of Meniere disease. Which action by the UAP will require follow-up by the nurse? 1. Assist the client in ambulating to the bathroom [23%] 2. Dim the room lights [11%] 3. Place the bed in low position with all side rails up [62%] 4. Turn off the television [2%] Omitted Correct answer 3 Answered correctly 62% Time: 1 seconds Updated: 12/25/2016 Explanation: Meniere disease (endolymphatic hydrops) results from excess fluid accumulation in the inner ear. Clients have episodic attacks of vertigo, tinnitus, hearing loss, and aural fullness. The vertigo can be severe and is associated with nausea and vomiting. Clients report feeling being pulled to the ground (drop attacks). Fall precautions that should be instituted include assisting the client when arising and ambulating (Option 1), placing the bed in low position, and raising side rails. However, raising all side rails is considered a restraint and would be inappropriate. The nurse would need to intervene and instruct the UAP that 2 or 3 side rails lifted up would be sufficient (Option 3). (Options 2 and 4) Vertigo may be minimized by staying in a quiet, dark room and avoiding sudden head movements. The client should reduce stimulation by not watching television and not looking at flickering lights. Educational objective: Safety is a priority for the client experiencing an acute attack of Meniere disease. Fall precautions include placing the bed in low position, raising 2 or 3 side rails, and assisting the client with arising and ambulating. Vertigo can be minimized by staying in a quiet, dark room without a television or flickering lights. Copyright © UWorld. All rights reserved.

Block Time Remaining: 00:21:47 TUTOR Test Id: 81114149 QId: 30350 (921666) 41 of 73 A A A A client with an acute head injury cannot accurately identify the sensation felt when the nurse touches the intact skin with a cotton ball or paper clip. The nurse is aware that the deficit reflects injury to which area of the brain? Hotspot Question Incorrect Correct answer Refer to Hotspot Answered correctly 46% Time: 20 seconds Updated: 01/28/2017 Explanation: The parietal lobe of the brain integrates somatic and sensory input. Injury to the parietal lobe could result in a deficit with sensation. The nurse would verify the client's injuries and documented imaging studies to confirm that this was an expected deficit and document it accordingly. If it is a new or unexpected deficit, the nurse should inform the health care provider immediately. The frontal lobe controls higher-order processing, such as executive function and personality. Injury to the frontal lobe often results in behavioral changes. The temporal lobe integrates visual and auditory input and past experiences. Temporal lobe injury clients cannot understand verbal or written language. The occipital lobe of the brain registers visual images. Injury to the occipital lobe could result in a deficit with vision. Educational objective: The frontal lobe controls executive function and personality. The temporal lobe receives auditory input. The parietal lobe receives sensory input. The occipital lobe receives visual images. Copyright © UWorld. All rights reserved.

Block Time Remaining: 00:28:46 TUTOR Test Id: 81114149 QId: 31949 (921666) 58 of 73 A A A The nurse is assessing a newly admitted client on a neurological inpatient unit. Which assessment findings are abnormal and require follow-up by the nurse? Select all that apply. 1. Cannot touch chin to chest 2. Eyes roll in opposite direction when turning head side to side 3. Muscle strength of lower extremities is 3/5 4. Pupils are 8 mm in diameter 5. Toes point downward when noxious stimuli are applied to the sole Incorrect Correct answer 1,3,4 Answered correctly 11% Time: 47 seconds Updated: 12/09/2016 Explanation: The neck should be supple and flex easily, allowing the chin to touch the chest. Nuchal rigidity requires investigation as it is a classic sign of meningeal irritation (Option 1). Normal muscle strength is 5/5. A score of 4 indicates that the muscle is responding but weakened. A score of 3 indicates ability to lift against gravity without resistance applied. A score of 2 indicates lateral movement and inability to lift against gravity. A score of 1 indicates flickering. The reason for the weakness needs to be evaluated (Option 3). Normal pupils are 3-5 mm in diameter. Pupil dilation can be the result of medication, but other serious etiologies must be considered (Option 4). (Option 2) Oculocephalic reflex (doll's eyes) is an expected finding that indicates an intact brain stem. The test is not performed if spinal trauma is suspected. (Option 5) In the Babinski reflex (a normal finding up to age 18 months), the toes fan outward and upward when the sole is stroked. The normal finding in adults is the toes pointing downward with the stimuli (no Babinski). The toes going upward in an adult (Babinski) indicates an upper motor neuron (brain or spinal cord) lesion. Educational objective: Normal pupils are 3-5 mm in diameter. Normal muscle strength is 5/5. The neck should be supple and flex easily, allowing the chin to touch the chest. Copyright © UWorld. All rights reserved.

Block Time Remaining: 00:42:27 TUTOR Test Id: 81012973 QId: 30501 (921666) 58 of 75 A A A A neonate requires respiratory resuscitation. Which is the proper head position of the neonate for rescue breathing? 1. [37%] 2. [6%] 3. [51%] 4. [4%] Incorrect Correct answer 1 Answered correctly 37% Time: 20 seconds Updated: 03/31/2017 Explanation: The neonate should be placed on the back with the neck slightly extended. This is a neutral or "sniffing" position. A blanket or towel roll can be placed under the shoulders, elevating them 0.75-1.0 in (19-25.4 mm) off the mattress. This is particularly useful if the infant has a large occiput from molding or edema. The nurse must watch that the infant's head does not shift to an improper position during caregiving activities. (Option 2) This is underextension of the neck, which can decrease air entry. (Option 3) This is hyperextension of the neck, which can decrease air entry. (Option 4) A blanket or towel under the head results in neck flexion and decreases air entry. Educational objective: Position a neonate with the head in a slightly extended position, as if the infant is "sniffing" a flower, to allow proper air entry for rescue breathing. Copyright © UWorld. All rights reserved.

Block Time Remaining: 00:43:09 TUTOR Test Id: 81012973 QId: 30149 (921666) 59 of 75 A A A The nurse cares for a client with type 1 diabetes mellitus. Which laboratory result is most important to report to the primary health care provider? 1. Fasting blood glucose 99 mg/dL (5.5 mmol/L) [6%] 2. Serum creatinine 2.0 mg/dL (177 µmol/L) [86%] 3. Serum potassium 3.9 mEq/L (3.9 mmol/L) [4%] 4. Serum sodium 140 mEq/L (140 mmol/L) [2%] Correct Answered correctly 86% Time: 41 seconds Updated: 03/22/2017 Explanation: The normal serum creatinine for an adult is 0.6-1.3 mg/dL (53-115 µmol/L). It provides an estimation of the glomerular filtration rate and is an indicator of kidney function. A level of 2 mg/dL (177 µmol/L) is clearly abnormal. The client with diabetes mellitus is at risk for diabetic nephropathy, a complication associated with microvascular blood vessel damage in the kidney. Early treatment and tight control of blood glucose levels are indicated to prevent progressive renal injury in a client with diabetic nephropathy. (Option 1) Normal serum fasting blood glucose is 70-99 mg/dL (3.9-5.5 mmol/L). (Option 3) Normal serum potassium is 3.5-5.0 mEq/L (3.5-5.0 mmol/L). (Option 4) Normal serum sodium is 135-145 mEq/L (135-145 mmol/L). Educational objective: The normal serum creatinine for an adult is 0.6-1.3 mg/dL (53-115 µmol/L). Serum creatinine provides an estimation of the glomerular filtration rate and is an indicator of kidney function. Copyright © UWorld. All rights reserved.

Block Time Remaining: 00:03:01 TUTOR Test Id: 72458431 QId: 33582 (921666) 9 of 10 A A A The clinic nurse reinforces teaching to a client with systemic lupus erythematosus. Which instructions will the nurse include? Select all that apply. 1. Avoid annual influenza vaccination 2. Avoid situations that cause physical and emotional stress 3. Avoid sun exposure and ultraviolet light when possible 4. Notify the health care provider if you have fever 5. Use antibiotic soap to cleanse skin rashes Correct Answered correctly 59% Time: 38 seconds Updated: 05/09/2017 Explanation: Systemic lupus erythematosus (SLE) is an autoimmune disorder (the body's immune system erroneously attacks body tissues) that results in inflammation and damage to many body parts. Symptoms vary widely among affected individuals, but most experience painful/swollen joints, extreme fatigue, skin rashes, and kidney problems. The symptoms typically appear for periods of time (called flares) alternating with periods of remission. There is no cure for SLE, but it can be treated with immunosuppressants (eg, corticosteroids) or immunomodulators (eg, hydroxychloroquine). Pneumonia and annual influenza vaccinations are recommended for those with SLE as they are more susceptible to infections. These individuals should avoid contact with sick people and report fever to their health care provider (Options 1 and 4). Both physical and emotional stress can exacerbate SLE. Therefore, clients should follow a healthy lifestyle (eg, 7-8 hours of sleep, no smoking). Balanced exercise with alternating periods of rest is recommended (Option 2). Sunlight is known to worsen the rash of SLE and should be avoided when possible (especially between 10 AM-4 PM); protective clothing and sunscreen application are recommended during periods of sun exposure (Option 3). (Option 5) The rash of SLE should be cleansed only with mild soap. Harsh soap and chemicals should be avoided. The rash is not due to bacterial infection. Educational objective: Clients with SLE should be advised to avoid harsh sunlight and ultraviolet light exposure as well as harsh soaps and chemicals. These clients often receive corticosteroids and are susceptible to infection; therefore, annual influenza vaccination (eg, killed vaccines) is important. Copyright © UWorld. All rights reserved.

Block Time Remaining: 00:00:00 TUTOR Test Id: 72490738 QId: 30516 (921666) 1 of 75 A A A A client is admitted with an exacerbation of asthma following a respiratory viral illness. Which clinical manifestations characteristic of a severe asthma attack does the nurse expect to assess? Select all that apply. 1. Accessory muscle use 2. Chest tightness 3. Diminished breath sounds bilaterally 4. High-pitched wheezing on expiration 5. Prolonged inspiratory phase 6. Tachypnea Incorrect Correct answer 1,2,3,4,6 Answered correctly 13% Time: 30 seconds Updated: 03/26/2017 Explanation: Asthma is a disease characterized by airway hyper-reactivity and chronic inflammation, resulting in bronchial muscle spasm, mucosal edema, and hypersecretion of mucus. The airways narrow, resulting in increased airway resistance, air trapping, and lung hyperinflation. Characteristic clinical manifestations include the following: Accessory muscle use related to increased work of breathing and diaphragm fatigue Chest tightness related to air trapping Diminished breath sounds related to hyperinflation High-pitched, sibilant wheezing on expiration caused by increased airway resistance; as the condition worsens, wheezing may be heard on both inspiration and expiration Tachypnea, which typically causes respiratory alkalosis initially Cough from inflamed airways and hypersecretion of mucus (Option 5) During an asthma attack, the expiratory phase is prolonged due to air trapping. Educational objective: Asthma exacerbation is characterized by accessory muscle use; chest tightness; diminished breath sounds; high-pitched, sibilant wheezing on expiration; tachypnea; and cough. Copyright © UWorld. All rights reserved.

Labor/Delivery Process of labor, position, presentation, station The nurse is admitting a pregnant client who is experiencing intense "back labor." The nurse suspects the fetus is in which position? 1. [40%] 2. [15%] 3. [7%] 4. [37%] Correct Answered correctly Explanation: A laboring client may experience lower back pain with contractions, or "back labor," when the fetus is in the right occiput posterior (ROP) position. This variation of vertex presentation causes the fetal occiput to exert added pressure on the woman's sacrum during contractions. Positioning the woman on her hands and knees often helps decrease back pain and facilitates fetal rotation into an anterior position. (Option 2) This fetus is in the right occiput anterior (ROA) position, which is optimal for birth as it allows for rotation of the fetal head through the birth canal. (Option 3) This fetus is in the right occiput transverse (ROT) position. When the fetus remains in the OP or OT position, labor is often prolonged. Most fetuses in these positions will rotate spontaneously to the OA position during labor. Manual rotation may be attempted with persistent OP or OT position. (Option 4) Breech presentation, with the fetal feet or buttocks presenting first in the maternal pelvis, does not cause back labor. Potential complications from breech presentation include ineffective dilation of the cervix and increased risk of umbilical cord prolapse. Educational objective: The occiput posterior position is associated with intense "back labor" due to increased pressure on the maternal sacrum and may result in prolonged labor. Occiput anterior position is optimal for fetal rotation and birth. Breech presentation is associated with ineffective cervical dilation and increased risk of umbilical cord prolapse.

Safety/Infection Control Labor/Delivery Fluid and electrolyte care The nurse answers a call light on a client not assigned to the nurse. The client, who was just admitted from the emergency department, requests a cup of coffee. What is the appropriate intervention? 1. Allow a family member to bring the client a cup of coffee from the cafeteria [0%] 2. Ask the client to wait until the health care provider's (HCP's) prescriptions can be verified [64%] 3. Delegate the task to the unlicensed assistive personnel (UAP) assigned to the client [4%] 4. Suggest water instead until admission assessment can be completed [31%] Correct Answered correctly ` Explanation: Because the nurse is unfamiliar with the client, the prescriptions from the HCP should be reviewed before giving any fluids. It is common for clients admitted from the emergency department to be designated nothing by mouth (NPO) until appropriate diagnostics have been completed or in case of possible surgery. Caffeine would be questionable as it can interfere with certain diagnostic tests, such as nuclear cardiac studies. (Option 1) Once it has been verified that the client can have oral fluids and caffeine, a family member could get coffee from the cafeteria. (Option 3) The nurse should verify the HCP's prescriptions; delegation to the UAP is then appropriate. (Option 4) If the client is not NPO, water is a good option until blood glucose can be obtained and if diagnostics affected by caffeine are prescribed. Educational objective: Before giving any client oral fluids, the nurse should verify HCP prescriptions related to oral intake and prescribed diagnostics or procedures.

Postpartum Labor/Delivery Postpartum Complications Test Id: 72315460 QId: 30442 (921666) 3 of 10 A A A The nurse reviews the chart of a client who gave birth 4 hours ago. Which contributing factor indicates that the client has an increased risk of postpartum hemorrhage? 1. Infant birth weight of 9 lb 2 oz (4139 g) [58%] 2. Labor and birth without pain medication [1%] 3. Labor that lasted 8 hours [19%] 4. Third stage of labor lasting 20 minutes [19%] Incorrect Correct answer 1 Answered correctly 58% Time: 53 seconds Updated: 03/29/2017 Explanation: Postpartum hemorrhage (PPH) is usually defined as maternal blood loss of >500 mL after a vaginal birth or >1000 mL after a cesarean birth. Uterine atony, characterized by a soft, "boggy," and poorly contracted uterus, is the most common cause of early PPH (occurring ≤24 hours after birth). Delayed PPH (>24 hours after birth) usually results from retained placental fragments associated with a long third stage of labor (ie, time from birth of baby to expulsion of placenta, lasting >30 minutes). Risk factors for PPH include: History of PPH in prior pregnancy Uterine distension due to: Multiple gestation Polyhydramnios (ie, excessive amniotic fluid) Macrosomic infant (≥8 lb 13 oz [4000 g]) (Option 1) Uterine fatigue (labor lasting >24 hours) High parity Use of certain medications: Magnesium sulfate Prolonged use of oxytocin during labor Inhaled anesthesia (ie, general anesthesia) (Option 2) Natural, unmedicated labor and birth reduces the chance of PPH. (Option 3) Labor lasting <24 hours does not increase the risk for PPH. (Option 4) A third stage of labor lasting <30 minutes does not increase the risk for PPH. Educational objective: Postpartum hemorrhage is defined as maternal blood loss of >500 mL after a vaginal birth or >1000 mL after a cesarean birth. Uterine atony (ie, "boggy" uterus) is the most common cause of early postpartum hemorrhage (occurring ≤24 hours after birth). Risk factors include uterine distension, uterine fatigue, high parity, and certain medications. .

Safety/Infection Control Postpartum Labor/Delivery Isolation Test Id: 72315460 QId: 30107 (921666) 4 of 10 A A A The nurse caring for a client with tuberculosis (TB) transports the client to the radiology department for a chest x-ray. The nurse ensures that the client uses which personal protective equipment when out of the negative-pressure room? 1. Isolation gown, surgical mask, goggles, and gloves [1%] 2. Isolation gown and surgical mask [4%] 3. N95 respirator mask [49%] 4. Surgical mask [45%] Incorrect Correct answer 4 Answered correctly 45% Time: 11 seconds Updated: 12/13/2016 Explanation: Clients with airborne infections such as TB, measles, or chickenpox (varicella) are confined to a negative-pressure room except when traveling to various departments for essential diagnostic procedures or surgery. While being transported through the health care facility, the client on airborne transmission-based precautions wears a surgical mask to protect health care workers (HCWs) and other clients from respiratory secretions. (Options 1 and 2) The client on airborne transmission-based precautions must wear a surgical mask to contain exhaled respiratory secretions. The other personal protective equipment is not necessary. (Option 3) The Centers for Disease Control and Prevention recommends that HCWs who transport clients wear N95 respirator masks as protection against exposure to airborne droplets. N95 respirator masks protect HCWs by removing particles from inhaled air. The client is already infectious and does not require protection from inhaled air. Educational objective: While away from the negative-pressure isolation room, all clients on airborne transmission-based precautions must wear a surgical mask to contain exhaled respiratory secretions. .

Block Time Remaining: 02:46:05 TUTOR Test Id: 72348754 QId: 31676 (921666) 13 of 25 A A A The nurse is preparing a nutritional teaching plan for a client planning to become pregnant. Which foods would best prevent neural tube defects? 1. Calcium-rich snacks [7%] 2. Fortified cereals [68%] 3. Organ meats [16%] 4. Wild salmon [7%] Correct Answered correctly 68% Time: 10 seconds Updated: 01/23/2017 Explanation: Women who are planning on becoming pregnant should consume 400-800 mcg of folic acid daily. Food options that are rich in folic acid include fortified grain products (eg, cereals, bread, pasta) and green, leafy vegetables (Option 2). Inadequate maternal intake of folic acid during the critical first 8 weeks after conception (often before a woman knows she is pregnant) increases the risk of fetal neural tube defects (NTDs), which inhibit proper development of the brain and spinal cord. Common NTDs are spina bifida and anencephaly (lack of cerebral hemispheres and overlying skull). (Option 1) Adequate calcium intake is especially important during the last trimester for mineralization of fetal bones and teeth, but it does not prevent NTDs. (Option 3) Organ meats (eg, liver) may contain moderately high levels of folate but are consumed more for their high iron content, which can promote red blood cell formation and prevent maternal anemia. (Option 4) A prenatal diet rich in omega-3 fatty acids is important for fetal neurologic function and is linked to a lower risk of preterm birth. Dietary sources include wild salmon, anchovies, flaxseed, and walnuts. Educational objective: Women who are planning to become pregnant should consume 400-800 mcg of folic acid daily to prevent neural tube defects (eg, spina bifida, anencephaly). Food options that are rich in folic acid include fortified grain products (eg, cereals, bread, pasta) and green, leafy vegetables. Copyright © UWorld. All rights reserved.

Block Time Remaining: 02:46:05 TUTOR Test Id: 72348754 QId: 31595 (921666) 14 of 25 A A A The obstetric nurse is reviewing phone messages. Which client should the nurse call first? 1. Client at 18 weeks gestation taking ceftriaxone and reporting mild diarrhea [10%] 2. Client at 22 weeks gestation with twins who is taking acetaminophen twice a day [23%] 3. Client at 28 weeks gestation taking metronidazole and reporting dark-colored urine [27%] 4. Client at 32 weeks gestation taking ibuprofen for moderate back pain [38%] Incorrect Correct answer 4 Answered correctly 38% Time: 46 seconds Updated: 01/03/2017 Explanation: Nonsteroidal anti-inflammatory drugs (NSAIDs) (eg, ibuprofen, indomethacin, naproxen) inhibit prostaglandin synthesis and can be taken to decrease pain and inflammation or to reduce fever. NSAIDs are pregnancy category C in the first and second trimesters and pregnancy category D in the third trimester. NSAIDs must be avoided during the third trimester due to the risk of causing premature closure of the ductus arteriosus in the fetus (Option 4). During the first and second trimesters, NSAIDs should be taken only if benefits outweigh risks and under the supervision of a health care provider (HCP). (Option 1) Beta lactam antibiotics (eg, amoxicillin, ceftriaxone [Rocephin]) are pregnancy category B. Diarrhea is a common side effect of beta lactams. Although diarrhea should be reported to the HCP, as it could indicate pseudomembranous (Clostridium difficile) colitis or lead to dehydration if prolonged, this client is not the priority. (Option 2) Acetaminophen (pregnancy category B) is a common pain reliever and/or fever reducer used during pregnancy. Acetaminophen intake should not exceed 4 g per day, including any over-the-counter or prescription combination medications that contain acetaminophen. (Option 3) Metronidazole (Flagyl) is an anti-infective and pregnancy category B. Dark-colored urine is an expected side effect of metronidazole and not cause for concern. Educational objective: Nonsteroidal anti-inflammatory drugs (NSAIDs) should be avoided in the third trimester due to risk of premature closure of the fetal ductus arteriosus. NSAIDs should be taken only under the direction and supervision of a health care provider during the first and second trimesters. Copyright © UWorld. All rights reserved.

Block Time Remaining: 00:30:14 TUTOR Test Id: 81130759 QId: 30341 (921666) 59 of 75 A A A The nurse working in an intensive care unit cares for a client with a left triple lumen subclavian central venous catheter (CVC). The nurse should call the primary health care provider (HCP) for clarification prior to implementation when recognizing that which prescription is an error? 1. Administer intravenous (IV) total parenteral nutrition (TPN) at 50 mL/hr [11%] 2. Change occlusive central line dressing every 7 days [20%] 3. Flush unused lumens of the CVC with 1000 units heparin every 12 hours [53%] 4. Use distal port of CVC to monitor central venous pressure (CVP) [14%] Correct Answered correctly 53% Time: 36 seconds Updated: 05/22/2017 Explanation: Most CVC lumens require anticoagulation in the form of a heparin flush to maintain patency and prevent clotting when not in use. The nurse should check the institution's protocol and the HCP prescription to determine the correct dose. Doses of 2-3 mL containing 10 units/mL-100 units/mL are the standard of care for flushing a CVC. Doses of 1000-10,000 units are given for cases of venous thromboembolism; therefore, this prescription is an error and should be clarified by the nurse. The Centers for Disease Control and Prevention (CDC) recommend that a single-dose vial or prefilled syringe be used to reduce infection risk. Heparin is a high-alert medication (at high risk for causing significant harm to the client if given in error). (Option 1) TPN should be administered through a CVC. Because of its viscosity and high glucose, lipids, electrolytes, vitamins and minerals, it is safest when administered through a CVC or peripherally inserted central catheter. (Option 2) According to the CDC, an occlusive dressing should be changed every 7 days. The nurse should check the institution's protocol for frequency of dressing changes. (Option 4) The distal port of a triple lumen CVC is the largest lumen (tube) and should be used for CVP (right atrium pressure) monitoring. The distal end of the CVC is in reverse as regards the client; therefore, the distal end is at the tip of the catheter in the superior vena cava vein, closest to the right atrium of the heart. Educational objective: Most CVCs require intravenous heparin flushes to maintain patency and prevent clotting. Single-dose vials of 2-3 mL of 10 units/mL or 100 units/mL are the standard of care. A dose of 1000-10,000 units is given for cases of thromboembolism. Copyright © UWorld. All rights reserved.

Block Time Remaining: 00:00:22 TUTOR Test Id: 81142136 QId: 30447 (921666) 1 of 75 A A A Four clients in labor are requesting pain medication from the nurse. Which client can safely receive an opioid agonist-antagonist analgesic intravenous (IV) push at this time? 1. Gravida 1, 2 cm dilated, 50% effaced, contractions 7-10 minutes apart, crying [47%] 2. Gravida 1, 6 cm dilated, 75% effaced, contractions 2-4 minutes apart, has history of heroin use [10%] 3. Gravida 2, 5 cm dilated, 100% effaced, contractions 3-4 minutes apart, moaning and shaking [31%] 4. Gravida 4, 10 cm dilated, 100% effaced, contractions 2-3 minutes apart, wants to push [11%] Incorrect Correct answer 3 Answered correctly 31% Time: 22 seconds Updated: 03/04/2017 Explanation: Stages of labor Stage Definition 1 Latent 0-5 cm cervical dilation Active 6-10 cm cervical dilation 2 10 cm (complete) cervical dilation to birth 3 Birth of baby to expulsion of placenta 4 1-4 hours after birth, maternal physiologic readjustment Systemic analgesia may be administered to the laboring client who is in the active phase of stage 1 labor. Systemic analgesia crosses the blood-brain barrier to provide a central analgesic effect. These medications also cross the placental barrier, with a resulting effect on the fetus depending on dose and time of administration prior to delivery. Parameters for safer administration include the following: Stable maternal vital signs Fetus with heart rate of 110-160 beats/min Well-established labor contractions Cervix dilated to at least 4-5 cm in primipara and 4 cm in multipara Opioid agonist-antagonist medications commonly used in labor are butorphanol tartrate (Stadol) and nalbuphine hydrochloride (Nubain). IV push is the preferred route and is given over the peak of 2 contractions to decrease the bolus of medication to the fetus. During contractions, the uterine muscle is very tense and blood flow to the fetus is slowed. Therefore, medication reaches the fetus at a slower rate. This class of medications has a ceiling effect—after a certain dosage, subsequent or higher doses will not be effective or produce pain relief. Therefore, usually no more than 3 doses will be prescribed. The medications can precipitate withdrawal in opioid-dependent clients and should not be used. This gravida 2 client meets the criteria for medication administration. The client is in the active phase of stage 1 labor and contractions are well established. (Option 1) This client is in the early phase of stage 1 labor. Dilation is 2 cm, and the client is only 50% effaced. The contraction pattern is not well established at 7-10 minutes apart. This phase of labor can continue for up to 8 hours. The nurse can assist this client by using nonpharmacologic methods of pain relief until labor has progressed to the active phase of stage 1. (Option 2) This client has met the criteria for the active phase of stage 1 labor with a well-established contraction pattern and dilation. However, this client is a heroin user, and use of the opioid agonist-antagonist medication could cause withdrawal symptoms in both the client and fetus. This client will benefit from an epidural anesthetic for pain relief or use of nonpharmacologic pain relief methods. (Option 4) This client has now entered stage 2 of labor, which is not the appropriate time to administer IV or intramuscular pain relief methods. Being gravida 4, this client has a risk for delivery at about the time when an IV medication would peak in the fetus. Administering IV medication now will create a risk for respiratory distress at delivery and the need to administer naloxone (Narcan) to the neonate to reverse the effects of the medication. Educational objective: Opioid agonist-antagonist medications commonly used in labor are butorphanol tartrate and nalbuphine hydrochloride. These medications can precipitate withdrawal in opioid-dependent clients and should not be used. They are used in the active phase of stage 1 labor when labor contractions are well established and the cervix is dilated to at least 4 cm. Copyright © UWorld. All rights reserved.

Block Time Remaining: 00:02:01 TUTOR Test Id: 72797956 QId: 31263 (921666) 58 of 75 A A A Which measures will help prevent falls in the elderly clients of a long-term care facility? Select all that apply. 1. Exercise programs 2. Good room lighting 3. Handrails in stairwell 4. Smooth-soled shoes 5. Staff hourly rounds Omitted Correct answer 1,2,3,5 Answered correctly 36% Time: 6 seconds Updated: 01/30/2017 Explanation: Falls are a leading predictor of mortality and morbidity in older adults. General exercise programs, especially those including gait, balance, and strength training, not only reduce the risk of falls but also prevent injuries from falls (Option 1). Vision impairment can contribute to fall risks; most adults need additional light by age 50. The nurse should ensure that clients are wearing needed prescription glasses (Option 2). Handrails, particularly in stairwells, hallways, and bathrooms, have been shown to reduce falls (Option 3). Studies show that staff rounds at regular intervals (hourly or every other hour) decrease falls and call light use. The practice allows staff to intervene early in needs. Typically staff checks on the "Ps": potty, position, pain, and placement/proximity of personal items (eg, bed height, call light, water, tissues, urinal). A common reason clients get out of bed unassisted is to use the bathroom (Option 5). (Option 4) Non-slip rubber-soled shoes are recommended to prevent falls. Educational objective: Client falls can be prevented with exercise programs, good lighting, handrails, and hourly staff rounds. Copyright © UWorld. All rights reserved.

Block Time Remaining: 00:01:10 TUTOR Test Id: 81012973 QId: 30205 (921666) 2 of 75 A A A Exhibit A client with diabetes and an infected heel ulcer is transferred to the intensive care unit because of deteriorating condition. Based on the admission assessment, what does the nurse identify as the most likely condition? Click the exhibit button for additional information. 1. Multiple organ dysfunction syndrome (MODS) [5%] 2. Sepsis [20%] 3. Septic shock [64%] 4. Systemic inflammatory response syndrome (SIRS) [10%] Correct Answered correctly 64% Time: 57 seconds Updated: 02/03/2017 Explanation: The presence of infection with a gram-positive wound culture increases the risk for developing sepsis and septic shock. Sepsis-induced hypotension despite adequate fluid resuscitation (30 mL/kg) is defined as "septic shock." Hypotension and inadequate tissue perfusion (ie, elevated serum lactate level) despite fluid resuscitation and decreased central venous pressure (ie, decreased circulating volume) and pulmonary artery wedge pressure (ie, decreased preload) indicate the presence of septic shock in this client. Based on the alterations in hemodynamic parameters, septic shock is the most likely condition in this client. (Option 1) MODS is the failure of 2 or more body organs (eg, acute kidney injury, acute respiratory distress syndrome). Septic shock can progress to multiorgan dysfunction (ie, severe end of sepsis and septic shock). However, the assessment data do not indicate that it is the most likely condition at this time. (Option 2) Sepsis is a systemic inflammatory response (ie, increased heart rate, respirations, temperature, and decreased systolic blood pressure) to a documented or suspected infection and is present in this client. However, it is not the most likely condition because the assessment data support progression along the sepsis continuum to septic shock. (Option 4) SIRS is a generalized inflammatory response to an infectious or noninfectious insult to the body. It is often difficult to distinguish from early sepsis. When SIRS is suspected, a source for sepsis should be sought. Educational objective: SIRS → Generalized inflammatory response to an infectious or noninfectious insult to the body Sepsis → Presence (probable or documented) of infection along with systemic manifestations of infection Septic shock → Sepsis-induced hypotension despite adequate fluid resuscitation (30 mL/kg) Copyright © UWorld. All rights reserved.

Block Time Remaining: 00:02:47 TUTOR Test Id: 72797956 QId: 31380 (921666) 73 of 75 A A A The nurse is drawing a blood specimen from a client's central line. Identify the steps necessary to prevent transmission of infection while performing this procedure. Select all that apply. 1. Discard the first 6-10 mL of blood drawn from the line 2. Flush the line with sterile normal saline before and after collection 3. Perform hand hygiene 4. Place the specimen in a biohazard bag 5. Scrub the catheter hub with antiseptic prior to use Omitted Correct answer 3,4,5 Answered correctly 19% Time: 7 seconds Updated: 06/02/2017 Explanation: Blood and bodily fluids are considered hazardous materials and must be placed in containers identifying them as biohazards (eg, biohazard bag) (Option 4). This alerts staff to take the necessary precautions to prevent infection transmission when handling the specimen. Other procedures to prevent transmission of infection include: Meticulous hand hygiene (Option 3) Use of disposable gloves during collection and handling of specimen Cleaning the specimen bag with a disinfecting wipe Proper and immediate transport of specimen to the lab Avoiding placing specimen in clean areas (eg, nursing station) An appropriate antiseptic (eg, 70% alcohol) scrub of the catheter hub prior to use inhibits microorganism entry and prevents transmission of infection to the client (Option 5). (Option 1) When drawing a blood specimen from a central line, the nurse should discard the first blood drawn to prevent an inaccurate lab result, but this will not prevent the transmission of infection. (Option 2) Flushing the line prior to specimen collection will clear any previous infusions and assist in checking patency. It is important to flush the line after collection to remove blood and prevent clotting. Neither action prevents infection transmission. Educational objective: When handling blood or body fluid specimens, the nurse should adhere strictly to protocols that will prevent the transmission of infection, including meticulous hand hygiene and use of gloves. All body fluid specimens should be transported immediately in a container labeled with the biohazard symbol. Copyright © UWorld. All rights reserved.

Block Time Remaining: 00:01:36 TUTOR Test Id: 81012973 QId: 31645 (921666) 3 of 75 A A A The nurse is caring for a client with surgical complications who requires continuous total parenteral nutrition (TPN). The nurse assists the health care provider with the insertion of a subclavian triple lumen central venous access device. What is the nurse's priority action before initiating the TPN infusion? 1. Attach a filter to the IV tubing [3%] 2. Check baseline fingerstick glucose levels [22%] 3. Check the results of the portable chest x-ray [71%] 4. Program the electronic infusion pump [2%] Correct Answered correctly 71% Time: 26 seconds Updated: 05/27/2017 Explanation: The priority action after placing a subclavian central venous catheter is to check the results of the chest x-ray to ensure that the catheter tip is placed correctly in the superior vena cava. Obtain verification before using the catheter as perforation of the visceral pleura can occur during insertion and lead to an iatrogenic pneumothorax or hemothorax. Although these complications are rare, due to the use of ultrasound to guide insertion, if present, the TPN would infuse into the pleural space. (Option 1) Filters are used during TPN infusion to remove particulate matter, precipitates, or microorganisms. Because TPN uses a hypertonic solution (eg, dextrose concentration >10%), it increases the risk for infection. Therefore, the use of a 0.22-micron filter is recommended to remove microorganisms when administering a lipid-free formula. The use of a 1.2-micron filter is recommended when administering TPN with lipids. Although this action is appropriate, it is not the priority. (Option 2) The nurse should monitor the baseline blood glucose (BG) level and fingerstick BG every 6 hours while the client is receiving TPN; it should be maintained in the range of 140-180 mg/dL (7.8-10.0 mmol/L) for a hospitalized adult client. This action is appropriate, but it is not the priority. (Option 4) TPN is always administered using an electronic infusion pump to ensure an accurate and consistent hourly infusion rate to help avoid fluctuations in BG levels. This action is appropriate, but it is not the priority. Educational objective: Incorrect placement of a subclavian central venous catheter can result in an iatrogenic pneumothorax or hemothorax. The priority is to check the results of the chest x-ray to verify that the catheter tip has been placed correctly in the superior vena cava. Other appropriate actions include attaching a filter to the IV tubing, monitoring baseline and fingerstick BG levels every 6 hours, and programming the electronic infusion device to ensure an accurate and consistent hourly infusion rate. Copyright © UWorld. All rights reserved.

Block Time Remaining: 00:01:49 TUTOR Test Id: 72797956 QId: 30869 (921666) 53 of 75 A A A A client receives intermittent bolus enteral feedings through a nasogastric tube. Which are appropriate nursing actions prior to starting the feeding? Select all that apply. 1. Discard aspirated residual volume in a biohazard container 2. Flush the tube before and after the feeding 3. Place the client in the semi-Fowler position 4. Start the feeding after obtaining a gastric residual volume <100 mL 5. Start the feeding when the residual volume has pH of 6 Omitted Correct answer 2,3,4 Answered correctly 46% Time: 1 seconds Updated: 01/03/2017 Explanation: The head of the bed should be elevated to a minimum of 30 degrees (semi-Fowler position) during enteral feedings and for 30-60 minutes afterward, thereby decreasing aspiration risk. Many institutions have policies that require the nurse to hold the feeding if the client must be supine (eg, diagnostic tests). Gastric residual volumes are checked every 4 hours with continuous feeding or before each intermittent feeding and medication administration. Continuing feedings despite a large volume residual increases the client's risk for emesis and aspiration. Recent evidence suggests that holding the feeding for a residual volume >100 mL is not necessary, and some institutional policies allow a residual volume of up to >500 mL as long as the client is asymptomatic. Flush the tube before and after bolus feedings to keep the tube patent and avoid contamination of the stagnant feeding solution. Sterile fluid is used to help prevent infection in vulnerable clients. (Option 1) Aspirated residual volume should be returned to the stomach. If acidic gastric juices are repeatedly discarded (2,500 mL secreted daily), there is risk for metabolic alkalosis and hypokalemia. (Option 5) Gastric pH should be acidic (pH ≤5). A pH ≥6 requires x-ray confirmation of tube placement. In addition, any newly inserted nasogastric tube requires x-ray confirmation of tube location. Educational objective: Care of a client receiving enteral feedings requires elevating the head of the bed, monitoring for acidic pH ≤5, assessing for excessive residual volume, returning aspirated residual volume to the stomach, and flushing the tubing before and after bolus intermittent feeding. Copyright © UWorld. All rights reserved.

Block Time Remaining: 00:01:50 TUTOR Test Id: 72797956 QId: 30746 (921666) 54 of 75 A A A The nurse notes muffled heart tones in a client with a pericardial effusion. How would the nurse assess for a pulsus paradoxus? 1. Check for variation in amplitude of QRS complexes on the electrocardiogram strip [7%] 2. Compare apical and radial pulses for any deficit [48%] 3. Measure the difference between Korotkoff sounds auscultated during expiration and throughout the respiratory cycle [26%] 4. Multiply diastolic blood pressure (DBP) by 2, add systolic blood pressure (SBP), and divide the result by 3; [(DBP x 2) + (SBP)]/3 [17%] Omitted Correct answer 3 Answered correctly 26% Time: 1 seconds Updated: 01/26/2017 Explanation: Muffled heart tones in a client with pericardial effusion can indicate the development of cardiac tamponade. This results in the build-up of fluid in the pericardial sac, which leads to compression of the heart. Cardiac output begins to fall as cardiac compression increases, resulting in hypotension. Additional signs and symptoms of tamponade include tachypnea, tachycardia, jugular venous distension, narrowed pulse pressure, and the presence of a pulsus paradoxus. Pulsus paradoxus is defined as an exaggerated fall in systemic BP >10 mm Hg during inspiration. The procedure for measurement of pulsus paradoxus is as follows: Place client in semirecumbent position Have client breathe normally Determine the SBP using a manual BP cuff Inflate the BP cuff to at least 20 mm Hg above the previously measured SBP Deflate the cuff slowly, noting the first Korotkoff sound during expiration along with the pressure Continue to slowly deflate the cuff until you hear sounds throughout inspiration and expiration; also note the pressure Determine the difference between the 2 measurements in steps 5 and 6; this equals the amount of paradox The difference is normally <10 mm Hg, but a difference >10 mm Hg may indicate the presence of cardiac tamponade. (Option 1) Variation in QRS amplitude is termed electrical alternans. It could be present in cardiac tamponade, but it is not how pulsus paradoxus is determined. Electrical alternans is due to the swinging motion of the heart in a fluid-filled pericardial sac. (Option 2) An apical/radial pulse deficit may be present during certain dysrhythmias, but this is not the procedure for measuring pulsus paradoxus. (Option 4) This is the formula for calculating mean arterial pressure. Educational objective: The nurse should assess the client for pulsus paradoxus when cardiac tamponade is suspected. The amount of paradox is the difference between the pressure heard at the first Korotkoff sound during expiration and the Korotkoff sounds heard throughout inspiration and expiration. A difference of <10 mm Hg is normal, but if it is >10 mm Hg, this may indicate cardiac tamponade. Copyright © UWorld. All rights reserved.

Block Time Remaining: 00:08:57 TUTOR Test Id: 81142136 QId: 32840 (921666) 14 of 75 A A A Which client should the nurse assess first after receiving the hand-off morning report? 1. Client 1 day postoperative exploratory abdominal laparotomy who has a nasogastric tube and absent bowel sounds in 4 quadrants [15%] 2. Client with a peripherally inserted central catheter who has a 5-cm (2-in) increase in external catheter length since yesterday [70%] 3. Client with chronic diarrhea from malabsorption syndrome who is receiving 10% dextrose in water via a peripheral IV line [5%] 4. Client with type 2 diabetes mellitus who is scheduled for discharge and has a hemoglobin A1C level of 9% [7%] Correct Answered correctly 70% Time: 41 seconds Updated: 04/08/2017 Explanation: A peripherally inserted central catheter (PICC) is inserted via the basilic or cephalic veins into the superior vena cava. The nurse should measure and document the external length of the PICC during dressing changes. A change in the length of the external portion of the catheter can indicate migration of the tip of the catheter from its original position. The nurse should hold IV fluids and medications, secure the PICC to prevent further movement, and notify the health care provider for x-ray evaluation of catheter tip placement. (Option 1) After abdominal surgery, placement of a nasogastric tube to decompress the stomach and the absence of bowel sounds for 24-72 hours due to postoperative paralytic ileus would be expected. (Option 3) The client with malabsorption syndrome is unable to digest and absorb nutrients by the gastrointestinal tract. Peripheral parenteral nutrition with 10% dextrose is an expected treatment. (Option 4) The hemoglobin A1C level of 9% is above the recommended level (ie, <7%) and reflects inadequate glycemic control, which can be expected in a client with diabetes mellitus. Educational objective: A change in the length of the external portion of a peripherally inserted central catheter (PICC) can indicate migration of the catheter from its original position. If migration is suspected, the nurse should hold IV fluids and medications, secure the PICC to prevent further movement, and notify the health care provider immediately for evaluation and x-ray verification of placement of the catheter tip. Copyright © UWorld. All rights reserved.

Block Time Remaining: 00:09:20 TUTOR Test Id: 81142136 QId: 30787 (921666) 15 of 75 A A A The nurse administers IV vancomycin to a client with a methicillin-resistant Staphylococcus aureus infection. Which nursing actions are most appropriate? Select all that apply. 1. Assess client for lethargy and decreased deep tendon reflexes 2. Assess skin for flushing and red rash on face and torso 3. Infuse medication over at least 60 minutes 4. Monitor blood pressure during infusion 5. Observe IV site every 30 minutes for pain, redness, and swelling Incorrect Correct answer 2,3,4,5 Answered correctly 11% Time: 23 seconds Updated: 06/03/2017 Explanation: When administering IV vancomycin, the nurse should assess for and work to prevent possible complications by performing the following: Draw the prescribed trough level prior to administration. Therapeutic vancomycin levels range from 10-20 mg/L (6.9-13.8 µmol/L) for hemodynamically stable clients. Adverse effects of vancomycin toxicity include nephrotoxicity (eg, elevated creatinine levels) and ototoxicity (eg, hearing loss, vertigo, tinnitus). Infuse medication over at least 60 minutes (≤10 mg/min). Faster rates increase the likelihood of complications (Option 3). Monitor blood pressure during the infusion. Hypotension is a possible adverse effect (Option 4) Assess for hypersensitivity. Red man syndrome is a nonallergic histamine reaction characterized by sudden onset of severe hypotension, flushing, and/or maculopapular rash of the face, neck, chest, and upper extremities (Option 2). Monitor for anaphylaxis (eg, rash, pruritus, laryngeal edema, wheezing). Observe IV site every 30 minutes for pain, redness, or swelling. Vancomycin is a vesicant and may cause thrombophlebitis or, if extravasation occurs, tissue necrosis. Administration using a central venous catheter is preferred; however, a peripheral IV may be used for short-term therapy (Option 5). (Option 1) Assessment of deep tendon reflexes is appropriate with magnesium sulfate administration. Manifestations of hypermagnesemia include lethargy, nausea, vomiting, and decreased deep tendon reflexes. Educational objective: Nursing care of clients receiving IV vancomycin includes drawing prescribed trough levels before drug administration, infusing the drug over at least 60 minutes, monitoring the client during administration (eg, blood pressure, respiratory status, signs of hypersensitivity/anaphylaxis), and assessing the IV site during and after administration. Copyright © UWorld. All rights reserved.

Block Time Remaining: 00:08:33 TUTOR Test Id: 81012973 QId: 31723 (921666) 10 of 75 A A A The charge nurse should intervene if the new graduate nurse performs which action when caring for a jaundiced newborn being treated with phototherapy? 1. Allowing the parents to feed the newborn [2%] 2. Applying a shirt while the newborn is exposed to phototherapy [88%] 3. Assessing the temperature of the incubator while the newborn is inside [5%] 4. Covering the newborn's eyes with protective shields [3%] Correct Answered correctly 88% Time: 18 seconds Updated: 02/26/2017 Explanation: Phototherapy is the use of fluorescent lights to treat hyperbilirubinemia or jaundice in newborns. The light is absorbed by the newborn's skin and converts bilirubin into a water-soluble form, allowing it to be excreted in the stool and urine. The newborn should be fully exposed, except for a diaper, when placed under the phototherapy lights. Lotions and ointments should not be applied as they can absorb the heat and cause burns. Maintaining skin integrity is important as bilirubin products in the stool can cause loose stool with frequency and produce skin excoriation and breakdown. (Option 1) Allowing parents to feed the newborn promotes bonding. The newborn should not be removed from the lights except during feedings for optimal effect of the phototherapy. Adequate hydration with human milk or infant formula (not water) is important as infants are prone to dehydration from phototherapy. (Option 3) Temperature should be monitored closely, with the incubator placed on a low-heat setting. (Option 4) The newborn's eyes should be covered with patches or guards to prevent retinal damage or cataracts when under the phototherapy lights. Educational objective: The newborn should be fully exposed, except for a diaper, when placed under phototherapy lights. Lotions and ointments should not be applied as they can absorb heat and cause burns. Newborns should wear eye shields and be monitored for adequate hydration and urine output. Copyright © UWorld. All rights reserved.

Block Time Remaining: 00:09:52 TUTOR Test Id: 81012973 QId: 31587 (921666) 12 of 75 A A A A client who reports sudden-onset severe right lower abdominal pain and dizziness is being evaluated for suspected ectopic pregnancy. Which assessment findings should the nurse anticipate? Select all that apply. 1. Blood pressure 82/64 mm Hg 2. Crackles on auscultation 3. Distended jugular veins 4. Pulse 120/min 5. Shoulder pain Incorrect Correct answer 1,4,5 Answered correctly 46% Time: 45 seconds Updated: 02/23/2017 Explanation: Ectopic pregnancy occurs when a fertilized ovum implants outside the uterine cavity. The majority of ectopic pregnancies occur in the fallopian tubes. Risk factors are sexually transmitted infections, tubal damage or infection, intrauterine devices, and tubal surgeries. Clinical manifestations are lower-quadrant abdominal pain on one side, mild to moderate vaginal bleeding, and missed or delayed menses. Signs of subsequent hypovolemic (hemorrhagic) shock from tubal rupture include dizziness, hypotension, tachycardia, and decreased urinary output to <30 mL/hr. Free intraperitoneal blood pooling under the diaphragm can cause referred shoulder pain. Peritoneal signs (eg, tenderness, rigidity, low-grade fever) develop subsequently. (Options 2 and 3) Distended jugular veins and lung crackles indicate volume overload conditions. The main risk with ectopic pregnancy is hypovolemic (internal hemorrhage) shock. Jugular veins would be flat in hypovolemic shock. Educational objective: The most common site for an ectopic pregnancy is the fallopian tubes. As the ovum grows and expands, tubal rupture may occur and result in active bleeding that progresses to life-threatening hypovolemic (hemorrhagic) shock. Copyright © UWorld. All rights reserved.

Block Time Remaining: 00:11:13 TUTOR Test Id: 81012973 QId: 31774 (921666) 14 of 75 A A A The nurse is conducting intake interviews at the clinic. Which client situations would require the nurse to intervene? Select all that apply. 1. Client with iron deficiency anemia takes iron supplements with milk 2. Client takes levothyroxine early in the morning on an empty stomach 3. Client taking phenazopyridine for urine infection states that the urine has turned orange 4. Client taking metronidazole mentions going to a wine-tasting party tonight 5. Client with closed-angle glaucoma takes over-the-counter diphenhydramine for a cold Correct Answered correctly 59% Time: 64 seconds Updated: 04/06/2017 Explanation: Iron is absorbed better on an empty stomach; ascorbic acid (vitamin C), such as found in citrus fruits and juices, increases the absorption of iron. However, milk products decrease iron absorption and should be avoided (Option 1). Metronidazole (Flagyl) is used to treat trichomoniasis and amebiasis. Consuming alcohol while taking the medication may elicit a disulfiram (Antabuse)-like reaction. Alcohol should be avoided for at least 48 hours after treatment is completed (Option 4). Many antihistamines also have anticholinergic effects. Anticholinergics have an antimuscarinic effect that can increase intraocular pressure and are therefore contraindicated in closed-angle glaucoma. Other contraindications include urinary retention (benign prostatic hyperplasia) and bowel obstruction related to the anticholinergic drug's effect on the smooth muscle in the urinary and gastrointestinal tract (Option 5). (Option 2) Enteral nutrition decreases levothyroxine absorption; as a result, it should be taken early in the morning on an empty stomach (at least 30 minutes before food intake). (Option 3) Phenazopyridine (Pyridium) is used as a local anesthetic in the treatment of urinary tract infection. The azo dye turns the urine an orange-red color. The client needs to be reassured that this is an expected result and could stain clothing. Educational objective: Clients taking metronidazole (Flagyl) should avoid alcohol. Those with glaucoma or urinary retention should avoid anticholinergic drugs. Oral iron is better absorbed on an empty stomach and with vitamin C. Phenazopyridine (Pyridium) will turn urine an orange-red color. Copyright © UWorld. All rights reserved.

Block Time Remaining: 00:13:12 TUTOR Test Id: 81012973 QId: 30035 (921666) 17 of 75 A A A Which instructions should the nurse include when providing discharge teaching to a client with peptic ulcer disease due to Helicobacter pylori infection? (Select all that apply.) 1. "Avoid foods that may cause epigastric distress such as spicy or acidic foods." 2. "It is best if you refrain from consuming alcohol products." 3. "Report black tarry stools to your health care provider immediately." 4. "Take your amoxicillin, clarithromycin, and omeprazole for the next 14 days." 5. "You may take over-the-counter drugs such as aspirin if you have mild epigastric pain." Correct Answered correctly 49% Time: 37 seconds Updated: 04/26/2017 Explanation: Client teaching related to peptic ulcer disease (PUD) includes lifestyle changes (eg, dietary modifications, stress reduction), PUD complications, and medication administration. Helicobacter pylori infection and treatment with nonsteroidal anti-inflammatory drugs (NSAIDs) are risk factors for complicated PUD. H pylori treatment includes antibiotics and proton-pump inhibitors for acid suppression. The recommended initial treatment is 7-14 days of triple-drug therapy with omeprazole (Prilosec), amoxicillin, and clarithromycin (Biaxin). (Option 5) Clients with PUD should avoid NSAIDs [eg, aspirin, ibuprofen (Motrin)] as they inhibit prostaglandin synthesis, increase gastric secretion, and reduce the integrity of the mucosal barrier. Educational objective: Clients with peptic ulcer disease should avoid NSAIDs, smoking, and excess use of alcohol or caffeine. Copyright © UWorld. All rights reserved.

Block Time Remaining: 00:07:02 TUTOR Test Id: 81114149 QId: 31012 (921666) 15 of 73 A A A The nurse completes a neurological examination on a client who has suffered a stroke to determine if damage has occurred to any of the cranial nerves. The nurse understands that damage has occurred to cranial nerve IX based on which assessment finding? 1. A tongue blade is used to touch the client's pharynx; gag reflex is absent [65%] 2. Only one side of the mouth moves when the client is asked to smile and frown [12%] 3. The absence of light touch and pain sensation on the left side of the client's face [8%] 4. When the client shrugs against resistance, the left shoulder is weaker than the right [13%] Correct Answered correctly 65% Time: 29 seconds Updated: 12/19/2016 Explanation: Cranial nerves IX (glossopharyngeal) and X (vagus) are related to the movement of the pharynx and tongue. To evaluate cranial nerves IX and X, the nurse assesses for the presence of a gag reflex and symmetrical movements of the uvula and soft palate, and listens to voice quality. A tongue blade can be used to touch the posterior pharyngeal wall to assess for a gag reflex. Asking the client to say "ah" will allow assessment of the uvula and soft palate. Harsh or brassy voice quality indicates dysfunction with the vagus nerve (X) (Option 1). (Option 2) The facial nerve (VII) is assessed by observing for symmetrical movements during facial expressions (eg, smile, frown, close eyes). (Option 3) The trigeminal nerve (V) has both sensory and motor functions. The nurse assesses for equal jaw strength by palpating the masseter muscle while the client clenches the jaw. To assess sensory function, the nurse touches the client's face with the client's eyes closed to determine if sensations are equal. (Option 4) The spinal accessory nerve innervates the sternocleidomastoid and part of the trapezius muscles. The nurse applies resistance during shrugging and head turning and assesses for equal strength. Educational objective: Cranial nerves IX and X work together to create movement of the pharynx and tongue. An absent gag reflex, asymmetrical uvular and palate movement, or a change in voice quality indicates damage. Copyright © UWorld. All rights reserved.

Block Time Remaining: 00:13:49 TUTOR Test Id: 81114149 QId: 30864 (921666) 29 of 73 A A A A client is brought to the emergency department with stroke symptoms that began 7 hours ago. A CT scan confirms the presence of an ischemic stroke. The client's current blood pressure is 202/108 mm Hg. Which nursing action is most appropriate? 1. Anticipate IV labetalol to keep blood pressure <140/90 mm Hg [50%] 2. Document the current findings in the client's chart [9%] 3. Prepare to administer thrombolytic therapy [28%] 4. Request a prescription for IV antiseizure medication [12%] Incorrect Correct answer 2 Answered correctly 9% Time: 31 seconds Updated: 02/07/2017 Explanation: After an acute ischemic stroke, the body senses the need for increased pressure to perfuse the penumbra (area of swollen and vulnerable but salvageable brain tissue) to keep the stroke from extending. This phenomenon is a protective mechanism ("permissive hypertension") that usually autocorrects within 24-48 hours and does not require treatment. The usual goal is to maintain systolic blood pressure ≥170 mm Hg to ensure adequate cerebral perfusion. Treatment would be indicated for blood pressure >220/120 mm Hg or for hemorrhagic strokes. (Option 1) IV antihypertensives (eg, labetalol [Normodyne], nicardipine [Cardene]) are often used if medication is required for acute stroke treatment. However, rapid blood pressure reduction may prolong the stroke. Blood pressure should be lowered cautiously by about 15% within the first 24 hours. (Option 3) This client is outside the time range for thrombolytic use for stroke treatment (<3-4.5 hours for systemic tissue plasminogen activator [tPA] and 6 hours for intra-arterial tPA). Also, blood pressure must be <185/110 mm Hg prior to administering tPA and must be maintained at ≤180/105 mm Hg for ≥24 hours after starting fibrinolytic therapy. (Option 4) Seizure medication is not routinely administered prophylactically to stroke victims. A more important concern is to monitor for signs of increased intracranial pressure, which could lead to seizures. Educational objective: "Permissive hypertension" is allowed within the first 24-48 hours of an acute ischemic stroke provided that the blood pressure is <220/120 mm Hg. This practice allows adequate cerebral perfusion to keep the stroke from extending. Copyright © UWorld. All rights reserved.

Block Time Remaining: 00:04:19 TUTOR Test Id: 81075711 QId: 32209 (921666) 13 of 75 A A A The nurse reviews the medication administration records and laboratory results for assigned clients. Which medication requires that the health care provider be notified before administration? 1. Calcium acetate for a client with a phosphate level of 8.5 mg/dL (2.75 mmol/L) [11%] 2. Clopidogrel for a client with a platelet count of 70,000/mm3 (70 × 109/L) [72%] 3. Magnesium sulfate for a client with a magnesium level of 1.0 mEq/L (0.5 mmol/L) [8%] 4. Metformin for a client with a glycosylated hemoglobin level of 11% [7%] Omitted Correct answer 2 Answered correctly 72% Time: 31 seconds Updated: 02/17/2017 Explanation: Clopidogrel (Plavix) is a platelet aggregation inhibitor used to prevent blood clot formation in clients with recent myocardial infarction, acute coronary syndrome, cardiac stents, stroke, or peripheral vascular disease. Because it can cause thrombocytopenia and increase the risk for bleeding, the nurse should notify the health care provider (HCP) of the low platelet count (normal: 150,000-400,000/mm3 [150-400 × 109/L]) before administering clopidogrel. (Option 1) Calcium acetate (PhosLo) is used to control hyperphosphatemia in clients with end-stage kidney disease by binding to phosphate in the intestines and excreting it in the stool. Because the phosphate level is high (normal adult: 2.4-4.4 mg/dL [0.78-1.42 mmol/L]), it is not necessary to notify the HCP. (Option 3) Magnesium sulfate is used to correct hypomagnesemia and treat torsades de pointes and seizures associated with eclampsia. Because the magnesium level is low (normal adult: 1.5-2.5 mEq/L [0.75-1.25 mmol/L]), it is not necessary to notify the HCP. (Option 4) Metformin (Glucophage) is a first-line drug for the control of blood sugar in clients with type 2 diabetes mellitus. Glycosylated hemoglobin (A1C) measures the total hemoglobin that has glucose attached to it, expressed as a percentage. Glucose remains attached to the red blood cell for the life of the cell (about 120 days) and reflects glycemic control over an extended period. The recommended A1C level for a client with diabetes is <7%. Although the A1C level is elevated, the medication would be administered regardless of the result (unless the client is hypoglycemic), so it is not necessary to notify the HCP. Educational objective: Clopidogrel (Plavix) can cause thrombocytopenia (platelet count <150,000/mm3 [150 × 109/L]) and increase a client's risk for bleeding. Copyright © UWorld. All rights reserved.

Block Time Remaining: 00:14:33 TUTOR Test Id: 81075711 QId: 30442 (921666) 27 of 75 A A A The nurse reviews the chart of a client who gave birth 4 hours ago. Which contributing factor indicates that the client has an increased risk of postpartum hemorrhage? 1. Infant birth weight of 9 lb 2 oz (4139 g) [58%] 2. Labor and birth without pain medication [1%] 3. Labor that lasted 8 hours [19%] 4. Third stage of labor lasting 20 minutes [19%] Correct Answered correctly 58% Time: 26 seconds Updated: 03/29/2017 Explanation: Postpartum hemorrhage (PPH) is usually defined as maternal blood loss of >500 mL after a vaginal birth or >1000 mL after a cesarean birth. Uterine atony, characterized by a soft, "boggy," and poorly contracted uterus, is the most common cause of early PPH (occurring ≤24 hours after birth). Delayed PPH (>24 hours after birth) usually results from retained placental fragments associated with a long third stage of labor (ie, time from birth of baby to expulsion of placenta, lasting >30 minutes). Risk factors for PPH include: History of PPH in prior pregnancy Uterine distension due to: Multiple gestation Polyhydramnios (ie, excessive amniotic fluid) Macrosomic infant (≥8 lb 13 oz [4000 g]) (Option 1) Uterine fatigue (labor lasting >24 hours) High parity Use of certain medications: Magnesium sulfate Prolonged use of oxytocin during labor Inhaled anesthesia (ie, general anesthesia) (Option 2) Natural, unmedicated labor and birth reduces the chance of PPH. (Option 3) Labor lasting <24 hours does not increase the risk for PPH. (Option 4) A third stage of labor lasting <30 minutes does not increase the risk for PPH. Educational objective: Postpartum hemorrhage is defined as maternal blood loss of >500 mL after a vaginal birth or >1000 mL after a cesarean birth. Uterine atony (ie, "boggy" uterus) is the most common cause of early postpartum hemorrhage (occurring ≤24 hours after birth). Risk factors include uterine distension, uterine fatigue, high parity, and certain medications. Copyright © UWorld. All rights reserved.

Block Time Remaining: 00:14:25 TUTOR Test Id: 81012973 QId: 31234 (921666) 18 of 75 A A A The nurse enters a client's room just as the unlicensed assistive personnel (UAP) is completing a bath and placing thigh-high anti-embolism stockings on the client. Which situation would cause the nurse to intervene? 1. UAP applies the anti-embolism stockings while maintaining the client in supine position [5%] 2. UAP carefully smoothes out any wrinkles over the length of the stockings [1%] 3. UAP checks that the toe opening of the stockings is located on the plantar side of the foot [10%] 4. UAP rolls down and folds over the excess material at the top of the stockings [82%] Incorrect Correct answer 4 Answered correctly 82% Time: 73 seconds Updated: 01/06/2017 Explanation: Anti-embolism stockings are part of venous thromboembolism (VTE) prophylaxis in hospitalized clients. Anti-embolism stockings improve blood circulation in the leg veins by applying graduated compression. When fitted properly and worn consistently, the stockings decrease VTE risk. The stockings should not be rolled down, folded down, cut, or altered in any way. If stockings are not fitted and worn correctly, venous return can actually be impeded. (Option 1) Anti-embolism stockings should be applied before ambulating while the client is in bed; this maximizes the compression effects of the stockings and promotes venous return. The UAP has performed this correctly. (Option 2) Wrinkles should be smoothed out to avoid impeding venous return. The UAP has performed this correctly. (Option 3) The toe opening should be located on the plantar side of the foot/under the toes. The UAP has performed this correctly. Educational objective: Anti-embolism stockings are worn by clients as part of VTE prophylaxis. It is important that the nurse verifies the stockings are correctly fitted and worn appropriately. Incorrect size and fit or alterations to the stockings can impede venous return. Copyright © UWorld. All rights reserved.

Block Time Remaining: 00:15:16 TUTOR Test Id: 81012973 QId: 31396 (921666) 19 of 75 A A A A charge nurse suspects that the unlicensed assistive personnel (UAP) is falsifying the documentation of clients' capillary glucose results rather than performing the test. What is the best action by the charge nurse to handle this situation? 1. Ask a client if the UAP has performed the test [18%] 2. Discuss the importance of task completion and accurate documentation in a staff meeting [13%] 3. Give the UAP a verbal warning not to falsify data [8%] 4. Take a client's capillary glucose personally and compare it to the recorded result [59%] Correct Answered correctly 59% Time: 51 seconds Updated: 01/16/2017 Explanation: The best initial result is to assess and validate the charge nurse's perception. Doing the test and comparing results randomly/intermittently will give data to prove/disprove this concern. (Option 1) It could cause concern to involve a client when there may be an issue about inadequate provider care. The nurse should handle it independently. (Option 2) It is good to reinforce policies in general announcements to the entire staff, especially if wide-spread compliance is a concern. However, there is only one person that is suspected of not adhering in this case. Speaking out is often a general step taken, but the intended individuals usually don't hear the information. In addition, this is information that the staff has known/heard before. (Option 3) The normal discipline process is a verbal warning, a written warning, suspension, and termination. To initiate the process, there has to be evidence of wrong doing. However, it is only a suspicion at this point. Educational objective: When deliberate inaccurate documentation is suspected, gather evidence before confronting the staff member. One way of doing this is by checking the data personally and comparing it to what has been documented. Copyright © UWorld. All rights reserved.

Block Time Remaining: 00:34:27 TUTOR Test Id: 81114149 QId: 30562 (921666) 69 of 73 A A A Assessment of a client with a history of stroke reveals that the client understands and follows commands but answers questions with incorrect word choices. The nurse documents the presence of which communication deficit? 1. Aphasia [49%] 2. Apraxia [23%] 3. Dysarthria [22%] 4. Dysphagia [4%] Correct Answered correctly 49% Time: 34 seconds Updated: 02/23/2017 Explanation: Aphasia refers to impaired communication due to a neurological condition (eg, stroke, traumatic brain injury). The term aphasia is interchangeable with dysphasia, although aphasia is used more commonly. Receptive aphasia refers to impaired comprehension of speech and writing. A client with receptive aphasia may speak full sentences, but the words do not make sense. The nurse should speak clearly, ask simple "yes" or "no" questions, and use gestures and pictures to increase understanding. Expressive aphasia refers to impaired speech and writing. A client with expressive aphasia may be able to speak short phrases but will have difficulty with word choice (Option 1). The nurse should listen without interrupting and give the client time to form words. A client may have one type of aphasia or a combination of both, and the severity will vary with the individual. (Option 2) Apraxia refers to loss of the ability to perform a learned movement (eg, whistling, clapping, dressing) due to neurological impairment. (Option 3) Dysarthria is weakness of the muscles used for speech. Pronunciation and articulation are affected. Comprehension and the meaning of words are intact, but speech is difficult to understand (eg, mumble, lisp). (Option 4) Dysphagia refers to difficulty swallowing. The term dysphagia is often confused with dysphasia. Clients with motor deficits after a stroke may have dysphagia, which requires swallowing precautions to prevent aspiration. Educational objective: Aphasia refers to a neurological impairment of communication. Clients may have impaired speech and writing, impaired comprehension of words, or a combination of both. Copyright © UWorld. All rights reserved.

Block Time Remaining: 00:15:49 TUTOR Test Id: 81130759 QId: 31755 (921666) 30 of 75 A A A The nurse is providing discharge teaching to several clients with new prescriptions. Which instructions by the nurse are correct in regard to medication administration? Select all that apply. 1. Avoid salt substitutes when taking valsartan for hypertension 2. Take levofloxacin with an aluminum antacid to avoid gastric irritation 3. Take sucralfate after meals to minimize gastric irritation associated with a gastric ulcer 4. When taking ethambutol, notify the health care provider (HCP) of any changes in vision 5. When taking rifampin, notify the HCP if the urine turns red-orange Incorrect Correct answer 1,4 Answered correctly 32% Time: 61 seconds Updated: 03/19/2017 Explanation: Both ACE inhibitors ("prils" - captopril, enalapril, lisinopril, ramipril) and angiotensin receptor blockers ("sartans" - valsartan, losartan, telmisartan) cause hyperkalemia. Salt substitutes contain high potassium and must not be consumed unless approved by the health care provider (HCP) (Option 1). Ethambutol (Myambutol) is used to treat tuberculosis but can cause ocular toxicity, resulting in vision loss and loss of red-green color discrimination. Vision acuity and color discrimination must be monitored regularly (Option 4). (Option 2) Levofloxacin (Levaquin) is a quinolone antibiotic. For this class of antibiotics, 2 hours should pass between drug ingestion and consumption of aluminum/magnesium antacids, iron supplements, multivitamins with zinc, or sucralfate. These substances can bind up to 98% of the drug and make it ineffective. (Option 3) Sucralfate (Carafate, Sulcrate), prescribed to treat gastric ulcers, should be administered before meals to coat the mucosa and prevent irritation of the ulcer during meals. It should also be given at least 2 hours before or after other medications to prevent interactions that reduce drug efficacy. (Option 5) Rifampin (Rifadin), used to treat tuberculosis, normally causes red-orange discoloration of all body fluids. The client should be alerted to expect this change but does not need to notify the HCP. Educational objective: The nurse should watch for vision changes with ethambutol. Potassium supplements or salt substitutes should not be given to a client taking an ACE inhibitor or angiotensin receptor blocker. Sucralfate must be given before meals to prevent irritation of the ulcer. Quinolone antibiotics should not be given with antacids or supplements that reduce drug efficacy. Rifampin commonly causes red-orange discoloration of body fluids. Copyright © UWorld. All rights reserved.

Block Time Remaining: 00:15:59 TUTOR Test Id: 81012973 QId: 31087 (921666) 20 of 75 A A A A staff nurse automatically takes a lunch break first every day instead of asking other staff nurses about their preferences. Another nurse would like to take the first lunch break time sometimes. Which is the best way for the staff nurse to handle the situation initially? 1. Ask the nurse manager to talk to the nurse who takes the first break time [16%] 2. Poll the other staff nurses to see what they think about the situation [5%] 3. Realize that some matters are not worth bringing up and say nothing [0%] 4. Tell the nurse directly of wanting to take the first lunch break sometimes [78%] Correct Answered correctly 78% Time: 43 seconds Updated: 06/04/2017 Explanation: Nurses should attempt to be assertive communicators. This involves owning wants and rights using "I" statements and speaking directly with the person involved. A classic format is to state, "I've noticed ___, I feel_____, and I want____." The nurse should ask directly for the desired action. Perhaps the other person never considered that others wanted that time. During this communication, other individual's values/considerations must be respected as well. Assertive communication involves a balance between passive and aggressive communication. Assertive communication is contrasted with aggressive communication, which attacks the other person and focuses on the person's inherent character rather than on a specific action (eg, calling the nurse selfish). Aggressive communication seeks to dominate others and is rarely effective in the long run. (Option 1) The manager can be brought in eventually if there is no response, if there is a pattern of behavior, or the situation involves essential issues related to client care. Manager involvement should not be the initial step. (Option 2) Polling other staff members does not deal with the issue directly. It does not matter what the other nurses think; this nurse wants to change the process of lunch assignments. Regardless of others' thoughts, the situation will not change unless the nurse is willing to deal with it. (Option 3) This is an example of avoidance. Although it may seem to work superficially, it does not resolve conflicts and differences. It can result in feelings that build up until they erupt inappropriately later over something entirely different. It is better to deal with the issue directly than to ignore it. Educational objective: Nurses should communicate assertively by owning needs, using "I" statements, and dealing with involved individuals directly. Aggressive communication uses loaded "you" statements and is hostile. Avoidance behavior results in issues remaining unresolved. Copyright © UWorld. All rights reserved.

Block Time Remaining: 00:17:21 TUTOR Test Id: 81012973 QId: 31730 (921666) 21 of 75 A A A The nurse on the step-down cardiothoracic unit receives the change-of-shift hand-off report. Which client should the nurse assess first? 1. 2 days postabdominal aortic aneurysm repair with a pedal pulse decreased from baseline [49%] 2. 2 days postcoronary bypass graft surgery with a white blood cell count of 18,000/mm3 (18.0 × 109/L) [16%] 3. Cardiomyopathy with an ejection fraction of 25% and dyspnea on exertion [19%] 4. Pneumothorax with a chest tube to negative suction and subcutaneous emphysema [14%] Correct Answered correctly 49% Time: 82 seconds Updated: 03/10/2017 Explanation: The nurse should assess the pulses (eg, femoral, posterior tibial, dorsalis pedis) and skin color and temperature of the lower extremities in the client with the abdominal aortic aneurysm (AAA) repair first. Pulses can be absent for 4-12 hours after surgery due to vasospasm. However, a pedal pulse decreased from the client's baseline or an absent pulse with a painful, cool, or mottled extremity 2 days postoperative can indicate the presence of an arterial or graft occlusion. This client's condition poses the greatest threat to survival. (Option 2) An elevated white blood cell count (>11,000/mm3 [11.0 x 109/L]) could be caused by an underlying infection or the stress of the surgery. This needs to be assessed as soon as possible, but it does not take priority over the possible limb loss with graft occlusion. (Option 3) A decreased ejection fraction (normal 55%-70%) results in decreased cardiac output and inability to meet oxygen demand, leading to shortness of breath and activity intolerance. The nurse should assess lung sounds. However, this is an expected finding, so the nurse does not need to assess this client first. (Option 4) Subcutaneous emphysema is air in the tissue surrounding the chest tube insertion site and can occur in a client with a pneumothorax. The nurse should assess lung sounds and palpate to determine the degree of emphysema. However, this is an expected finding, so the nurse does not need to assess this client first. Educational objective: A pedal pulse decreased from baseline or an absent pedal pulse and a cool or mottled extremity in a client who is postoperative abdominal aortic aneurysm repair can indicate the presence of an arterial or graft occlusion and poses the greatest threat to survival. Copyright © UWorld. All rights reserved.

Block Time Remaining: 00:10:27 TUTOR Test Id: 81040578 QId: 30254 (921666) 20 of 75 A A A A client with community-acquired pneumonia is receiving 0.9% normal saline (NS) at 50 mL/hr. Pulse oximetry shows 95% on nasal O2 at 3 L/min. The nurse identifies a nursing diagnosis of ineffective airway clearance. Which prescription would the nurse expect to best facilitate secretion removal? 1. Incentive spirometer every 2 hours [45%] 2. Increase 0.9% NS to 125 mL/hr [20%] 3. Increase nasal oxygen to 4 L/min [1%] 4. Place the client in semi-Fowler's position [33%] Incorrect Correct answer 2 Answered correctly 20% Time: 21 seconds Updated: 05/25/2017 Explanation: Airway clearance is impaired with inadequate hydration. The client is receiving NS at only 50 mL/hr (1200 mL/24 hr). In addition, fever and tachypnea increase insensible losses through the skin and respiration. A fluid intake of 2,500-3,000 mL/day is recommended in clients with pneumonia; additional fluids are needed to replace insensible losses. Low-pitched wheezing indicates the presence of secretions in the airways. Increasing hydration by increasing the infusion rate to 125 mL/hr (eg, 3,000 mL/day) will help thin secretions and facilitate expectoration of mucus, and is the best choice for this client. (Option 1) An incentive spirometer will increase ventilation, reverse atelectasis (crackles), and facilitate gas exchange. Although it encourages deep breathing and coughing, it is not the best choice for thick secretion removal. (Option 3) The saturation (95%) is adequate. Increasing the oxygen flow rate will not facilitate secretion removal and so is not the best choice for this client. (Option 4) Positioning in high-Fowler's (not semi-Fowler's) position would improve gas exchange. With the effects of gravity, it would be easier for the client to cough up thinner secretions. Semi-Fowler's position is not the best choice for this client. Educational objective: Airway clearance is impaired with inadequate hydration. A fluid intake of 2,500-3,000 mL/day is recommended in clients with pneumonia, and additional fluids are needed to replace insensible losses associated with fever and tachypnea. Increasing oral and intravenous hydration helps to thin secretions and facilitate expectoration of mucus. Copyright © UWorld. All rights reserved.

Block Time Remaining: 00:17:46 TUTOR Test Id: 81040578 QId: 30365 (921666) 30 of 75 A A A A client with chronic bronchitis tells the home health nurse of being exhausted all day due to coughing all night and being unable to sleep. The client can feel thick mucus in the chest and throat. Which interventions can the nurse suggest to help mobilize secretions and improve sleep? Select all that apply. 1. Increase fluids to at least 8 glasses (2-3 L) of water a day 2. Sleep with a cool mist humidifier 3. Take prescribed guaifenesin cough medicine before bedtime 4. Use abdominal breathing and the huff cough technique at bedtime 5. Use pursed-lip breathing during the night Incorrect Correct answer 1,2,3,4 Answered correctly 20% Time: 45 seconds Updated: 04/16/2017 Explanation: Chronic bronchitis is characterized by excessive mucus production, chronic cough, and recurrent respiratory tract infections. Interventions to help reduce viscosity of mucus, facilitate secretion removal, and promote comfort include the following: Increasing oral fluids to 2-3 L/day if not contraindicated prevents dehydration and keeps secretions thin Cool mist humidifier increases room humidity of inspired air Guaifenesin (Robitussin) is an expectorant that reduces the viscosity of thick secretions by increasing respiratory tract fluid; drinking a full glass of water after taking the medication is recommended. Abdominal breathing with the huff, a forced expiratory cough technique, is effective in mobilizing secretions into the large airways so that they can be expectorated Chest physiotherapy (postural drainage, percussion, vibration) Airway clearance handheld devices, which use the principle of positive expiratory pressure to help loosen secretions when the client exhales through the mouthpiece (Option 5) Pursed lip breathing prolongs exhalation, reduces air trapping in the lungs, and decreases dyspnea. It does not help to thin secretions. Educational objective: Interventions to help reduce viscosity of mucus, facilitate secretion removal, and promote comfort in clients with chronic bronchitis include the following: Increasing oral fluids to 2-3 L/day if not contraindicated Cool mist humidifier to increase room humidity Guaifenesin (Robitussin), an expectorant, to reduce viscosity of secretions Huff coughing Copyright © UWorld. All rights reserved.

Block Time Remaining: 00:16:30 TUTOR Test Id: 81075711 QId: 30994 (921666) 31 of 75 A A A Which prescriptions for these clients does the nurse question? Select all that apply. 1. Client with Clostridium difficile colitis, prescribed vancomycin 125 mg PO 2. Client with diabetes and elevated mealtime glucose, prescribed lispro insulin scale 6 units subcutaneously 3. Client with gastrointestinal bleed and nasogastric tube, prescribed pantoprazole 40 mg intravenous 4. Client with hypertension and blood pressure (BP) 94/40 mm Hg, prescribed metoprolol succinate SR 50 mg PO 5. Client with otitis media and penicillin allergy, prescribed ampicillin 500 mg PO Incorrect Correct answer 4,5 Answered correctly 41% Time: 68 seconds Updated: 01/15/2017 Explanation: The nurse would question the prescriptions for the following clients: Client with hypertension and BP 94/40 mm Hg, prescribed metoprolol succinate SR (Toprol-XL) 50 mg PO: This client's mean arterial pressure (MAP) is only 58 mm Hg ({[2x diastolic] + systolic} ÷ 3). A MAP >60-65 mm Hg is necessary to perfuse the vital organs (eg, brain, coronary arteries, kidneys). Toprol-XL is a long-acting beta blocker and will continue to drop the client's BP over a 24-hour period. Client with otitis media and penicillin allergy prescribed ampicillin 500 mg PO: Ampicillin is classified as a penicillin antibiotic and is contraindicated in clients with a penicillin allergy. (Option 1) C difficile colitis is treated with metronidazole or vancomycin, depending on severity and number of relapses. Vancomycin is typically given orally in this situation, unlike other nonintestinal infections in which IV is the standard route. There is no reason to question this prescription. (Option 2) A sliding insulin (correction) scale is used to prescribe rapid-acting lispro (Humalog) to control postprandial hyperglycemia. The nurse would not question this prescription. (Option 3) Proton pump inhibitors (eg, pantoprazole, omeprazole) are prescribed for gastroesophageal reflux disease, and ulcer treatment and prophylaxis. The IV preparation is administered when the oral route is contraindicated. The nurse would not question this prescription. Educational objective: IV proton pump inhibitors are used for gastric ulcer bleeding. Oral vancomycin can be used for C difficile colitis. Ampicillin or amoxicillin are contraindicated in clients with a penicillin allergy. Antihypertensives are held if the client has borderline low BP. Copyright © UWorld. All rights reserved.

Block Time Remaining: 00:17:49 TUTOR Test Id: 81075711 QId: 30500 (921666) 34 of 75 A A A A client is seen in the ambulatory care center for treatment of a second episode of acute gout. Which lifestyle modifications would help prevent future exacerbations? Select all that apply. 1. Achieve and maintain a healthy weight 2. Avoid foods containing protein 3. Drink plenty of fluids 4. Increase meat intake 5. Limit alcohol consumption Correct Answered correctly 63% Time: 17 seconds Updated: 01/17/2017 Explanation: Gout is an inflammatory condition caused by ineffective metabolism of purines, which causes uric acid accumulation in the blood. Uric acid crystals typically form in the joints. Kidney stones can also develop, increasing the risk of kidney damage. Clients with medical risk factors (eg, obesity, hypertension, dyslipidemia, insulin resistance) and other lifestyle factors (eg, poor diet, alcohol consumption, sedentary lifestyle) have increased risk for future gout attacks. Improvements in uric acid control are often seen when weight loss is accompanied by dietary modifications (Option 1). Suggested modifications include: Increasing fluid intake (2 L/day) to help eliminate excess uric acid (Option 3) Implementing a low-purine diet, particularly avoiding organ meats (eg, liver, kidney, brain) and certain seafood (eg, sardines, shellfish) Limiting alcohol intake, especially beer (Option 5) Following a healthy, low-fat diet, as excess dietary fats impair urinary excretion of urates (Option 2) It is unpalatable and impractical to avoid all foods containing protein. The risk of developing gout increases with high dietary purine intake but not necessarily with protein intake. Low-fat dairy products are good sources of protein that are associated with a reduced risk of gout. (Option 4) Increasing intake of meat, especially organ meats, will not prevent future gout attacks but may precipitate them. Educational objective: Weight loss and dietary modifications may reduce the frequency of acute episodes of gout. These strategies include increasing fluids, limiting daily alcohol consumption, and avoiding organ meats and seafood to reduce purine load. Copyright © UWorld. All rights reserved.

Block Time Remaining: 00:19:39 TUTOR Test Id: 81040578 QId: 30519 (921666) 34 of 75 A A A The nurse is assisting a client with a diagnosis of asthma in use of a peak flow meter. Place the steps for measuring peak expiratory flow using a peak flow meter in the correct order. All options must be used. Your Response/ Incorrect Response Correct Response Slide the indicator on the numbered scale on the flow meter to the 0 or lowest number and instruct the client to stand or sit as upright as possible Instruct the client to breathe in deeply, place the mouthpiece in the mouth, and close the lips tightly around it to form a seal Instruct the client to exhale as quickly and forcibly as possible and note the reading on the numbered scale Repeat the procedure 2 more times with a 5-10-second rest period between exhalations Record the highest reading achieved (personal best) Omitted Correct answer 5,1,2,4,3 Answered correctly 75% Time: 13 seconds Updated: 04/26/2017 Explanation: A peak flow meter is a portable, inexpensive, hand-held device used to measure the client's ability to push air out of the lungs. Flow meters are available in low ranges for measuring smaller volumes and for small children, and in a standard range for older children, teenagers, and adults. To obtain the most accurate readings to help guide, maintain, and evaluate treatment in clients with moderate to severe asthma, the procedure is carried out in the following order: Before using, slide the indicator on the numbered scale on the flow meter to 0 or lowest number and instruct the client to stand or sit as upright as possible Instruct the client to breathe in deeply, place the mouthpiece in the mouth, and the close lips tightly around it to form a seal Instruct the client to exhale as quickly and forcibly as possible and note the reading on the numbered scale Repeat the procedure 2 more times with a 5-10-second rest period between exhalations Record the highest reading achieved (personal best) Educational objective: A peak flow meter is a hand-held device used to measure the client's ability to push air out of the lungs. It is used for assessment of moderate to severe asthma. Copyright © UWorld. All rights reserved.

Block Time Remaining: 00:19:39 TUTOR Test Id: 81040578 QId: 30519 (921666) 34 of 75 A A A The nurse is assisting a client with a diagnosis of asthma in use of a peak flow meter. Place the steps for measuring peak expiratory flow using a peak flow meter in the correct order. All options must be used. Your Response/ Incorrect Response Correct Response Slide the indicator on the numbered scale on the flow meter to the 0 or lowest number and instruct the client to stand or sit as upright as possible Instruct the client to breathe in deeply, place the mouthpiece in the mouth, and close the lips tightly around it to form a seal Instruct the client to exhale as quickly and forcibly as possible and note the reading on the numbered scale Repeat the procedure 2 more times with a 5-10-second rest period between exhalations Record the highest reading achieved (personal best) Omitted Correct answer 5,1,2,4,3 Answered correctly 75% Time: 13 seconds Updated: 04/26/2017 Explanation: A peak flow meter is a portable, inexpensive, hand-held device used to measure the client's ability to push air out of the lungs. Flow meters are available in low ranges for measuring smaller volumes and for small children, and in a standard range for older children, teenagers, and adults. To obtain the most accurate readings to help guide, maintain, and evaluate treatment in clients with moderate to severe asthma, the procedure is carried out in the following order: Before using, slide the indicator on the numbered scale on the flow meter to 0 or lowest number and instruct the client to stand or sit as upright as possible Instruct the client to breathe in deeply, place the mouthpiece in the mouth, and the close lips tightly around it to form a seal Instruct the client to exhale as quickly and forcibly as possible and note the reading on the numbered scale Repeat the procedure 2 more times with a 5-10-second rest period between exhalations Record the highest reading achieved (personal best) Educational objective: A peak flow meter is a hand-held device used to measure the client's ability to push air out of the lungs. It is used for assessment of moderate to severe asthma. Copyright © UWorld. All rights reserved.

Block Time Remaining: 00:19:00 TUTOR Test Id: 81012973 QId: 30801 (921666) 22 of 75 A A A An elderly client with staphylococcal pneumonia treated with intravenous antibiotic therapy for 3 days becomes extremely short of breath and restless and is difficult to arouse. Which additional assessment findings indicate to the nurse that the client can be developing sepsis? Select all that apply. 1. Absent bowel sounds 2. Capillary refill 5 seconds 3. Diminished breath sounds in bases 4. Serum glucose level 180 mg/dL (10.0 mmol/L) 5. Urine output 1 mL/kg/hr Incorrect Correct answer 1,2,4 Answered correctly 4% Time: 99 seconds Updated: 02/28/2017 Explanation: Sepsis is a systemic inflammatory response to an infection and can occur as a complication of pneumonia in clients who do not respond to antibiotic therapy. It is caused by the entry of bacteria from the alveoli into the bloodstream. Manifestations characteristic of sepsis include heart rate >90 beats/min, temperature >100.9 F (38.3 C), systolic blood pressure <90 mm Hg, altered mental status, and hyperglycemia (>140 mg/dL [7.8 mmol/L]) in the absence of diabetes. The assessment findings most important for the nurse to report to the health care provider include the following: Absent bowel sounds. Paralytic ileus occurs in the presence of sepsis and hypoxia as blood is shunted away from the gastrointestinal tract to the vital organs. Capillary refill 5 seconds. Prolonged capillary refill (>3-4 seconds in an adult) indicates inadequate blood flow to peripheral tissues. Serum glucose >140 mg/dL (7.8 mmol/L). Gluconeogenesis occurs in response to the physiologic stress of infection. Insulin resistance is associated with anaerobic metabolism. (Option 3) Diminished breath sounds in the bases are expected in an elderly client with pneumonia. (Option 5) Urine output of 1 mL/kg/hr is within the normal range (0.5-1.0 mL/kg/hr). Educational objective: Sepsis is a complication of pneumonia that can progress to septic shock and/or multisystem organ dysfunction syndrome. To limit progression, the nurse assesses oxygenation (pulse oximeter, arterial blood gases), airway (patency), breathing (respiratory pattern and rate), circulation (vital signs), tissue perfusion (eg, level of consciousness, capillary refill, skin temperature and color, bowel sounds), and urine output. Copyright © UWorld. All rights reserved.

Block Time Remaining: 00:20:29 TUTOR Test Id: 81012973 QId: 32009 (921666) 23 of 75 A A A The nurse is performing a physical assessment on a 2-year-old with cold symptoms and a fever at home of 101.7 F (38.7 C). The parent is concerned about the child's ability to cooperate during the examination. Place the components of assessment in the order the nurse would perform them. All options must be used. Your Response/ Incorrect Response Interact with the parent in a friendly manner Play with the child using a finger puppet Take the child's vital signs Auscultate the child's heart and lungs Measure the child's height and weight Correct Response Interact with the parent in a friendly manner Play with the child using a finger puppet Measure the child's height and weight Auscultate the child's heart and lungs Take the child's vital signs Incorrect Correct answer 2,4,3,1,5 Answered correctly 25% Time: 89 seconds Updated: 01/07/2017 Explanation: Always complete the assessment by performing the least invasive parts first and then progressing to the most invasive. By first establishing a rapport with the parent (Option 2), the nurse will elicit the child's trust and cooperation. Playing with the child will help the child relax and perceive the nurse as less of a threat (Option 4). Measuring the child's height and weight should be performed next (Option 3). Auscultation of the heart and lungs should then be performed. Allowing the child to play with the equipment first will make this part of the assessment easier (Option 1). Taking vital signs can be difficult as a blood pressure cuff can be perceived as painful (Option 5); once the child is upset, it becomes difficult to continue with the assessment. A temperature of 101.7 F (38.7 C) is not serious in a child, especially if there are signs and symptoms of an upper respiratory infection. Educational objective: Performing a physical assessment in a toddler can be challenging. The nurse should establish a rapport with the parent and then attempt to gain the child's trust. Playing with the child can make the experience easier on the nurse, parents, and child. The nurse should always perform the least invasive procedures first, explain them in simple terms, and praise the child throughout the assessment. Copyright © UWorld. All rights reserved.

Block Time Remaining: 00:21:07 TUTOR Test Id: 81012973 QId: 30980 (921666) 24 of 75 A A A A nurse is discussing parallel play with the parent of a 2-year-old. Which statement by the parent indicates understanding of the discussion? 1. "I encourage working in a group to build towers with large blocks." [14%] 2. "I have a chalk board available to teach the alphabet and numbers." [5%] 3. "I set out a basket of various balls in the backyard when other children come to play." [76%] 4. "I try to organize games that involve a team approach." [3%] Correct Answered correctly 76% Time: 38 seconds Updated: 01/27/2017 Explanation: Parallel play is typical behavior of a toddler and involves activities focused on improving motor skills, imitative efforts, and the use of multiple senses. Toddlers play alongside, rather than with, other children. Having a variety of different balls for a group of children allows each child to be present with others and participate as they desire. Other examples of parallel play activities include pushing and pulling large toys; smearing paint; playing with dolls or toy cars; and digging in a sandbox. (Option 1) Working in groups is an appropriate play activity for children in the preschooler period. (Option 2) The classroom approach does not promote parallel play. Using large chalk to draw allows the child creative expression in an unstructured manner. (Option 4) A toddler is challenged by the concept of team games, which requires attention to the group's effort. Educational objective: Toddlers engage in parallel play, which involves playing alongside, not with, other children. Activities such as playing with dolls or toy cars, pushing and pulling large toys, smearing paint, and digging in a sandbox encourage parallel play. Copyright © UWorld. All rights reserved.

Block Time Remaining: 00:21:16 TUTOR Test Id: 81012973 QId: 30523 (921666) 25 of 75 A A A A client with chest pain is diagnosed with acute pericarditis by the health care provider. The nurse explains that the pain will improve with which of the following? 1. Coughing and deep breathing [7%] 2. Left lateral position [17%] 3. Pursed-lip breathing [7%] 4. Sitting up and leaning forward [66%] Correct Answered correctly 66% Time: 9 seconds Updated: 04/16/2017 Explanation: The most common cause of acute pericarditis is a recent viral infection. It is an inflammation of the visceral and/or parietal pericardium. Pericarditis is characterized by typical pleuritic chest pain that is sharp. It is aggravated during inspiration and coughing. Pain is typically relieved by sitting up and leaning forward. This position reduces pressure on the inflamed parietal pericardium, especially during lung inflation. The pain is different than that experienced during myocardial infarction. Assessment shows a pericardial friction rub (scratchy or squeaking sound). Treatment includes a combination of nonsteroidal anti-inflammatory drugs (NSAIDS) or aspirin plus colchicine. (Option 1) Pericarditis causes pain on inspiration, not expiration. This pleuritic-type pain also increases with coughing. (Option 2) The supine or lying-down position worsens pericarditis pain. (Option 3) The pursed-lip breathing technique helps to decrease shortness of breath by preventing airway collapse, promoting carbon dioxide elimination, and reducing air trapping in clients with chronic obstructive pulmonary disease. Educational objective: Pericarditis is characterized by typical pleuritic chest pain that is sharp. It is aggravated during inspiration and coughing. Pain is typically relieved by sitting up and leaning forward. Treatment includes a combination of NSAIDs or aspirin plus colchicine. Copyright © UWorld. All rights reserved.

Block Time Remaining: 00:22:11 TUTOR Test Id: 81012973 QId: 32403 (921666) 26 of 75 A A A The clinic nurse interviews the parents of a 6-month-old about the child's diet and feeding schedule. Which parent statement causes the nurse the most concern? 1. "Apples are a healthy food, so we often make apple pie for our child." [5%] 2. "Chopped pears are one of our child's favorite foods." [8%] 3. "Oatmeal with fresh honey is our child's favorite breakfast." [57%] 4. "We have found TV dinners to be convenient as they have both meat and vegetables." [28%] Correct Answered correctly 57% Time: 55 seconds Updated: 02/21/2017 Explanation: Although more than one of these parent comments are concerning, the most concerning is feeding honey to a child under age 1 year. Honey (especially raw or wild) is not recommended for children under age 1 due to the risk for infant botulism. An infant under age 1 has an immature gut system that can allow Clostridium botulinum spores contaminated in honey to colonize the gastrointestinal tract and release toxin that causes botulism. Botulinum toxin produces muscle paralysis by inhibiting the release of acetylcholine at the neuromuscular junction. Infants often present with constipation, diminished deep tendon reflexes, and generalized weakness. Additional symptoms are lack of head control, difficulty in feeding, and decreased gag reflex, which can progress to respiratory failure. Isolation of the organism from the child's stool can take several days; therefore, diagnosis is usually made by history, and treatment with botulism immune globulin is started before laboratory results are known. (Option 1) Apple pie is not the best way to serve apples to a 6-month-old as the other ingredients add too much fat and sugar. This would need to be addressed but is not a priority over the use of honey. (Option 2) Raw fruits are appropriate for a 6-month-old. (Option 4) Although TV dinners contain meat and vegetables, they are not the best source of food for an infant due to the high sodium content. This would need to be addressed after the use of honey is addressed. Educational objective: Due to the risk of infant botulism, honey should not be given to children under age 1 year. Copyright © UWorld. All rights reserved.

Block Time Remaining: 00:22:40 TUTOR Test Id: 81012973 QId: 34228 (921666) 27 of 75 A A A A 3-month-old infant has irritability, facial edema, a 1-day history of diarrhea with adequate oral intake, and seizure activity. During assessment, the parents state that they have recently been diluting formula to save money. Which is the most likely cause for the infant's symptoms? 1. Hypernatremia due to diarrhea [9%] 2. Hypoglycemia due to dilute formula intake [16%] 3. Hypokalemia due to excess gastrointestinal output [11%] 4. Hyponatremia due to water intoxication [63%] Correct Answered correctly 63% Time: 29 seconds Updated: 03/22/2017 Explanation: Water intoxication (water overload) resulting in hyponatremia may occur in infants when formula is diluted to "stretch" the feeding to save money. Hyponatremia may also result from ingestion of plain water (eg, caregiver attempting to rehydrate an infant who has been ill). Infants have immature renal systems with a low glomerular filtration rate, which decreases their ability to excrete excess water and makes them susceptible to water intoxication. Symptoms of hyponatremia include irritability, lethargy, and, in severe cases, hypothermia and seizure activity. Breast milk and/or formula are the only sources of hydration an infant needs for the first 6 months of life. Formula should be prepared per the manufacturer's instructions. (Option 1) Hypernatremia may be caused by dehydration (eg, decreased oral intake, vomiting, diarrhea) and presents with similar neurological symptoms (eg, restlessness, seizures). The infant's history indicates adequate oral intake and signs of fluid overload (eg, facial edema), not dehydration. (Option 2) Hypoglycemia may present with irritability and seizures, but facial edema and recent history of over-diluting the formula should alert the nurse that water intoxication with hyponatremia is the most likely cause. (Option 3) Hypokalemia secondary to diarrhea may present with irritability, muscle weakness, and cardiac arrhythmias. Educational objective: Infants are susceptible to hyponatremia secondary to water intoxication, which can present with neurological symptoms (eg, lethargy, irritability, seizures). Breast milk and/or formula provide sufficient hydration for the first 6 months of life. Formula should not be diluted to save money. Copyright © UWorld. All rights reserved.

Block Time Remaining: 00:22:40 TUTOR Test Id: 81012973 QId: 34228 (921666) 27 of 75 A A A A 3-month-old infant has irritability, facial edema, a 1-day history of diarrhea with adequate oral intake, and seizure activity. During assessment, the parents state that they have recently been diluting formula to save money. Which is the most likely cause for the infant's symptoms? 1. Hypernatremia due to diarrhea [9%] 2. Hypoglycemia due to dilute formula intake [16%] 3. Hypokalemia due to excess gastrointestinal output [11%] 4. Hyponatremia due to water intoxication [63%] Correct Answered correctly 63% Time: 29 seconds Updated: 03/22/2017 Explanation: Water intoxication (water overload) resulting in hyponatremia may occur in infants when formula is diluted to "stretch" the feeding to save money. Hyponatremia may also result from ingestion of plain water (eg, caregiver attempting to rehydrate an infant who has been ill). Infants have immature renal systems with a low glomerular filtration rate, which decreases their ability to excrete excess water and makes them susceptible to water intoxication. Symptoms of hyponatremia include irritability, lethargy, and, in severe cases, hypothermia and seizure activity. Breast milk and/or formula are the only sources of hydration an infant needs for the first 6 months of life. Formula should be prepared per the manufacturer's instructions. (Option 1) Hypernatremia may be caused by dehydration (eg, decreased oral intake, vomiting, diarrhea) and presents with similar neurological symptoms (eg, restlessness, seizures). The infant's history indicates adequate oral intake and signs of fluid overload (eg, facial edema), not dehydration. (Option 2) Hypoglycemia may present with irritability and seizures, but facial edema and recent history of over-diluting the formula should alert the nurse that water intoxication with hyponatremia is the most likely cause. (Option 3) Hypokalemia secondary to diarrhea may present with irritability, muscle weakness, and cardiac arrhythmias. Educational objective: Infants are susceptible to hyponatremia secondary to water intoxication, which can present with neurological symptoms (eg, lethargy, irritability, seizures). Breast milk and/or formula provide sufficient hydration for the first 6 months of life. Formula should not be diluted to save money. Copyright © UWorld. All rights reserved.

Block Time Remaining: 00:22:40 TUTOR Test Id: 81012973 QId: 34228 (921666) 27 of 75 A A A A 3-month-old infant has irritability, facial edema, a 1-day history of diarrhea with adequate oral intake, and seizure activity. During assessment, the parents state that they have recently been diluting formula to save money. Which is the most likely cause for the infant's symptoms? 1. Hypernatremia due to diarrhea [9%] 2. Hypoglycemia due to dilute formula intake [16%] 3. Hypokalemia due to excess gastrointestinal output [11%] 4. Hyponatremia due to water intoxication [63%] Correct Answered correctly 63% Time: 29 seconds Updated: 03/22/2017 Explanation: Water intoxication (water overload) resulting in hyponatremia may occur in infants when formula is diluted to "stretch" the feeding to save money. Hyponatremia may also result from ingestion of plain water (eg, caregiver attempting to rehydrate an infant who has been ill). Infants have immature renal systems with a low glomerular filtration rate, which decreases their ability to excrete excess water and makes them susceptible to water intoxication. Symptoms of hyponatremia include irritability, lethargy, and, in severe cases, hypothermia and seizure activity. Breast milk and/or formula are the only sources of hydration an infant needs for the first 6 months of life. Formula should be prepared per the manufacturer's instructions. (Option 1) Hypernatremia may be caused by dehydration (eg, decreased oral intake, vomiting, diarrhea) and presents with similar neurological symptoms (eg, restlessness, seizures). The infant's history indicates adequate oral intake and signs of fluid overload (eg, facial edema), not dehydration. (Option 2) Hypoglycemia may present with irritability and seizures, but facial edema and recent history of over-diluting the formula should alert the nurse that water intoxication with hyponatremia is the most likely cause. (Option 3) Hypokalemia secondary to diarrhea may present with irritability, muscle weakness, and cardiac arrhythmias. Educational objective: Infants are susceptible to hyponatremia secondary to water intoxication, which can present with neurological symptoms (eg, lethargy, irritability, seizures). Breast milk and/or formula provide sufficient hydration for the first 6 months of life. Formula should not be diluted to save money. Copyright © UWorld. All rights reserved.

Block Time Remaining: 00:18:56 TUTOR Test Id: 81075711 QId: 30917 (921666) 36 of 75 A A A The nurse removes personal protective equipment (PPE) after completing a wound dressing change for a client in airborne transmission-based precautions. Which PPE should the nurse remove first? 1. Face shield/goggles [15%] 2. Gloves [52%] 3. Gown [25%] 4. Mask/respirator [6%] Correct Answered correctly 52% Time: 21 seconds Updated: 01/19/2017 Explanation: Personal protective equipment (PPE) is necessary when a client is on contamination precautions (eg, droplet, airborne, contact). A gown is not normally required in an airborne precaution room; however, if contamination is probable (eg, dressing change, contact with bodily fluids), a gown is necessary. The proper removal of PPE limits self-contamination. The exact procedure for donning and removing PPE varies with the level of precautions and location of nursing practice. Gloves should be removed first and promptly after use to prevent contamination of other items or noncontaminated materials (Option 2). To remove gloves: Grasp the first glove by its palmar surface and pull off inside out. Next, slide fingers of the ungloved hand under the second glove at the wrist and peel off over the first glove. Discard gloves in an infectious waste container. (Options 1, 3, and 4) Face shield/goggles, gown, and mask/respirator can be removed after gloves, which are considered the most contaminated piece of PPE. Educational objective: The proper removal of personal protective equipment limits self-contamination. Gloves should be removed first and promptly after use to prevent contamination of other items or noncontaminated materials. Copyright © UWorld. All rights reserved.

Block Time Remaining: 00:22:57 TUTOR Test Id: 81075711 QId: 30997 (921666) 44 of 75 A A A Which nursing interventions for a client recovering from a gastroduodenostomy (Billroth I) are contraindicated? Select all that apply. 1. Applying a sequential compression device and antiembolism stockings 2. Encouraging turn, cough, and deep-breathing exercises every 2 hours 3. Keeping the head of the bed raised and positioned at a 45-degree angle 4. Repositioning and irrigating a clogged nasogastric tube as needed 5. Teaching the importance of small, frequent, high-carbohydrate meals Incorrect Correct answer 4,5 Answered correctly 14% Time: 55 seconds Updated: 03/22/2017 Explanation: A gastroduodenostomy (Biliroth I) involves removal of the distal two-thirds of the stomach; the remaining stomach is anastomosed to the duodenum. This technique is used to treat stomach cancer and peptic ulcer disease that does not respond to more conservative treatment. Following a gastroduodenostomy, clients should be taught to consume frequent, low-carbohydrate meals with moderate amounts of fat and protein. Due to the decreased size of the stomach, fluids and meal sizes should be reduced to prevent dumping syndrome (the rapid emptying of stomach contents into the small intestine). Other common postoperative interventions, such as deep venous thrombosis prophylaxis (eg, sequential compression device, antiembolism stockings); turning, coughing, and deep breathing; and elevating the head of the bed to prevent aspiration from reflux, also apply (Options 1, 2, and 3). (Option 4) Following the procedure, the nurse should take care not to introduce infection or disrupt the sutures; therefore, the nasogastric tube should not be moved or flushed unless prescribed by the health care provider. (Option 5) The client should avoid high-carbohydrate meals as they cause dumping syndrome and may lead to hypoglycemia. Educational objective: Clients undergoing a gastroduodenostomy (Billroth I) benefit from postoperative management such as deep venous thrombosis prophylaxis; turning, coughing, and deep breathing; and aspiration precautions (elevating the head of the bed). Dietary changes include eating smaller, frequent meals; decreasing carbohydrates; and eating moderate amounts of fat and protein. Copyright © UWorld. All rights reserved.

Block Time Remaining: 00:24:01 TUTOR Test Id: 81012973 QId: 30372 (921666) 29 of 75 A A A Which lifestyle and nutritional strategies can help clients reduce and manage the signs and symptoms of gastroesophageal reflux disease? Select all that apply. 1. Avoid caffeine, chocolate, and peppermint 2. Choose foods that are low in fat 3. Eat 3 meals a day with minimal or no snacking 4. Minimize intake of dairy products 5. Sip water with meals Incorrect Correct answer 1,2,5 Answered correctly 24% Time: 43 seconds Updated: 04/01/2017 Explanation: Gastroesophageal reflux disease (GERD) develops when the reflux of stomach contents causes inflammation of the esophageal mucosa. The pressure of the lower esophageal sphincter (LES) normally prevents stomach contents from entering the esophagus. Any factor that decreases LES pressure (eg, caffeine, alcohol, fatty foods) or increases gastric pressure (eg, large meals, tight clothing) can precipitate GERD. Common symptoms and complications include heartburn, regurgitation, persistent sore throat, painful swallowing, and chronic cough. Most individuals are treated effectively with acid suppression using antacids (eg, calcium carbonate, magnesium hydroxide), proton pump inhibitors (omeprazole), and histamine 2 receptor antagonists (ranitidine). Lifestyle and dietary measures that prevent GERD or reduce it effects include the following: Weight loss, as excessive belly fat can increase gastric pressure Abdominal breathing exercises to strengthen the LES Small, frequent meals (Option 3) with sips of water or fluids (Option 5) to help facilitate passage of stomach contents into the small intestine Avoiding GERD triggers such as caffeine, alcohol, nicotine, high-fat foods, chocolate, spicy foods, peppermint, and carbonated beverages (Options 1 and 2) Chewing gum to promote salivation, which may help neutralize and clear acid from the esophagus Elevating the head of the bed Refraining from eating at bedtime and/or lying down immediately after eating (Option 4) There is no reason to minimize or eliminate dairy products; however, clients with GERD should choose low- or no-fat products. Educational objective: Dietary interventions to help prevent or minimize the symptoms of gastroesophageal reflux disease include avoiding factors that decrease lower esophageal sphincter pressure (eg, alcohol, caffeine, chocolate, high-fat and spicy foods) and promoting factors that decrease gastric pressure (eg, small, frequent meals; loose clothing). Copyright © UWorld. All rights reserved.

Block Time Remaining: 00:24:33 TUTOR Test Id: 81012973 QId: 34229 (921666) 31 of 75 A A A The nurse receives report on 4 clients. Which client should be seen first? 1. Client with amyotrophic lateral sclerosis experiencing increased dysarthria [24%] 2. Client with chronic obstructive pulmonary disease reporting increasing leg edema [40%] 3. Client with strep throat and fever of 102 F (38.9 C) on antibiotics for 12 hours [11%] 4. Client with urolithiasis reporting wavelike flank pain and nausea [24%] Omitted Correct answer 1 Answered correctly 24% Time: 3 seconds Updated: 02/06/2017 Explanation: Amyotrophic lateral sclerosis (ALS) is characterized by the progressive loss of motor neurons in the brainstem and spinal cord. Clients have spasticity, muscle weakness, and atrophy. Neurons involved in swallowing and respiratory function are eventually impaired, leading to aspiration, respiratory failure, and death. Care of clients with ALS focuses on maintaining respiratory function, adequate nutrition, and quality of life. There is no cure, and death usually occurs within 5 years of diagnosis. The client with ALS and worsening ability to speak (dysarthria) may also have dysphagia and respiratory distress; this client should be seen first (Option 1). (Option 2) The client with chronic obstructive pulmonary disease and peripheral edema may have cor pulmonale, or right-sided heart failure, from vasoconstriction of the pulmonary vessels. Cor pulmonale is treated with long-term, low-flow oxygen; bronchodilators; and diuretics. This client should be seen second. Right-sided heart failure (peripheral edema) is not as dangerous as left-sided heart failure (pulmonary edema). (Option 3) Fever often occurs with strep throat and may persist for ≥24 hours after initiation of antibiotics. This client should be seen last and should receive an antipyretic. (Option 4) Wavelike flank pain is characteristic of urolithiasis (urinary stones). This client needs pain medication and, possibly, further treatment (eg, lithotripsy) and should be seen third. Educational objective: Amyotrophic lateral sclerosis causes progressive loss of motor neurons, resulting in muscle weakness and spasticity. Muscles involved in respiration and swallowing are affected, leading to aspiration and, ultimately, respiratory failure. Treatment focuses on maintaining respiratory function, adequate nutrition, and quality of life. Copyright © UWorld. All rights reserved.

Block Time Remaining: 00:26:02 TUTOR Test Id: 81012973 QId: 33402 (921666) 33 of 75 A A A The postpartum nurse receives report on 4 mother-baby couplets. Which tasks can be delegated to unlicensed assistive personnel? Select all that apply. 1. Assisting the mother with morning hygiene 2. Demonstrating neonate bathing technique 3. Documenting intake and output on the mother 4. Evaluating caregiver interaction with the neonate 5. Obtaining an axillary temperature on the neonate 6. Swaddling the neonate after diaper changes Incorrect Correct answer 1,3,5,6 Answered correctly 58% Time: 39 seconds Updated: 03/20/2017 Explanation: Scope of practice RN LPN/LVN UAP Clinical assessment Initial client education Discharge education Clinical judgment Initiating blood transfusion Monitoring RN findings Reinforcing education Routine procedures (eg, catheterization) Most medication administrations Ostomy care Tube patency & enteral feeding Specific assessments Activities of daily living Hygiene Linen change Routine, stable vital signs Documenting input/output Positioning LPN = licensed practical nurse; LVN = licensed vocational nurse; RN = registered nurse; UAP = unlicensed assistive personnel. *Limited assessments (eg, lung sounds, bowel sounds, neurovascular checks). Assisting clients with activities of daily living is within the scope of practice of unlicensed assistive personnel (UAP). Helping the mother with morning hygiene, documenting intake and output, taking vital signs of stable clients, and swaddling the neonate may be delegated to UAP. The registered nurse (RN) should follow the 5 rights of delegation when planning care. (Option 2) The RN assumes responsibility for initial client teaching and demonstration of home care. Once teaching and demonstration are complete, the UAP can assist the mother with bathing the neonate. (Option 4) The RN should assess caregiver interaction with the newborn to identify any attachment issues. Elements of the nursing process (assessment, planning, and evaluation) and tasks requiring nursing judgment cannot be delegated. Educational objective: The registered nurse is responsible for any care requiring clinical judgment. Unlicensed assistive personnel can assist with activities of daily living, documenting intake and output, positioning, and taking the vital signs of stable clients. Copyright © UWorld. All rights reserved.

Block Time Remaining: 00:27:22 TUTOR Test Id: 81012973 QId: 34811 (921666) 35 of 75 A A A Exhibit A client with type 1 diabetes has a prescription for 30 units of insulin glargine at bedtime. Fingerstick blood glucose measurements are prescribed before meals and at bedtime with regular insulin based on a sliding scale. At 9:00 PM, the client's blood glucose measurement is 180 mg/dL (10.0 mmol/L). What action should the nurse take? Click on the exhibit button for additional information. 1. Administer 30 units of glargine; give the client a snack, then administer 2 units of regular insulin [12%] 2. Administer 30 units of glargine and 2 units of regular insulin in 2 different injections [42%] 3. Mix 30 units of glargine with 2 units of regular insulin in the same syringe, drawing up the glargine first [7%] 4. Mix 30 units of glargine with 2 units of regular insulin in the same syringe, drawing up the regular insulin first [36%] Correct Answered correctly 42% Time: 48 seconds Updated: 03/19/2017 Explanation: A combination of long-acting insulin (eg, glargine, detemir) with rapid- (eg, lispro, aspart) or short-acting (eg, regular) insulin is often prescribed for clients with diabetes. The different onsets, peaks, and durations mimic the body's natural insulin levels and enhance glycemic control. Long-acting (basal) insulins have no peak and may last 24 hours or longer. Short-acting insulins peak 2-5 hours after administration and last approximately 5-8 hours. Regular or rapid-acting insulins may be given on a sliding scale at prescribed intervals (eg, before meals and at bedtime) and are dosed based on the client's blood glucose measurement. Insulin glargine and regular insulin may be safely given concurrently due to the differences in onset, peak, and duration (Option 2). (Option 1) Because long-acting insulins have no peak, they should not potentiate hypoglycemia. Regular insulin may cause hypoglycemia. However, concurrent administration of regular insulin with insulin glargine will not increase the probability of hypoglycemia as each medication has a different onset, peak, and duration; therefore, a snack is not required. (Options 3 and 4) Insulin glargine should not be mixed in a single syringe with any other insulin as the mixture may alter the pharmacodynamics of the drug. Educational objective: Sliding-scale regular insulin can be concurrently administered with scheduled insulin glargine without potentiating hypoglycemia as long as both medications are properly dosed and administered as separate injections. Insulin glargine should not be mixed in a syringe with any other insulin. Copyright © UWorld. All rights reserved.

Block Time Remaining: 00:27:56 TUTOR Test Id: 81012973 QId: 31884 (921666) 36 of 75 A A A Exhibit A client with heart failure is started on furosemide. The laboratory results are shown in the exhibit. The nurse is most concerned about which condition? Click on the exhibit button for additional information. 1. Atrial fibrillation [27%] 2. Atrial flutter [11%] 3. Mobitz II [7%] 4. Torsades de pointes [53%] Correct Answered correctly 53% Time: 34 seconds Updated: 03/12/2017 Explanation: Hypomagnesemia (normal: 1.5-2.5 mEq/L [0.75-1.25 mmol/L]) causes a prolonged QT interval that increases the client's susceptibility to ventricular tachycardia. Torsades de pointes is a type of polymorphic ventricular tachycardia coupled with a prolonged QT interval; it is a lethal cardiac arrhythmia that leads to decreased cardiac output and can develop quickly into ventricular fibrillation. The American Heart Association recommends treatment with IV magnesium sulfate. (Option 1) Characteristics of atrial fibrillation (AF) include an irregularly irregular rhythm and replacement of P waves by fibrillatory waves. Although electrolyte disturbances increase the likelihood of developing AF, clients can have this chronic condition managed with anticoagulation therapy. AF is usually associated with an underlying heart disease and is rarely immediately life-threatening. (Option 2) Atrial flutter is characterized by sawtooth-shaped flutter waves. There is no clinical evidence suggesting that hypomagnesemia leads to atrial flutter, which is associated with underlying heart disease (eg, mitral valve disorders, cardiomyopathy, cor pulmonale). (Option 3) Mobitz II (type II second-degree atrioventricular block) is usually not associated with electrolyte disturbances but is more often associated with conduction system disease or drug toxicity (eg, beta blockers, calcium channel blockers). Educational objective: In a client with hypomagnesemia, it is important to assess the QT interval. The client is most at risk for torsades de pointes, a serious complication that can develop quickly into ventricular fibrillation (lethal arrhythmia). Copyright © UWorld. All rights reserved.

Block Time Remaining: 00:28:34 TUTOR Test Id: 81012973 QId: 31409 (921666) 37 of 75 A A A The registered nurse and practical nurse are conducting a workshop on contraceptive methods for a group of outpatient clients. Which instructions should the nurses include when discussing combined estrogen-progestin oral contraceptives? Select all that apply. 1. Consult the health care provider (HCP) if you experience leg pain or swelling 2. Discontinue contraceptives if you experience spotting between menses 3. Do not smoke while taking combined contraceptives 4. Immediately report any breast tenderness to the HCP 5. Seek immediate medical treatment if you experience vision loss Correct Answered correctly 71% Time: 38 seconds Updated: 03/12/2017 Explanation: The use of hormonal contraception (ie, estrogen with or without progestin) places women at a 2- to 4-fold increased risk for developing blood clots due to resulting hypercoagulability. Hormone levels vary among contraceptives, and higher levels of hormone content correlate to an increased risk of adverse thrombotic events (eg, stroke, myocardial infarction). Clients who are prescribed oral contraceptive pills (OCPs) containing estrogen should be educated on potential warning signs (eg, chest pain, vision loss, severe leg pain) (Options 1 and 5). In addition, clients should be instructed not to smoke while taking combined OCPs due to an increased risk of blood clots (Option 3). (Option 2) Irregular bleeding and spotting between menses are common side effects of combined OCPs. These side effects may be bothersome but are not serious and may improve within 3 months of initiation. If the client cannot tolerate side effects, a different OCP may be considered. (Option 4) Clients should be counseled that breast tenderness is a common side effect of combined OCPs and does not warrant emergent reporting to the health care provider. Educational objective: Clients who are prescribed oral estrogen contraceptives (with or without progestin) have an increased risk for developing blood clots. Clients should be educated on warning signs to report to the health care provider (eg, severe leg pain, vision loss) versus common side effects (eg, breast tenderness, spotting). Copyright © UWorld. All rights reserved.

Block Time Remaining: 00:23:23 TUTOR Test Id: 81075711 QId: 31457 (921666) 45 of 75 A A A Which client in a prenatal clinic should the nurse assess first? 1. Client at 11 weeks gestation with backache and pelvic pressure [13%] 2. Client at 16 weeks gestation with earache and sinus congestion [3%] 3. Client at 27 weeks gestation with headache and facial edema [78%] 4. Client at 37 weeks gestation with white vaginal discharge and urinary frequency [4%] Correct Answered correctly 78% Time: 26 seconds Updated: 04/10/2017 Explanation: Gestational hypertension is new-onset high blood pressure (≥140/90 mm Hg) that occurs after 20 weeks gestation without proteinuria. The development of proteinuria with hypertension indicates preeclampsia, which may manifest with symptoms such as headache, visual disturbances, and facial swelling. This client is exhibiting symptoms of preeclampsia and should be assessed first (Option 3). Complications of preeclampsia may include thrombocytopenia, liver dysfunction, and renal insufficiency. Clients with preeclampsia must be monitored closely for sudden worsening, which can lead to serious complications, including eclampsia and/or HELLP syndrome (hemolysis, elevated liver enzymes, and low platelets). (Option 1) Backache and pelvic pressure are common discomforts of pregnancy. The enlarging uterus stretches the supporting ligaments and may cause increased backache. Fluid retention from increasing blood volume causes pelvic congestion, which may be reported as pelvic pressure. (Option 2) Earache and sinus congestion are common discomforts during pregnancy that result from increased blood volume and fluid retention. This client should be assessed to rule out sinus and/or ear infection, but this is not the highest priority. (Option 4) The increase in estrogen and progesterone during pregnancy often results in leukorrhea, a mucoid, white vaginal discharge. Pressure on the bladder from the fetal head during the third trimester may cause diminished bladder capacity and urinary frequency without dysuria. These are common and expected findings. Educational objective: The nurse should assess the client with symptoms of potentially serious complications first before assessing the remaining clients. Signs of hypertensive disorders during pregnancy may include headache and facial edema. Copyright © UWorld. All rights reserved.

Block Time Remaining: 00:29:44 TUTOR Test Id: 81075711 QId: 30814 (921666) 56 of 75 A A A The nurse assessing a client's pain would expect the client to make which statement when describing the abdominal pain associated with appendicitis? 1. "My pain is a burning sensation in my upper abdomen." [2%] 2. "My pain is an 8 out of 10 and on my left side below my belly button." [12%] 3. "My pain is excruciating in my lower abdomen above my right hip." [76%] 4. "My pain is intermittent in my abdomen and right shoulder." [9%] Correct Answered correctly 76% Time: 35 seconds Updated: 05/29/2017 Explanation: The appendix is a blind pouch located at the junction of the ileum of the small intestine and the beginning of the large intestine (cecum). When infected or obstructed (foreign body, fecal material, tumor, lymph tissue), the appendix becomes inflamed, causing acute appendicitis. Signs and symptoms of acute appendicitis include the following: Pain: Continuous; begins in the periumbilical region and then moves to the right lower quadrant centering at McBurney's point (one-third of the distance from the right anterior superior iliac spine to the umbilicus) (Option 3) Gastrointestinal symptoms: Anorexia, nausea, and vomiting Rebound tenderness and guarding Clients with acute appendicitis attempt to decrease pain by preventing increased intraabdominal pressure (eg, avoiding coughing, sneezing, deep inhalation) and lying still with the right leg flexed. (Option 1) Burning pain in the upper abdomen can be due to gastric or duodenal ulcers. If the ulcers are located posteriorly, the client may experience back pain. (Option 2) Pain in the left lower quadrant is associated with diverticulitis (often in the sigmoid colon). Other signs and symptoms include a palpable, tender abdominal mass and systemic symptoms of infection (fever, increased C-reactive protein, and leukocytosis with a left shift). (Option 4) Pain and tenderness in the epigastric or right upper quadrant of the abdomen that is referred to the right scapula is associated with acute cholecystitis. Clients may also experience indigestion, nausea, vomiting, restlessness, and diaphoresis. Educational objective: Pain associated with acute appendicitis typically begins in the periumbilical region and migrates to the area overlying the appendix (McBurney's point). The client will attempt to decrease pain by lying still with the right leg flexed and preventing increased intraabdominal pressure (eg, avoiding coughing, sneezing, deep inhalation). Copyright © UWorld. All rights reserved.

Block Time Remaining: 00:30:59 TUTOR Test Id: 81012973 QId: 32245 (921666) 38 of 75 A A A The nurse receives laboratory reports on 4 clients. Which report is most concerning and should be reported to the health care provider? 1. The client admitted with asthma exacerbation who has a PaCO2 of 32 mm Hg (4.26 kPa) [20%] 2. The client diagnosed with chronic obstructive pulmonary disease whose latest arterial blood gas shows a PaO2 of 85 mm Hg (11.33 kPa) [11%] 3. The client receiving warfarin for atrial fibrillation whose morning laboratory report includes an INR of 2.5 [7%] 4. The client who had a total knee replacement 2 hours ago and whose hemoglobin is 7 g/dL (70 g/L) [59%] Incorrect Correct answer 4 Answered correctly 59% Time: 145 seconds Updated: 01/31/2017 Explanation: Blood loss is a common complication of a total knee replacement, and a hemoglobin level of 7 g/dL (70 g/L) is very low (normal adult male: 13.2-17.3 g/dL [132-173 g/L]; normal adult female: 11.7-15.5 g/dL [117-155 g/L]). This client needs to be assessed for any active bleeding as well as for respiratory and cardiac complications (eg, rapid pulse, shortness of breath) resulting from the low hemoglobin level. The health care provider must be notified. (Option 1) Normal PaCO2 is 35-45 mm Hg (4.66-5.98 kPa). Clients with asthma exacerbations, as well as those with panic attacks, pulmonary embolism, and pneumonia, will have rapid breathing. In all of these conditions, this rapid breathing pushes more CO2 out of lungs, with a mild decrease in PaCO2 as the body's expected compensatory response. In these clients, retention of CO2 (or even normal PaCO2) is more dangerous as it indicates respiratory muscle fatigue (failure) resulting in retention of PaCO2. (Option 2) PaO2 >80 mm Hg (10.66 kPa) is considered a normal finding. In clients with chronic obstructive pulmonary disease (COPD), airflow out of the lungs is impeded, trapping CO2 in the lungs. The body adjusts to the higher CO2 level (which would cause an increase in respirations in a non-COPD client) and then uses the PaO2 as the drive for breathing. (Option 3) Warfarin is prescribed to prevent blood clotting in clients with atrial fibrillation. To be therapeutic and prevent clotting, the dosage of warfarin is adjusted to maintain an INR of 2-3. This client's INR is therapeutic for the diagnosis of atrial fibrillation. Educational objective: Blood loss is a common complication of a total knee replacement. Monitoring the client postoperatively for signs of blood loss and active bleeding is a priority. Copyright © UWorld. All rights reserved.

Block Time Remaining: 00:32:39 TUTOR Test Id: 81012973 QId: 33466 (921666) 39 of 75 A A A A new graduate nurse is preparing to administer the following analgesics to clients with postoperative pain. Which situation would require intervention by the precepting nurse? 1. Chooses to administer 50 mcg of the prescribed 50-100 mcg of IV fentanyl for the first dose [5%] 2. Dilutes hydromorphone with 5 mL of normal saline and injects IV push over 2 minutes [17%] 3. Injects 1 mg of morphine sulfate undiluted via IV push over 5 minutes [24%] 4. Selects a 25-gauge ½-inch (1.3-cm) needle to inject ketorolac intramuscularly [53%] Incorrect Correct answer 4 Answered correctly 53% Time: 100 seconds Updated: 04/19/2017 Explanation: Ketorolac is a nonsteroidal anti-inflammatory drug (NSAID) analgesic administered (orally, IV, or intramuscularly [IM]) for short-term relief of mild to moderate pain. Usage should not exceed 5 days due to adverse effects (eg, kidney injury, gastrointestinal ulcers, bleeding). Ketorolac IM should be administered into a large muscle using the Z-track method to mitigate burning and discomfort. A 1- to 1½-in (2.5- to 3.8-cm) needle is recommended to inject medication into the proper muscular space in average-weight individuals. (Option 1) The amount of analgesic to administer of a variable dose medication should be based on the client's pain level, level of consciousness, and history of narcotic use. Selecting a smaller first dose is appropriate if the nurse is unsure of how the client will respond to the medication. If needed, the larger amount can be given the next time a dose is requested or an additional one-time dose can be requested from the health care provider if breakthrough pain occurs (before the next scheduled medication dose is available). (Option 2) Hydromorphone IV push, given undiluted or diluted with 5 mL of sterile water or normal saline, should be administered slowly over 2-3 minutes; rapid infusion increases the risk of opioid-induced adverse reactions (eg, nausea, itching). (Option 3) Undiluted morphine IV push should be administered slowly over 4-5 minutes; rapid infusion increases the risk of opioid-induced adverse reactions (eg, hypotension, flushing). Educational objective: Ketorolac, a nonsteroidal anti-inflammatory drug, is used for short-term (≤5 days) pain relief due to risk of bleeding, gastrointestinal ulcers, and kidney injury. Intramuscular (IM) injections (using Z-track method) should be given deep into a large muscle due to burning and discomfort. A 1- to 1½-in (2.5- to 3.8-cm) needle is used to reach the proper muscle space. Copyright © UWorld. All rights reserved.

Sildenafil (Viagra) is a phosphodiesterase inhibitor used to treat erectile dysfunction

Block Time Remaining: 00:32:59 TUTOR Test Id: 81012973 QId: 31146 (921666) 40 of 75 A A A The nurse admits a client with newly diagnosed unstable angina. Which information obtained during the admission health history is most important for the nurse to report to the health care provider (HCP) immediately? 1. Drinks 6 cans of beers on the weekend [1%] 2. Gets up 4 times during the night to void [9%] 3. Smokes 1 pack of cigarettes daily [33%] 4. Uses sildenafil occasionally [55%] Incorrect Correct answer 4 Answered correctly 55% Time: 20 seconds Updated: 12/19/2016 Explanation: Sildenafil (Viagra) is a phosphodiesterase inhibitor used to treat erectile dysfunction. The use of sildenafil is most important for the nurse to report to the HCP. This must be communicated immediately as concurrent use of nitrate drugs (commonly prescribed to treat unstable angina) is contraindicated as it can cause life-threatening hypotension. Before any nitrate drugs can be administered, further action is necessary to determine when sildenafil was taken last (ie, half-life is about 4 hours). (Option 1) Clients do not always report the amount of alcohol they consume accurately. The nurse should monitor all clients for alcohol withdrawal syndrome as it is quite common in hospitalized clients. (Option 2) Getting up 4 times during the night to void can be associated with medication, an enlarged prostate gland, or drinking fluids at bedtime. Further action may be needed to determine the cause of the nocturia, but this is not the most significant information to report to the HCP. (Option 3) Smoking 1 pack of cigarettes daily needs to be addressed as tobacco causes vasoconstriction and decreased oxygen supply to the body tissues. Further action is needed regarding smoking cessation education. However, the client's tobacco history is not the most important information to report to the HCP. Educational objective: Nitrate drugs are prescribed to treat angina. The concurrent use sildenafil (Viagra) and nitrates is contraindicated as it can cause life-threatening hypotension. Copyright © UWorld. All rights reserved.

Block Time Remaining: 00:34:11 TUTOR Test Id: 81012973 QId: 30167 (921666) 45 of 75 A A A In the intensive care unit, the nurse cares for a client who develops diabetes insipidus (DI) 2 days after pituitary adenoma removal via hypophysectomy. Which intervention should the nurse implement? 1. Administer desmopressin [65%] 2. Assess fasting blood glucose [16%] 3. Institute fluid restriction [14%] 4. Place the client in the Trendelenburg position [3%] Incorrect Correct answer 1 Answered correctly 65% Time: 29 seconds Updated: 04/28/2017 Explanation: Diabetes insipidus (DI) is a condition that occurs due to insufficient production/suppression of antidiuretic hormone (ADH). Neurogenic DI is a type of DI that results from impaired ADH secretion, transport, or synthesis. It sometimes occurs after manipulation of the pituitary or other parts of the brain during surgery, brain tumors, head injury, or central nervous system infections. DI is characterized by polydipsia (increased thirst) and polyuria (increased urine output) with low urine specific gravity (dilute urine). As a result, fluids should be replaced orally/intravenously to prevent dehydration (Option 3). ADH release is impaired in neurogenic DI. As a result, ADH replacement with vasopressin (Pitressin) can be used to treat DI. However, it also has vasoconstrictive properties. Therefore, desmopressin (DDAVP), an analog without vasopressor activity, is the preferred therapy. Clients on this treatment should be monitored for urine output, urine specific gravity, and serum sodium (to avoid hyponatremia due to excess DDAVP). (Option 2) DI is not associated with low/high blood glucose and should not be confused with diabetes mellitus (DM) as both DI and DM involve symptoms of excessive urination (polyuria). (Option 4) The Trendelenburg position (body laid flat and supine with feet higher than the head by at least 15-30 degrees) is contraindicated in most neurological conditions. Educational objective: DI occurs when there is insufficient production/suppression of ADH. It is characterized by polydipsia and polyuria with diluted urine. Oral and/or intravenous fluid replacement is imperative to prevent dehydration. DI is treated with ADH replacement drugs (eg, desmopressin acetate [DDAVP]). Clients should be monitored for urine output, urine specific gravity, and serum sodium. Copyright © UWorld. All rights reserved.

Block Time Remaining: 00:35:46 TUTOR Test Id: 81012973 QId: 30739 (921666) 48 of 75 A A A Which of the following findings reflect concerning vital signs that require further nursing assessment and intervention? Select all that apply. 1. After albuterol administration, 5-year-old client reports tremor and has pulse 120/min 2. After hydromorphone 1 mg intravenous push (IVP), BP decreases from 130/80 mm Hg to 110/70 mm Hg 3. Client receiving blood transfusion; pre-infusion: BP 120/80 mm Hg and pulse 80/min; now: 90/70 mm Hg and 100/min 4. Fetal heart rate monitored during labor changes from 140/min to 100/min with decelerations 5. Nurse preparing to administer prescribed nifedipine; blood pressure is 90/60 mm Hg Correct Answered correctly 53% Time: 56 seconds Updated: 03/20/2017 Explanation: Acute hemolytic reaction during a blood transfusion usually develops within the first 15 minutes. Signs/symptoms include chills, fever, lower back pain (from damaged cells in the kidneys), tachycardia, tachypnea, and hypotension. Acute hemolytic reaction is an emergency that requires the nurse to stop the transfusion and treat shock. Normal fetal heart tones are 110-160/min. Decrease in heart rate with decelerations could indicate uteroplacental insufficiency and must be assessed. Nifedipine (Procardia) is a potent calcium channel blocker antihypertensive. It should not be administered when the client's BP is on the lower end of the acceptable range, as this may result in hypotension. (Option 1) Albuterol (Ventolin) is a bronchodilator beta-adrenergic agonist. Expected side effects include tremor, tachycardia, and palpitation. Normal pulse rate in a 5-year-old can be 70-120/min and averages 100/min. This is the upper limit of expected findings. (Option 2) This is the upper normal dosing limit for initial IVP administration of hydromorphone (Dilaudid), a potent narcotic. Hypotension and bradycardia are expected adverse effects. Orthostatic hypotension occurs most often with ambulation or positioning in the semi-Fowler's position. Clients are not ordinarily allowed to be ambulatory for 20-30 minutes after IVP administration of narcotics. This one-time reading is not significant enough to require emergency intervention. Educational objective: Beta-adrenergic agents have an expected side effect of tremor or palpitations, and narcotics can cause mild hypotension or bradycardia. Intervention is not necessary unless the values are significant or the client becomes symptomatic. Copyright © UWorld. All rights reserved.

Block Time Remaining: 00:35:46 TUTOR Test Id: 81012973 QId: 30739 (921666) 48 of 75 A A A Which of the following findings reflect concerning vital signs that require further nursing assessment and intervention? Select all that apply. 1. After albuterol administration, 5-year-old client reports tremor and has pulse 120/min 2. After hydromorphone 1 mg intravenous push (IVP), BP decreases from 130/80 mm Hg to 110/70 mm Hg 3. Client receiving blood transfusion; pre-infusion: BP 120/80 mm Hg and pulse 80/min; now: 90/70 mm Hg and 100/min 4. Fetal heart rate monitored during labor changes from 140/min to 100/min with decelerations 5. Nurse preparing to administer prescribed nifedipine; blood pressure is 90/60 mm Hg Correct Answered correctly 53% Time: 56 seconds Updated: 03/20/2017 Explanation: Acute hemolytic reaction during a blood transfusion usually develops within the first 15 minutes. Signs/symptoms include chills, fever, lower back pain (from damaged cells in the kidneys), tachycardia, tachypnea, and hypotension. Acute hemolytic reaction is an emergency that requires the nurse to stop the transfusion and treat shock. Normal fetal heart tones are 110-160/min. Decrease in heart rate with decelerations could indicate uteroplacental insufficiency and must be assessed. Nifedipine (Procardia) is a potent calcium channel blocker antihypertensive. It should not be administered when the client's BP is on the lower end of the acceptable range, as this may result in hypotension. (Option 1) Albuterol (Ventolin) is a bronchodilator beta-adrenergic agonist. Expected side effects include tremor, tachycardia, and palpitation. Normal pulse rate in a 5-year-old can be 70-120/min and averages 100/min. This is the upper limit of expected findings. (Option 2) This is the upper normal dosing limit for initial IVP administration of hydromorphone (Dilaudid), a potent narcotic. Hypotension and bradycardia are expected adverse effects. Orthostatic hypotension occurs most often with ambulation or positioning in the semi-Fowler's position. Clients are not ordinarily allowed to be ambulatory for 20-30 minutes after IVP administration of narcotics. This one-time reading is not significant enough to require emergency intervention. Educational objective: Beta-adrenergic agents have an expected side effect of tremor or palpitations, and narcotics can cause mild hypotension or bradycardia. Intervention is not necessary unless the values are significant or the client becomes symptomatic. Copyright © UWorld. All rights reserved.

Block Time Remaining: 00:36:28 TUTOR Test Id: 81012973 QId: 34357 (921666) 49 of 75 A A A The student nurse is preparing to perform a heel stick on a neonate to collect blood for diagnostic testing. Which statement by the student nurse indicates a need for further education? 1. "I can perform the stick on either the medial or lateral side of the outer aspect of the heel." [35%] 2. "Sucrose and a pacifier can help alleviate the infant's pain and stress during the puncture." [15%] 3. "The heel area should be warmed for 3-5 minutes prior to puncture." [17%] 4. "Venipuncture should be reserved only for failed heel sticks because it is more painful." [32%] Correct Answered correctly 32% Time: 42 seconds Updated: 04/05/2017 Explanation: The neonatal heel stick (heel lancing) is used to collect a blood sample to assess capillary glucose and perform newborn screening for inherited disorders (eg, congenital hypothyroidism, phenylketonuria). Proper technique is essential for minimizing discomfort and preventing complications and includes: Select a location on the medial or lateral side of the outer aspect of the heel (Option 1). Avoid the center of the heel to prevent accidental insult to the calcaneus. Puncture should not occur over edematous or infected skin. Warm the heel for several minutes with a warm towel compress or approved single-use instant heat pack to promote vasodilation (Option 3). Cleanse the intended puncture site with alcohol. Sucrose and nonnutritive sucking on a pacifier may reduce procedural pain (Option 2). Use an automatic lancet, which controls the depth of puncture. Lancing the heel too deeply can result in penetration of the calcaneus bone, leading to osteochondritis or osteomyelitis. An acceptable alternate method of blood collection in the neonate is venipuncture (ie, drawing blood from a vein). Venipuncture is considered less painful and often requires fewer punctures to obtain a sample, especially if a larger volume is needed (Option 4). Educational objective: To perform a neonatal heel stick, select a location on the medial or lateral side of the outer aspect of the heel to avoid insult to the calcaneus bone. Provide comfort measures (eg, nonnutritive sucking), warm the selected puncture site to promote vasodilation, cleanse with alcohol, and puncture using an automatic lancet. Copyright © UWorld. All rights reserved.

Block Time Remaining: 00:31:19 TUTOR Test Id: 81075711 QId: 30576 (921666) 60 of 75 A A A Exhibit The new emergency department nurse is caring for a client with a gunshot wound. The cardiac monitor alarms. The rhythm seen on the monitor is displayed in the exhibit. The nurse is unable to palpate a pulse. Which action by the new nurse would cause the charge nurse to intervene? Click on the exhibit button for additional information. 1. Administer IV epinephrine [17%] 2. Initiate chest compressions [22%] 3. Provide defibrillator shock [40%] 4. Provide oxygen with bag-mask [19%] Incorrect Correct answer 3 Answered correctly 40% Time: 27 seconds Updated: 05/15/2017 Explanation: The client in asystole has a total absence of ventricular electrical activity and is pulseless, apneic, and unresponsive. The nurse should first assess the client to verify the monitor reading. The nurse would next call for help and initiate cardiopulmonary resuscitation and oxygenated ventilation (Options 2 and 4). Advanced cardiovascular life-support measures for asystole include epinephrine IV every 3-5 minutes, possible placement of an advanced airway (ie, intubation), and treatment of reversible causes (eg, hypovolemia) (Option 1). Unfortunately, clients in asystole often cannot be resuscitated. (Option 3) Defibrillation is not indicated when electrical activity is absent (asystole) or when pulseless electrical activity is present. Defibrillation is used for treating ventricular fibrillation and pulseless ventricular tachycardia. Educational objective: Asystole is characterized by the absence of all ventricular electrical activity. The client is pulseless, apneic, and unresponsive. Treatment includes cardiopulmonary resuscitation, oxygenated ventilation, and advanced cardiovascular life-support measures (eg, epinephrine IV, advanced airway, treatment of reversible causes). Copyright © UWorld. All rights reserved.

Block Time Remaining: 00:36:53 TUTOR Test Id: 81075711 QId: 30859 (921666) 70 of 75 A A A The nurse is conducting a pain assessment on a client with dysuria. Which pain description is most likely associated with pyelonephritis? 1. Constant; increased by pressure over the suprapubic area [6%] 2. Dull and continuous; occasional spasms over the suprapubic area [9%] 3. Dull flank pain; extending toward the umbilicus [40%] 4. Excruciating; sharp flank pain radiating to the groin [43%] Correct Answered correctly 40% Time: 46 seconds Updated: 05/15/2017 Explanation: Urinary tract infections (UTIs) can occur in the kidneys (pyelonephritis), bladder (cystitis), and/or urethra (urethritis). Pyelonephritis (inflammation of the kidney parenchyma) causes flank pain that is experienced in the back at the costovertebral angle (the angle between the lower ribs and adjacent vertebrae) and may spread toward the umbilicus. Cystitis alone does not cause nausea/vomiting or chills. Presence of these, fever, and signs and symptoms of a lower UTI (dysuria, urgency, and frequency) indicate pyelonephritis. (Option 1) The client with a distended bladder experiences constant pain increased by any pressure over the bladder. Bladder distension is found through palpation (firmness, pain, urgency) and percussion (dullness) over the suprapubic area. (Option 2) Bladder and urethral pain is usually dull and continuous and may be experienced as spasms. The detrusor muscle of the bladder may spasm if cystitis is present. (Option 4) Renal colic pain (in response to renal calculi) is excruciating, sharp, and stabbing; the client would be tossing in the bed unable to find a comfortable position. Pain radiates down to the groin area as the stone travels down the ureter. Educational objective: Pain in pyelonephritis is dull, constant, and maximal at the costovertebral angle area. Pain from renal stones is excruciating, sharp, and often radiates toward the groin from the flank. Suprapubic pain indicates bladder distension or cystitis. Spasms can be seen with infection (cystitis) or manipulation of the bladder. Copyright © UWorld. All rights reserved.

Block Time Remaining: 00:37:00 TUTOR Test Id: 81012973 QId: 31960 (921666) 50 of 75 A A A A client is newly admitted to the mental health unit with a diagnosis of schizophrenia with persecutory delusions. Which nursing interventions should the nurse include in the client's plan of care with regard to the delusional thinking? Select all that apply. 1. Explore the meaning behind the client's delusions 2. Focus on reality and verbally reinforce it 3. Focus on the client's feelings secondary to the delusions 4. Gently confront the client about the false beliefs 5. Present logical explanations to discredit the delusions Incorrect Correct answer 2,3 Answered correctly 11% Time: 32 seconds Updated: 12/18/2016 Explanation: Clients with persecutory delusions (paranoid delusions) believe that they are being persecuted or harmed (eg, spied on, cheated, followed, poisoned). Focusing on the client's feelings secondary to the delusion is an example of empathy, one of the most important parts of the therapeutic nurse-client relationship. When nurses attempt to understand clients' feelings and their meaning, clients realize that someone is trying to understand them and the nurse-client relationship grows (Option 3). Focusing on reality and verbally reinforcing it will decrease the time that the client spends thinking about the delusions (Option 2). For example, the nurse may focus on the client's feelings by stating, "I understand that it is frightening to know that someone is trying to poison you." Reality orientation may also be helpful by telling the client, "What you are thinking is part of your disease and not real." (Option 1) Attempting to explore the meaning behind a delusion will encourage the client to focus/think more on this delusion. (Option 4) Confronting the client about the delusion is not therapeutic because arguing will not eradicate the delusion. It also hinders the development of a trusting nurse-client relationship. (Option 5) Clients believe that their delusions are real despite proof otherwise. Presenting logical explanations to discredit the delusions will not help. Educational objective: When communicating with a delusional client, the nurse must focus on the client's feelings and reinforce reality rather than argue or present evidence that the delusion is false or irrational. Copyright © UWorld. All rights reserved.

Block Time Remaining: 00:37:23 TUTOR Test Id: 81012973 QId: 30758 (921666) 51 of 75 A A A The health care provider (HCP) has prescribed amitriptyline 25 mg orally every morning for an elderly client with recent herpes zoster infection (shingles) and severe postherpetic neuralgia. What is the priority nursing action? 1. Encourage increased fluid intake [19%] 2. Provide frequent rest periods [10%] 3. Teach the client to get up slowly from the bed or a sitting position [55%] 4. Tell the client to wear sunglasses when outdoors [14%] Incorrect Correct answer 3 Answered correctly 55% Time: 23 seconds Updated: 05/07/2017 Explanation: Tricyclic antidepressants (eg, amitriptyline, nortriptyline, desipramine, imipramine) are commonly used for neuropathic pain. Side effects are especially common in elderly clients. Due to the increased risk of falling, the priority nursing action is to teach the client to get up slowly from the bed or a sitting position. (Options 1, 2, and 4) These are important instructions but not priority ones. Educational objective: The most common side effects experienced by clients taking tricyclic antidepressants include dizziness, drowsiness, dry mouth, constipation, photosensitivity, urinary retention, and blurred vision. The priority nursing action is to teach caution in changing positions due to the increased risk for falls from dizziness and orthostatic hypotension, especially in elderly clients. Copyright © UWorld. All rights reserved.

Block Time Remaining: 00:37:50 TUTOR Test Id: 81012973 QId: 30287 (921666) 52 of 75 A A A Which client statement demonstrates mental health well-being when considering stress and anxiety? 1. "I know that relaxation techniques help me deal with my life's stress and anxiety." [74%] 2. "I understand stress and anxiety because my family has a history of depression." [0%] 3. "You must understand that stress and anxiety affect everyone's life." [4%] 4. "You should identify and then avoid those things that cause you stress and anxiety." [20%] Correct Answered correctly 74% Time: 27 seconds Updated: 04/26/2017 Explanation: Well-being is demonstrated by the ability to cope with routine stress and continue to function adequately even with the resulting anxiety. (Option 2) Understanding the concepts of stress and anxiety does not ensure ability to cope effectively to maintain mental health well-being. (Option 3) Understanding that stress and anxiety are present in daily life does not ensure the ability to cope effectively to maintain mental health well-being. (Option 4) Although it may be possible to identify one's stress and anxiety triggers, it is not possible to avoid them all. Coping techniques are required to maintain mental health well-being. Educational objective: The mental health continuum uses a continuous line to represent the transition from mental health to mental illness. This passage varies from person to person and within an individual over time but is demonstrated by the ability to cope and function effectively with routine stress and anxiety. Copyright © UWorld. All rights reserved.

Block Time Remaining: 00:38:18 TUTOR Test Id: 81012973 QId: 31743 (921666) 53 of 75 A A A A blood transfusion is prescribed for a client with sickle cell exacerbation and a hemoglobin level of 6 g/dL (60 g/L). Which are appropriate actions by the registered nurse? Select all that apply. 1. Administer O negative (O-) blood to the AB positive (AB+) client 2. Delegate the fourth set of vital signs to the unlicensed assistive personnel 3. Prime line with normal saline prior to hanging the blood 4. Time the blood infusion to occur over a 6-hour period 5. Validate the client's name and room number with a licensed practical nurse Correct Answered correctly 20% Time: 28 seconds Updated: 04/14/2017 Explanation: The blood type O- is the "universal donor" as it has no anti-A or anti-B antigens; AB+ is the "universal recipient" as the lack of antibodies allows any blood type to be transfused (Option 1). Blood is always transfused with normal saline, not dextrose. The line should be established prior to obtaining the blood. Most facilities have a policy to start the blood within 30 minutes of obtaining it to prevent bacterial growth (Option 3). The most likely time for a serious ABO incompatibility/transfusion reaction is when the infused blood first enters the client's body. The registered nurse (RN) should remain in the room for the first 15 minutes/50 mL of the transfusion. However, the fourth set of vital signs would be taken after 1 hour of infusion; it would be safe to delegate this data collection to the unlicensed assistive personnel (the RN will analyze the vital signs) (Option 2). (Option 4) Most facilities want the transfusion completed in 2-4 hours. "Old" blood is more likely to break apart and cause hyperkalemia from the intracellular potassium leak. (Option 5) Most policies have the RN checking with another RN or qualified health care professional prior to blood administration. At least 2 identifiers such as name, medical record number, or date of birth can be used. Client identifiers never include a room number. Educational objective: Safe blood transfusion protocol includes checking of at least 2 client identifiers by 2 qualified health professionals, using normal saline to prime, and giving the infusion in 2-4 hours. The unlicensed assistive personnel can take vital signs during the later part of the transfusion. O- is the universal donor blood type and AB+ is the universal recipient. Copyright © UWorld. All rights reserved.

Block Time Remaining: 00:02:28 TUTOR Test Id: 72490738 QId: 30588 (921666) 9 of 75 A A A The nurse provides instruction to a community group about lung cancer prevention, health promotion, and smoking cessation. Which statement made by a member of the group indicates the need for further instruction? 1. "Even though I am getting nicotine in my patches, I am not being exposed to all of the other toxic stuff in cigarettes." [7%] 2. "I can't get lung cancer because I don't smoke." [90%] 3. "My husband needs to take smoking cessation classes." [0%] 4. "We installed a radon detector in our home." [2%] Omitted Correct answer 2 Answered correctly 90% Time: 24 seconds Updated: 04/26/2017 Explanation: Smoking is responsible for 80%-90% of all lung cancers. Although the risk is greater among smokers, former smokers and nonsmokers can develop lung cancer as well. Risk factors include secondhand smoke, air pollution, genetic predisposition, and exposure to radon, asbestos, and chemicals in the workplace. (Option 1) Smoking cessation is the best way to prevent lung cancer. Nicotine replacement therapy (eg, patches, gum, inhalers, lozenges) is effective in helping smokers quit by reducing cravings. Although users receive a low dose of nicotine, they do not receive the other toxins that cigarettes include. (Option 3) The best way to reduce the risk of lung cancer is to avoid both firsthand and secondhand smoke. Smoke from someone else's burning cigarette contains the same carcinogens as those found in mainstream smoke and creates a health risk to those inhaling it. (Option 4) Exposure to high levels of radon can cause lung cancer. Radon levels must be tested before a home can be sold. Educational objective: Although the risk is greater among smokers, former smokers and nonsmokers can develop lung cancer. Risk factors include secondhand smoke, air pollution, genetic predisposition, and exposure to radon, asbestos, and chemicals in the workplace. Copyright © UWorld. All rights reserved.

Block Time Remaining: 00:02:47 TUTOR Test Id: 72490738 QId: 33811 (921666) 10 of 75 A A A During a camping trip, a camp counselor falls and gets a small splinter of wood embedded in the right eye. What action should the volunteer camp nurse take first? 1. Gently flush the eye with cool water [45%] 2. Instill optic antibiotic ointment [2%] 3. Patch both eyes with eye shields [46%] 4. Remove the splinter using tweezers [5%] Incorrect Correct answer 3 Answered correctly 46% Time: 19 seconds Updated: 01/07/2017 Explanation: The camp nurse protects the injured eye using an eye shield (eg, small Styrofoam or paper cup), ensuring the shield does not touch the foreign body. The eyes work in synchrony with each other; therefore, the non-injured eye is patched to prevent further eye movement. The nurse also facilitates transport to the nearest emergency care center for assessment and treatment by an ophthalmologist. (Option 1) Flushing the eye with cool water is contraindicated as it may cause further damage by moving the splinter and/or introducing potential wound pathogens. (Option 2) Instilling optic antibiotic ointment would interfere with ophthalmologic medical examination. Optic antibiotic ointment may be prescribed by the health care provider to reduce the risk of infection once the object is removed from the eye. (Option 4) The nurse should not attempt to remove a foreign body embedded in the eye. An ophthalmologist, a health care provider who specializes in the surgical and nonsurgical evaluation and treatment of eye conditions, should remove the embedded object as soon as possible. Educational objective: When a foreign body becomes accidentally embedded in the eye, both eyes should be shielded to prevent eye movement and additional injury. The nurse should immediately refer the client to an ophthalmologist for further evaluation and treatment. Copyright © UWorld. All rights reserved.

Safety/Infection Control Postpartum Labor/Delivery Medication administration Test Id: 72315460 QId: 32014 (921666) 7 of 10 A A A The nurse is administering a pink pill to a hospitalized medical-surgical client. The alert, oriented client says, "This is a pill I haven't seen before." What follow-up action should the nurse take next? 1. Check the health care provider's prescription in the medical record [69%] 2. Explain that the health care provider has prescribed the medication [1%] 3. Look up the medication in the pharmacology reference [9%] 4. Teach the client about the purpose of the medication [18%] Correct Answered correctly Explanation: Safe medication administration is conducted according to 6 rights: Right client using 2 identifiers Right medication Right dose Right route Right time Right documentation When a mentally competent client questions a drug administration, the safest option is to first check the prescription to verify the 6 rights of medication administration (Option 1). If an error is ruled out (eg, different brand, new order) the nurse should follow up with appropriate teaching. (Option 2) The nurse must first verify all aspects of proper medication administration. If they are correct, the nurse should provide appropriate teaching on why the health care provider prescribed the medication. Explaining that the nurse is just following orders is rarely the correct answer. (Option 3) A pharmacology reference can verify information about the medication but will not confirm that the client is the correct recipient. Acceptable identifiers include first and last name, medical record number, and birth date. (Option 4) The nurse can teach the client about the purpose of the medication after the 6 rights have been verified. Educational objective: When a competent client questions a new medication, the nurse should first verify the 6 rights of safe medication administration: right client, medication, dose, route, time, and documentation. If safe administration has been confirmed, the nurse should then provide appropriate teaching to the client.

Safety/Infection Control Postpartum Labor/Delivery Crutch walking The home health nurse visits a client who is rehabilitating after a tibial fracture. Which interventions are appropriate to include in the client's teaching plan to promote safety in the home when using crutches? Select all that apply. 1. Keep a clear path to the bathroom 2. Look down at the feet when walking 3. Remove scatter rugs from floors 4. Use a small backpack/shoulder bag to hold personal items 5. Wear rubber-soled shoes, preferably without laces Incorrect Correct answer 1,3,4,5 Answered correctly Explanation: Interventions to promote safety when using crutches in the home include the following: Keep the environment free of clutter and remove scatter rugs to reduce fall risk (Options 1 and 3) Look forward, not down at the feet, when walking to maintain an upright position, which will help prevent muscle and joint strain, maintain balance, and reduce fall risk (Option 2) Use a small backpack, fanny pack, or shoulder bag to hold small personal items (eg, eyeglasses, cell phone), which will keep hands free when walking (Option 4) Wear rubber- or non-skid-soled slippers or shoes without laces to reduce fall risk (Option 5) Rest crutches upside down on the axilla crutch pads when not in use to prevent them from falling and becoming a trip hazard Keep crutch rubber tips dry. Replace them if worn to prevent slipping. Educational objective: Interventions to promote safety and reduce the risk of falling when using axillary crutches in the home include looking forward when walking, maintaining a clutter-free environment, resting crutches upside down on the axilla pads when not in use, using a small bag to hold personal items, wearing sturdy rubber-soled shoes, and keeping crutches in good repair.

Safety/Infection Control Postpartum Labor/Delivery Room Assignment For Client With Cellulitis The charge nurse must assign a semi-private room to a client with diabetes mellitus admitted for IV antibiotic therapy to treat leg cellulitis. Which of the 4 room assignments is the best option for this client? 1. Room 1: Client 1 day postoperative laparoscopic cholecystectomy who is awaiting discharge [31%] 2. Room 2: Client with dementia and urinary incontinence wearing an external urine collection device [26%] 3. Room 3: Client with history of splenectomy 15 years ago, now admitted for pulmonary embolism [19%] 4. Room 4: Client with lupus nephritis who is prescribed treatment with azathioprine [21%] Incorrect Correct answer 2 Explanation: Cellulitis is a common skin bacterial infection that is usually treated with IV antibiotics in clients with diabetes mellitus. Room 2 is the best assignment option for this client with cellulitis. The client with dementia and urinary incontinence who has an external urinary condom catheter is the least susceptible to infection compared to those in rooms 1, 3, and 4. (Option 1) The client who is 1 day postoperative laparoscopic cholecystectomy (surgical procedure with small incisions) is at increased risk for infection. The client with cellulitis should not be placed in room 1. (Option 3) Although this client has pulmonary embolism, the history of prior splenectomy leads to a very high lifelong risk of rapid sepsis. Splenectomy clients need vaccination against encapsulated organisms (eg, pneumococcus, meningococcus, and Haemophilus influenzae type B). Even a low-grade fever should be taken seriously in these clients. The client with cellulitis should not be placed in room 3. (Option 4) Lupus nephritis is a serious renal complication of systemic lupus erythematosus (SLE), an inflammatory autoimmune disease that can lead to end-stage kidney disease. The systemic disease and the immunosuppressant (azathioprine [Imuran]) prescribed to slow its progression increase infection risk. The client with cellulitis should not be placed in room 4. Educational objective: A client with an infection should not be assigned to a semi-private room with a client who had surgery or is immunocompromised and receiving immunosuppressants as these clients are highly susceptible to infection. Post-splenectomy clients are also at lifelong risk for rapid sepsis. .

Skills/Procedures Postpartum Labor/Delivery Medication administration The nurse observes a student nurse administer ear drops to an elderly client to help loosen cerumen. The nurse intervenes when the student performs which action? 1. Instills ear drops at room temperature [1%] 2. Instills ear drops with dropper by occluding the ear canal [69%] 3. Places a cotton ball loosely in outermost auditory canal after the instillation [14%] 4. Pulls pinna up and back and instills drops [14%] Correct Answered correctly Explanation: Otic medications are used to treat infection, soften cerumen for later removal, and facilitate removal of an insect trapped in the ear canal. They are contraindicated in a client with a perforated eardrum. The general procedure for instilling ear drops includes the following steps: Perform hand hygiene and don clean gloves. The ear canal is not sterile, but aseptic technique is used Position the client side-lying with the affected ear up (if not contraindicated). This facilitates administration and prevents drops from leaking out of the ear Warm ear drops to room temperature (ie, use hand or warm water) to help avoid vertigo, dizziness, or nausea as the internal ear is sensitive to temperature extremes (Option 1) Pull the pinna up and back to straighten the ear canal in clients >4 years old and adults. Pull the pinna down and back in clients <3 years old (Option 4) Support hand on the client's head and instill the prescribed number of drops by holding the dropper 1 cm (1/2 in) above the ear canal. This avoids damaging the ear canal with the dropper (Option 2) Apply gentle pressure to the tragus (fleshy part of external ear canal) if it does not cause pain, which facilitates the flow of medication into the ear canal Instruct the client to remain side-lying for at least 2-3 minutes to facilitate medication distribution and prevent leakage Place a cotton ball loosely in the client's outermost ear canal for 15 minutes, only if needed, to absorb excess medication. Perform this with caution and avoid in infants or very young clients as it is a choking hazard (Option 3) Educational objective: To administer otic medications in an adult client, follow these steps: (1) Perform hand hygiene, (2) position the client side-lying with the affected ear up, (3) pull pinna up and back, (4) administer prescribed number of ear drops, (5) instruct the client to remain side-lying for 2-3 minutes, and (6) place cotton ball loosely in the outer ear canal for 15 minutes (if needed).

Block Time Remaining: 00:39:02 TUTOR Test Id: 72490738 QId: 30737 (921666) 36 of 75 A A A When caring for a client with pneumonia, which nursing activities are most appropriate for the registered nurse (RN) to delegate to the licensed practical nurse (LPN) working under RN supervision? Select all that apply. 1. Administering metered-dose inhaled medications 2. Monitoring lung sounds 3. Evaluating use of the incentive spirometer 4. Nasotracheal suctioning to collect a sputum specimen 5. Teaching the importance of fluid intake Omitted Correct answer 1,2,4 Answered correctly 45% Time: 1 seconds Updated: 02/06/2017 Explanation: Scope of practice RN LPN/LVN UAP Clinical assessment Initial client education Discharge education Clinical judgment Initiating blood transfusion Monitoring RN findings Reinforcing education Routine procedures (eg, catheterization) Most medication administrations Ostomy care Tube patency & enteral feeding Specific assessments Activities of daily living Hygiene Linen change Routine, stable vital signs Documenting input/output Positioning LPN = licensed practical nurse; LVN = licensed vocational nurse; RN = registered nurse; UAP = unlicensed assistive personnel. *Limited assessments (eg, lung sounds, bowel sounds, neurovascular checks). The RN can safely delegate certain aspects of nursing care for stable clients to the LPN. The scope of nursing practice for the LPN includes routine procedures, such as administering most medications, completing basic assessment tasks (eg, auscultating lung or bowel sounds after initial assessment by RN), and performing sterile procedures (eg, suctioning, catheterization). The RN can delegate these tasks to the LPN because they do not involve the functions of higher-level assessment, planning, evaluation, or clinical nursing judgment. (Options 3 and 5) The RN is responsible for initial instruction on the importance of fluid intake and for evaluation of proper incentive spirometer use. Educational objective: The RN can safely delegate tasks to the LPN that do not involve the functions of high-level assessment, evaluation, or clinical nursing judgment. Copyright © UWorld. All rights reserved.

Block Time Remaining: 00:39:03 TUTOR Test Id: 72490738 QId: 30738 (921666) 37 of 75 A A A An obese 85-year-old client, who is an avid gardener and eats only home-grown fruits, legumes, and vegetables, is admitted to the hospital with pneumonia after having an upper respiratory tract infection for a week. Which factor puts the client at greatest risk for developing pneumonia? 1. Advanced age [58%] 2. Environmental exposure [25%] 3. Nutritional deficit [7%] 4. Obesity [8%] Omitted Correct answer 1 Answered correctly 58% Time: 1 seconds Updated: 05/05/2017 Explanation: Pneumonia is an inflammatory process in the alveoli and interstitium of the lung usually caused by an infectious or noninfectious agent. Any condition, such as advanced age (>65), that compromises the respiratory system's protective mechanical or immune mechanisms to maintain the sterility of the lower airway can increase the risk for pneumonia. (Options 2, 3, and 4) Working in the garden and being exposed to environmental factors (eg, pollen), eating a vegetarian diet, and obesity do not pose the greatest risks for development of pneumonia. Educational objective: Any condition that compromises the respiratory system's protective mechanical or immune mechanisms, which maintain the sterility of the lower airway, can increase the risk for pneumonia. Copyright © UWorld. All rights reserved.

Block Time Remaining: 00:05:59 TUTOR Test Id: 72419393 QId: 31165 (921666) 73 of 75 A A A The nurse admits an elderly client with an altered level of consciousness and a history of stroke and left-sided weakness. The client is dehydrated from multiple episodes of diarrhea. Which interventions should the nurse implement to keep the client free of injury? Select all that apply. 1. Place a bedside commode on the client's right side 2. Place the client in a room closest to the nursing station 3. Put restraints on the client to decrease the chance of getting up without assistance 4. Raise the bed rails on leaving the room 5. Use a bed alarm to alert staff when the client gets up Omitted Correct answer 1,2,5 Answered correctly 48% Time: 5 seconds Updated: 03/09/2017 Explanation: Several factors increase the risk of falling in this elderly client, including: Unfamiliar surroundings Unsteady gait Cognitive changes Orthostatic hypotension (related to dehydration) Decreased strength and coordination Most falls are unobserved and occur in the client's room. Evidence confirms an association between falling and hurrying to the bathroom due to bowel or urinary urgency and/or frequency. Placing a commode by the right (stronger) side of the bed decreases the number of steps and time needed to get to a toilet (Option 1). It also decreases the chance of tripping on equipment (eg, IV pump, tubing). Placing the client in a room closest to the nursing station allows frequent observation and a faster response time to calls for assistance (Option 2). A bed alarm alerts staff when the client attempts to get out of bed, which allows for prompt response (Option 5). (Option 3) Restraints can increase injury risk in a confused client. The client should be oriented to the call bell and instructed to ask for assistance when using the bedside commode. (Option 4) Clients with impaired mobility and mentation can become entrapped if they attempt to climb over raised bed rails, resulting in possible injury. Educational objective: Most falls are unobserved and occur in the client's room, especially when the client hurries to the bathroom due to bowel or urinary urgency/frequency. Fall risk precautions include placing the client in a room close to the nursing station, placing a commode by the client's stronger side, and using a bed alarm. Copyright © UWorld. All rights reserved.

Block Time Remaining: 00:00:00 TUTOR Test Id: 72458431 QId: 31611 (921666) 1 of 10 A A A The charge nurse is preparing for the admission of an elderly client with delirium and agitation associated with urinary tract infection. To promote client safety, which intervention is most important for the charge nurse to implement? 1. A bed near the nursing station [29%] 2. Four-point leather restraints [0%] 3. Minimizing environmental stimuli [17%] 4. One-on-one supervision from a sitter [52%] Incorrect Correct answer 4 Answered correctly 52% Time: 26 seconds Updated: 02/25/2017 Explanation: Client and staff safety is an ongoing concern when working with clients who are confused and agitated. The least restrictive restraint should be used. One-on-one supervision provided by a trained staff member who stays with the client at all times can promote safety while reducing or eliminating the use of restraints on a client who is confused and agitated. Frequent reassurance, touch, and verbal orientation (regarding name, location, time, and the client's situation) can lessen disruptive behaviors. Placing a large clock and calendar within the client's visibility would also help. (Option 1) Ideally, the client will be placed in a room near the nursing station. However, the client with delirium and agitation will also require ongoing supervision to minimize harm to self or others. (Option 2) Four-point leather restraints are one of the most restrictive restraint options. These are not appropriate as a first-line option for promoting safety. (Option 3) Reducing environmental stimuli is important for an agitated client, but these alone are not most helpful. Educational objective: When caring for clients with behavioral issues that may compromise safety, less restrictive restraint and safety options should be implemented before more restrictive restraint options are used. Copyright © UWorld. All rights reserved.

Block Time Remaining: 00:00:05 TUTOR Test Id: 72419393 QId: 30632 (921666) 17 of 75 A A A The nurse plans to administer 9:00 AM medications via the nasogastric (NG) route to a client with an NG tube. The nurse contacts the primary health care provider (PHCP) to clarify which prescriptions that are contraindicated using this route? Select all that apply. 1. Enteric-coated ibuprofen 200-mg tablet 2. Extra-strength acetaminophen 500-mg tablet 3. Metoprolol extended-release 50-mg tablet 4. Sulfamethoxazole double-strength 800-mg tablet 5. Tamsulosin 0.4-mg slow-release capsule Correct Answered correctly 60% Time: 53 seconds Updated: 12/12/2016 Explanation: Enteric-coated drugs have a barrier coating that dissolves at a slower rate (usually in the small intestine) to protect the stomach from irritant effects. Crushing enteric-coated medications (eg, ibuprofen) disrupts the barrier coating and may cause stomach irritation. In addition, the particles from the coating may clog the NG tube, particularly small-bore NG tubes. Slow-, extended-, or sustained-release drug formulations are designed to dissolve very slowly within a specific time frame. Crushing these medications alters this property and introduces the risk of adverse effects from toxic blood levels due to more rapid drug absorption. Therefore, the nurse should first contact the PHCP for clarification. (Options 2 and 4) Double- and extra-strength drugs such as sulfamethoxazole and acetaminophen may be crushed and administered separately through an NG tube as long as they are not enteric-coated. The nurse should flush the tube with water before and after each drug administration. Educational objective: Crushing an enteric-coated, slow-release, extended-release, or sustained-release drug disrupts its designed time of release and is contraindicated. The nurse should contact the PHCP for an alternate prescription if such a drug is prescribed via NG route. Copyright © UWorld. All rights reserved.

Block Time Remaining: 00:00:05 TUTOR Test Id: 72419393 QId: 30697 (921666) 18 of 75 A A A The nurse is caring for a client with end-stage liver disease who was admitted for bleeding esophageal varices. The bleeding varices were banded successfully, but the client declined having a transjugular intrahepatic portal-systemic shunt (TIPS) procedure and opted for do not resuscitate (DNR) status. Which topic is most important for the nurse to discuss with the client and family at discharge? 1. Complete abstinence from alcohol [9%] 2. Proper use of medications including lactulose [10%] 3. The importance of calling the healthcare provider (HCP) immediately if bleeding recurs [30%] 4. The purpose and use of the DNR bracelet [48%] Correct Answered correctly 48% Time: 60 seconds Updated: 01/16/2017 Explanation: A client with end-stage liver disease is at high risk for life-threatening events such as bleeding esophageal varices and hepatic encephalopathy. This client continues to be at risk for bleeding varices due to the declined TIPS procedure, which could have prevented further esophageal varices by treating the portal hypertension. This client who is DNR in the hospital should be discharged with a DNR bracelet or an active Physician Orders for Life-Sustaining Treatment (POLST) form in the community setting. This should be done to ensure that the client's wishes for emergency care will be carried out by first responders. (Option 1) Abstinence from alcohol will help delay the progression of end-stage liver disease and its complications. However, this is not always realistic for a client with long-term alcohol addiction. In addition, this client with end-stage liver disease who has chosen to be DNR may also choose to continue drinking if this is deemed important to quality of life. Even though the nurse may not approve of this choice, the client is the one who ultimately makes personal lifestyle and health management decisions. (Option 2) Lactulose and other medications are necessary for managing end-stage liver disease. However, this topic is less important than emergency response and advance care planning issues, particularly in a client with a new DNR order and recent history of bleeding esophageal varices. (Option 3) Although the client and family should know what to do if bleeding recurs, it would be more appropriate to call 911 than the HCP in this emergency situation. In addition, this topic is not as important as the discussion on DNR bracelet use which already covers emergency care for any type of situation. Educational objective: Discharge planning and teaching for the client with a new DNR order should include a method of ensuring that the DNR order will be carried out in the community and home. DNR bracelets and POLST forms are community-based systems that provide emergency responders with the legal documentation needed to withhold resuscitation. Copyright © UWorld. All rights reserved.

Block Time Remaining: 00:00:05 TUTOR Test Id: 72419393 QId: 30327 (921666) 15 of 75 A A A An experienced nurse precepts a graduate nurse in the intensive care unit while caring for a client with a right subclavian triple-lumen central venous catheter (CVC). Which statement by the graduate nurse indicates understanding of the CVC? 1. "All 3 lumens come together, so all drugs infused through the CVC must be compatible." [14%] 2. "It is used to provide enteral nutrition to the client who cannot eat." [5%] 3. "Sterile gloves must be worn when administering drugs through the CVC." [11%] 4. "The lumen hub should be cleaned thoroughly with antiseptic prior to drug administration." [67%] Incorrect Correct answer 4 Answered correctly 67% Time: 56 seconds Updated: 05/30/2017 Explanation: A central line or central venous catheter (CVC) is inserted by the health care provider in a "central" vein (eg, subclavian, internal jugular, femoral) and is used to administer fluids, medications, and parenteral nutrition and for hemodynamic monitoring. Proper hand hygiene should be performed when caring for a CVC to prevent infection, and nonsterile gloves should be worn to protect the nurse from blood or body fluids at the port site as one or more lumens are often used to draw blood (Option 3). The Centers for Disease Control and Prevention recommend that catheter hubs always be handled aseptically to prevent catheter-associated infections. The hubs should be disinfected with a hospital-approved antiseptic (eg, 70% alcohol sterile pads; > 0.5% chlorhexidine with alcohol; 10% povidone-iodine). Always allow the antiseptic to dry before using the hub/port (Option 4). (Option 1) CVCs may have multiple lumens. These are used to administer incompatible drugs simultaneously, for blood draws, and for hemodynamic monitoring. (Option 2) Enteral nutrition is given only through the GI tract (orally or through a feeding tube). Parenteral nutrition is administered through the IV route via a central vein. Educational objective: A central venous catheter is used to administer fluids, for simultaneous infusion of incompatible drugs, for parenteral nutrition, and for hemodynamic monitoring. The nurse should always handle the lumen ports and hubs aseptically with facility-approved antiseptics to prevent catheter-associated infections. Copyright © UWorld. All rights reserved.

Block Time Remaining: 00:00:05 TUTOR Test Id: 72419393 QId: 31180 (921666) 16 of 75 A A A A client is being discharged after having a coronary artery bypass grafting (CABG) x 5. The client asks questions about the care of chest and leg incisions. Which instructions should the registered nurse include? Select all that apply. 1. Report any itching, tingling, or numbness around your incisions 2. Report any redness, swelling, warmth, or drainage from your incisions 3. Soak incisions in the tub once a week then clean with hydrogen peroxide and apply lotion 4. Wash incisions daily with soap and water in the shower and gently pat them dry 5. Wear an elastic compression hose on your legs and elevate them while sitting Incorrect Correct answer 2,4,5 Answered correctly 30% Time: 94 seconds Updated: 01/01/2017 Explanation: Incisions may take 4-6 weeks to heal. The nurse should instruct clients on how to care for their incisions; these instructions are as follows: Wash incisions daily with soap and water in the shower. Gently pat dry (Option 4). Itching, tingling, and numbness around the incisions may be present for several weeks due to damage to the local nerves (Option 1). Tub baths should be avoided due to risk of introducing infection (Option 3). Do not apply powders or lotions on incisions as these trap the bacteria at the incision (Option 3). Report any redness, swelling, and increase in drainage or if the incision has opened (Option 2). Wear a supportive elastic hose on the legs. Elevate legs when sitting to decrease swelling (Option 5). Educational objective: The nurse should instruct the client with chest and leg incisions from CABG to wash them daily with soap and water in the shower. In addition, the client must be instructed not to apply any powders or lotions to the incisions, to report any redness, swelling or increase in drainage, and to wear an elastic compression hose on the legs. Copyright © UWorld. All rights reserved.

Block Time Remaining: 00:00:05 TUTOR Test Id: 72419393 QId: 30477 (921666) 13 of 75 A A A A client with a tracheostomy is alert and oriented and able to tolerate oral intake. Which action would be appropriate to reduce the client's risk of aspiration pneumonia? 1. Fully inflate the cuff before feeding [29%] 2. Have the client sit in an upright position with the neck hyperextended [24%] 3. Partially or fully deflate the cuff [21%] 4. Provide a modified diet of pureed foods [24%] Incorrect Correct answer 3 Answered correctly 21% Time: 40 seconds Updated: 06/02/2017 Explanation: A tracheostomy tube with inflated cuff is used in clients who are at risk for aspiration (eg, who are unconscious or on mechanical ventilation). However, an inflated cuff is uncomfortable for clients who are awake because it is difficult to swallow or talk. The cuff is deflated when the client is improving, is determined not to be at risk of aspiration, and is awake. Before the cuff is deflated, the client is asked to cough (if possible) to expectorate the oropharyngeal secretions that have built up above the inflated cuff. In addition, suction is applied through the tracheostomy tube and then the mouth; the cuff is then deflated. Additional interventions to decrease the risk of aspiration include the following: Having the client sit upright with the chin flexed slightly toward the chest Monitoring for a wet or garbled-sounding voice Monitoring for signs of fever (Option 1) Inflating the cuff makes it difficult for a client who is awake to swallow and talk. In addition, more secretions can accumulate above the inflated cuff due to difficulty swallowing. The inflated cuff may not provide a 100% seal and the accumulated secretions can slide through it, causing aspiration. For these reasons, the deflated cuff is beneficial in awake clients with no risk of aspiration. (Option 2) Having the client sit upright will help reduce the risk of aspiration. However, the chin should be flexed toward the chest; hyperextension of the neck increases the risk of aspiration. (Option 4) There is no reason to give pureed foods just because the client has a tracheostomy. The client's diet should be determined by a swallowing evaluation. Educational objective: The risk of aspiration in a conscious, alert, and oriented client with a tracheostomy can be reduced by partially or fully deflating the tracheostomy cuff, having the client in an upright position, monitoring for a wet cough or voice quality, and monitoring vital signs. Copyright © UWorld. All rights reserved.

Block Time Remaining: 00:00:05 TUTOR Test Id: 72419393 QId: 31268 (921666) 14 of 75 A A A A young Spanish-speaking client is experiencing a spontaneous abortion (miscarriage). Which illustrates the best use of an interpreter to explain the situation to the client? Select all that apply. 1. Ask the client to nod so the nurse can confirm the client understands the situation 2. Attempt to use a female interpreter to avoid gender sensitivity 3. Make good eye contact with the client (rather than the interpreter) when speaking 4. Preferably use a personal friend or relative to facilitate client privacy under HIPAA 5. Teach about one intervention at a time and in the order it will occur Incorrect Correct answer 2,3,5 Answered correctly 48% Time: 71 seconds Updated: 01/03/2017 Explanation: Clients from many cultures will be more responsive if the interpreter is the same gender, especially when the condition is highly personal or sensitive (Option 2). The nurse should maintain good eye contact when communicating with the client. The interpreter should translate the client's words literally. Communication is with the client, not the interpreter. The nurse should use basic English rather than medical terms, speak slowly, and pause after 1-2 sentences to allow for translation (Option 3). Providing simple instructions about upcoming actions in the order they will occur will be easier for the client to understand. For example, the nurse can indicate that there will be surgery and then a follow-up visit as opposed to, "You'll follow up with the health care provider after your procedure" (Option 5). (Option 1) The nurse should obtain feedback to be certain that the client understands. This feedback should extend beyond nodding as some people nod to indicate that they are listening or nod in agreement to "save face" even though they do not understand. It is better to use a tactic such as having the client repeat back information (which is then translated into English). (Option 4) Using a fee-based agency or language line is preferred if an appropriate bilingual employee is not available. The client may not want the friend/relative to know about this personal situation, or the person may not be able to adequately translate medical concepts and/or understand client rights. Educational objective: When an interpreter is needed, the nurse should attempt to use a trained, proficient, same-sex individual rather than a family member or personal friend. The nurse should speak slowly and directly to the client, not the interpreter; provide information in the sequence it will occur; and obtain feedback of comprehension beyond merely nodding. Copyright © UWorld. All rights reserved.

Block Time Remaining: 00:00:05 TUTOR Test Id: 72419393 QId: 31446 (921666) 8 of 75 A A A The nurse is caring for a client who develops Clostridium difficile colitis after multiple days of antibiotic therapy. Which infection control measures are appropriate to implement? Select all that apply. 1. Disinfect surfaces with diluted bleach solution 2. Hand hygiene with alcohol-based hand rub 3. Wear a face mask 4. Wear a protective gown 5. Wear nonsterile gloves Incorrect Correct answer 1,4,5 Answered correctly 48% Time: 63 seconds Updated: 05/14/2017 Explanation: C difficile poses a unique hazard in health care settings. This infection of the colon may develop/spread through contact with the organism or after prolonged antibiotic therapy alters normal bowel flora, allowing for C difficile overgrowth. Clients with C difficile infection should be placed on strict contact precautions in private rooms. These precautions require staff to wear protective gowns and gloves when entering the client's room (Options 4 and 5). Hand hygiene using soap and water is the only effective method for removing C difficile spores (Option 2). In addition, alcohol is not an effective agent for killing C difficile spores; therefore, a diluted bleach solution must be used to disinfect contaminated equipment and surfaces (Option 1). (Option 3) Contact precautions require the caregiver to wear a gown and gloves. A face mask must be worn as personal protective equipment if an organism is spread via droplets. However, it is not required to prevent the spread of a contact-transmissible infection. The nurse should not wear a mask solely to avoid the unpleasant odor associated with C difficile diarrhea as this may be offensive and embarrassing to the client. Educational objective: C difficile infection requires strict contact precautions, including wearing a gown and gloves at all times. Alcohol cannot kill C difficile spores, so caregivers must use soap and water in place of alcohol-based hand sanitizers. Contaminated surfaces and equipment should be disinfected using a diluted bleach solution. Copyright © UWorld. All rights reserved.

Block Time Remaining: 00:00:05 TUTOR Test Id: 72419393 QId: 32849 (921666) 9 of 75 A A A A graduate nurse is caring for a client with a triple-lumen peripherally inserted central catheter in the right arm. Which actions by the graduate nurse indicate that more education is needed? Select all that apply. 1. Flushing the line before and after each medication administration 2. Pausing the parenteral nutrition prior to drawing blood from a different port 3. Reinforcing a torn peripherally inserted central catheter line dressing with tape 4. Scrubbing the port with alcohol for 5 seconds before use 5. Taking the client's blood pressure in the left arm Incorrect Correct answer 3,4 Answered correctly 27% Time: 66 seconds Updated: 04/08/2017 Explanation: A peripherally inserted central catheter (PICC) is a venous access device that is inserted via the cephalic or basilic vein and terminates in the superior vena cava. It is indicated for administration of noxious medications (eg, parenteral nutrition, chemotherapy), for long-term IV therapy, or in clients with poor venous access. Proper care and aseptic technique are important to maintain lumen patency and eliminate the risk of life-threatening central line-associated bloodstream infection (CLABSI). The nurse should inspect the insertion site for signs of infection (redness, drainage) and dressing integrity. Routine care includes sterile dressing changes every 48 hours with a gauze dressing or 7 days with a transparent semipermeable dressing (biopatch) as well as immediately if dressing is loose/torn, soiled, or damp. The line should be flushed before and after medication administration and per facility protocol (Option 1). Blood pressure and venipuncture should not be performed on the affected arm as compression of the vein can alter its integrity (Option 5). All infusing medications (except vasopressors) must be paused before drawing blood from the PICC to prevent false interpretation of the client's serum levels (Option 2). (Option 3) Dressings that no longer occlude the insertion site must be changed immediately. Loose corners may be temporarily reinforced with tape. (Option 4) The nurse should "scrub the hub" with alcohol or chlorhexidine/alcohol for 10-15 seconds. This should be done before flushing, drawing blood, or administering medication. Educational objective: Peripherally inserted central catheter lines provide central venous access for clients who require long-term medication administration or infusion of noxious substances. Maintaining the line integrity with aseptic technique and routine care (sterile dressing changes, flushing the line, blood pressures/venipunctures on unaffected arm) is important for continued use and prevention of central line-associated bloodstream infections. Copyright © UWorld. All rights reserved.

Block Time Remaining: 00:00:05 TUTOR Test Id: 72419393 QId: 31017 (921666) 19 of 75 A A A An unconscious client is brought to the emergency department by the paramedics after being hit by a car. An emergency craniotomy is required. The client has no identification. What action should be taken next? 1. Contact the national database to see if the client has a healthcare proxy [1%] 2. Contact the police to help identify the client and locate family members [2%] 3. Obtain a court order for the client's surgical procedure [3%] 4. Transport the client to the operating room under implied consent [92%] Correct Answered correctly 92% Time: 28 seconds Updated: 12/22/2016 Explanation: Implied consent in emergency situations includes the following criteria: There is an emergency Treatment is required to protect the client's health It is impractical to obtain consent It is believed that the client would want treatment if able to consent In this case, it would be assumed that the client would want life-saving surgery; the health care provider should proceed. (Option 1) This client's name is not known and there is no national database of healthcare proxy names/power of attorney. (Option 2) This should also be done but results may not be obtained in a timely manner. The client needs immediate surgery and this should proceed with the client as a "John Doe" (placeholder name) in the meantime. (Option 3) This would cause considerable delay. Court orders are used for protective custody to take control of the care of a minor when the adult parent is refusing necessary life-saving care. Educational objective: Emergency life-saving care can proceed for a client who cannot give consent if it is essential and believed that the client would want treatment if able to consent. Care is rendered under the principle of implied consent. Copyright © UWorld. All rights reserved.

Block Time Remaining: 00:00:05 TUTOR Test Id: 72419393 QId: 33130 (921666) 20 of 75 A A A Unlicensed assistive personnel report 4 situations to the registered nurse. Which situation warrants the nurse's intervention first? 1. Room 1: Client on a 24-hour urine collection had a specimen discarded by mistake [14%] 2. Room 2: Client and family request clergy to administer last rites [31%] 3. Room 3: Puncture-resistant sharps disposal container on the wall is full [33%] 4. Room 4: Client with diabetes mellitus has an 8 AM fingerstick glucose of 80 mg/dL (4.4 mmol/L) [20%] Incorrect Correct answer 3 Answered correctly 33% Time: 61 seconds Updated: 04/03/2017 Explanation: Health care workers are required to abide by Occupational Safety and Health Administration standards and regulations to reduce work-related injuries (eg, sharps) and exposure to bloodborne pathogens (eg, HIV, hepatitis B and C). A sharps disposal container should not be overfilled and should be replaced on a regular basis to reduce the risk for a needle stick during disposal. (Option 1) If any urine is discarded by accident during a 24-hour collection test, the procedure must be restarted. A new container will need to be labeled with the appropriate times and date, but immediate intervention is not required. (Option 2) The nurse will arrange for a visit from clergy to administer the last rites (Sacrament of the Sick), a religious ceremony for Roman Catholic clients who are extremely or terminally ill. Although the situation requires prompt intervention, it does not involve a safety hazard. (Option 4) A fingerstick glucose of 80 mg/dL (4.4 mmol/L) is normal (70-110 mg/dL [3.9-6.1 mmol/L]) and requires no intervention unless the client received insulin and refuses or is unable to eat. Educational objective: Prevention of injury and safety in the workplace should be a priority when the nurse is delegating, planning, or providing nursing care. Copyright © UWorld. All rights reserved.

Block Time Remaining: 00:00:16 TUTOR Test Id: 72419393 QId: 31356 (921666) 21 of 75 A A A Which positions are correct when caring for clients undergoing therapeutic procedures? Select all that apply. 1. High-Fowler's for a paracentesis in cirrhosis 2. Left side after liver biopsy in hepatitis 3. Semi-Fowler's after a cardiac catheterization 4. Sims for soap-suds enema administration 5. Supine position after lumbar puncture Omitted Correct answer 1,4,5 Answered correctly 18% Time: 11 seconds Updated: 05/27/2017 Explanation: A therapeutic/comfort paracentesis in cirrhosis requires the client to be upright so that the fluid is in the lower abdomen where the trocar will be placed for draining it (Option 1). When in the Sims, or left lateral recumbent, position, the client is lying on the left side with the left leg straight and the right hip and knee flexed. It is a common position for enema administration (Option 4). Before a lumbar puncture (spinal tap), the client is placed in the fetal position or bent over a table to separate the vertebrae. Afterwards, the client is placed flat in bed in the prone or supine position for 4 to 8 hours. This will minimize the risk for a "spinal" headache from the loss of cerebrospinal fluid (Option 5). (Option 2) Before a liver biopsy, the client is placed supine with the right arm above the head. The client is instructed to exhale fully and to not breathe when the needle is inserted. The risk after a liver biopsy is for internal bleeding as liver pathology affects coagulation factors. After the biopsy, the client is placed supine on the right side for 12-14 hours so that the heavy liver falls down on itself and provides internal direct pressure to minimize bleeding. (Option 3) The client is laid flat for hours after a percutaneous coronary intervention (PCI) to prevent pressure at the insertion site of a major vessel so that there is no hemorrhage or hematoma. Educational objective: Use Sims position for enema administration, sitting upright before a paracentesis, and supine after a lumbar puncture. Place a client on right side after a liver biopsy, and keep the client supine after a PCI. Copyright © UWorld. All rights reserved.

Block Time Remaining: 00:00:16 TUTOR Test Id: 72419393 QId: 30105 (921666) 26 of 75 A A A A client is being admitted to the health care facility with a new diagnosis of Clostridium difficile colitis. Which elements of infectious disease precautions are necessary when providing routine care for this client? Select all that apply. 1. Alcohol-based sanitizers for hand cleaning 2. Client in single-room (private) isolation 3. Nurse using N95 respirator 4. Nurse using sterile gloves 5. Nurse using surgical mask 6. Nurse wearing disposable gown Incorrect Correct answer 2,6 Answered correctly 42% Time: 33 seconds Updated: 05/19/2017 Explanation: Clostridium difficile requires contact precautions under the guidelines published by the Centers for Disease Control and Prevention. Place the client in single-room isolation (preferred) or cohort with other C difficile-infected clients All surfaces within 3 feet of the bed are considered contaminated Personal protective equipment (gown and gloves) must be discarded before leaving the room Hand hygiene must be performed with soap and water Alcohol-based hand sanitizers do not kill C difficile spores (Option 1) Dedicated medical equipment (stethoscope, blood pressure cuff) should remain in the room (Option 3) For client care involving airborne precautions, a class N95 or higher grade respirator must be used in lieu of a surgical mask to avoid potential exposure to aerosolized particles. Surgical masks are rated for barrier protection for droplet splashing and filtration of large respiratory particles only. (Option 4) Regular clean gloves (not sterile) are needed. (Option 5) Surgical masks are rated for barrier protection from droplet-based disease transmission and provide only filtration for large respiratory particles. Masks are required during any activity with the possibility of fluids splashing (suctioning, wound care). Educational objective: Clostridium difficile requires contact-based precautions - private isolation room preferred, disposable gown, and nitrile gloves. C difficile requires the use of soap and water, not alcohol-based hand sanitizers, to prevent transmission. Copyright © UWorld. All rights reserved.

Block Time Remaining: 00:00:00 TUTOR Test Id: 72490738 QId: 31294 (921666) 2 of 75 A A A The nurse responds to the call light of a client with chronic obstructive pulmonary disease (COPD) who says, "I can't breathe." The client seems to be having difficulty breathing and is nervous and tremulous. Vital signs are stable, oxygen saturation is 92% on 2 L, and there are clear breath sounds bilaterally. Which intervention would be most appropriate at this time? 1. Administer albuterol nebulizer [16%] 2. Assist the client in identifying the trigger and ways to avoid it [6%] 3. Coach the client through controlled breathing exercises [73%] 4. Continue to monitor oxygen saturation [3%] Correct Answered correctly 73% Time: 64 seconds Updated: 01/05/2017 Explanation: Anxiety is an emotional reaction to a perceived threat. For the client with COPD, the fear of having difficulty breathing can actually trigger difficulty breathing, which worsens as the client's anxiety increases. This client is stable, with no obvious cause of shortness of breath. The nurse should intervene by calmly coaching the client through breathing exercises, which will promote relaxation and help alleviate the anxiety that is causing the client to feel short of breath. (Option 1) The client's lung sounds are clear bilaterally and so albuterol, a bronchodilator used for wheezing, will not be helpful. Its action as an adrenergic agonist may cause tachycardia and tremulousness and actually worsen the client's anxiety. (Option 2) Trigger avoidance and problem solving are appropriate strategies for long-term control of anxiety and shortness of breath. However, these are not appropriate at this time as the client has acute symptoms that need to be controlled. (Option 4) This client has normal oxygen saturation. Constant monitoring is not likely to alleviate the symptoms unless the client is reassured by this knowledge. However, the client's anxiety may actually be worsened by worrying about the saturation results and the alarms that are likely to be triggered by monitoring. Educational objective: Anxiety is common in clients with COPD and can contribute to difficulty breathing. In the client with acute shortness of breath and normal assessment findings, appropriate interventions are controlled breathing and relaxation. Copyright © UWorld. All rights reserved.

Block Time Remaining: 00:00:16 TUTOR Test Id: 72490738 QId: 31195 (921666) 3 of 75 A A A Exhibit A client with pneumonia is transferred from the medical unit to the intensive care unit due to sepsis and worsening respiratory failure. Based on the nurse's progress note, which assessment data are most important for the nurse to report to the health care provider (HCP)? Click on the exhibit button for additional information. 1. Cough with mucus production [1%] 2. Refractory hypoxemia [82%] 3. Scattered rhonchi and crackles [9%] 4. Temperature 101 F (38.3 C) [6%] Omitted Correct answer 2 Answered correctly 82% Time: 16 seconds Updated: 03/04/2017 Explanation: Refractory hypoxemia is the hallmark of acute respiratory distress syndrome (ARDS), a progressive form of acute respiratory failure that has a high mortality rate. It can develop following a pulmonary insult (eg, aspiration, pneumonia, toxic inhalation) or nonpulmonary insult (eg, sepsis, multiple blood transfusions, trauma) to the lung. The insult triggers a massive inflammatory response that causes the lung tissue to release inflammatory mediators (leukotrienes, proteases) that cause damage to the alveolar-capillary (A-C) membrane. As a result of the damage, the A-C membrane becomes more permeable, and intravascular fluid then leaks into the alveolar space, resulting in a noncardiogenic pulmonary edema. The lungs become stiff and noncompliant, which makes ventilation and oxygenation less than optimal and results in increased work of breathing, tachypnea and alkalosis, atelectasis, and refractory hypoxemia. Profound hypoxemia despite high concentrations of oxygen is a key sign of ARDS and is the most important assessment finding to report to the HCP. (Options 1 and 3) Cough with mucus production and scattered rhonchi and crackles are expected findings in a client with pneumonia. (Option 4) Temperature is an expected finding in a client with pneumonia who is receiving antibiotic therapy. The white blood cell count can still be elevated after 2 days of antibiotic therapy. Educational objective: Refractory hypoxemia is the inability to improve oxygenation with increases in oxygen concentration. It is the hallmark of ARDS, a progressive form of acute respiratory failure that has a high mortality rate. Copyright © UWorld. All rights reserved.

Block Time Remaining: 00:00:17 TUTOR Test Id: 72419393 QId: 30595 (921666) 27 of 75 A A A A comatose client in the intensive care unit has an indwelling urinary catheter. Which action(s) should the nurse implement to reduce the incidence of catheter-associated urinary tract infections (UTIs)? Select all that apply. 1. Cleanse periurethral area with antiseptics every shift 2. Ensure each client has a separate container to empty collection bag 3. Keep catheter bag below the level of the bladder 4. Routinely irrigate the catheter with antimicrobial solution 5. Use sterile technique when collecting a urine specimen Omitted Correct answer 2,3,5 Answered correctly 27% Time: 2 seconds Updated: 01/16/2017 Explanation: Health care catheter-associated UTIs are prevalent among hospitalized clients with indwelling urinary catheters. Steps to prevent infections in clients with urinary catheters include the following: Wash hands thoroughly and regularly Perform routine perineal hygiene with soap and water each shift and after bowel movements Keep drainage system off the floor or contaminated surfaces Keep the catheter bag below the level of the bladder Ensure each client has a separate, clean container to empty collection bag and measure urine Use sterile technique when collecting a urine specimen Facilitate drainage of urine from tube to bag to prevent pooling of urine in the tube or backflow into the bladder Avoid prolonged kinking, clamping, or obstruction of the catheter tubing Encourage oral fluid intake in clients who are awake and if not contraindicated Secure the catheter in accordance with hospital policy (tape or Velcro device) Inspect the catheter and tubing for integrity, secure connections, and possible kinks (Option 1) Perineal hygiene is performed using soap and water only every shift and as needed. Routine use of antiseptic cleansers is not shown to prevent infection and may lead to the development of drug-resistant bacteria. (Option 4) Routine irrigation with antimicrobial solution or systemic administration of antimicrobials is not recommended for routine catheter care and infection prevention. Educational objective: Routine catheter care to prevent health care catheter-associated UTIs includes routine hand hygiene, cleansing the perineal area with soap and water routinely, keeping the catheter bag below the bladder and off the ground, keeping the catheter and tubing free of kinks and facilitating urine into the bag, and using sterile technique when collecting urine specimens. Copyright © UWorld. All rights reserved.

Block Time Remaining: 00:00:17 TUTOR Test Id: 72419393 QId: 31936 (921666) 28 of 75 A A A The nurse is teaching a client of American Indian heritage how to self-administer insulin. As the nurse describes the necessary steps in the injection process, the client continuously avoids eye contact and occasionally turns away from the nurse. Which action is most appropriate for the nurse to take in this situation? 1. Continue teaching the client and verify understanding by return demonstration [90%] 2. Discuss how important it is for the client to pay attention during the teaching [2%] 3. Maintain eye contact during the teaching by following the client's movements [2%] 4. Provide written instructions and a private place for the client to learn independently [5%] Correct Answered correctly 90% Time: 44 seconds Updated: 12/11/2016 Explanation: Communication with individuals of various cultures may be difficult for the nurse at times due to cultural language differences (ie, verbal and nonverbal communication styles including the use of silence). The mainstream American and European cultures value direct eye contact, believing that it is a sign of attention and trustworthiness. People of American Indian and Asian cultures view direct eye contact as rude and disrespectful and will likely move the eyes away, not allowing the nurse to maintain eye contact. If the client avoids eye contact during a teaching episode, the most appropriate action is to continue with the instruction and verify understanding by return demonstration. (Option 2) Lecturing the client about the importance of listening to the instructions for insulin self-injection would most likely be interpreted as degrading and disrespectful. (Option 3) In the American Indian culture, it is disrespectful to maintain eye contact during a conversation. (Option 4) A client learning the process of self-administration of insulin requires guidance and evaluation from the registered nurse before, during, and after the teaching session. The client should not be sent to a quiet place to learn the procedure independently. Educational objective: Individuals of American Indian and Asian cultures view direct eye contact as rude and disrespectful and will likely move the eyes away during conversations in an attempt to prevent it. The nurse demonstrates culturally competent care by respecting and accepting this cultural communication pattern. Copyright © UWorld. All rights reserved.

Block Time Remaining: 00:00:17 TUTOR Test Id: 72419393 QId: 34809 (921666) 29 of 75 A A A Exhibit A client with type 1 diabetes has a prescription for 20 units of neutral protamine Hagedorn (NPH) insulin daily at 7:30 AM and regular insulin before meals, based on a sliding scale. At 7:00 AM, the client's blood glucose level was 220 mg/dL (12.2 mmol/L). What action should the nurse take? Click on the exhibit button for additional information. 1. Administer 20 units of NPH insulin now and then 6 units of regular insulin after the morning meal [8%] 2. Administer 26 units of insulin: 20 units of NPH insulin and 6 units of regular insulin in 2 separate injections [16%] 3. Administer 26 units of insulin: 20 units of NPH mixed with 6 units of regular insulin in the same syringe, drawing up the NPH into the syringe first [13%] 4. Administer 26 units of insulin: 20 units of NPH mixed with 6 units of regular insulin in the same syringe, drawing up the regular insulin first [61%] Incorrect Correct answer 4 Answered correctly 61% Time: 236 seconds Updated: 04/03/2017 Explanation: Long-acting insulins, with the exception of insulin glargine, or intermediate-acting insulins (neutral protamine Hagedorn [NPH]) can be safely mixed with short- (regular) and rapid-acting (lispro, aspart) insulins in a single syringe (Option 4). Six units of regular insulin are needed to address the client's blood glucose reading of 220 mg/dL (12.21 mmol/L) in addition to the scheduled 20 units of NPH insulin. In preparing the mixed dose, first inject the NPH insulin vial with 20 units of air without inverting the vial or passing the needle into the solution. Then, inject 6 units of air into the regular insulin vial and withdraw the dose, leaving no air bubble in the syringe. The NPH should then be drawn into the syringe, totaling 26 units. Any overdraw of NPH into the syringe will necessitate wasting the total quantity of insulin in the syringe. (Option 1) The 2 insulins may be safely given together before the meal because regular insulin has a rapid onset of action, whereas the NPH has a slower onset but longer duration. (Option 2) The insulins can be given as 2 separate injections; however, this increases client discomfort and risk of infection. (Option 3) Regular insulin should be drawn up first to avoid contaminating the vial of regular insulin with NPH insulin. Educational objective: Neutral protamine Hagedorn (NPH) insulin and regular insulin may be safely mixed and administered as a single injection. Regular insulin should be drawn into the syringe before intermediate-acting insulin to decrease the risk of cross-contaminating multidose vials (mnemonic - RN: Regular comes before NPH). Copyright © UWorld. All rights reserved.

Block Time Remaining: 00:00:17 TUTOR Test Id: 72419393 QId: 32163 (921666) 30 of 75 A A A A female nurse in the intensive care unit is caring for a client who is intubated and has subclavian central venous access. Which nursing intervention is most important to prevent the spread of infection to this client? 1. Frequent hand hygiene [86%] 2. No artificial nails [4%] 3. Use of chlorhexidine bath wipes [3%] 4. Wearing personal protective equipment [5%] Correct Answered correctly 86% Time: 41 seconds Updated: 01/07/2017 Explanation: Hand hygiene is the single most important factor in preventing the spread of infection and microorganisms. It includes hand washing with soap and water, alcohol-based hand rubs, and surgical hand antisepsis. Hand hygiene is indicated before client contact and donning gloves. It is also indicated after client contact, glove removal, or contact with bodily fluids. Alcohol-based hand rubs should cover the entire surface of the hands, which should then be allowed to dry. Proper hand washing procedure involves wetting the hands; applying soap; scrubbing all hand surfaces, wrists, and beneath the nails for at least 15-30 seconds; rinsing; drying hands with a paper towel; and then using the paper towel to turn off the faucet. (Option 2) Artificial nails or extenders have been found to harbor microorganisms before and after hand washing. They may lead to the spread of hospital-acquired infections and should not be worn, especially in high-risk areas (eg, intensive care unit, operating room). (Option 3) Use of chlorhexidine bathing wipes may be indicated the day before and the morning of a planned surgical procedure or for a client with methicillin-resistant Staphylococcus aureus. (Option 4) Personal protective equipment is appropriate, but it is not as important and does not replace hand hygiene to prevent the spread of infection. Educational objective: Hand hygiene is the single most important nursing intervention to prevent the spread of infection to clients. Copyright © UWorld. All rights reserved.

Block Time Remaining: 00:00:08 TUTOR Test Id: 72797956 QId: 30325 (921666) 8 of 75 A A A A nurse preparing to insert a peripheral IV catheter dons clean gloves, applies a tourniquet to the client's arm, and immediately identifies a site for venipuncture. Place in order the remaining steps that the nurse should take. All options must be used. Your Response/ Incorrect Response Correct Response Cleanse selected site using an antiseptic swab Anchor vein by holding skin taut Insert needle bevel-side up until blood return is observed Advance catheter hub while retracting stylet Remove stylet and attach extension or infusion set Apply a transparent dressing Omitted Correct answer 4,2,5,1,6,3 Answered correctly 71% Time: 61 seconds Updated: 04/01/2017 Explanation: Steps to promote safety and reduce infection risk when initiating IV therapy include the following: Perform hand hygiene using Centers for Disease Control and Prevention guidelines Prepare equipment: Open IV tray, prime tubing with prescribed IV solution for infusion, set IV pump if indicated, prepare tape, and open the over-the-needle catheter (ONC) with safety device Don clean (non-sterile) gloves Identify a possible venipuncture site Apply a tourniquet, ensuring it is tight enough to impede venous return but not tight enough to occlude the artery Select a venipuncture site after palpating the vein. Ask the client to open and close the hand several times to promote vein distension. The tourniquet may need to be released temporarily to restore blood flow and prevent trauma from extended application. Clean the site with chlorhexidine, alcohol, or povidone iodine. Use friction and clean per facility protocol, either back and forth or in a circular motion from insertion site to outward area (clean to dirty direction). Stretch the skin taut using the nondominant hand to stabilize the vein Insert the IV ONC bevel up at a 10- to 30-degree angle and watch for blood backflow as the catheter enters the vein lumen, advancing ¼ inch into the vein to release the stylet. On visualization of blood return, lower the ONC almost parallel with the skin and thread the plastic cannula completely into the vein to the insertion site. Never reinsert the stylet after it is loosened. Use the push-tab safety device to advance the catheter. Apply firm but gentle pressure about 1¼ inch above the catheter tip, release the tourniquet, and retract the stylet from the ONC On removal, guide the protective guard over the stylet for safety and feel for a click as the device is locked. Never try to recap a stylet. Attach a sterile connection of primed IV tubing to the hub of the catheter and stabilize the catheter with tape and dressing using sterile technique. Dispose of the stylet in the sharps container. Educational objective: When initiating IV therapy, the nurse should wash hands thoroughly and don clean gloves, identify the appropriate venipuncture site, apply the tourniquet, select the venipuncture site after palpating the vein, clean the site, stretch skin taut, insert the IV ONC until blood returns, thread the cannula, apply firm pressure above the catheter tip, release the tourniquet, and retract the stylet safely. Copyright © UWorld. All rights reserved.

Block Time Remaining: 00:00:21 TUTOR Test Id: 72797956 QId: 30610 (921666) 11 of 75 A A A A client who is intubated and on mechanical ventilation is receiving continuous enteral tube feedings at 30 mL/hr via a small-bore nasogastric tube. Which actions should the nurse take to prevent aspiration in this client? Select all that apply. 1. Assess abdominal distension every 4 hours 2. Check gastric residual every 12 hours 3. Keep head of the bed at ≥30 degrees 4. Maintain endotracheal cuff pressure 5. Use caution when administering sedatives Incorrect Correct answer 1,3,4,5 Answered correctly 13% Time: 76 seconds Updated: 04/13/2017 Explanation: Clients who are critically ill are at increased risk for aspiration of oropharyngeal secretions and gastric contents, particularly when they are receiving enteral feedings. Nursing interventions to reduce aspiration risk for clients receiving enteral tube feedings include: Assess client for gastrointestinal intolerance to feedings every 4 hours by monitoring gastric residual and assessing for abdominal distension, abdominal pain, bowel movements, and flatus (Option 1) Assess feeding tube placement at regular intervals Keep head of the bed at ≥30 degrees, with 30-45 degrees being optimal to reduce gastroesophageal reflux and aspiration risk unless otherwise indicated (Option 3) Keep endotracheal cuff inflated at appropriate pressure (about 25 cm H20) for intubated clients, as low cuff pressure increases the risk for aspirating oropharyngeal secretions and/or gastric contents (Option 4) Suction any secretions that may have collected above the endotracheal tube before deflating the cuff if deflation is necessary Use caution when giving sedatives and frequently monitor for over-sedation, which can slow gastric emptying and reduce gag reflex (Option 5) Avoid bolus tube feedings for clients at high risk for aspiration (Option 2) Gastric residual should be checked no less than every 4 hours in intubated clients. Educational objective: Precautions to prevent aspiration in the client receiving continuous tube feedings include assessing for gastric intolerance (ie, residual, distension) every 4 hours, keeping the head of the bed at ≥30 degrees, using sedation cautiously, and regular assessment of tube placement. If the client is intubated, the nurse should also keep the endotracheal tube cuff inflated and suction appropriately. Copyright © UWorld. All rights reserved.

Block Time Remaining: 00:00:22 TUTOR Test Id: 72419393 QId: 31117 (921666) 34 of 75 A A A A client in the mental health unit picks up a fire extinguisher and throws it at a nurse standing by the nurses' station. What is the most important intervention by the nurse? 1. Facilitate immediate removal of people from the area [78%] 2. Inform the client that the client cannot act that way [9%] 3. Pull the fire alarm to get additional immediate help [6%] 4. State that the nurse can see the client is upset [4%] Correct Answered correctly 78% Time: 27 seconds Updated: 06/04/2017 Explanation: When a situation is out of control, safety is the primary concern. The nurse and everyone else should leave the area, and security should be called immediately. (Option 2) The situation is no longer diffusible. Quoting authoritative rules will not likely have the desired effect as the client has lost control (and may not be in touch with reality). The nurse's priority is to move out of harm's way. (Option 3) Staff members should call security immediately and/or institute a back-up staff/takedown protocol. The fire alarm will activate a call to a fire department, which is not the type of help needed. However, when security arrives, the "best-trained brain" remains in control and the nurse should direct the actions of the team. (Option 4) When violence (eg, throwing a fire extinguisher) occurs, trying to defuse the situation verbally is no longer the priority. Educational objective: Safety is the priority when violence occurs. People should leave the area and call security immediately. Copyright © UWorld. All rights reserved.

Block Time Remaining: 00:00:22 TUTOR Test Id: 72419393 QId: 30333 (921666) 35 of 75 A A A The nurse is teaching a client with insomnia techniques to improve sleep habits. Which statement by the client indicates a need for further teaching? 1. "I will avoid caffeine with dinner." [3%] 2. "I will avoid naps later in the day." [5%] 3. "I will keep my bedroom cool." [25%] 4. "I will read in bed if I can't fall asleep." [64%] Correct Answered correctly 64% Time: 35 seconds Updated: 04/26/2017 Explanation: Clients with trouble sleeping should be encouraged to keep good sleep habits, which include the following: Reducing stimuli in the bedroom (eg, reading, television). Reading in bed is not recommended. A client wanting to read before bed should do so in a different setting and then go to bed when ready to sleep. Avoiding naps later in the day. Keeping the bedroom slightly cool, quiet, and dark for comfort. Avoiding caffeine, nicotine, and alcohol (stimulants) within 6 hours of sleep. Avoiding exercise or strenuous activity within 6 hours of going to bed to avoid brain stimulation. Avoiding going to bed hungry. Practicing relaxation techniques if stress is causing insomnia. Educational objective: Clients with insomnia should be taught good sleep hygiene - using the bed only for sleep (not reading or watching television), avoiding stimulants and exercise before bedtime, and keeping the room cool and dark. Copyright © UWorld. All rights reserved.

Block Time Remaining: 00:00:22 TUTOR Test Id: 72419393 QId: 31602 (921666) 36 of 75 A A A The charge nurse on the orthopedic unit has 4 semiprivate room beds available. Which room should the nurse assign to a client being transferred from the post anesthesia recovery unit following a total knee replacement? 1. Room 1 - client in skeletal traction following a fracture of the femur, who has erythema at the pin sites [10%] 2. Room 2 - client with cellulitis and osteomyelitis following blunt trauma of the tibia [2%] 3. Room 3 - client with compartment syndrome following a crush injury, who is 1 day post fasciotomy [12%] 4. Room 4 - client with a long leg cast following open reduction of a fractured tibia [75%] Correct Answered correctly 75% Time: 108 seconds Updated: 02/18/2017 Explanation: A client who is postoperative total knee replacement is at risk for infection. No postoperative client should be assigned to a room with a client who has an actual infection or the potential for infection. This client should be assigned to room 4 as the client with the cast has the lowest potential risk for infection (Option 4). (Option 1) This client has erythema at the pin sites; this can be a sign of infection, a complication of skeletal traction. (Option 2) This client has cellulitis, a bacterial infection of the skin, and osteomyelitis, an infection of the bone. (Option 3) This client has a fasciotomy wound, which is usually kept open for several days to relieve the pressure in the myofascial compartment. This client is a potential source of infection and is susceptible to infection as well. Educational objective: A client who is postoperative total knee replacement is at increased risk for infection. This client should not be assigned to a room with a client who has an actual (eg, cellulitis, osteomyelitis) or potential (eg, skeletal traction, fasciotomy) infection. Copyright © UWorld. All rights reserved.

Block Time Remaining: 00:00:22 TUTOR Test Id: 72419393 QId: 30917 (921666) 39 of 75 A A A The nurse removes personal protective equipment (PPE) after completing a wound dressing change for a client in airborne transmission-based precautions. Which PPE should the nurse remove first? 1. Face shield/goggles [15%] 2. Gloves [52%] 3. Gown [25%] 4. Mask/respirator [6%] Correct Answered correctly 52% Time: 10 seconds Updated: 01/19/2017 Explanation: Personal protective equipment (PPE) is necessary when a client is on contamination precautions (eg, droplet, airborne, contact). A gown is not normally required in an airborne precaution room; however, if contamination is probable (eg, dressing change, contact with bodily fluids), a gown is necessary. The proper removal of PPE limits self-contamination. The exact procedure for donning and removing PPE varies with the level of precautions and location of nursing practice. Gloves should be removed first and promptly after use to prevent contamination of other items or noncontaminated materials (Option 2). To remove gloves: Grasp the first glove by its palmar surface and pull off inside out. Next, slide fingers of the ungloved hand under the second glove at the wrist and peel off over the first glove. Discard gloves in an infectious waste container. (Options 1, 3, and 4) Face shield/goggles, gown, and mask/respirator can be removed after gloves, which are considered the most contaminated piece of PPE. Educational objective: The proper removal of personal protective equipment limits self-contamination. Gloves should be removed first and promptly after use to prevent contamination of other items or noncontaminated materials. Copyright © UWorld. All rights reserved.

Block Time Remaining: 00:00:17 TUTOR Test Id: 72419393 QId: 34357 (921666) 31 of 75 A A A The student nurse is preparing to perform a heel stick on a neonate to collect blood for diagnostic testing. Which statement by the student nurse indicates a need for further education? 1. "I can perform the stick on either the medial or lateral side of the outer aspect of the heel." [35%] 2. "Sucrose and a pacifier can help alleviate the infant's pain and stress during the puncture." [15%] 3. "The heel area should be warmed for 3-5 minutes prior to puncture." [17%] 4. "Venipuncture should be reserved only for failed heel sticks because it is more painful." [32%] Correct Answered correctly 32% Time: 41 seconds Updated: 04/05/2017 Explanation: The neonatal heel stick (heel lancing) is used to collect a blood sample to assess capillary glucose and perform newborn screening for inherited disorders (eg, congenital hypothyroidism, phenylketonuria). Proper technique is essential for minimizing discomfort and preventing complications and includes: Select a location on the medial or lateral side of the outer aspect of the heel (Option 1). Avoid the center of the heel to prevent accidental insult to the calcaneus. Puncture should not occur over edematous or infected skin. Warm the heel for several minutes with a warm towel compress or approved single-use instant heat pack to promote vasodilation (Option 3). Cleanse the intended puncture site with alcohol. Sucrose and nonnutritive sucking on a pacifier may reduce procedural pain (Option 2). Use an automatic lancet, which controls the depth of puncture. Lancing the heel too deeply can result in penetration of the calcaneus bone, leading to osteochondritis or osteomyelitis. An acceptable alternate method of blood collection in the neonate is venipuncture (ie, drawing blood from a vein). Venipuncture is considered less painful and often requires fewer punctures to obtain a sample, especially if a larger volume is needed (Option 4). Educational objective: To perform a neonatal heel stick, select a location on the medial or lateral side of the outer aspect of the heel to avoid insult to the calcaneus bone. Provide comfort measures (eg, nonnutritive sucking), warm the selected puncture site to promote vasodilation, cleanse with alcohol, and puncture using an automatic lancet. Copyright © UWorld. All rights reserved.

Block Time Remaining: 00:00:22 TUTOR Test Id: 72419393 QId: 31293 (921666) 33 of 75 A A A An elderly client with end-stage renal disease who has refused dialysis is admitted to a long-term care facility for rehabilitation following hospitalization. The next day, the client becomes agitated and says to the nurse, "I've got to get back home to my things. I have so much to do." Which is the most likely interpretation of this client's behavior? 1. The client has been admitted to the facility without the client's consent [4%] 2. The client is becoming delirious and should be assessed for infection [37%] 3. The client is concerned that someone might steal possessions [2%] 4. The client wants to take care of business before imminent death [55%] Omitted Correct answer 4 Answered correctly 55% Time: 5 seconds Updated: 01/29/2017 Explanation: This client with advanced renal failure who decides not to start dialysis treatments may have only a few weeks to live. Toxins will build up in the body and soon lead to increased weakness and cognitive decline. This client knows there is a limited time left to live and wants to ensure that possessions will be taken care of appropriately after the client's death (Option 4). (Option 1) The client has probably been admitted to the facility due to concerns about safe management at home. However, the statement does not indicate that the client has been admitted against the client's will. (Option 2) Clients with end-stage renal disease are at risk for delirium due to a buildup of toxins, which may manifest as agitation and statements about needing to go somewhere. However, the nurse should not automatically assume that the client is delirious. Instead, it is important to assess the client's concern with an open mind so that appropriate interventions can be planned. (Option 3) The client's statement about having "so much to do" suggests that this is not the concern prompting the behavior. Educational objective: The client with a limited life expectancy will have concerns about completing personal business, such as ensuring that possessions go to the appropriate people. The nurse should assess the client's needs and ensure that the plan of care will facilitate the client's life closure activities (eg, legacy building). Copyright © UWorld. All rights reserved.

Block Time Remaining: 00:00:22 TUTOR Test Id: 72419393 QId: 31392 (921666) 40 of 75 A A A A nurse on the telemetry unit is preparing client medications in the medication room at the nurse's station. The nurse should perform which actions to be consistent with client safety practices related to medication administration? Select all that apply. 1. Check laboratory values before administering anticoagulants 2. Compare medication, dosage, and route to prescription orders prior to administration 3. Discard any unlabeled medications 4. Open unit dose packages and place medications in a dispensing cup to take to the bedside 5. Wear gloves to handle unopened individual unit dose medication packages Incorrect Correct answer 1,2,3 Answered correctly 55% Time: 79 seconds Updated: 01/07/2017 Explanation: The nurse must follow the 6 rights of medication administration: The right client The right medication The right dose The right time The right route The right documentation Additionally, one of the National Patient Safety Goals (NPSGs) is to "improve the safety of using medications." This includes labeling all medications as soon as prepared, discarding any medications that are found unlabeled, and taking extra care for clients who take anticoagulant drugs. (Option 4) Individual dose packages should be opened at the client's bedside and should be placed in a medication cup only immediately prior to administration. (Option 5) Gloves are generally not required during medication preparation or handling of unopened packages or vials, although hand hygiene should be performed both prior to preparation or handling and again prior to administration. The nurse should wear gloves during medication administration when coming into contact with a route that is potentially contaminated by blood or body fluids (eg, administering intramuscular or subcutaneous injections, accessing a closed IV tubing system, placing a pill into a client's mouth using fingers). Educational objective: The nurse should follow the 6 rights of medication administration when preparing and administering drugs to a client. Additionally, the NPSGs of improving the safety of using medications should be followed, including labeling all medications, discarding medications found unlabeled, and taking extra care for clients taking anticoagulant drugs. Copyright © UWorld. All rights reserved.

Block Time Remaining: 00:00:22 TUTOR Test Id: 72419393 QId: 31382 (921666) 41 of 75 A A A An adolescent client is brought to the emergency department after being in a serious motor vehicle crash. The client is undergoing cardiopulmonary resuscitation. The nurse calls the family to inform them to come to the hospital and a family member asks how the client is doing. Which is an example of the ethical principle of beneficence when responding to the client's family? 1. "He is critically ill and we are caring for his needs." [46%] 2. "His heart has stopped and we are attempting to revive him." [13%] 3. "I don't know how he is doing but you need to come." [1%] 4. "I will have the health care provider talk to you once you arrive." [38%] Correct Answered correctly 46% Time: 165 seconds Updated: 06/02/2017 Explanation: Beneficence is the ethical principle of doing good. It involves helping to meet the client's (including the family) emotional needs through understanding. This can involve withholding information at times. Stating that the client is critically ill and is being cared for meets the ethical principle of veracity (telling the truth) but also avoids overwhelming the family before they travel to the hospital. The nurse does not want the family to be too distressed to process the situation and arrive safely. (Option 2) This is a true statement but it is being given abruptly to the family without support or gradual adjustment. It might be so distressing that they cannot travel to the hospital safely. (Option 3) This is not a true statement and violates the principle of veracity. It will do nothing to help the family and might even cause them alarm that a nurse there is not informed about what is going on with their child. (Option 4) Although this is an option, it does nothing to deal with the situation and the family's needs adequately. It also "passes the buck" to another provider, and even though this provider can speak to them, the nurse should deal with the family's immediate needs at this point. Once they arrive, the health care provider is usually the one to tell family members about the client's prognosis. Educational objective: The ethical principle of beneficence means doing good. It can involve not saying all known information immediately but delaying notification until appropriate support is in place. Copyright © UWorld. All rights reserved.

Block Time Remaining: 00:00:24 TUTOR Test Id: 72797956 QId: 32356 (921666) 13 of 75 A A A A nurse is making a home visit when a fire starts in the client's kitchen trash can. The client has a fire extinguisher. The nurse should take which actions to properly operate the fire extinguisher? Select all that apply. 1. Aim the nozzle at the base of the fire 2. Pull out the pin on the handle 3. Shake the canister prior to use 4. Squeeze the handle to spray 5. Sweep the spray from side to side Omitted Correct answer 1,2,4,5 Answered correctly 78% Time: 1 seconds Updated: 02/12/2017 Explanation: A small fire can quickly become very dangerous. During an emergency situation, such as a fire, anxiety can narrow a person's focus, causing hesitation or difficulty in responding to the situation, especially when operation of unfamiliar equipment (eg, fire extinguisher) is involved. The mnemonic PASS is often used to help people remember the steps used in operating a fire extinguisher: P - Pull the pin on the handle to release the extinguisher's locking mechanism A - Aim the spray at the base of the fire S - Squeeze the handle to release the contents/extinguishing agent S - Sweep the spray from side to side until the fire is extinguished (Option 3) The extinguisher does not need to be shaken before use, and doing so would delay extinguishing the fire. Educational objective: PASS is a mnemonic to help people remember the steps used in operating a fire extinguisher: P - Pull the pin; A - Aim the spray at the base of the fire; S - Squeeze the handle; and S - Sweep the spray. Copyright © UWorld. All rights reserved.

Block Time Remaining: 00:00:33 TUTOR Test Id: 72797956 QId: 30600 (921666) 16 of 75 A A A The nurse prepares equipment for insertion of a large-bore nasogastric (NG) tube for a hospitalized client. Which actions should the nurse take to measure and mark the tube? Select all that apply. 1. Fold tube in half and mark at the halfway point 2. Extend tape measure from naris to stomach 3. Measure from tip of nose to earlobe to xiphoid process 4. Place a small piece of tape at the point of measurement 5. Use rubber clamp after measuring to mark the point of measurement Omitted Correct answer 3,4 Answered correctly 64% Time: 3 seconds Updated: 02/18/2017 Explanation: Because distance from the nares to the stomach varies with each client, it is important to measure and mark the NG tube prior to insertion to ensure its correct placement in the stomach. The Traditional Method is most commonly used for large-bore NG tube placement. Traditional Method: Using the end of the tube that will eventually rest in the stomach, measure from the tip of the nose, extend the tube to the earlobe and then down to the xiphoid process (Options 1, 2, and 3). Mark the distance with a small piece of tape that can be easily removed (Options 4 and 5). Educational objective: Ensure proper measurement prior to inserting a large-bore NG tube by measuring from the tip of the nose, extending the tube to the earlobe, and then down to the xiphoid process. Mark the distance with a small piece of tape that can be easily removed. Copyright © UWorld. All rights reserved.

Block Time Remaining: 00:00:22 TUTOR Test Id: 72490738 QId: 30252 (921666) 4 of 75 A A A The nurse provides an in-service for hospital staff on how to prevent pressure ulcers in clients with limited mobility. Which instructions are appropriate for the nurse to include? Select all that apply. 1. Apply moisture barrier cream to dry skin 2. Clean perineal area after incontinent episodes 3. Massage bony prominences frequently 4. Place foam-padded seat cushions on chairs 5. Reposition clients in bed every 6 hours Omitted Correct answer 1,2,4 Answered correctly 60% Time: 6 seconds Updated: 03/08/2017 Explanation: National Pressure Ulcer Advisory Panel pressure ulcer prevention guidelines Skin care Barriers for incontinence Hydration Moisturizer Repositioning Pad bony prominences Pad medical devices Lift, do not pull Limit chair time Minimize shearing & frictional forces Turn every 2-4 hours Nutrition Calorie counting (30-35 kcal/kg/day) Enteral nutrition High-protein nutritional supplements (1.25-1.5 g/kg/day) Deficiency assessment Support surfaces Alternating pressure Avoid donut-type devices & synthetic sheepskins Heel protection Mattress Overlay Pressure ulcers (eg, decubitus ulcers, bed sores) develop from external pressure compressing capillaries and underlying soft tissue, or from friction and shearing forces. The nurse should assess every client's risk for pressure ulcers (using the Braden scale) upon admission and at least once daily during hospitalization. To prevent pressure ulcers: Use emollients and barrier creams to hydrate, protect, and strengthen the skin (Option 1). Use foam padding on chairs, commode seats, and other surfaces to help reduce pressure on bony prominences (Option 4). Provide prompt incontinence care and use additional barrier cream to keep skin clean and dry; this will further help reduce irritation and associated breakdown of the skin (Option 2). Reposition clients with a turn sheet every 2 hours using devices (eg, pillows, foam wedges) to maintain position; avoid pulling/dragging the client up in bed, as shearing can occur. (Option 3) Massage is not an acceptable intervention for pressure ulcer prevention as it can lead to deep tissue damage. It is contraindicated in the presence of inflammation, damaged blood vessels, or fragile skin. (Option 5) Clients must be repositioned and turned every 2 hours. Turning clients every 6 hours is too infrequent and will not confer the same protection against pressure and associated tissue ischemia. Educational objective: Skin assessment, proper skin care, repositioning every 2 hours, adequate nutrition, and proper support surfaces are effective in helping prevent pressure ulcers. Massage over the bony prominences is not recommended for pressure ulcer prevention. Copyright © UWorld. All rights reserved.

Block Time Remaining: 00:00:36 TUTOR Test Id: 72490738 QId: 30687 (921666) 5 of 75 A A A The nurse is caring for a client with a chest tube to evacuate a hemopneumothorax after a motor vehicle accident. The drainage has been consistently 25-50 mL/hr for the majority of the shift. However, over the past 2 hours there has been no drainage. Which actions should the nurse take? Select all that apply. 1. Auscultate breath sounds 2. Increase amount of suction 3. Instruct client to cough and deep breathe 4. Milk the chest tube 5. Reposition the client Correct Answered correctly 62% Time: 14 seconds Updated: 01/13/2017 Explanation: When chest drainage stops abruptly, the nurse must perform assessments and interventions to ascertain if this is an expected finding. Auscultating breath sounds (Option 1) helps the nurse detect whether breath sounds are audible in all lung fields, potentially indicating that the lung has re-expanded and there is no more drainage. Other interventions to facilitate drainage include having the client cough and deep breathe (Option 3) and repositioning the client (Option 5). If a client has been in one position for a prolonged period, drainage may accumulate and a position change may facilitate improved drainage. (Option 2) A change in suction level should be performed only after obtaining a health care provider (HCP) prescription. The nurse should perform the assessment of breath sounds, coughing and deep breathing, and client repositioning before notifying the HCP about a change in suction level. In general, suction above 20 cm H2O is not indicated. (Option 4) Milking chest tubes to maintain patency is performed only if prescribed. It is generally contraindicated due to potential tissue damage from highly increased pressure changes in the pleural space. Educational objective: The nurse should assess breath sounds, encourage coughing and deep breathing, and reposition the client who has a decrease in chest tube drainage. Copyright © UWorld. All rights reserved.

Block Time Remaining: 00:00:35 TUTOR Test Id: 72797956 QId: 31405 (921666) 18 of 75 A A A A 3-month-old infant is treated in the emergency department for a spiral femur fracture. The parent reports that the infant sustained the injury after rolling off the bed. What is the priority nursing action? 1. Document a description of the injury [15%] 2. Question the mother about where the infant sleeps [26%] 3. Report the injury per facility protocol [52%] 4. Separate the mother from the infant [4%] Omitted Correct answer 3 Answered correctly 52% Time: 1 seconds Updated: 05/08/2017 Explanation: The parent's account of this injury is inconsistent with the developmental milestones of a 3-month-old infant, as the muscles required for rolling over do not develop until age 4-5 months. Additionally, spiral femur fractures indicate that pressure was applied to the leg in opposite directions (torsion), which is an unlikely accidental injury in a nonambulatory child. Fractures in young children, especially nonambulatory infants, are always of concern and suspicious of child abuse. The nurse's priority is to report suspected child maltreatment to the appropriate authorities following facility protocol as required by law in the United States and Canada (Option 3). However, the nurse should also be aware of cultural health practices (eg, cupping, coining) and physiologic conditions (eg, hemophilia, Mongolian spots) mimicking maltreatment. After reporting suspected maltreatment, the nurse should: Facilitate a complete physical evaluation (eg, skeletal survey, growth/development comparisons, radiographic studies, neurologic examination) Document facts and observations objectively, using medical terms when possible (Option 1). Include the history provided by the parent or caregiver and the time period from injury occurrence to evaluation. Perform a review of child-care practices with the caregiver (Option 2). (Option 4) A child and caregiver should only be separated when the child is in immediate physical danger or if authorities must interview a verbal child without the parent present. Educational objective: Injuries in a nonambulatory child, especially fractures, warrant suspicion. The nurse has a duty to report suspected child maltreatment to the appropriate authorities as required by law. Copyright © UWorld. All rights reserved.

Block Time Remaining: 00:00:40 TUTOR Test Id: 72797956 QId: 31646 (921666) 21 of 75 A A A A student nurse prepares to change a large wet-to-damp sterile wound dressing and uses a disposable moisture-proof sterile drape to set up the sterile field. The precepting nurse intervenes when the student performs which action? 1. Holds the package 6" (15 cm) above the sterile field and drops the sterile gauze onto the field [3%] 2. Opens the sterile gauze package with ungloved hands [13%] 3. Places the sterile gauze dressings within 2" (5 cm) from the edge of the sterile drape [7%] 4. Pours sterile normal saline solution (NSS) into a sterile basin from a bottle opened 30 hours ago [75%] Omitted Correct answer 4 Answered correctly 75% Time: 2 seconds Updated: 03/23/2017 Explanation: The sterility of an opened bottle of sterile saline cannot be guaranteed. Some institutions' policies permit recapped bottles of solution to be reused within 24 hours of opening, and some require disposal of the remaining solution. Therefore, the nurse should intervene when the student uses sterile saline from a bottle that was opened >24 hours ago. The general steps for preparing the sterile field for a wet-to-damp dressing change include: Perform hand hygiene. Open a sterile gauze package that has a partially sealed edge with ungloved hands by grasping both sides of the edge, one with each hand, and pull them apart while being careful not to contaminate the gauze (Option 2). Hold the inverted opened gauze package 6" (15 cm) above the waterproof sterile field so it does not touch the field, and then drop the gauze dressing onto the sterile field (Option 1). Place the sterile dressings on the sterile field 2" (5 cm) from the edge; the 1" (2.5 cm) margin at each edge is considered unsterile because it is in contact with unsterile surfaces (Option 3). Use sterile NSS from a recapped bottle that was opened <24 hours ago (if policy permits). Educational objective: The general steps for preparing the sterile field for a wet-to-damp dressing change include: Perform hand hygiene. Open a sterile gauze package with ungloved hands. Hold the inverted opened gauze package 6" (15 cm) above the sterile field. Place the sterile gauze dressing more than 1" (2.5 cm) from the edge of the sterile field. Use sterile NSS from a recapped bottle that was opened <24 hours ago (if policy permits). Copyright © UWorld. All rights reserved.

Block Time Remaining: 00:00:43 TUTOR Test Id: 72797956 QId: 30093 (921666) 24 of 75 A A A The nurse is performing a central line tubing change when the client suddenly begins gasping for air and writhing. Order the interventions by priority. All options must be used. Your Response/ Incorrect Response Correct Response Clamp the catheter tubing Place the client in Trendelenburg position on the left side Administer oxygen as needed Notify the health care provider (HCP) Stay with the client and provide reassurance Omitted Correct answer 2,4,1,3,5 Answered correctly 28% Time: 1 seconds Updated: 02/04/2017 Explanation: Leakage of more than 500 mL of air into a central venous catheter is potentially fatal. An air embolism in the small pulmonary capillaries obstructs blood circulation. A central venous catheter leaks air rapidly at 100 mL/sec. This client requires immediate intervention to prevent further complications (eg, cardiac arrest, death). The nurse should not delay emergency treatment, not even to stop and contact the HCP or the rapid response team (RRT). Priority interventions for active or suspected air embolism are as follows: Clamp the catheter to prevent more air from embolizing into the venous circulation. Place the client in Trendelenburg position on the left side, causing any existing air to rise and become trapped in the right atrium. Administer oxygen if necessary to relieve dyspnea. Notify the HCP or call an RRT to provide further resuscitation measures. Stay with the client to provide reassurance and monitoring as the air trapped in the right atrium is slowly absorbed into the bloodstream over the course of a few hours. Educational objective: Any delay in treatment of an air embolism could prove fatal. There is no time to call the HCP. Seal off the source of the leak, and ensure stabilization of the air bubble via left lateral positioning. Copyright © UWorld. All rights reserved.

Block Time Remaining: 00:00:44 TUTOR Test Id: 72797956 QId: 34067 (921666) 25 of 75 A A A The nurse is interviewing a non-English-speaking client. Which best practices will the nurse use when working with a professional medical interpreter for clients of limited English proficiency? Select all that apply. 1. Address the interpreter directly 2. Ask the client's adult child to translate 3. Hold a pre-conference with the interpreter 4. Identify any gender or age preferences 5. Speak in short sentences Omitted Correct answer 3,4,5 Answered correctly 38% Time: 1 seconds Updated: 03/27/2017 Explanation: Title IV of the Civil Rights Act of 1964 initiated national standards for appropriate care of culturally diverse clients. Clients with limited English proficiency have the right to receive medical interpreter services free of charge. When working with an interpreter, the nurse should apply the following best practices to maximize communication and understanding with the client: Address the client directly in the first person Speak in short sentences, pausing to allow the interpreter to speak (Option 5) Ask only one question at a time Avoid complex issues, idioms, jokes, and medical jargon Hold a pre-conference with the medical interpreter to review the goals of the interview (Option 3) Use a qualified professional interpreter whenever possible The nurse should avoid using interpreters from conflicting cultures (eg, Palestinian, Jewish) and be mindful of any cultural, gender, or age preferences (Option 4). (Option 1) The nurse should speak directly to the client, not the interpreter. (Option 2) A family member or friend may not have the vocabulary, knowledge, or skills to provide the best communication for the client. Untrained interpreters may omit or simplify critical pieces of information if they do not understand the terminology. Educational objective: When working with a medical interpreter, the nurse should apply best practices to maximize communication and understanding with the client. Key practices include speaking to the client directly; using short, simple sentences; avoiding the use of family members as interpreters; and being mindful of cultural, gender, or age preferences. Copyright © UWorld. All rights reserved.

Block Time Remaining: 00:00:45 TUTOR Test Id: 72797956 QId: 32149 (921666) 26 of 75 A A A The nurse is caring for a postoperative client who has D5W/0.45% normal saline with 10 mEq potassium chloride infusing through a peripheral IV catheter. What are appropriate reasons for the nurse to change the site? Select all that apply. 1. Area around the insertion site feels cool to the touch 2. Client reports mild arm discomfort since the infusion was started 3. Edema is observed on the dependent side of the involved arm 4. Intraoperative peripheral IV catheter is placed in the left antecubital region 5. Serous fluid leaks from the site despite secure connections Omitted Correct answer 1,3,5 Answered correctly 47% Time: 1 seconds Updated: 04/17/2017 Explanation: Peripheral IV (PIV) catheter sites should be changed usually no more frequently than every 72-96 hours unless signs of complications develop. Signs of phlebitis include erythema, edema, warmth, pain, and palpable venous cord. Manifestations of infiltration include edema and coolness to the touch around the insertion site (Option 1). The nurse should also monitor for edema related to infiltration under the involved limb. Infiltrated fluid may leak into loose skin, causing edema in dependent areas without obvious signs of infiltration at the PIV site, particularly in the elderly (Option 3). If a PIV site is leaking fluid, the tubing and catheter connections should be assessed. If all connections are intact, possible problems include infiltration/extravasation, a thrombus at the catheter tip, or damage to the catheter; all of these issues require a site change (Option 5). (Option 2) Potassium is a known irritant to veins. Discomfort is not a sign of infiltration, although the site should be regularly monitored for complications. (Option 4) Locations where flexion occurs (eg, antecubital region) are generally avoided; however, these sites may be required for certain medications or situations. Unless a problem develops, PIV sites are not changed based solely on location. Educational objective: Peripheral IV catheter sites should be changed no more frequently than every 72-96 hours unless signs of complications develop. The nurse should check for signs of infiltration by assessing the insertion site and areas dependent from it (ie, edema, cool skin). Copyright © UWorld. All rights reserved.

Block Time Remaining: 00:00:47 TUTOR Test Id: 72797956 QId: 31625 (921666) 27 of 75 A A A A nurse administers an intramuscular (IM) injection using the Z-track technique. Place the steps in chronological order. All options must be used. Your Response/ Incorrect Response Correct Response Pull the skin 1-1 ½" (2.5-3.5 cm) laterally and away from the injection site Hold the skin taut with non-dominant hand and insert needle at a 90-degree angle Inject medication slowly with dominant hand while maintaining traction Wait 10 seconds after injecting the medication and withdraw the needle Release the hold on the skin, allowing the layers to slide back to their original position Apply gentle pressure at the injection site but do not massage Omitted Correct answer 4,3,2,6,5,1 Answered correctly 56% Time: 2 seconds Updated: 01/15/2017 Explanation: The Z-track technique prevents tracking (leakage) of the medication into the subcutaneous tissue and is universally recommended for the administration of IM injections. Displacing the skin while injecting the medication, and then releasing the skin back to its normal position after removing the needle creates a zigzag track. The procedure for administering an IM injection using the Z-track technique includes these steps: Pull the skin 1-1 ½" (2.5-3.5 cm) laterally away from the injection site (Option 4). Hold the skin taut with the nondominant hand, and insert the needle at a 90-degree angle - taut skin facilitates entry of the needle and this angle ensures that the needle will reach the muscle (Option 3). Inject the medication slowly into the muscle while maintaining traction - slow injection promotes comfort and allows time for tissue expansion to facilitate absorption of the medication (Option 2). Wait 10 seconds after injecting the medication and withdraw the needle while maintaining traction on the skin; this allows the medication to diffuse before needle removal and helps to prevent tracking (Option 6). Release the hold on the skin - this allows the tissue layers to slide back to their original position, sealing off the needle track (Option 5). Apply gentle pressure at the injection site, but do not massage as this can cause the medication to seep back up to the skin surface and cause local tissue irritation (Option 1). There is no clear evidence to support the need for aspiration prior to IM injection. Aspiration may be indicated if the dorsogluteal site (last resort) is used for IM injection due to its proximity to the gluteal artery. The preferred areas for IM injection are the ventrogluteal site in adults and the vastus lateralis site in children. Educational objective: The Z-track technique for administering IM injections prevents tracking (leakage) of the medication into the subcutaneous tissue. Copyright © UWorld. All rights reserved.

Block Time Remaining: 00:00:48 TUTOR Test Id: 72797956 QId: 31358 (921666) 28 of 75 A A A Which interventions should the nurse perform when assisting the health care provider with removal of a client's chest tube? Select all that apply. 1. Ensure the client is given an analgesic 30-60 minutes before tube removal 2. Instruct the client to breathe in, hold it, and bear down while the tube is being removed 3. Place the client in the Trendelenburg position 4. Prepare a sterile airtight petroleum jelly gauze dressing 5. Provide the health care provider with sterile suture removal equipment Omitted Correct answer 1,2,4,5 Answered correctly 34% Time: 1 seconds Updated: 01/19/2017 Explanation: A chest tube is removed when drainage is minimal (<200 mL/24 hr) or absent, an air leak (if present) is resolved, and the lung has reexpanded. The general steps for chest tube removal include: Premedicate the client with analgesic (eg, IV opioid, nonsteroidal anti-inflammatory drug [ketorolac]) 30-60 minutes before the procedure to promote comfort as evidence indicates that most clients report significant pain during removal (Option 1). Provide the health care provider (HCP) with sterile suture removal equipment (Option 5). Instruct the client to breathe in, hold it, and bear down (Valsalva maneuver) while the tube is removed to decrease the risk for a pneumothorax. Most HCPs use this technique to increase intrathoracic pressure and prevent air from entering the pleural space (Option 2). Apply a sterile airtight occlusive dressing to the chest tube site immediately; this will prevent air from entering the pleural space (Option 4). Perform a chest x-ray within 2-24 hours after chest tube removal as a post-procedure pneumothorax or fluid accumulation usually develops within this time frame. (Option 3) The client should be placed in semi-Fowler's position or on the unaffected side to promote comfort and facilitate access for tube removal. Educational objective: Before chest tube removal, the client is given an analgesic and then asked to perform Valsalva during the procedure. The nurse should also bring sterile suture removal equipment and a sterile airtight occlusive dressing. Post-procedure chest x-ray is necessary within 2-24 hours. Copyright © UWorld. All rights reserved.

Block Time Remaining: 00:00:49 TUTOR Test Id: 72797956 QId: 34229 (921666) 29 of 75 A A A The nurse receives report on 4 clients. Which client should be seen first? 1. Client with amyotrophic lateral sclerosis experiencing increased dysarthria [24%] 2. Client with chronic obstructive pulmonary disease reporting increasing leg edema [40%] 3. Client with strep throat and fever of 102 F (38.9 C) on antibiotics for 12 hours [11%] 4. Client with urolithiasis reporting wavelike flank pain and nausea [24%] Omitted Correct answer 1 Answered correctly 24% Time: 1 seconds Updated: 02/06/2017 Explanation: Amyotrophic lateral sclerosis (ALS) is characterized by the progressive loss of motor neurons in the brainstem and spinal cord. Clients have spasticity, muscle weakness, and atrophy. Neurons involved in swallowing and respiratory function are eventually impaired, leading to aspiration, respiratory failure, and death. Care of clients with ALS focuses on maintaining respiratory function, adequate nutrition, and quality of life. There is no cure, and death usually occurs within 5 years of diagnosis. The client with ALS and worsening ability to speak (dysarthria) may also have dysphagia and respiratory distress; this client should be seen first (Option 1). (Option 2) The client with chronic obstructive pulmonary disease and peripheral edema may have cor pulmonale, or right-sided heart failure, from vasoconstriction of the pulmonary vessels. Cor pulmonale is treated with long-term, low-flow oxygen; bronchodilators; and diuretics. This client should be seen second. Right-sided heart failure (peripheral edema) is not as dangerous as left-sided heart failure (pulmonary edema). (Option 3) Fever often occurs with strep throat and may persist for ≥24 hours after initiation of antibiotics. This client should be seen last and should receive an antipyretic. (Option 4) Wavelike flank pain is characteristic of urolithiasis (urinary stones). This client needs pain medication and, possibly, further treatment (eg, lithotripsy) and should be seen third. Educational objective: Amyotrophic lateral sclerosis causes progressive loss of motor neurons, resulting in muscle weakness and spasticity. Muscles involved in respiration and swallowing are affected, leading to aspiration and, ultimately, respiratory failure. Treatment focuses on maintaining respiratory function, adequate nutrition, and quality of life. Copyright © UWorld. All rights reserved.

Block Time Remaining: 00:01:10 TUTOR Test Id: 72797956 QId: 31771 (921666) 32 of 75 A A A A client with suspected foot osteomyelitis is scheduled for magnetic resonance imaging (MRI). Which assessment findings should the nurse notify the health care provider (HCP) about before the test? Select all that apply. 1. Cardiac pacemaker 2. Colostomy 3. Retained metal foreign body in eye 4. Total hip replacement 5. Transdermal testosterone patch Omitted Correct answer 1,3,4 Answered correctly 73% Time: 10 seconds Updated: 04/06/2017 Explanation: Clients must be screened for contraindications before exposure to the magnetic field (MRI), as it can damage implanted devices or metallic implants. Absolute contraindications can preclude testing, and relative contraindications can pose a hazard to the client's devices or implants, affect the quality of the images, or cause discomfort. Therefore, the imaging HCP determines the client's eligibility for MRI. Absolute contraindications: Cardiac pacemaker (Option 1) Implantable cardioverter defibrillator (ICD) Cochlear implant Retained metallic foreign body, especially in organs such as the eye (Option 3) Relative contraindications: Prosthetic heart valve Metal plate, pins, brain aneurysm clip, or joint prosthesis (Option 4). Some of these devices have nonferrous MRI-safe materials and should be verified. Implanted device (eg, insulin pump, medication port) Other factors that can affect the client's eligibility include inability to remain supine for 30-60 minutes and claustrophobia (sedation can be prescribed, open MRI machine can be used). (Option 2) A colostomy is not a contraindication for MRI. (Option 5) Transdermal metal-containing medication patches (clonidine, nicotine, scopolamine, testosterone, or fentanyl) are not a contraindication for MRI. However, these must be removed before and replaced after testing. Educational objective: Usual contraindications for MRI include metallic implants (eg, pacemaker, ICD, plates, pins, brain aneurysm clips), implanted devices (eg, insulin pumps, medication ports), and prostheses (eg, joints, heart valves). However, some of these devices are manufactured with MRI-safe materials that should be verified. Copyright © UWorld. All rights reserved.

Block Time Remaining: 00:01:12 TUTOR Test Id: 72797956 QId: 31616 (921666) 33 of 75 A A A A home health nurse is supervising a home health aide who is changing the dressing for a client with a chronic heel wound. Which actions by the aide indicate adherence to appropriate infection control procedures? Select all that apply. 1. Open a sterile container of 4 x 4's using the outermost corner to peel back the cover 2. Pull glove off over the soiled dressing to encase it before disposal 3. Save unused sterile 4 x 4's by taping original package shut for the next dressing change 4. Wash hands prior to putting on gloves and after removing them 5. Wrap soiled dressing in paper towels before disposing of it in the trash can Omitted Correct answer 1,2,4 Answered correctly 56% Time: 7 seconds Updated: 02/25/2017 Explanation: The nurse is responsible for observing the home health aide periodically during delegated tasks. The aide should wash the hands prior to gloving and after glove removal (Option 4). Sterile dressing supplies should be opened prior to the dressing change; this should be done by carefully peeling from the outermost corner of the package to expose the contents without contaminating the sterile product (Option 1). A contaminated used dressing should be placed in impervious plastic or a paper bag before disposal in the household trash (Option 2). (Option 3) Unused sterile supplies should not be saved as it is not possible to ensure their sterility. (Option 5) Paper towels are not impervious and infectious waste from the dressing can seep through and into other items in the trash can. Educational objective: In the home care setting, infection control procedures for changing a dressing include washing the hands before and after gloving, opening sterile supplies carefully to avoid contamination, and placing old dressings inside a glove or plastic bag before disposal in the household trash. Copyright © UWorld. All rights reserved.

Block Time Remaining: 00:00:52 TUTOR Test Id: 72419393 QId: 31452 (921666) 43 of 75 A A A The same-day surgery nurse performs the preoperative assessment for a client with a history of coronary artery disease scheduled for an elective laparoscopic cholecystectomy. Which statement made by the client is critical to report to the health care provider (HCP) before the surgery? 1. "I didn't take the clopidogrel pill for my heart yesterday or today." [52%] 2. "I know I should stop smoking completely, but at least I didn't have a cigarette yesterday or today." [8%] 3. "I stopped taking my gingko biloba 2 weeks ago even though it really helps relieve leg cramps when I walk." [9%] 4. "I stopped taking naproxen for my arthritis pain 1 week ago and have been taking acetaminophen instead." [30%] Correct Answered correctly 52% Time: 26 seconds Updated: 01/23/2017 Explanation: Clopidogrel (Plavix) is an antiplatelet medication that should be discontinued 5-7 days before surgery to decrease the risk for excessive bleeding. The client took this drug 48 hours ago. Therefore, the nurse must notify the HCP. The surgery may be postponed due to the increased risk for intra- and post-operative bleeding (Option 1). (Option 2) All clients should try not to smoke for at least 24 hours before surgery to help prevent oxygenation problems. (Option 3) The client takes gingko biloba to relieve symptoms of intermittent claudication; it was discontinued 2 weeks ago because it can increase the risk for excessive bleeding. (Option 4) Nonsteroidal anti-inflammatory drugs (NSAIDS) such as naproxen (Naprosyn) should be discontinued 7 days before scheduled surgery as they can increase the risk for excessive bleeding. Acetaminophen can be taken to control pain up until surgery. Educational objective: Medications (eg, anticoagulants, antiplatelets, NSAIDS, herbal drugs) that increase the risk for excessive bleeding should be discontinued at least 5-7 days before surgery. Copyright © UWorld. All rights reserved.

Block Time Remaining: 00:01:13 TUTOR Test Id: 72419393 QId: 33555 (921666) 44 of 75 A A A The charge nurse must assign a room for a client with dementia who was transferred from a long-term care facility and is scheduled for extensive surgical debridement of a stage 4 pressure ulcer. Which room assignment is the most appropriate for this client? 1. Room A: Client with multiple myeloma who is being treated with corticosteroids [9%] 2. Room B: Client with diabetes mellitus and osteomyelitis receiving IV antibiotics [15%] 3. Room C: Client with a gastrointestinal bleed who has a nasogastric tube set to low suction [29%] 4. Room D: Client with an acute migraine headache attack who requires IV analgesia every 2 hours [45%] Incorrect Correct answer 3 Answered correctly 29% Time: 21 seconds Updated: 06/03/2017 Explanation: Surgical debridement of a stage 4 pressure ulcer involves using a scalpel to remove necrotic (eschar) or infected tissue from the wound to promote healing. The most appropriate room assignment for this client is Room C, as the client with a gastrointestinal bleed and nasogastric tube is the least susceptible to infection when compared with the clients in rooms A and B. (Option 1) Multiple myeloma is a cancer that involves proliferation of malignant plasma cells (monoclonal antibodies), which are ineffective in providing protection against infection and suppress normal bone marrow cell production (eg, red blood cells, platelets, white blood cells). This client is especially vulnerable to infection due to immunosuppression related to the disease process and to drug therapy with corticosteroids. (Option 2) The postoperative client should not be assigned to a room with a client who has osteomyelitis, an infection of bone. (Option 4) The client with a severe migraine requires frequent administration of analgesia and antiemetic medications, and a quiet dark environment with minimal stimuli to help induce sleep. This is not an appropriate room assignment, as surgery and change of environment can increase agitation, hyperactivity, and disorientation in a client with dementia; these require frequent nursing interventions to maintain safety. Educational objective: A client with dementia undergoing an extensive surgical debridement for an infected pressure ulcer should not be assigned to a room with a client who is vulnerable to infection (eg, immunocompromised), has an active infection, or requires a quiet dark environment. Copyright © UWorld. All rights reserved.

Block Time Remaining: 00:00:23 TUTOR Test Id: 72458431 QId: 31161 (921666) 4 of 10 A A A Exhibit The nurse is forming a plan of care for an 82-year-old client with a history of anxiety, hypertension, urinary incontinence, and arthritic back pain. Which nursing diagnosis should be addressed first? Click on the exhibit button for additional information. 1. Anxiety [5%] 2. Chronic pain [9%] 3. Risk for acute confusion [13%] 4. Risk for falls [70%] Correct Answered correctly 70% Time: 23 seconds Updated: 02/09/2017 Explanation: Fall risk precautions Standard Orientation to room & call light Call light within reach Bed in lowest position Uncluttered room Nonslip socks or shoes Well-lit room Belongings within reach High fall risk Bed alarm High fall risk signs Room close to nurses' station Color-coded socks & wristbands When determining which nursing diagnosis to address first, the nurse should consider factors that affect client safety. Risk for falls is an immediate safety concern (Option 4). Nursing diagnoses that relate to chronic conditions (eg, anxiety, chronic pain) are addressed after risk for falls. The nurse should immediately implement fall risk precautions by placing the bed in the lowest position, ensuring that the call light is within reach, and turning on the bed alarm. Interventions for addressing other client needs may be carried out after measures to ensure client safety. Advanced age is associated with decreased visual acuity, muscle mass, strength, and reaction time. Medications that cause dizziness or drowsiness increase the risk for falls. Diuretics (eg, furosemide) increase urinary frequency and may cause hypotension. Antihypertensive medications (eg, lisinopril, metoprolol) may cause bradycardia and dizziness. (Option 1) Safety needs are addressed before love and belonging needs (eg, anxiety). Anxiety interventions (eg, therapeutic touch, medication) may be implemented after safety interventions. (Option 2) Safety is the immediate concern for a client with a high fall risk. Arthritic joint changes are a source of chronic pain. Pain interventions (eg, medication, repositioning) may be implemented after safety interventions. (Option 3) A client with advanced age in an unfamiliar environment may develop acute confusion during the hospital course, but a high fall risk is a more immediate concern on admission. Educational objective: The nursing diagnosis of risk for falls should be addressed first for a client who has multiple risk factors for falls. Advanced age, incontinence, certain medications, and limited mobility increase fall risk. Copyright © UWorld. All rights reserved.

Block Time Remaining: 00:01:27 TUTOR Test Id: 72458431 QId: 30118 (921666) 5 of 10 A A A A registered nurse is precepting a new nurse in the intensive care unit. The client is sedated with propofol, on a mechanical ventilator, and is receiving enteral feeding via nasogastric tube. The new nurse performs interventions to prevent aspiration. The preceptor should intervene if the new nurse performs which of the following actions? 1. Assesses gastric residual volumes every 4 hours [10%] 2. Measures the number of centimeters the feeding tube is secured at the nare every 4 hours [8%] 3. Requests that the physician change the client from continual to bolus feedings [67%] 4. Uses a sedation scale to titrate down the sedation (if possible) [14%] Correct Answered correctly 67% Time: 64 seconds Updated: 04/26/2017 Explanation: Critically ill clients are at increased risk for aspiration of oropharyngeal secretions and gastric content. It is common in clients who are intubated, sedated, on a mechanical ventilator, and receiving enteral feedings. The nurse must provide nursing interventions to prevent aspiration and monitor for its signs and symptoms. Clients are at increased risk when receiving bolus rather than continual enteral feedings. Bolus feedings should be avoided in critically ill clients, who are already at increased risk for aspiration. (Option 1) Assessing gastric residual volumes according to institution policy (at least every 4 hours) is standard for clients receiving continual enteral feedings. Increased volumes may indicate poor absorption and increase the risk of regurgitation and aspiration. (Option 2) Measuring the number of centimeters at the nare every 4 hours can help determine if the tube has moved, but it can increase aspiration risk. X-ray confirmation may be necessary if the tube has moved. (Option 4) A sedation scale such as the Ramsay Scale is used to assess level of sedation. It is preferable to keep the client minimally sedated (asleep but arousable). This helps decrease the risk of aspiration. Educational objective: Assessing gastric residual volumes and level of sedation at regular intervals, checking enteral feeding tube placement, and administering continual rather than bolus tube feeding are interventions that help prevent aspiration in critically ill high-risk clients. Copyright © UWorld. All rights reserved.

Block Time Remaining: 00:00:36 TUTOR Test Id: 72490738 QId: 30687 (921666) 5 of 75 A A A The nurse is caring for a client with a chest tube to evacuate a hemopneumothorax after a motor vehicle accident. The drainage has been consistently 25-50 mL/hr for the majority of the shift. However, over the past 2 hours there has been no drainage. Which actions should the nurse take? Select all that apply. 1. Auscultate breath sounds 2. Increase amount of suction 3. Instruct client to cough and deep breathe 4. Milk the chest tube 5. Reposition the client Correct Answered correctly 62% Time: 14 seconds Updated: 01/13/2017 Explanation: When chest drainage stops abruptly, the nurse must perform assessments and interventions to ascertain if this is an expected finding. Auscultating breath sounds (Option 1) helps the nurse detect whether breath sounds are audible in all lung fields, potentially indicating that the lung has re-expanded and there is no more drainage. Other interventions to facilitate drainage include having the client cough and deep breathe (Option 3) and repositioning the client (Option 5). If a client has been in one position for a prolonged period, drainage may accumulate and a position change may facilitate improved drainage. (Option 2) A change in suction level should be performed only after obtaining a health care provider (HCP) prescription. The nurse should perform the assessment of breath sounds, coughing and deep breathing, and client repositioning before notifying the HCP about a change in suction level. In general, suction above 20 cm H2O is not indicated. (Option 4) Milking chest tubes to maintain patency is performed only if prescribed. It is generally contraindicated due to potential tissue damage from highly increased pressure changes in the pleural space. Educational objective: The nurse should assess breath sounds, encourage coughing and deep breathing, and reposition the client who has a decrease in chest tube drainage. Copyright © UWorld. All rights reserved.

Block Time Remaining: 00:01:29 TUTOR Test Id: 72490738 QId: 30716 (921666) 7 of 75 A A A The nurse assesses a client with fever and productive cough for the last 10 days. Which findings support the presence of pneumonia? Select all that apply. 1. Coarse crackles 2. Hyperresonance 3. Pleuritic chest pain 4. Shortness of breath 5. Trachea deviating from midline Correct Answered correctly 46% Time: 18 seconds Updated: 04/13/2017 Explanation: Pneumonia is an acute infection of the lungs. Findings in a client with pneumonia include: Crackles - Fine or coarse crackling sounds caused by air passing through alveoli and small airways obstructed with mucus (Option 1) Fever, chills, productive cough, dyspnea, and pleuritic chest pain (Options 3 and 4) Increased vocal/tactile fremitus - Transmission of palpable vibrations (fremitus) is increased when transmitted through consolidated versus normal lung tissue. Bronchial breath sounds in peripheral lung fields - High-pitched, harsh sounds conducted through consolidated lung tissue, which are abnormal when heard in an area distant from where normally heard (ie, trachea); this finding can be an early sign of pneumonia. Unequal chest expansion - Decreased expansion of affected lung on palpation Dullness - Percussion of medium-pitched sounds over consolidated lung tissue (pneumonia) or fluid-filled space (eg, pleural effusion, a complication of pneumonia) (Option 2) Hyperresonance is percussed over a hyperinflated lung (eg, asthma, emphysema) or air in the pleural space (eg, pneumothorax). (Option 5) A trachea deviating from midline is not a symptom of pneumonia but instead indicates a tension pneumothorax where the trachea deviates away from the tension. Educational objective: Physical examination of a client with pneumonia can reveal crackles, increased vocal/tactile fremitus, unequal chest expansion, and bronchial breath sounds in peripheral areas. Clients often report fever, chills, productive cough, dyspnea, and pleuritic chest pain. Copyright © UWorld. All rights reserved.

Block Time Remaining: 00:01:38 TUTOR Test Id: 72419393 QId: 30135 (921666) 45 of 75 A A A A student nurse asks why enteral (tube) feedings, rather than total parenteral nutrition (TPN), are being administered to a client with sepsis and respiratory failure. Which is the best response by the registered nurse? 1. "Enteral feedings have no complications." [1%] 2. "Enteral feedings maintain gut integrity and help prevent stress ulcers." [46%] 3. "Enteral feedings provide higher calorie content." [9%] 4. "Risk of hyperglycemia is lower with enteral feedings than with TPN." [42%] Incorrect Correct answer 2 Answered correctly 46% Time: 25 seconds Updated: 03/24/2017 Explanation: Stress ulcers are a common complication in critically ill clients because the gastrointestinal tract is not a preferential organ. In the presence of hypoxemia, blood is shunted to the more vital organs, increasing the risk of stress ulcers. The early initiation of enteral feedings helps preserve the function of the gut mucosa, limits movement of bacteria (translocation) from the intestines into the bloodstream, and prevents stress ulcers. Enteral feedings are also associated with lower risk of infectious complications compared with TPN. However, the mortality is the same. (Option 1) Complications/problems commonly associated with enteral feedings include aspiration, tube displacement, hyperglycemia, diarrhea, abdominal distension, enteral tube misconnections, and clogged tubes. (Option 3) Caloric and metabolic needs can usually be met adequately using enteral feedings or TPN. Multiple enteral or TPN formulas are available to meet individual client needs. If metabolic demands are not being met using enteral feedings alone, TPN can be added. (Option 4) Illness-related stress hyperglycemia (gluconeogenesis) occurs in clients receiving both enteral feedings and TPN. Educational objective: The enteral route is preferred for feeding. Enteral feedings maintain the integrity of the gut, prevent stress ulcers, and help prevent the translocation of bacteria into the bloodstream. Copyright © UWorld. All rights reserved.

Block Time Remaining: 00:01:52 TUTOR Test Id: 72419393 QId: 30567 (921666) 46 of 75 A A A The nurse should consider which of the following client reports as an indication of an allergic reaction? 1. "I can't eat broccoli or cabbage when I take my warfarin." [0%] 2. "I get a headache when using my nitroglycerine patch." [0%] 3. "My feet swell when I take felodipine." [0%] 4. "My lips swell when I eat bananas or avocados." [98%] Correct Answered correctly 98% Time: 14 seconds Updated: 04/26/2017 Explanation: People with latex allergy usually have a cross-allergy to foods such as bananas, kiwis, avocados, tomatoes, peaches, and grapes because some proteins in rubber are similar to food proteins. Latex sensitivity increases with exposure and should be suspected in the following situations: Allergic contact dermatitis (rash, itching, vesicles) developing 3-4 days after exposure to a rubber latex product. This is a type IV hypersensitivity reaction (delayed onset). Anaphylaxis - many cases of anaphylaxis have been reported in both medical and non-medical settings. These represent a type I hypersensitivity reaction and should be treated with intramuscular epinephrine injections. Some common settings include: Glove use Procedures involving balloon-tipped catheters (eg, arterial catheterization) Blowing up toy balloons Use of bottle nipples, pacifiers Use of condoms or diaphragms during sex Clients with severe allergies should wear a Medic Alert bracelet and carry an injectable epinephrine pen due to cross-sensitivity with many food and industrial products that can be impossible to avoid. (Option 1) Foods rich in vitamin K reduce the effects of warfarin (which works by inhibiting vitamin K-dependent clotting factors). Consumption of these foods decreases the effectiveness of warfarin; clients must be taught to eat the same amount of or avoid dark, green, leafy vegetables. (Option 2) Nitroglycerine is a vasodilator and a headache from dilating cerebral vessels is an expected finding. The side effect is treated with acetaminophen (Tylenol). (Option 3) Peripherally acting calcium channel blockers (eg, nifedipine, amlodipine, felodipine) cause vasodilation, and clients may develop peripheral edema. This is an expected, frequent side effect and is not an allergic reaction. Clients are advised to elevate the legs when lying down and to use stockings. Educational objective: Latex allergy is suspected when there is a food allergy to banana, kiwis, or avocados. Peripheral edema is an expected side effect of peripherally acting calcium channel blockers. Headache is an expected side effect of nitroglycerine. Clients taking warfarin (Coumadin) should consume the same amounts of food high in vitamin K. Copyright © UWorld. All rights reserved.

Block Time Remaining: 00:01:29 TUTOR Test Id: 72490738 QId: 32349 (921666) 6 of 75 A A A The nurse is changing the dressing, injection caps, and IV tubing of a client who is receiving total parenteral nutrition through a right peripherally inserted central venous catheter. The nurse should implement what actions to prevent complications during this procedure? Select all that apply. 1. Instruct the client to hold the breath when changing the injection caps and tubing 2. Instruct the client to keep the head to the right side during the dressing change 3. Perform hand hygiene before and after the procedure 4. Place the client in the Trendelenburg position before the procedure 5. Wear sterile gloves and a surgical mask when changing the dressing Correct Answered correctly 22% Time: 35 seconds Updated: 04/08/2017 Explanation: Peripherally inserted central venous catheters (PICC) are commonly used for long-term antibiotic administration, chemotherapy treatments, and nutritional support with total parenteral nutrition (TPN). Complications related to the PICC are occlusion of the catheter, phlebitis, air embolism, and infection due to bacterial contamination. Prior to a central line dressing change, the nurse performs hand hygiene (Option 3). The central line dressing change is performed using sterile technique with the nurse wearing a mask to prevent contamination of the site with microorganisms or respiratory secretions (Option 5). During injection cap and tubing changes, the client is instructed to hold the breath (or perform the Valsalva maneuver) to prevent air from entering the line, traveling to the heart, and forming an air embolism (Option 1). (Option 2) When performing the dressing change, the client should be instructed to turn the head away from the PICC site to prevent potential contamination of the insertion site by microorganisms from the client's respiratory tract. (Option 4) During dressing, injection caps, and tubing changes, the client is placed in the supine position. If an air embolism is suspected, the client should be placed in the Trendelenburg position (head down) on the left side, causing any existing air to rise and become trapped in the right atrium. Educational objective: The central line dressing change is performed using a sterile technique that includes wearing sterile gloves and mask to prevent contamination of the site with microorganisms or respiratory secretions. During injection cap, tubing, and dressing changes, the client is instructed to turn the head away from the peripherally inserted central venous catheter site to prevent site contamination by the client's respiratory secretions. During cap/tubing changes, the client is instructed to hold the breath (or perform the Valsalva maneuver) to prevent air from entering the line, traveling to the heart, and forming an air embolism. Copyright © UWorld. All rights reserved.

Block Time Remaining: 00:02:04 TUTOR Test Id: 72490738 QId: 30603 (921666) 8 of 75 A A A A client with chronic kidney disease has a large pleural effusion. What findings characteristic of a pleural effusion does the nurse expect to assess? Select all that apply. 1. Bronchial breath sounds 2. Decreased fremitus 3. Diminished lung sounds 4. Hyperresonance on percussion 5. Wheezing Incorrect Correct answer 2,3 Answered correctly 9% Time: 35 seconds Updated: 04/26/2017 Explanation: A pleural effusion is an abnormal collection of fluid (>15-20 mL) in the pleural space between the parietal and visceral pleurae that prevents the lung from expanding fully. This results in decreased lung volume, atelectasis, and ineffective gas exchange. Clients commonly have dyspnea on exertion and non-productive cough. Examination shows diminished breath sounds, dullness to percussion, and decreased tactile fremitus. If the effusion is large, the trachea (mediastinum) is deviated to the opposite side. Palpable vibration felt on the chest wall is known as fremitus. Sound travels faster in solids (consolidation) than in an aerated lung, resulting in increased fremitus in pneumonia. The presence of egophony, bronchophony, or whispered pectoriloquy also suggests a consolidative process. Fluid or air outside the lung interrupts the transmission of sound, resulting in decreased fremitus in pleural effusion and pneumothorax (Option 2). (Option 1) Bronchial breath sounds are normally present over the trachea, and vesicular breath sounds are present over the lung parenchyma. However, in consolidation (lobar pneumonia), bronchial breath sounds are present over the lung parenchyma due to over-transmission of sound. Over-transmission is due to airway patency leading to the affected area. Breath sounds are diminished or absent over a pleural effusion or pneumothorax. (Option 4) Percussion shows hyperresonance in clients with pneumothorax and dullness in those with pleural effusions or pneumonia. (Option 5) Wheezing indicates an obstructive process (eg, asthma, chronic obstructive pulmonary disease) and is not typical in pleural effusion. Educational objective: A pleural effusion is an abnormal collection of fluid (>15-20 mL) in the pleural space between the parietal and visceral pleurae that prevents the lung from expanding fully, resulting in decreased lung volume, atelectasis, and ineffective gas exchange. Examination shows diminished breath sounds, dullness to percussion, and decreased tactile fremitus. Copyright © UWorld. All rights reserved.

Block Time Remaining: 00:01:59 TUTOR Test Id: 72419393 QId: 30258 (921666) 47 of 75 A A A A home health nurse visits a client with chronic obstructive pulmonary disease. The nurse teaches the client to use abdominal breathing to perform the "huff" cough technique to facilitate secretion removal. Place the steps in the correct order. All options must be used. Your Response/ Incorrect Response Correct Response Position upright Inhale through the nose using abdominal breathing and prolong the exhalation through pursed lips for 3 breaths Hold breath for 2-3 seconds following an inhalation while keeping the throat open, then exhale Deeply inhale and, while leaning forward, force the breath out gently using the abdominal muscles while making a "ha" sound (huff cough); repeat 2 more times Inhale deeply using abdominal breathing and give one forced huff cough Omitted Correct answer 5,4,2,1,3 Answered correctly 14% Time: 7 seconds Updated: 03/19/2017 Explanation: Coughing is an important lung defense mechanism. Clients with chronic obstructive pulmonary disease (COPD) have weakened muscles and narrowed airways that are prone to collapse when under increased pressure. They are therefore unable to generate the high pressure needed to create the explosive rush of air to cough effectively. The low-pressure "huff" cough, which uses a series of mini-coughs, is more effective in mobilizing and expectorating secretions in clients with COPD. When this technique is done correctly, there is less airway collapse, less energy and oxygen consumption, and greater secretion removal. The steps are as follows: Position upright - maximizes lung expansion and gas exchange Inhale through the nose using abdominal breathing and prolong the exhalation through pursed lips for 3 breaths - deflates excess air from lungs Hold breath for 2-3 seconds following an inhalation, keeping the throat open - opens glottic structures and prevents a high-pressure cough Deeply inhale and, while leaning forward, force the breath out gently using the abdominal muscles while making a "ha" sound (huff cough); repeat 2 more times (eg, "ha, ha, ha") - keeps airways open while moving secretions up and out of the lungs. Inhale deeply using abdominal breathing and give one forced huff cough - the last, increased force ("ha") usually results in mucus being expectorated from the larger airways. Educational objective: The normal cough reflex creates high pressure in the airways. Because the airways of clients with chronic obstructive pulmonary disease are prone to collapse with increased airway pressure, clients are taught the low-pressure cough technique (huff) to expectorate mucus. Copyright © UWorld. All rights reserved.

Block Time Remaining: 00:02:08 TUTOR Test Id: 72419393 QId: 30857 (921666) 48 of 75 A A A Place the nursing actions for performing a renal system physical assessment in the correct order. All options must be used. Your Response/ Incorrect Response Correct Response Advise client to empty the bladder completely Observe skin and contour of abdomen and lower back Auscultate the renal arteries in right and left upper quadrants Percuss and palpate both the right and left kidneys Document the assessment of renal system function Omitted Correct answer 1,4,2,5,3 Answered correctly 75% Time: 9 seconds Updated: 03/26/2017 Explanation: Examination of the urinary system requires an abdominal assessment. Therefore, assessment techniques must be reordered to optimize the examination. The steps for a renal system assessment are: Empty the bladder to avoid discomfort during percussion and palpation and to provide a clean-catch sample (if prescribed) (Option 1) Inspect the abdomen and lower back for color, contour, symmetry, distension, and movements (eg, visible peristalsis). Inspection is always done first during physical examination (Option 4). The nurse should auscultate immediately after inspection as percussion or palpation may increase bowel motility and interfere with sound transmission during auscultation. Listen for renal artery bruits in the right and left upper abdominal quadrants (Option 2). Percuss for kidney borders, costovertebral angle tenderness, and bladder distension. A dull percussion sound indicates solid structures or fluid-filled cavities (eg, distended bladder). Palpate for bladder distension, masses, and tenderness. A distended bladder may be palpated at any point from the symphysis pubis to the umbilicus and is felt as a firm, rounded organ. A normal kidney is not usually palpable; a palpable kidney may indicate hydronephrosis or polycystic kidney disease (Option 5). Document all renal assessment findings immediately after the examination (Option 3). Educational objective: Physical assessment of the renal system includes the techniques of inspection, auscultation, percussion, and palpation, in that order. Allow the client to empty the bladder before beginning the assessment and auscultate immediately after inspection as percussion or palpation may increase bowel motility and interfere with sound transmission during auscultation. Always document the findings. Copyright © UWorld. All rights reserved.

Block Time Remaining: 00:02:06 TUTOR Test Id: 72458431 QId: 31015 (921666) 7 of 10 A A A Which emergency department client would be allowed to leave against medical advice (AMA) after discussing the risks with the primary health care provider? 1. 5-year-old child who needs antibiotics for meningitis and was brought in by a Christian parent [14%] 2. Client with coffee-ground emesis from chronic use of high-dose aspirin [67%] 3. Client who is disoriented to time and place due to urinary tract infection [10%] 4. Client who tried to commit suicide by taking a handful of acetaminophen 1 hour ago after a fight with a significant other [7%] Incorrect Correct answer 2 Answered correctly 67% Time: 33 seconds Updated: 12/19/2016 Explanation: In order to leave AMA, a client must have the risks explained and be able to understand them. Issues that can make a client ineligible to leave AMA include danger to self or others, lack of consciousness, altered consciousness, mental illness, being under chemical influence, or a court decision. In this case, despite it not being in the client's best health interest, the client with GI bleeding can leave. (Option 1) Parents may not refuse life-, limb-, or organ-saving treatment on behalf of their minor children for religious reasons. They can only make that decision for themselves. If the parents are not willing to allow this crucial treatment, the hospital will have to go to court and take protective custody of the child. (Option 3) The client is not oriented x3 and is therefore not competent. There are 3 orientation categories (time, place, and person); orientation to time is lost first. In order to be oriented the client must answer all questions in each category correctly: Time day, month, year, season Place current location, city, state Person outside "self" (eg, asking the client "Who is the president?") (Option 4) Possible suicidal ideation is a criterion that prevents the client from being allowed to leave. The danger with acetaminophen (Tylenol) overdose is that the client initially may not feel any adverse effects. Major symptoms or signs, such as right upper quadrant pain and elevated liver enzymes, usually do not start until 24 hours after ingestion. Treatment includes activated charcoal and acetylcysteine (Mucomyst). Educational objective: A client must be competent to sign AMA, but can leave AMA if an informed refusal is given after an explanation of the risks. A client with suicidal ideation or altered consciousness is not competent. Parents may not refuse required limb- or life-saving treatment for their minor child based on their own religious beliefs. Copyright © UWorld. All rights reserved.

Block Time Remaining: 00:02:23 TUTOR Test Id: 72458431 QId: 31641 (921666) 8 of 10 A A A A nurse is preparing a client for below-the-knee amputation surgery. Which actions should the nurse complete? Select all that apply. 1. Administer a preoperative IV antibiotic 2. Ensure that the correct limb to be amputated is marked appropriately 3. Place a red "no known allergies" bracelet on the client 4. Place operative permits in the client's chart 5. Replace the current 20G IV catheter with an 18G IV catheter Omitted Correct answer 1,2,4 Answered correctly 22% Time: 17 seconds Updated: 05/15/2017 Explanation: The Joint Commission's National Patient Safety goals include preventing mistakes in surgery. The goals state, "Make sure that the correct surgery is done on the correct patient and at the correct place on the patient's body." The correct surgical site is marked preoperatively with a permanent marker (Option 2). A measure by The Centers for Medicare & Medicaid and the Joint Commission states that a prophylactic antibiotic is to be given within 1 hour prior to surgical incision (Option 1). It is also a standard of practice for the nurse to ensure that appropriate operative permits have been signed and placed on the client's chart (Option 4). (Option 3) A red allergy band should be placed on the client only if the client has an allergy. (Option 5) If the client does not have an IV line started, an 18G would be preferable. However, if the client has a functioning IV line present, then a 20G is most likely acceptable. Blood can be transfused through a 20G if necessary. Educational objective: When preparing a client for surgery, the nurse needs to ensure that operative permits are signed and on the chart, an allergy band is placed if allergies are present, the operative site is marked on a limb, and IV prophylactic antibiotics are infused within 1 hour prior to surgery. Copyright © UWorld. All rights reserved.

Block Time Remaining: 00:02:45 TUTOR Test Id: 72419393 QId: 31750 (921666) 49 of 75 A A A A student nurse performs the morning assessment and obtains a urine specimen from a client with methicillin-resistant Staphylococcus aureus (MRSA) who is on contact precautions. The registered nurse intervenes when the student performs which action? 1. Cleans the disposable stethoscope with chlorhexidine solution before use with another client [50%] 2. Removes the urine specimen cup from the room in a sealed biohazard bag [4%] 3. Scrubs the Foley catheter collection port with alcohol for 15 seconds before withdrawing a urine specimen [9%] 4. Uses an alcohol-based hand antiseptic solution after removing gloves [36%] Correct Answered correctly 50% Time: 37 seconds Updated: 01/21/2017 Explanation: Clients with a health care-associated infection, such as methicillin-resistant Staphylococcus aureus, are placed on contact precautions to limit the transmission of microorganisms. In addition to standard precautions implemented for all clients, contact precautions include use of a gown and gloves and equipment that is designated for use with a single client only (eg, stethoscopes, blood pressure cuffs, glucometer). Disposable stethoscopes and other dedicated equipment should be kept in the room after each client use and then disinfected or discarded after no longer needed (eg, upon discharge) (Option 1). (Option 2) The urine specimen should be placed in a leak-proof specimen cup and then in a sealed biohazard bag before transport to the laboratory. (Option 3) To prevent contaminating the urine specimen or introducing bacteria into the client's urinary tract, the nurse should scrub the Foley collection port with alcohol or chlorhexidine solution for 15 seconds before withdrawing a specimen. (Option 4) Hand hygiene with an alcohol-based antiseptic solution is appropriate except when caring for clients with Clostridium difficile - for these clients, the nurse should instead perform hand hygiene with soap and water to remove spores from hands. Educational objective: Contact precautions require use of a gown and gloves, and equipment that is designated for single client use. Hand hygiene using soap and water is necessary when caring for a client with Clostridium difficile. For all clients, the nurse should practice hand hygiene, use aseptic technique when collecting a urine specimen from a Foley catheter, and transport specimens in sealed biohazard bags. Copyright © UWorld. All rights reserved.

Block Time Remaining: 00:03:14 TUTOR Test Id: 72419393 QId: 30511 (921666) 52 of 75 A A A Which of the following drug administrations should be reported as a practice error? Select all that apply. 1. Cephalexin administered; client has history of anaphylaxis from penicillin 2. Hydromorphone 2 mg administered; client reports pruritus 3. Immunization for 3-month-old administered in ventrogluteal site 4. Oral niacin (nicotinic acid) administered; client has facial flushing 5. Warfarin administered; client at 12 weeks gestation Omitted Correct answer 1,3,5 Answered correctly 31% Time: 12 seconds Updated: 12/17/2016 Explanation: Warfarin (Coumadin) is generally contraindicated in pregnancy. Warfarin is a teratogen and exposure during early pregnancy can result in fetal malformations (warfarin embryopathy). It crosses the placenta, resulting in fetal anticoagulation; dangerous fetal bleeding, including intracranial hemorrhage, can occur. As a result, a client on warfarin is taught to use effective contraception (Option 5). For children age <7 months, the site for immunizations is the anterolateral thigh (vastus lateralis). The gluteus medius muscle (muscle injected with a ventrogluteal injection) is developed through crawling and walking. The muscles are not developed enough at this age to be used as an acceptable site (Option 3). History of penicillin hypersensitivity should be determined prior to administration. Clients who are truly allergic to penicillins (eg, anaphylaxis) have an increased risk of allergy to other beta-lactam antibiotics. The incidence of cross-reactivity is 1%-4% (Option 1). (Option 2) Pruritus (itching) is a known side effect of narcotic administration, particularly if the client is opioid naïve. It does not represent true allergy and is often treated with an antihistamine. Nausea is also quite common when opioid therapy is initiated, but clients quickly develop tolerance. (Option 4) Niacin (nicotinic acid or B3) is used in large doses for lipid-lowering properties. In large doses, it may produce cutaneous vessel vasodilation. The resulting warm sensation within the first 2 hours after oral ingestion is uncomfortable but harmless. It may last for several hours. Effects usually subside as therapy continues. Educational objective: Do not administer warfarin if the client is pregnant. Intramuscular injections are given in the vastus lateralis to children age <7 months. Penicillins and cephalosporins can have a cross-sensitivity response. Narcotic-induced pruritus is not a true allergy. Copyright © UWorld. All rights reserved.

Block Time Remaining: 00:03:27 TUTOR Test Id: 72490738 QId: 30834 (921666) 11 of 75 A A A An 86-year-old client with diabetes and gastroparesis has had repeated hospitalizations for aspiration pneumonia following a stroke and is now hospitalized with altered level of consciousness. Which nursing action is most appropriate to decrease the client's risk for developing aspiration pneumonia? 1. Assessing client's breath sounds every 2 hours [10%] 2. Placing client in the side lying position in bed [43%] 3. Titrating client's oxygen to maintain saturation ≥93% [3%] 4. Turning and repositioning the client every 2 hours [41%] Incorrect Correct answer 2 Answered correctly 43% Time: 40 seconds Updated: 02/12/2017 Explanation: Clients with decreased level of consciousness may not be alert enough to protect their own airways; therefore, a side lying or lateral position is used to decrease the risk for developing aspiration pneumonia. If vomiting were to occur, this position promotes drainage of emesis out of the mouth instead of down the pharynx where it can be aspirated into the lungs. Maintaining an upright position during and after meals will allow remaining food particles to clear from the pharynx. (Option 1) Assessing breath sounds can help identify the presence of pneumonia (eg, diminished, bronchial, crackles) but does not prevent the client from aspirating. (Option 3) Titrating oxygen to maintain saturation ≥93% provides for adequate oxygenation but does not prevent the client from aspirating. (Option 4) Turning and repositioning every 2 hours helps to prevent stasis of secretions in the lungs but does not prevent the client from aspirating. Educational objective: Placing a client with a decreased level of consciousness in a position that uses gravity to help drain oropharyngeal and gastric secretions can be effective in preventing aspiration and reducing the risk for aspiration pneumonia. Copyright © UWorld. All rights reserved.

Block Time Remaining: 00:04:22 TUTOR Test Id: 72490738 QId: 34703 (921666) 12 of 75 A A A Exhibit The nurse auscultates the lung sounds of a client with shortness of breath. Based on the sounds heard, which action would the nurse anticipate? Listen to the audio clip. 1. Administer albuterol via nebulizer [23%] 2. Administer furosemide IV push [59%] 3. Instruct to use pursed-lip breathing [6%] 4. Prepare for chest tube insertion [10%] Correct Answered correctly 59% Time: 55 seconds Updated: 03/09/2017 Explanation: Coarse crackles (loud, low-pitched bubbling) are heard primarily during inspiration and are not cleared by coughing. The sound is similar to that of Velcro being pulled apart. Coarse crackles may be confused with fine crackles (eg, atelectasis), which have a high-pitched popping sound. Coarse crackles are present when fluid or mucus collects in the lower respiratory tract (eg, pulmonary edema, pulmonary fibrosis). In heart failure, the left ventricle fails to eject enough blood, causing increased pressure in the pulmonary vasculature. As a result, fluid leaks into the alveoli (pulmonary edema). Diuretics (eg, furosemide) treat pulmonary edema by increasing fluid excretion by the kidneys (Option 2). (Option 1) Clients with asthma or chronic obstructive pulmonary disease (eg, emphysema) develop wheezing due to bronchospasm. Bronchodilators (eg, albuterol, ipratropium) are indicated for these clients. (Option 3) Emphysema is a chronic hyperinflation of the alveoli. Clients with emphysema are taught the pursed-lip breathing technique to prevent alveolar collapse during exhalation. Emphysema causes diminished lung sounds, prolonged expiration, and wheezing. (Option 4) Chest tubes are inserted into the pleural space to remove trapped air (eg, pneumothorax) or fluids (eg, hemothorax, pleural effusion). Lung sounds are diminished or absent when lung tissue is compressed by air or fluids in the pleural space. Educational objective: Auscultation of coarse crackles indicates the presence of fluid or mucus in the lower respiratory tract. This may indicate pulmonary edema or pulmonary fibrosis. Diuretic administration (eg, furosemide) is used to treat pulmonary edema. Copyright © UWorld. All rights reserved.

Block Time Remaining: 00:04:34 TUTOR Test Id: 72490738 QId: 30818 (921666) 13 of 75 A A A A previously healthy client is hospitalized with left lower lobe (LLL) bacterial pneumonia. The nurse assesses chest pain with inspiration, productive cough of thick rusty sputum, and LLL fine inspiratory crackles and low-pitched expiratory wheezing. Which of the medications that the health care provider prescribes should the nurse question? 1. Furosemide 20 mg intravenous (IV) push every day [37%] 2. Guaifenesin ER 600 mg PO every 12 hours [27%] 3. Ibuprofen 600 mg PO every 6 hours PRN [24%] 4. Levofloxacin 500 mg IV every day [10%] Correct Answered correctly 37% Time: 12 seconds Updated: 12/11/2016 Explanation: Medications commonly prescribed to treat bacterial pneumonia include antibiotics, expectorants, mucolytics, antipyretics, analgesics, and anti-inflammatories. Furosemide (Lasix) is a diuretic and is not appropriate for treating the fine crackles associated with pneumonia. The crackles result from alveolar filling and atelectasis, not from heart failure or pulmonary edema. (Option 2) Extended-release guaifenesin (Mucinex) is an expectorant medication that increases respiratory fluids and thins secretions to facilitate mobilization and expectoration. It is appropriate to prescribe in clients who have pneumonia with a productive cough and low-pitched wheezing (rhonchi). (Option 3) Ibuprofen is an anti-inflammatory medication administered to relieve pleuritic chest pain associated with pneumonia and is appropriate to prescribe in clients with pneumonia. (Option 4) Recommended antibiotic therapy for hospitalized clients with community-acquired streptococcal pneumonia includes monotherapy with a fluoroquinolone or combined therapy with a macrolide (eg, azithromycin [Zithromax]) plus a beta-lactam (eg, a cephalosporin [ceftriaxone]) for 1-2 days before transitioning to oral antibiotics. Educational objective: Medications prescribed to treat hospitalized clients with community-acquired bacterial pneumonia include IV antibiotics, expectorants, mucolytics, antipyretics, analgesics, and anti-inflammatories. Copyright © UWorld. All rights reserved.

Block Time Remaining: 00:04:35 TUTOR Test Id: 72490738 QId: 30253 (921666) 14 of 75 A A A The nurse is caring for a 72-year-old client 1 day postoperative colectomy. The nurse assesses an increased work of breathing, diminished breath sounds at the bases with fine inspiratory crackles, respirations 12/min and shallow, and pulse oximetry 96% on 2 L oxygen. There is no jugular venous distension or peripheral edema. Pain is regulated with client-controlled morphine. Which prescription does the nurse anticipate? 1. Bolus dose of IV morphine [2%] 2. Incentive spirometer [56%] 3. IV furosemide [29%] 4. Non-rebreather mask [11%] Omitted Correct answer 2 Answered correctly 56% Time: 1 seconds Updated: 04/18/2017 Explanation: During the initial postoperative period, a client needs respiratory interventions to keep the lungs expanded and prevent atelectasis and postoperative pneumonia. Atelectasis is maximal during the second postoperative night. Clients can be asymptomatic or have increased work of breathing, hypoxia, and basal crackles. Postoperative pain, opioid respiratory depression, limited mobility, and reluctance to take a deep breath due to anticipated pain contribute to postoperative atelectasis. The elderly and postoperative abdominal and thoracic surgery clients are at increased risk for atelectasis. The incentive spirometer encourages the client to breathe deeply with maximum inspiration. This action improves ventilation and oxygenation by expanding the lungs, encourages coughing, and prevents or improves atelectasis. It is the most appropriate prescription for this client. (Option 1) In a client whose pain is regulated with client-controlled analgesia (eg, morphine), administration of a bolus dose is not indicated and may increase the risk for respiratory depression. (Option 3) Fine crackles in the lungs usually indicate atelectasis. The presence of coarse crackles, elevated jugular venous distension, and peripheral edema usually indicates volume overload (fluid in the alveoli). In addition, clients with fluid overload breathe at a rapid rate (tachypnea) rather than take slow, shallow breaths. IV furosemide (Lasix) is an appropriate intervention for volume overload but not for atelectasis. (Option 4) As-needed oxygen may be prescribed postoperatively, especially with blood loss. A non-rebreather mask, which has 100% oxygen, is not indicated in this client as the pulse oximeter shows 96% saturation, indicating adequate oxygenation. Educational objective: The incentive spirometer is a handheld, inexpensive breathing device. It encourages the client to breathe deeply with maximum inspiration, which improves ventilation and oxygenation and encourages coughing. The incentive spirometer is used to prevent or improve atelectasis in clients who are postoperative, have respiratory problems (eg, pneumonia), or have experienced trauma. Copyright © UWorld. All rights reserved.

Block Time Remaining: 00:03:45 TUTOR Test Id: 72419393 QId: 30918 (921666) 56 of 75 A A A The nurse dons personal protective equipment (PPE) before providing care for a client in airborne transmission-based precautions. Place the steps for donning PPE in the appropriate sequence. All options must be used. Your Response/ Incorrect Response Correct Response Hand hygiene Gown Mask or respirator Goggles or face shield Gloves Omitted Correct answer 4,3,5,2,1 Answered correctly 35% Time: 3 seconds Updated: 05/20/2017 Explanation: PPE for the health care worker protects the mucous membranes, airways, skin, and clothing from contact with potentially infectious agents. The category of transmission-based precautions (eg, contact, droplet, airborne) required determines the type of PPE that the health care worker will wear. The exact procedure for donning and removing PPE varies with the level of precautions required. Guidelines are provided by the Centers for Disease Control and Prevention (CDC) and by institution policy and procedure. The sequence for donning PPE includes: Hand hygiene Gown - fully cover torso from neck to knees, arms to end of wrists, and wrap around back; fasten in back of neck and waist Mask or respirator - secure ties or elastic bands at middle of head and neck; fit flexible band to nose bridge; fit snugly to face and below chin; fit-check respirator Goggles or face shield - place over face and eyes and adjust fit; may be combined with mask (visor) Gloves - don and extend to cover wrist of isolation gown Educational objective: The CDC suggests the following sequence for donning PPE: hand hygiene, gown, mask or respirator, goggles or face shield, and gloves. Copyright © UWorld. All rights reserved.

Block Time Remaining: 00:04:53 TUTOR Test Id: 72419393 QId: 30160 (921666) 57 of 75 A A A The nurse auscultates rhonchi in a client with a tracheostomy tube and performs endotracheal suctioning to clear the secretions. Which nursing interventions are most appropriate to limit the risks associated with suctioning? Select all that apply. 1. Apply suction only while withdrawing catheter 2. Instill sterile normal saline to loosen secretions 3. Limit aspiration time to 10 seconds with each suction pass 4. Maintain sterile technique throughout suctioning procedure 5. Pre-oxygenate with 100% oxygen Omitted Correct answer 1,3,4,5 Answered correctly 52% Time: 68 seconds Updated: 05/20/2017 Explanation: Endotracheal suctioning is performed to maintain a patent airway if a client cannot mobilize secretions independently. Inserting a catheter into the airway compromises the sterility of the lower airway and increases the risk for infection. Suctioning removes oxygen in addition to secretions, placing the client at risk for hypoxemia. High suction levels or the contact of the catheter with the trachea can cause trauma, such as barotrauma, damage to tracheal mucosa, and microatelectasis. In order to decrease the occurrence of these complications: Use strict sterile technique throughout suctioning process. Pre-oxygenate with 100% oxygen (hyperoxygenation) for 3-4 breaths. Aspirate during withdrawal of catheter only, limiting each suction pass to 10 seconds. Allow client 4-5 recovery breaths between suction passes to replenish oxygen. (Option 2) Instilling 5-10 mL of sterile normal saline solution (NSS) is thought to help loosen thick secretions and stimulate cough. Although saline lavage is a common practice in some facilities, the installation of NSS into the airway prior to suctioning is not recommended. It can dislodge bacteria, causing increased bacterial colonization, and can stimulate excessive coughing. Educational objective: Endotracheal suctioning increases risks for pulmonary infection, hypoxemia, microatelectasis, and mucosal tissue damage. To decrease these suctioning-associated risks, use sterile technique, preoxygenate, apply suction only while withdrawing catheter, and limit aspiration time. Avoid the use of saline lavage and frequent suction passes without adequate rest between. Copyright © UWorld. All rights reserved.

Block Time Remaining: 00:05:03 TUTOR Test Id: 72419393 QId: 30755 (921666) 60 of 75 A A A The nurse collects a sputum specimen from a client with pneumonia. Which directions should the nurse give to the client before collecting the specimen? Select all that apply. 1. "Cough deeply and expectorate into the clean specimen container." 2. "Cough deeply and expectorate into the sterile specimen container." 3. "Inhale deeply several times." 4. "Rinse your mouth with mouthwash." 5. "Rinse your mouth with water." 6. "Sit upright or on the side of the bed." Omitted Correct answer 2,3,5,6 Answered correctly 38% Time: 3 seconds Updated: 03/12/2017 Explanation: Sputum collection is prescribed to identify respiratory pathogens (eg, in the setting of bacterial pneumonias or tuberculosis). Collection should be done in the morning, as secretions accumulate overnight. A nebulizer treatment may be prescribed to help mobilize secretions. To collect a sputum specimen, the nurse should instruct the client to: Rinse the mouth with water to reduce specimen contamination by oral flora Sit on the side of the bed, if possible, or in a high or semi-Fowler position to allow maximum lung ventilation and expansion Inhale deeply several times to provide enough air to force secretions from the lower airways to the pharynx Cough deeply to raise enough sputum (4-10 mL), and expectorate into the sterile specimen container The nurse should immediately close and label the specimen container as this will prevent contamination or transmission of microorganisms and assure proper client information. The specimen and requisition are transported to the laboratory per policy; some specimens must be sent immediately, and others may be refrigerated. The nurse should then provide oral care for the client and document pertinent information (eg, sputum characteristics, tolerance of procedure). (Option 1) Sputum collection is a sterile procedure. (Option 4) Mouthwash can alter flora in the sample and affect the results. Educational objective: Collection of a sputum specimen by expectoration is a sterile procedure that requires the client to be able to breathe deeply and cough effectively. The client should be instructed to rinse the mouth with water, sit upright, inhale deeply several times, and cough prior to expectorating. Copyright © UWorld. All rights reserved.

Block Time Remaining: 00:05:09 TUTOR Test Id: 72419393 QId: 34107 (921666) 61 of 75 A A A A client is receiving normal saline 75 mL/hr and morphine sulfate via patient-controlled analgesia (PCA) bolus doses. The PCA and normal saline tubing are connected at the "Y" site. The nurse reviews a prescription from the health care provider to discontinue the normal saline. What is the most appropriate nursing action? 1. Change the rate of the normal saline to 10 mL/hr [3%] 2. Clarify the prescription with the health care provider [33%] 3. Flush the IV with normal saline and then convert it to a saline lock [24%] 4. Turn off the normal saline and disconnect it from the "Y" site [37%] Omitted Correct answer 2 Answered correctly 33% Time: 6 seconds Updated: 03/01/2017 Explanation: Patient-controlled analgesia (PCA) delivers a set amount of IV analgesic each time the client presses the administration button. With many PCA pumps, a continuous IV solution (eg, normal saline) is required to keep the vein open and flush the PCA medication through the line so that the boluses reach the client. Many facilities have a policy regarding IV fluid for use with PCA; however, a prescription may be required. To ensure uninterrupted delivery of this client's PCA, the nurse should contact the health care provider to clarify the prescription to discontinue the normal saline. (Option 1) A "keep-vein-open" rate (eg, 5-20 mL/hr) may be appropriate; however, a prescription is necessary before the nurse can implement this. (Option 3) This client is still receiving PCA, so it is inappropriate to convert the IV to a saline lock. In addition, this does not address the need to flush the PCA medication through the line. (Option 4) Continuous IV fluids may be required to deliver the PCA boluses; before discontinuing the normal saline, the nurse should receive clarification from the health care provider. Educational objective: Continuous IV fluids are often necessary with use of a patient-controlled analgesia (PCA) pump; the fluids maintain an open vein and provide a vehicle for PCA delivery. Copyright © UWorld. All rights reserved.

Block Time Remaining: 00:04:49 TUTOR Test Id: 72490738 QId: 30593 (921666) 15 of 75 A A A The client with malignant left pleural effusion undergoes a thoracentesis and 900 mL of excess pleural fluid is removed. Which of these manifestations, if noted on the post-procedure assessment, should the nurse report to the health care provider immediately? 1. Asymmetrical chest expansion and decreased breath sounds on the left [82%] 2. Blood pressure 100/65 mm Hg (mean arterial pressure 77 mm Hg) [7%] 3. Client complains of 6/10 pain at the needle insertion site [1%] 4. Respiratory rate 24/min, pulse oximetry 94% on oxygen 2 L/min [8%] Incorrect Correct answer 1 Answered correctly 82% Time: 14 seconds Updated: 02/23/2017 Explanation: A thoracentesis involves the insertion of a large-bore needle through an intercostal space to remove excess fluid. The procedure has the following advantages: Diagnostic - analysis of fluid to diagnose the underlying cause of the pleural effusion (eg, infection, malignancy, heart failure), including cytology, bacterial culture, and related testing Therapeutic - removal of excess fluid (>1 L) improves dyspnea and client comfort Complications from insertion of the needle and removal of large amounts of fluid include iatrogenic pneumothorax, hemothorax, pulmonary edema, and infection. After the procedure, the nurse assesses for pain and difficulty breathing; monitors vital signs and oxygen saturation; and observes for changes in respiratory rate and depth, symmetry of chest expansion, and breath sounds. If any abnormalities are noted, a post-procedure chest x-ray is obtained. Decreased chest expansion with inspiration and breath sounds on the affected side, tachypnea, tracheal deviation to the opposite side, and hyperresonance (air) on the affected side are manifestations of a pneumothorax. These should be reported immediately. (Option 2) Hypotension, pulmonary edema, and tachycardia can occur as the result of removal of large amounts of pleural fluid (>1.5 L). This client's blood pressure is adequate (mean arterial pressure 77 mm Hg), and the nurse should continue to monitor. However, this blood pressure does not need to be reported immediately. (Option 3) Mild to moderate pain is common after the procedure. It does not need to be reported immediately. (Option 4) Difficulty breathing, tachypnea, and hypoxemia are pulmonary complications that can occur after thoracentesis. Saturation (94%) and respiratory rate (24/min) are adequate and do not need to be reported immediately. Educational objective: Complications of thoracentesis include iatrogenic pneumothorax, hemothorax, and infection. Post-procedure, the nurse assesses for pain and difficulty breathing; monitors vital signs and oxygen saturation; and observes for changes in respiratory rate and depth, symmetry of chest expansion, and breath sounds. Copyright © UWorld. All rights reserved.

Block Time Remaining: 00:05:29 TUTOR Test Id: 72490738 QId: 34609 (921666) 17 of 75 A A A The nurse is caring for a client admitted with incomplete fractures of right ribs 5-7. The nurse notes shallow respirations, and the client reports deep pain on inspiration. What is the priority at this time? 1. Administer prescribed IV morphine [34%] 2. Facilitate hourly client use of incentive spirometry [6%] 3. Instruct client on gently splinting injury during coughing [19%] 4. Notify the health care provider immediately [39%] Incorrect Correct answer 1 Answered correctly 34% Time: 21 seconds Updated: 02/05/2017 Explanation: Rib fractures are often the result of blunt thoracic trauma (eg, motor vehicle collision). In the absence of significant internal injuries (eg, pneumothorax, pulmonary contusion, spleen laceration), interventions focus on pain management and pulmonary hygiene techniques (eg, coughing, deep breathing, incentive spirometry). Breaths may become shallow as the client experiences pain with inspiration, which can result in a buildup of secretions, atelectasis, and pneumonia. The nurse should ensure adequate pain control prior to encouraging pulmonary hygiene techniques (Option 1). (Options 2 and 3) Interventions focused on removing secretions to improve gas exchange (eg, ambulation, coughing, incentive spirometry) are appropriate after the client's pain is controlled. (Option 4) Rib fractures are very painful. Shallow breathing and reports of pain on inspiration are expected findings that do not require immediate notification of the health care provider. Educational objective: Client management for rib fractures focuses on pain control followed by pulmonary hygiene techniques (eg, coughing, deep breathing, incentive spirometry). Without adequate pain control, breathing can become shallow, which may lead to buildup of secretions, atelectasis, and pneumonia. Copyright © UWorld. All rights reserved.

Block Time Remaining: 00:05:37 TUTOR Test Id: 72490738 QId: 32839 (921666) 18 of 75 A A A The medical surgical nurse cares for a client who had a mediastinal tumor removed 2 days ago and reports difficulty breathing. The client becomes confused and restless, and respirations are 30/min. What is the nurse's next action? 1. Administer a dose of prescribed prn anti-anxiety medication [8%] 2. Call the health care provider who performed the surgery [8%] 3. Call the rapid response team [42%] 4. Place the client in the left lateral recovery position [40%] Omitted Correct answer 3 Answered correctly 42% Time: 8 seconds Updated: 03/30/2017 Explanation: The rapid response team (RRT) consists of a group of health care providers who bring critical care expertise to the bedside of clients demonstrating early signs of deterioration such as dyspnea, confusion, and restlessness. This team differs from the "Code" team that is called when a client stops breathing or goes into cardiac arrest. Any health care worker can call the RRT. (Option 1) The client's restlessness and confusion are likely secondary to low oxygenation. Anxiety will cause hyperventilation, which will only exacerbate the situation. However, administering anti-anxiety medication is not the priority over obtaining help quickly. In addition, the client's oxygenation could deteriorate depending on the prescribed anti-anxiety medication, which could depress respirations. (Option 2) The health care provider who performed the surgery must be notified of the client's deteriorating condition; however, this should be done after calling the RRT. Stabilizing the client is the priority. (Option 4) The recovery position is used as a first aid measure for an unconscious client who is still breathing. The client is placed on the left or right side in a three-fourths prone position with the top leg flexed. This position maintains the airway and ensures that the client does not choke on vomit. Educational objective: When a client is demonstrating clinical deterioration, the nurse's priority is to prevent full respiratory or cardiac arrest by calling the rapid response team. Copyright © UWorld. All rights reserved.

Block Time Remaining: 00:05:40 TUTOR Test Id: 72490738 QId: 30568 (921666) 19 of 75 A A A Exhibit The nurse admits a client to the medical unit. Based on the admission data, which priority nursing diagnosis does the nurse identify? Click on the exhibit button for additional information. 1. Activity intolerance [1%] 2. Impaired gas exchange [39%] 3. Ineffective airway clearance [10%] 4. Ineffective breathing pattern [48%] Omitted Correct answer 4 Answered correctly 48% Time: 3 seconds Updated: 01/30/2017 Explanation: Ineffective breathing pattern is a change in rate, rhythm, timing, depth, or chest and abdominal wall excursion during inspiration and/or expiration that affects a client's ability to maintain adequate ventilation. Characteristic manifestations include dyspnea, tachypnea, bradypnea, altered chest excursion, respiratory depth (shallow) and rhythm changes (2-3-second periods of apnea), use of accessory muscles, paradoxical breathing, orthopnea, and use of the tripod position. (Option 1) Activity intolerance is insufficient physiological or psychological energy to complete required or desired daily activities. Common characteristics include abnormal heart rate, blood pressure, or respiratory response to activity; exertional discomfort or dyspnea; reports of fatigue or weakness; and inability to complete activities of daily living. (Option 2) Impaired gas exchange is an alteration in the normal exchange of oxygen and carbon dioxide at the alveolar capillary membrane. Characteristic manifestations include restlessness, irritability, hypoxemia, hypoxia, confusion, somnolence, dyspnea, abnormal arterial blood gases, tachycardia, and pale and dusky skin (cyanosis). (Option 3) Ineffective airway clearance is the inability to clear secretions or obstructions from the respiratory tract to maintain a clear airway. Common characteristic manifestations include abnormal breath sounds (eg, crackles, wheezes, rhonchi), dyspnea, ineffective cough, excessive secretions, and orthopnea. Educational objective: Ineffective breathing pattern is a change in rate, rhythm, timing, depth, or chest and abdominal wall excursion during inspiration and/or expiration that affects a client's ability to maintain adequate ventilation. Copyright © UWorld. All rights reserved.

Block Time Remaining: 00:05:15 TUTOR Test Id: 72419393 QId: 32214 (921666) 64 of 75 A A A What nursing care related to peripherally inserted IV catheters can reduce the incidence of catheter-related infections? Select all that apply. 1. After insertion, apply sterile tape over the catheter hub 2. Clean ports with 70% alcohol prior to accessing the catheter system 3. Prior to insertion, apply chlorhexidine in a back-and-forth motion using friction 4. Prior to insertion, shave excess hair over selected site 5. Replace or remove the catheter every 48 hours Omitted Correct answer 1,2,3 Answered correctly 10% Time: 2 seconds Updated: 01/24/2017 Explanation: The nurse should select an IV catheter site on an upper extremity, preferably the hand or forearm. To reduce the incidence of catheter-related infections, the selected site should be cleaned with antiseptic solution using friction (preferably chlorhexidine, using a back-and-forth motion) and then allowed to air dry completely (Option 3). Chlorhexidine is preferred over povidone-iodine as it achieves an antimicrobial effect within 30 seconds whereas povidone-iodine takes ≥2 minutes. After insertion, the catheter hub should be taped down with a narrow strip of sterile tape to prevent accidental removal or excessive back-and-forth motion, which can introduce microorganisms into the vein (Option 1). The Centers for Disease Control and Prevention recommends cleaning access ports with 70% alcohol (Option 2). (Option 4) Excessive hair may be clipped but never shaved as shaving may cause microabrasions and possible portals of entry for microorganisms. (Option 5) Peripheral IV catheters should not be removed or replaced more frequently than every 72-96 hours unless signs of phlebitis or other complications (eg, infiltration, infection) occur. Educational objective: To reduce the incidence of catheter-related infections from peripheral IV catheters, the nurse should clip excessive hair if needed, clean the site with chlorhexidine in a back-and-forth motion using friction, and allow it to dry completely. The catheter hub should be secured with sterile tape, and access ports should be cleaned with 70% alcohol prior to use. Copyright © UWorld. All rights reserved.

Block Time Remaining: 00:05:43 TUTOR Test Id: 72419393 QId: 31576 (921666) 66 of 75 A A A Which client finding is most important for the nurse to follow up? 1. Client with distinct liver edge even with right costal margin [13%] 2. Client with pyelonephritis who has costovertebral angle tenderness [12%] 3. Client with rash that has purplish blotches that do not blanch [31%] 4. Client with spinal injury whose toes point downward with the Babinski test [42%] Incorrect Correct answer 3 Answered correctly 31% Time: 22 seconds Updated: 01/07/2017 Explanation: Purpura refers to reddish-purple blotches on the skin that do not blanch with pressure due to bleeding underneath the skin. Further assessment must be done to evaluate for a potentially serious etiology, such as blood dyscrasia. (Option 1) The normal finding is a soft, distinct liver edge that is even with the bottom of the right rib cage or right costal margin. An abnormal finding would be a boggy liver edge below the rib cage (hepatomegaly). (Option 2) Kidney inflammation (pyelonephritis) results in positive costovertebral angle (CVA) tenderness tenderness in the back/flank. It is an expected finding that is elicited when the examiner places the hand over the client's lower back and places the other hand on top and makes a fist to gently "thump" or tap the area. (Option 4) The Babinski sign can indicate an upper motor neuron lesion from damage to the corticospinal tract. A normal finding for an adult is for the toes to point downward. Educational objective: Purpura refers to purplish blotches indicating bleeding underneath the skin; it is a significant finding that requires further assessment. Copyright © UWorld. All rights reserved.

Block Time Remaining: 00:05:47 TUTOR Test Id: 72490738 QId: 33387 (921666) 20 of 75 A A A The nurse plans discharge teaching for a client with active herpes lesions who has a new prescription for oral acyclovir and topical lidocaine. What information will the nurse include in the teaching plan? 1. Adhesive bandaging should remain on the lesions to prevent virus shedding [4%] 2. Blood tests will be drawn to ensure the virus is eradicated [3%] 3. Condoms should be used during intercourse until the lesions are healed [25%] 4. Gloves should be used to apply the medication to the lesions [67%] Omitted Correct answer 4 Answered correctly 67% Time: 7 seconds Updated: 04/24/2017 Explanation: Acyclovir (Zovirax), famciclovir, and valacyclovir are commonly used to treat herpes infection as they shorten the duration and severity of active lesions. Genital herpes is a sexually transmitted infection caused by a herpes simplex virus and is highly contagious, especially when lesions are active. It remains dormant in the body even when active lesions are healed. There is no cure for genital herpes; treatment is aimed at relieving symptoms and preventing the spread of infection. Touching the lesions and then rubbing or scratching another part of the body can spread the infection. Therefore, gloves should be used when applying topical antiviral or analgesic (eg, lidocaine) medications. (Option 1) Herpetic lesions should be kept clean and dry. They can be cleansed with warm water and soap or other solutions. Bandages are not applied to the lesions. (Option 2) There is no cure for herpes infection. Genital herpes often leads to local recurrence. Some clients may need long-term suppressive therapy. (Option 3) During periods of active lesions, abstinence from sexual intercourse is indicated. Condoms should be used during periods of dormancy due to viral shedding. Educational objective: Clients should be taught to use gloves when applying topical medication to herpes lesions to avoid the spread of infection. There is no cure for genital herpes infection; recurrences are common. Complete abstinence from sexual intercourse is recommended when active lesions are present as barrier contraception alone is insufficient to prevent the spread of infection. Copyright © UWorld. All rights reserved.

Block Time Remaining: 00:05:49 TUTOR Test Id: 72490738 QId: 30242 (921666) 21 of 75 A A A The nurse teaches safety precautions of home oxygen use to a client with emphysema being discharged with a nasal cannula and portable oxygen tank. Which client statement indicates the need for further teaching? Select all that apply. 1. "I can apply Vaseline to my nose when my nostrils feel dry from the oxygen." 2. "I can cook on my gas stove as long as I have a fire extinguisher in the kitchen." 3. "I can increase the liter flow from 2 to 6 liters a minute whenever I feel short of breath." 4. "I should not polish my nails when using my oxygen." 5. "I should not use a wool blanket on my bed." Omitted Correct answer 1,2,3 Answered correctly 21% Time: 2 seconds Updated: 04/26/2017 Explanation: Oxygen is a colorless, odorless gas that supports combustion and makes up about 21% of the atmosphere. Oxygen is not combustible itself, but it can feed a fire if one occurs. When using home oxygen, safety precautions are imperative. Vaseline is an oil-based, flammable product and should be avoided. A water-soluble lubricant may be used instead. Oxygen canisters should be kept at least 5-10 feet away from gas stoves, lighted fireplaces, wood stoves, candles, or other sources of open flames. Clients should use precautions as cooking oils and grease are highly flammable. The prescribed concentration of oxygen, usually 24%-28% for clients with COPD, should be maintained. Oxygen is prescribed to raise the PaO2 to 60-70 mm Hg and the saturations from 90%-93%. A flow rate of 2 L/min provides approximately 28% oxygen concentration, and 6 L/min provides approximately 44%. Higher rates usually do not help and can even be dangerous in clients with COPD as they can decrease the drive to breathe. The client should notify the care provider about excessive shortness of breath as additional treatment may be indicated. (Option 4) The client understands that nail polish remover and nail polish contain acetone, which is highly combustible. (Option 5) Clients should avoid synthetic and wool fabrics because they can cause static electricity, which may ignite a fire in the presence of oxygen. Clients should use cotton blankets and wear cotton fabrics. Educational objective: Safety precautions for home oxygen use include the following: no smoking; electrical devices in good condition and plugs grounded; avoiding volatile, flammable products and materials that generate static electricity; staying at least 5-10 feet away from open sources of flame; keeping fire extinguishers readily available; and regularly testing smoke detectors. Copyright © UWorld. All rights reserved.

Block Time Remaining: 00:05:49 TUTOR Test Id: 72419393 QId: 31089 (921666) 67 of 75 A A A The registered nurse walks into the client's inpatient room and sees a fire in the wastebasket. The nurse should take which action first? 1. Activate the fire alarm [68%] 2. Close the door [14%] 3. Pour water on the fire [4%] 4. Use a fire extinguisher [12%] Omitted Correct answer 1 Answered correctly 68% Time: 6 seconds Updated: 12/26/2016 Explanation: RACE is the acronym used to remember how to deal with fires in inpatient settings. RACE represents the following actions: Rescue (remove clients from immediate danger) Alarm (activate the fire alarm, call "code red," alert nearby appropriate personnel) Confine (close the doors and windows) Extinguish the fire or evacuate clients—first horizontally, then vertically In the provided options, the client rescue has been done or is not needed (ie, no one is in the room). The next priority is to pull the fire alarm to get assistance to help rescue other clients or contain the fire. (Option 2) Closing the doors (this room, all client rooms, and the fire doors between the units) is part of containment and should be done immediately after activating the alarm. (Option 3) Extinguishing the fire is performed after rescue, alarm, and containment. As a rule, water is not used on a fire in a hospital. It is used on Class A fires (paper, wood, and cloth) only. Instead, the nurse should use a fire extinguisher, which is designated for the types of material seen in hospitals. (Option 4) PASS is the acronym representing how to use an extinguisher and is part of the fourth step of RACE. It stands for Pull the pin, Aim the nozzle, Squeeze the handle, and Sweep back and forth over the fire. Educational objective: RACE is the acronym that represents the steps to take when dealing with fires at inpatient settings. The order of actions is Rescue, Alarm, Confine, and Extinguish/Evacuate. PASS is the acronym representing how to use a fire extinguisher (Pull, Aim, Squeeze, and Sweep). Copyright © UWorld. All rights reserved.

Block Time Remaining: 00:05:50 TUTOR Test Id: 72419393 QId: 34319 (921666) 68 of 75 A A A The inpatient hospice nurse is caring for a Muslim client newly admitted with terminal cancer. Which of the following interventions would the nurse anticipate for this client? Select all that apply. 1. Arrange for health care workers of the same sex to provide care for the client 2. Coordinate with the registered dietician to provide halal meals 3. Reposition the immobile client to face the city of Mecca during daily prayer times 4. Restrict the number of visitors from the family to preserve the client's privacy 5. Upon death, provide the family with supplies for postmortem care Omitted Correct answer 1,2,3,5 Answered correctly 42% Time: 1 seconds Updated: 12/21/2016 Explanation: Spirituality, religious beliefs, and traditions are important to include in client care. Aspects of care for Muslim clients include: Facilitating client to face Kaaba in the holy city of Mecca, generally northeastward from North America, during prayer (Option 3) - Ritual daily prayers occur 5 times a day, and dying clients may pray more often. Modesty - Care providers should be the same sex as the client whenever possible (Option 1). The female client may require a hijab (traditional head covering) and/or gown to cover most of the body. Providing foods that are halal (lawful), or acceptable for consumption (eg, no pork) - Kosher and vegetarian meals are acceptable if a specific halal menu is unavailable (Option 2). During Ramadan, the sick and dying are not required to fast with other Muslims from dawn until sunset. If the client chooses to fast, meals and medications should be rescheduled accordingly. Postmortem care of the Muslim client involves ritual washing, usually performed by family members, in preparation for burial. Burial occurs quickly after death, sometimes the same day (Option 5). (Option 4) In Islam, the family is the most important unit, and family presence brings strength to the individual. Multiple visitors should be accommodated unless they interfere with care. Educational objective: Important aspects of care for Muslim clients include accommodating the following client needs: Facing Kaaba in the holy city of Mecca for prayer, modesty considerations, adherence to dietary practices (halal or kosher meals and possibly fasting during Ramadan), and involvement of family. Copyright © UWorld. All rights reserved.

Block Time Remaining: 00:05:51 TUTOR Test Id: 72419393 QId: 31735 (921666) 69 of 75 A A A The nurse in the intensive care unit (ICU) is giving unlicensed assistive personnel (UAP) directions for bathing a client who has a surgical incision infected with methicillin-resistant Staphylococcus aureus (MRSA). Which instructions would be most effective for reducing infection? 1. Assist the client to the shower and provide directions to use antibacterial soap [8%] 2. Delay the bath until the client has received antibiotic therapy for 24 hours [13%] 3. Use a bath basin with warm water and a new wash cloth for each body area [35%] 4. Use packaged pre-moistened cloths containing chlorhexidine to bathe the client [42%] Omitted Correct answer 4 Answered correctly 42% Time: 1 seconds Updated: 05/04/2017 Explanation: Current evidence supports the recommendation for clients with MRSA or other drug-resistant organisms to be bathed with pre-moistened cloths or warm water containing chlorhexidine solution. Bathing clients in this way can significantly reduce MRSA infection. (Option 1) This action may be appropriate for a client in the home setting. However, most clients in the ICU are unable to go to the shower or have monitoring equipment and/or invasive lines that would make bathing difficult. Chlorhexidine is recommended in the hospital setting. (Option 2) It is not appropriate to delay bathing as the client's skin and incision need to be cleaned. Delay should only occur if the client is unstable. (Option 3) This option would be appropriate if the bath water contained a solution of chlorhexidine. Educational objective: Pre-moistened cloths or warm water with a chlorhexidine solution should be used when bathing clients infected with MRSA or other drug-resistant organisms. Copyright © UWorld. All rights reserved.

Block Time Remaining: 00:05:52 TUTOR Test Id: 72419393 QId: 32629 (921666) 70 of 75 A A A The charge nurse is instructing a new graduate nurse on performing postmortem care. Which client situations might cause the nurse to delay or not perform postmortem care? Select all that apply. 1. Client died following a prolonged illness 2. Client's family was not present when death occurred 3. Client's religious background requires special ceremonial treatment of the body 4. Death occurred in the emergency department following a suicide attempt 5. Family requests a priest to perform last rites Omitted Correct answer 3,4,5 Answered correctly 37% Time: 1 seconds Updated: 03/06/2017 Explanation: Postmortem care typically is performed immediately following the pronouncement of death to allow visitation of the deceased by the family. There are several circumstances in which postmortem care may be delayed or not performed. Certain cultural or religious beliefs require that care be performed by the family or clergy (Option 3). The family also may want religious ceremonies performed or last rites given before the body is cleaned or disturbed in any way (Option 5). Postmortem care can also be delayed, altered, or not performed in accordance with state law and agency policies. These situations include deaths that are considered non-natural, traumatic, or associated with criminal activity (Option 4). (Option 1) Death following a prolonged illness may be expected and would not cause a delay in postmortem care unless the family requests it. (Option 2) Unless family members notify the agency of any religious or cultural needs related to the deceased, they do not have to be present for postmortem care to take place. Educational objective: Postmortem care may be delayed or not performed if the family has certain cultural or religious beliefs or if the death is considered to be non-natural, traumatic, or associated with criminal activity. Copyright © UWorld. All rights reserved.

Block Time Remaining: 00:05:53 TUTOR Test Id: 72419393 QId: 30672 (921666) 71 of 75 A A A The nurse performs tracheostomy care for a client with a disposable inner cannula and tracheostomy dressing. Place the steps in the correct order. All options must be used. Your Response/ Incorrect Response Correct Response Gather supplies and position client Don mask, goggles, and clean gloves Remove soiled dressing Don sterile gloves; remove old disposable cannula and replace with a new one Clean around stoma with sterile water or saline; dry and replace sterile gauze pad Omitted Correct answer 4,2,5,3,1 Answered correctly 60% Time: 1 seconds Updated: 05/22/2017 Explanation: When performing tracheostomy care, the nurse follows institution policy and observes principles of infection control and client safety. Sterile technique is used to prevent infection of the lower airway. The steps for performing the procedure for a client with a disposable inner cannula include the following: Gather supplies to the bedside, then place client in semi-Fowler's position, if not contraindicated, to promote lung expansion and oxygenation and prevent aspiration of secretions. Don personal protective equipment (mask, goggles, and clean gloves) to maintain universal precautions. Auscultate lungs and suction secretions if necessary. Remove soiled dressing and also remove clean gloves. Don sterile gloves; remove old disposable cannula and replace with a new one. While stabilizing the back plate with the nondominant hand, unlock (unclip) the old cannula with the dominant hand; remove gently by pulling it out in line with its curvature; pick up the new cannula, touching only the outer locking portion (to prevent contamination and maintain asepsis); insert; and lock (clip) into place. Clean around stoma with sterile water or saline, dry and replace sterile gauze pad to remove dried secretions, and dry around stoma well to limit the growth of microorganisms. Some tracheostomy tubes are sutured in place and do not require a dressing. If secretions are copious, apply a dressing. Copyright © UWorld. All rights reserved.

Block Time Remaining: 00:05:54 TUTOR Test Id: 72419393 QId: 31912 (921666) 72 of 75 A A A A client diagnosed with hypertension has been prescribed a clonidine patch. Which instructions should the nurse include? Select all that apply. 1. Apply patch to the upper arm or chest 2. Fold used patches in half with sticky sides together before discarding 3. Remove patch if dizziness occurs when getting up 4. Rotate sites each time a new patch is applied 5. Shave hair before applying patch Omitted Correct answer 1,2,4 Answered correctly 32% Time: 1 seconds Updated: 03/27/2017 Explanation: Clonidine is a potent antihypertensive agent and is available as a transdermal patch. The patches should be replaced every 7 days and can be left in place during bathing. Instructions for using the clonidine (transdermal) patch: Apply the patch to a dry hairless area on the upper outer arm or chest once every 7 days (Option 1). Do not shave the area before applying the patch. The skin should be free from cuts, scrapes, calluses, or scars (Option 5). Wash hands with soap and water before and after applying the patch as some medication may remain on the hands after application. Wash the area with soap and water, then rinse and wipe with a clean, dry tissue. Remove the patch from the package. Do not touch the sticky side. Rotate sites of patch application with each new patch (Option 4). Remove the old patch only when applying a new one. Do not wear more than 1 patch at a time unless directed by your health care provider (HCP). When removing the patch, fold it in half with the sticky sides together. Discard the patch out of the reach of children and pets. Even after it has been used, the patch contains active medicine that may be harmful if accidentally applied or ingested (Option 2). Notify the HCP if you are experiencing side effects such as dizziness or slow pulse rate. Do not remove the patch without discussing this with the HCP as rebound hypertension can occur (Option 3). Educational objective: The nurse should teach a client receiving a clonidine patch to: Apply patch to a dry hairless area on the upper arm or chest Wash hands before and after application Rotate sites with each new patch application Discard patch away from children or pets with sticky sides folded together Never wear more than 1 patch at a time Never stop using the patch abruptly Copyright © UWorld. All rights reserved.

Block Time Remaining: 00:06:26 TUTOR Test Id: 72490738 QId: 30574 (921666) 22 of 75 A A A The nurse is caring for a client who has been receiving mechanical ventilation (MV) for 4 days. During multidisciplinary morning rounds, the health care provider questions the development of a ventilator-associated pneumonia (VAP). Which of the following manifestations does the nurse assess as the best indicator of VAP? 1. Blood-tinged sputum [4%] 2. Positive blood cultures [11%] 3. Positive, purulent sputum culture [53%] 4. Rhonchi and crackles [30%] Omitted Correct answer 3 Answered correctly 53% Time: 37 seconds Updated: 04/26/2017 Explanation: VAP is the second most common health care-associated infection (HAI) in the United States and is associated with increased mortality, hospital cost, and length of stay. Because it is a nosocomial infection, signs and symptoms associated with VAP usually present within ≥2-3 days after initiation of mechanical ventilation (MV). Characteristic clinical manifestations of VAP include purulent sputum, positive sputum culture, leukocytosis (12,000 mm3), elevated temperature (>100.4 F [38 C]), and new or progressive pulmonary infiltrates suggestive of pneumonia on chest x-ray. (Option 1) Blood-tinged sputum may occur but is not the best indicator of VAP. (Option 2) Positive blood cultures may identify the microorganism causing the infection but are not the best indicator of VAP. Positive blood cultures could be from another source of infection. (Option 4) Rhonchi and crackles are adventitious lung sounds associated with pneumonia but can be present in pulmonary edema or just from increased mucous secretions. They are not the best indicator of VAP. Educational objective: VAP is an HAI that usually occurs within ≥2-3 days after the initiation of mechanical ventilation. Characteristic manifestations of VAP include purulent secretions, positive sputum culture, leukocytosis, elevated temperature, and new or progressive pulmonary infiltrates on chest x-ray. Copyright © UWorld. All rights reserved.

Block Time Remaining: 00:07:08 TUTOR Test Id: 72490738 QId: 34205 (921666) 23 of 75 A A A The hospice nurse is assisting a client's family in managing anorexia during end-of-life care. Which interventions would be most supportive? Select all that apply. 1. Administer nausea medication prior to meals 2. Involve the client in daily meal planning 3. Offer food items the client desires 4. Plan for loved ones to share meal time with the client 5. Prepare 3 highly nutritious meals a day on a schedule Omitted Correct answer 1,2,3,4 Answered correctly 32% Time: 42 seconds Updated: 04/01/2017 Explanation: Anorexia is an expected complication in clients nearing the end of life and is exacerbated by many variables, including medications, stress, and progression of disease. Caregivers can implement strategies to manage these factors, including the following: Administering prescribed analgesia, antiemetic medications, and appetite stimulants (eg, dexamethasone, megestrol acetate) to enhance client comfort and increase intake (Option 1) Involving the client in meal planning to encourage autonomy and a sense of purpose (Option 2) Promoting foods that are preferred and well tolerated, regardless of nutritional value, to stimulate appetite and increase intake (Option 3) Providing meals with friends/family outside of the "sick room," if possible, to promote stimulation and enjoyment (Option 4) Providing frequent oral care, especially after eating, and using topical treatments to minimize oral discomfort and dry mouth Offering adequate fluid and fiber intake and implementing a bowel regimen to help prevent constipation (Option 5) Caregivers should not focus on "eating healthy" or meal scheduling, as stress can decrease appetite. Small portions of desired foods should be offered frequently throughout the day but not forced on the client. Educational objective: Managing anorexia during end-of-life care includes involving the client in meal planning, including friends/family at meals, offering preferred foods whenever the client is hungry, providing frequent oral care, administering pain/nausea medications and appetite stimulants, and preventing constipation with adequate fluid and fiber intake. Copyright © UWorld. All rights reserved.

Block Time Remaining: 00:07:10 TUTOR Test Id: 72490738 QId: 30934 (921666) 24 of 75 A A A Which teaching instructions should the nurse provide to a client with advanced chronic obstructive pulmonary disease (COPD)? Select all that apply. 1. Follow a low-calorie diet 2. Obtain a pneumococcal vaccine 3. Report increased sputum 4. Take iron to improve anemia 5. Use an incentive spirometer Omitted Correct answer 2,3 Answered correctly 5% Time: 2 seconds Updated: 05/15/2017 Explanation: The client with COPD is very susceptible to pulmonary infections. Most exacerbations are caused by bacterial or viral infection. Clients with COPD typically have some mucus production and cough at baseline. However, the presence of increased dyspnea, increased sputum volume, or sputum purulence indicates bacterial infection and benefit from antibiotics. It is important for the client to treat an exacerbation as soon as possible (Option 3). The current recommendations are that clients with COPD, regardless of age, should receive the pneumococcal vaccine. Clients should also receive a yearly influenza virus vaccine (Option 2). (Option 1) Clients with advanced COPD are underweight. The loss of muscle mass is attributed to aging, steroid use (catabolic effect), and the effort to eat with dyspnea. Extra intake of protein and calories is encouraged. (Option 4) Clients with COPD are at risk for polycythemia. The kidney is stimulated to make erythropoietin and increase the red blood cells due to the hypoxia. Anemia is not expected in this condition. (Option 5) Incentive spirometry gives the client visual feedback and encouragement to take deep breaths. It is used routinely for postoperative atelectasis. In COPD, the pathophysiology involves structural changes and permanent airflow limitations that result in trapped air. The client needs help to expel the air, not get more air into the lungs. Educational objective: Clients with COPD need to report any signs of infection (including change in their normal sputum) as infection is a primary cause of exacerbation. Any client with COPD should have a pneumococcal vaccine and an annual influenza virus vaccine. Copyright © UWorld. All rights reserved.

Block Time Remaining: 00:07:11 TUTOR Test Id: 72490738 QId: 31297 (921666) 25 of 75 A A A A client is experiencing an asthma attack. The nurse assesses extreme anxiety, dyspnea, nonproductive cough, inspiratory and expiratory wheezing, and diminished breath sounds. Respirations are 36/min, pulse is 122/min, and pulse oximeter shows 87% on room air. Which is the priority nursing diagnosis (ND) for this client? 1. Anxiety related to hypoxia and fear of suffocation [10%] 2. Impaired gas exchange related to alveolar hypoventilation [49%] 3. Ineffective airway clearance related to abnormal viscosity of mucus [11%] 4. Ineffective breathing pattern related to decreased lung expansion [29%] Omitted Correct answer 2 Answered correctly 49% Time: 1 seconds Updated: 02/03/2017 Explanation: The priority ND is impaired gas exchange related to alveolar hypoventilation and reduced oxygen available for exchange as evidenced by tachycardia, tachypnea, dyspnea, and hypoxemia. The assessment data that support a deficit in oxygenation and gas exchange include: High-pitched wheezing on inspiration and expiration. Wheezing is usually heard on expiration; when heard on inspiration as well, it indicates an even greater degree of bronchospasm, airway resistance, and hypoxemia. Diminished breath sounds during an asthma attack indicate reduced delivery of inspired oxygen due to hyperinflation, air trapping, and alveolar hypoventilation. Decreased oxygen saturation (pulse oximeter of 87% on room air) in the presence of tachycardia and tachypnea indicates significant impaired gas exchange and hypoxemia. (Option 1) Anxiety related to hypoxia and fear of suffocation as evidenced by tachycardia and tachypnea is an appropriate ND for a client with asthma, but it is not the priority ND. (Option 3) Low-pitched wheezing (rhonchi) and cough can indicate the presence of excessive secretions in the airway. These symptoms are expected findings in asthmatic clients. Based on the client's assessment data, there is no immediate risk of airway compromise. Ineffective airway clearance is not the priority ND. (Option 4) Ineffective breathing pattern related to anxiety, and edema and constriction of the bronchi, as evidenced by dyspnea and tachypnea, are appropriate NDs for a client with asthma but are not the priority ND. Educational objective: Impaired gas exchange related to alveolar hypoventilation (eg, air trapping, hyperinflation) is a priority ND for a client experiencing an asthma attack. It is evidenced by decreased oxygen saturation (<90%), increased airway resistance (high-pitched wheezing), and diminished breath sounds. Copyright © UWorld. All rights reserved.

Block Time Remaining: 00:07:24 TUTOR Test Id: 72490738 QId: 31183 (921666) 26 of 75 A A A The nurse in the postanesthesia care unit (PACU) is caring for an unresponsive client who just came from the operating room following surgery under general anesthetic for colorectal cancer. The nurse chooses what as the highest priority nursing diagnosis (ND)? 1. Acute pain [7%] 2. Impaired physical mobility [5%] 3. Ineffective airway clearance [78%] 4. Risk for fluid volume deficit [8%] Correct Answered correctly 78% Time: 13 seconds Updated: 01/02/2017 Explanation: Ineffective airway clearance, which is the inability to clear secretions or obstructions from the respiratory tract to maintain a clear airway, is the priority ND as it poses the greatest threat to survival. The most common causes of respiratory complications in the immediate postoperative period include the following: Airway obstruction, which can be due to retained secretions or the tongue falling backward against the soft palate in sedated clients. Suctioning and an artificial oral airway can be used to prevent obstruction until the client becomes more responsive. Hypoxemia, which can be due to atelectasis from increased retained secretions or hypoventilation, aspiration, or bronchospasm. Pulse oximetry and supplemental oxygen are used to maintain pulse oximeter readings >92%; placing the client in side-lying position and administrating antiemetic medications help to decrease aspiration. Hypoventilation, which can be due to depression of the respiratory drive as a result of anesthesia, pain, and opioid analgesia. (Option 1) Most clients receive continual or bolus epidural or patient-controlled opioid analgesia to control postoperative pain. Pain assessment after surgery is a priority, and acute pain is an appropriate ND. However, it does not pose the greatest threat to survival and is not the priority diagnosis at this time. (Option 2) Ambulation and ability to reposition are important interventions to prevent multiple postoperative complications. However, impaired physical mobility is not the priority ND at this time. (Option 4) Risk for fluid volume deficit related to fluid losses during surgery is an appropriate diagnosis, but it does not pose the greatest threat to survival and is not the priority ND at this time. Educational objective: Acute pain, impaired physical mobility, ineffective airway clearance, and risk for fluid volume deficit are appropriate NDs in a client in the PACU who is immediately postoperative. The highest priority ND is the one that poses the greatest threat to survival based on the client's current health status. Copyright © UWorld. All rights reserved.

Block Time Remaining: 00:07:35 TUTOR Test Id: 72490738 QId: 30532 (921666) 27 of 75 A A A A licensed practical nurse (LPN) is assigned to care for a client who was admitted to the medical unit last night with a moderate asthma exacerbation and an upper respiratory infection. Which tasks are appropriate for the registered nurse to delegate to the LPN? Select all that apply. 1. Administering albuterol metered-dose inhaler medication 2. Auscultating lung sounds to determine the client's response to a bronchodilator 3. Checking oxygen saturation with the pulse oximeter 4. Measuring morning peak expiratory flow with the client's peak flow meter 5. Monitoring for labored breathing and use of accessory muscles of respiration Omitted Correct answer 1,3,4,5 Answered correctly 13% Time: 11 seconds Updated: 03/08/2017 Explanation: Scope of practice RN LPN/LVN UAP Clinical assessment Initial client education Discharge education Clinical judgment Initiating blood transfusion Monitoring RN findings Reinforcing education Routine procedures (eg, catheterization) Most medication administrations Ostomy care Tube patency & enteral feeding Specific assessments Activities of daily living Hygiene Linen change Routine, stable vital signs Documenting input/output Positioning LPN = licensed practical nurse; LVN = licensed vocational nurse; RN = registered nurse; UAP = unlicensed assistive personnel. *Limited assessments (eg, lung sounds, bowel sounds, neurovascular checks). When delegating tasks, the registered nurse (RN) should consider the 5 rights of delegation along with the scope of practice. The scope of practice for a licensed practical nurse (LPN) includes administering medications (although regulations related to narcotics and IV medications vary by state) (Option 1). Based on staff member availability, it can also be appropriate to perform or delegate tasks below the scope of a given staff member (eg, delegating vital signs to an LPN, an RN performing ostomy care) (Option 3). Measuring peak expiratory flow with a peak flow meter and monitoring for labored breathing and use of accessory muscles of respiration are also within the scope of practice for an LPN (Options 4 and 5). (Option 2) The LPN can collect data, such as auscultating lung sounds, but evaluation of the collected data (ie, determining the client's response to bronchodilator medication) requires use of the nursing process, which is the responsibility of the RN. Educational objective: Administering most medications and collecting data related to adequacy of oxygenation and ventilation are tasks within the scope of practice for a licensed practical nurse. The overall assessment, analysis, and evaluation of the collected data to develop the client plan of care are the responsibilities of the supervising registered nurse. Copyright © UWorld. All rights reserved.

Block Time Remaining: 00:07:57 TUTOR Test Id: 72490738 QId: 30817 (921666) 28 of 75 A A A The home health nurses visits a 72-year-old client with pneumonia who was discharged from the hospital 3 days ago. The client has less of a productive cough at night but now reports sharp chest pain with inspiration. Which finding is most important for the nurse to report to the health care provider? 1. Bronchial breath sounds [3%] 2. Increased tactile fremitus [13%] 3. Low-pitched wheezing (rhonchi) [11%] 4. Pleural friction rub [72%] Incorrect Correct answer 4 Answered correctly 72% Time: 22 seconds Updated: 01/13/2017 Explanation: Pleurisy is characterized by stabbing chest pain that usually increases on inspiration or with cough. It is caused by inflammation of the visceral pleura (over the lung) and the parietal pleura (over the chest cavity). The pleural space (between the 2 layers) normally contains about 10 mL of fluid to help the layers glide easily with respiration. When inflamed, they rub together, causing pleuritic pain. A pleural friction rub is auscultated in the lateral lung fields over the area of inflammation. The sound is produced by the 2 layers rubbing together and can indicate pleurisy, a complication of pneumonia. It is characterized by squeaking, crackling, or the sound heard when the palm is placed over the ear and the back of the hand is rubbed with the fingers. Complications of pneumonia are more prevalent in elderly clients with underlying chronic disease. (Option 1) Clients with consolidative lung processes (pneumonia) may also have bronchial breath sounds due to over-transmission of sound over the chest wall. Breath sounds are diminished or absent over a pleural effusion or pneumothorax. (Option 2) Palpable vibration felt on the chest wall is known as fremitus. Sound travels faster in solids (consolidation) than in an aerated lung, resulting in increased fremitus in pneumonia. It is an expected finding in clients with pneumonia. It is concerning if decreased because the client may be developing pleural effusion. (Option 3) Low-pitched wheezing (rhonchi) is a continuous adventitious breath sound heard over the large airways, usually during expiration. It indicates the presence of secretions in the larger airways and is an expected finding as pneumonia resolves. Educational objective: Pleurisy is characterized by stabbing pleuritic chest pain that increases on inspiration. It is a complication of pneumonia caused by inflamed parietal and visceral pleurae rubbing together. Copyright © UWorld. All rights reserved.

Block Time Remaining: 00:07:57 TUTOR Test Id: 72490738 QId: 30817 (921666) 28 of 75 A A A The home health nurses visits a 72-year-old client with pneumonia who was discharged from the hospital 3 days ago. The client has less of a productive cough at night but now reports sharp chest pain with inspiration. Which finding is most important for the nurse to report to the health care provider? 1. Bronchial breath sounds [3%] 2. Increased tactile fremitus [13%] 3. Low-pitched wheezing (rhonchi) [11%] 4. Pleural friction rub [72%] Incorrect Correct answer 4 Answered correctly 72% Time: 22 seconds Updated: 01/13/2017 Explanation: Pleurisy is characterized by stabbing chest pain that usually increases on inspiration or with cough. It is caused by inflammation of the visceral pleura (over the lung) and the parietal pleura (over the chest cavity). The pleural space (between the 2 layers) normally contains about 10 mL of fluid to help the layers glide easily with respiration. When inflamed, they rub together, causing pleuritic pain. A pleural friction rub is auscultated in the lateral lung fields over the area of inflammation. The sound is produced by the 2 layers rubbing together and can indicate pleurisy, a complication of pneumonia. It is characterized by squeaking, crackling, or the sound heard when the palm is placed over the ear and the back of the hand is rubbed with the fingers. Complications of pneumonia are more prevalent in elderly clients with underlying chronic disease. (Option 1) Clients with consolidative lung processes (pneumonia) may also have bronchial breath sounds due to over-transmission of sound over the chest wall. Breath sounds are diminished or absent over a pleural effusion or pneumothorax. (Option 2) Palpable vibration felt on the chest wall is known as fremitus. Sound travels faster in solids (consolidation) than in an aerated lung, resulting in increased fremitus in pneumonia. It is an expected finding in clients with pneumonia. It is concerning if decreased because the client may be developing pleural effusion. (Option 3) Low-pitched wheezing (rhonchi) is a continuous adventitious breath sound heard over the large airways, usually during expiration. It indicates the presence of secretions in the larger airways and is an expected finding as pneumonia resolves. Educational objective: Pleurisy is characterized by stabbing pleuritic chest pain that increases on inspiration. It is a complication of pneumonia caused by inflamed parietal and visceral pleurae rubbing together. Copyright © UWorld. All rights reserved.

Block Time Remaining: 00:30:31 TUTOR Test Id: 72325665 QId: 31457 (921666) 1 of 10 A A A Which client in a prenatal clinic should the nurse assess first? 1. Client at 11 weeks gestation with backache and pelvic pressure [13%] 2. Client at 16 weeks gestation with earache and sinus congestion [3%] 3. Client at 27 weeks gestation with headache and facial edema [78%] 4. Client at 37 weeks gestation with white vaginal discharge and urinary frequency [4%] Correct Answered correctly 78% Time: 33 seconds Updated: 04/10/2017 Explanation: Gestational hypertension is new-onset high blood pressure (≥140/90 mm Hg) that occurs after 20 weeks gestation without proteinuria. The development of proteinuria with hypertension indicates preeclampsia, which may manifest with symptoms such as headache, visual disturbances, and facial swelling. This client is exhibiting symptoms of preeclampsia and should be assessed first (Option 3). Complications of preeclampsia may include thrombocytopenia, liver dysfunction, and renal insufficiency. Clients with preeclampsia must be monitored closely for sudden worsening, which can lead to serious complications, including eclampsia and/or HELLP syndrome (hemolysis, elevated liver enzymes, and low platelets). (Option 1) Backache and pelvic pressure are common discomforts of pregnancy. The enlarging uterus stretches the supporting ligaments and may cause increased backache. Fluid retention from increasing blood volume causes pelvic congestion, which may be reported as pelvic pressure. (Option 2) Earache and sinus congestion are common discomforts during pregnancy that result from increased blood volume and fluid retention. This client should be assessed to rule out sinus and/or ear infection, but this is not the highest priority. (Option 4) The increase in estrogen and progesterone during pregnancy often results in leukorrhea, a mucoid, white vaginal discharge. Pressure on the bladder from the fetal head during the third trimester may cause diminished bladder capacity and urinary frequency without dysuria. These are common and expected findings. Educational objective: The nurse should assess the client with symptoms of potentially serious complications first before assessing the remaining clients. Signs of hypertensive disorders during pregnancy may include headache and facial edema. Copyright © UWorld. All rights reserved.

Block Time Remaining: 00:30:31 TUTOR Test Id: 72325665 QId: 30400 (921666) 2 of 10 A A A A 37-weeks-pregnant woman comes to the emergency department with a fractured ankle. Which assessment finding is most concerning and requires the nurse to follow up? 1. Fetal heart rate remains 206/min [84%] 2. Fetus kicked 4 times in the past hour [2%] 3. Mother reports feeling 2 contractions every hour [5%] 4. Mother's hemoglobin is 11 g/dL (110 g/L) [7%] Correct Answered correctly 84% Time: 48 seconds Updated: 04/19/2017 Explanation: Fetal tachycardia is a baseline of >160 beats/min for >10 minutes. Tachycardia needs evaluation and continued surveillance. The most sensitive indicators of fetus health are fetal movement and fetal heart rate. (Option 2) This is an expected finding. Monitoring fetal movement/kick counts is a primary method of fetal surveillance. The reassuring finding is when the movement equals or exceeds the established baseline. In general, 4 movements/hour or 10 distinct fetal movements within 2 hours is a reassuring finding. (Option 3) Braxton-Hicks contractions are felt mid-pregnancy onward. These painless, occasional physiological contractions are normal. The contractions are a concern if they become regular and persist. (Option 4) During pregnancy, hemoglobin can drop to 11 g/dL (110 g/L), a condition known as physiological anemia of pregnancy. Due to the increased oxygen requirements of pregnancy, the red blood cell count increases 30%. However, anemia can result from an increase in the plasma volume that is relatively larger than the increase in red blood cells. This lowered maternal hemoglobin is within the expected range. Educational objective: Sustained fetal tachycardia (>160/min for >10 minutes) is a concerning finding that requires further follow-up. Copyright © UWorld. All rights reserved.

Block Time Remaining: 00:30:31 TUTOR Test Id: 72325665 QId: 31669 (921666) 7 of 10 A A A The nurse is documenting assessments of pregnant clients in the antepartum unit. Which client's assessment findings are most important to report to the health care provider? 1. Client at 28 weeks gestation with an asymptomatic systolic murmur [9%] 2. Client at 34 weeks gestation with 1+ edema of bilateral lower extremities [5%] 3. Client at 35 weeks gestation with painful genital lesions [53%] 4. Client at 39 weeks gestation with brownish, mucoid vaginal discharge [31%] Correct Answered correctly 53% Time: 39 seconds Updated: 12/20/2016 Explanation: Painful genital lesions can be indicative of an outbreak of genital herpes simplex virus (HSV) and are a priority assessment finding to report to the health care provider. Herpes in pregnant women can be transmitted to the infant in utero (congenital HSV), perinatally, or postnatally as a result of direct contact with virus particles shed from the infected vulva, vagina, cervix, or perineum. Neonatal HSV infection has serious morbidity (eg, permanent neurologic sequelae) and mortality. Immediate antiviral therapy (eg, acyclovir) should be initiated to treat the active infection. Vaginal birth is not recommended in the presence of active lesions; cesarean birth helps reduce the risk of transmission to the newborn (Option 3). (Option 1) A systolic murmur (swooshing sound heard during S1) is a common finding in pregnancy. It results from increased total body volume (flow murmur) and usually resolves after birth. No further intervention is necessary if the client is asymptomatic. (Option 2) Edema of the lower extremities is common in the third trimester, especially after long periods of standing. Pressure from the enlarged uterus slows venous return, contributing to dependent edema, which usually improves with rest and elevation of the legs. (Option 4) Vaginal discharge increases at the end of pregnancy and may become mucoid and blood-tinged (pink/brownish) in the days preceding labor. This assessment finding may be a sign of approaching labor in this client at term gestation (39 weeks), but it is not the priority to report to the health care provider. Educational objective: Painful genital lesions during pregnancy are a priority assessment finding to report to the health care provider. Active herpes lesions that are present at the onset of labor indicate the need for cesarean birth. Copyright © UWorld. All rights reserved.

Block Time Remaining: 00:30:31 TUTOR Test Id: 72325665 QId: 31545 (921666) 8 of 10 A A A Which finding is most important for the nurse to report to the health care provider? 1. 24 weeks gestation, hemoglobin is 9 g/dL (90 g/L) [44%] 2. 24 weeks gestation, 1-hour (50 g) oral glucose screen is 120 mg/dL (6.7 mmol/L) [8%] 3. 27 weeks gestation, vaginal secretion is pH 5 [10%] 4. 36 weeks gestation, white blood cell count is 13,000/mm3 (13.0 x 109/L) [35%] Correct Answered correctly 44% Time: 52 seconds Updated: 05/30/2017 Explanation: Iron deficiency anemia is a common complication during pregnancy. It is related to low iron stores and diets low in iron. During pregnancy, clients are considered anemic when hemoglobin is <11 g/dL (110 g/L) in the first or third trimesters, or <10.5 g/dL (105 g/L) in the second trimester. A pregnant client with a hemoglobin level of 9 g/dL (90 g/L) should be evaluated for symptoms of anemia (eg, fatigue, weakness) and may require additional laboratory work (eg, serum ferritin level) to determine the cause. An iron supplement may need to be prescribed. (Option 2) A 1-hour (50 g) oral glucose screen is considered abnormal if the result is ≥140 mg/dL (7.8 mmol/L). (Option 3) During pregnancy, the vagina maintains an acidic state to inhibit the growth of bacteria. (Option 4) During pregnancy, it is normal for the white blood cell count to increase; counts can be as high as 15,000/mm3 (15.0 x 109/L). Educational objective: Clients are diagnosed with anemia when hemoglobin is <11 g/dL (110 g/L) in the first or third trimesters, or <10.5 g/dL (105 g/L) in the second trimester. Iron deficiency is the most common cause. Copyright © UWorld. All rights reserved.

Block Time Remaining: 00:30:31 TUTOR Test Id: 72325665 QId: 30521 (921666) 3 of 10 A A A The nurse is preparing to assess a client visiting the women's health clinic. The client's obstetric history is documented as G5T1P2A1L2. Which interpretation of this notation is correct? 1. The client had 1 birth at 37 wk 0 d gestation or beyond [66%] 2. The client had 3 births between 20 wk 0 d and 36 wk 6 d gestation [16%] 3. The client has 3 currently living children [8%] 4. The client is currently not pregnant [8%] Correct Answered correctly 66% Time: 491 seconds Updated: 01/08/2017 Explanation: The GTPAL system is a shorthand system of documenting a client's obstetric history. This client (G5T1P2A1L2) has been pregnant 5 times (G5); had 1 term birth (T1), 2 preterm births (P2), and 1 abortion (A1); and has 2 currently living children (L2). The client's term birth is indicated by the T1 portion of the GTPAL notation (Option 1). (Option 2) The client had 2 preterm births, indicated by the P2 portion of the GTPAL notation. (Option 3) The client has 2 currently living children, as indicated by the L2 portion of the GTPAL notation. If a child born full- or preterm is not living (due to stillbirth from 20 wk 0 d and beyond or infant/child death after birth), that birth and subsequent death is counted toward T or P (term or preterm) but is not notated under L (currently living children); T and P record total number of births without regard to current living status. This client has 2 currently living children (L2), which is 1 less than the client's total notation for term + preterm (T1 + P2 = 3). Therefore, the client has experienced the death of 1 child who had been born at 20 wk 0 d gestation or beyond. (Option 4) If a client is currently pregnant, the number of pregnancies (gravida) will be greater than the number of births (term, preterm, and abortions combined). This client is a G5, and T1 + P2 + A1 = 4. Therefore, the client is currently pregnant. Educational objective: The GTPAL system notational components are G - gravida (number of pregnancies, regardless of outcome and including current pregnancies), T - term (37 wk 0 d gestation and beyond), P - preterm (20 wk 0 d through 36 wk 6 d gestation), A - abortions (before 20 wk 0 d gestation; spontaneous or induced), and L - currently living children. Copyright © UWorld. All rights reserved.

Block Time Remaining: 00:30:31 TUTOR Test Id: 72325665 QId: 31552 (921666) 4 of 10 A A A A client at 20 weeks gestation states that she started consuming an increased amount of cornstarch about 3 weeks ago. Based on this assessment, the nurse should anticipate that the health care provider will order which laboratory test(s)? 1. Hemoglobin and hematocrit levels [37%] 2. Human chorionic gonadotropin level [11%] 3. Serum folate level [49%] 4. White blood cell count [2%] Correct Answered correctly 37% Time: 34 seconds Updated: 01/29/2017 Explanation: Pica is the abnormal, compulsive craving for and consumption of substances normally not considered nutritionally valuable or edible. Common substances include ice, cornstarch, chalk, clay, dirt, and paper. Although the condition is not exclusive to pregnancy, many women only have pica when they are pregnant. Pica is often accompanied by iron deficiency anemia due to insufficient nutritional intake or impaired iron absorption. However, the exact relationship between pica and anemia is not fully understood. The health care provider would likely order hemoglobin and hematocrit levels to screen for the presence of anemia. (Option 2) Human chorionic gonadotropin is the hormone detected in a urine or serum pregnancy test to determine if a client is pregnant. It is not affected by iron deficiency anemia or pica. (Option 3) Increased folic acid consumption is necessary during pregnancy to reduce the risk for neural tube defects in the developing fetus. However, folate levels are not related to pica. (Option 4) A white blood cell count should be assessed when a client is suspected of having an infection. There is no indication that this client has an infection. Educational objective: Pica is the constant craving for and consumption of nonfood and/or nonnutritive food substances that may occur in pregnancy. It may be accompanied by iron deficiency anemia. Hemoglobin and hematocrit levels are useful in these clients to screen for anemia. Copyright © UWorld. All rights reserved.

Block Time Remaining: 00:30:31 TUTOR Test Id: 72325665 QId: 30565 (921666) 5 of 10 A A A A client comes to the clinic indicating that a home pregnancy test was positive. The client's last menstrual period was September 7. Today is December 7. Which are true statements for this client? Select all that apply. 1. According to Naegele's rule, the expected date of delivery is June 14 2. Detection of the fetal heart rate via Doppler is possible 3. Fundal height should be 24 cm above the symphysis pubis 4. The client should be feeling fetal movement 5. Urinary frequency is a common symptom Incorrect Correct answer 1,2,5 Answered correctly 45% Time: 158 seconds Updated: 12/22/2016 Explanation: Naegele's rule, which is the last menstrual period minus 3 months plus 7 days, can be used to calculate a client's expected date of delivery. The accuracy of this method may be influenced by the regularity and length of the client's menstrual cycle. September 7 minus 3 months is June 7, plus 7 days is June 14 (Option 1). Detection of a fetal heart rate is possible using a Doppler by 10-12 weeks gestation (Option 2). Urinary frequency, a presumptive sign of pregnancy common in the first trimester, occurs primarily due to hormonal changes and anatomical changes in the renal system (Option 5). However, clients also reporting dysuria, fever/chills, or back pain should be evaluated for a urinary tract infection. (Option 3) Uterine growth is assessed by measuring fundal height using a measuring tape. After 20 weeks gestation, the fundal height measurement in centimeters should correlate closely with the number of weeks pregnant (eg, 24 cm = 24 weeks). The client should empty the bladder before having fundal height measured, as a full bladder can displace the uterus and affect measurement accuracy. (Option 4) Quickening, the awareness of fetal movements, occurs around 18-20 weeks gestation in primigravidas and at 14-16 weeks in multigravidas. Educational objective: Naegele's rule for estimating date of delivery is the last menstrual period minus 3 months plus 7 days. Fetal heart rate is detectable by Doppler at 10-12 weeks gestation. Urinary frequency is a presumptive sign of pregnancy in the first trimester. Copyright © UWorld. All rights reserved.

Block Time Remaining: 00:30:31 TUTOR Test Id: 72325665 QId: 34441 (921666) 6 of 10 A A A A client who is 8 weeks pregnant reports morning sickness. What is the most appropriate response by the nurse? 1. Advise the client to consume hot, versus cold, foods [7%] 2. Instruct the client to drink 2 glasses of water with each meal [7%] 3. Suggest the client consume high-protein snacks on awakening [53%] 4. Tell the client that morning sickness should pass in a few weeks [32%] Correct Answered correctly 53% Time: 39 seconds Updated: 05/21/2017 Explanation: Morning sickness, characterized by nausea with or without vomiting, is a common problem during the first trimester of pregnancy. Although it is referred to as "morning" sickness, it can happen anytime throughout the day. It is thought to be due to rising hormone levels (ie, estrogen, progesterone, human chorionic gonadotropin). Initial interventions, focusing on diet management and triggering avoidance, include: Eating several small meals during the day (ie, high in protein or carbohydrates and low in fat) Drinking fluids (preferably clear, cold, carbonated beverages) between, rather than with, meals Having a high-protein snack before bedtime and on awakening (Option 3) Consuming foods/drinks with ginger (eg, ginger tea, ginger lollipops, ginger chews) Consuming foods high in vitamin B6 (eg, nuts, seeds, legumes) (Option 1) Hot foods will not relieve morning sickness; cold foods/drinks are usually recommended. (Option 2) Fluids should be consumed at least 30 minutes before or after meals to avoid abdominal distension. (Option 4) This response may provide false reassurance and does not help the client manage symptoms. Morning sickness typically resolves after the first trimester, although some clients experience nausea through the second or third trimester. Educational objective: Morning sickness can usually be relieved through lifestyle and dietary changes, including eating small and frequent meals, drinking cold fluids between meals, having a high-protein snack before bedtime and on awakening, and consuming foods/drinks containing ginger and vitamin B6. Copyright © UWorld. All rights reserved.

Block Time Remaining: 00:07:57 TUTOR Test Id: 72490738 QId: 30817 (921666) 28 of 75 A A A The home health nurses visits a 72-year-old client with pneumonia who was discharged from the hospital 3 days ago. The client has less of a productive cough at night but now reports sharp chest pain with inspiration. Which finding is most important for the nurse to report to the health care provider? 1. Bronchial breath sounds [3%] 2. Increased tactile fremitus [13%] 3. Low-pitched wheezing (rhonchi) [11%] 4. Pleural friction rub [72%] Incorrect Correct answer 4 Answered correctly 72% Time: 22 seconds Updated: 01/13/2017 Explanation: Pleurisy is characterized by stabbing chest pain that usually increases on inspiration or with cough. It is caused by inflammation of the visceral pleura (over the lung) and the parietal pleura (over the chest cavity). The pleural space (between the 2 layers) normally contains about 10 mL of fluid to help the layers glide easily with respiration. When inflamed, they rub together, causing pleuritic pain. A pleural friction rub is auscultated in the lateral lung fields over the area of inflammation. The sound is produced by the 2 layers rubbing together and can indicate pleurisy, a complication of pneumonia. It is characterized by squeaking, crackling, or the sound heard when the palm is placed over the ear and the back of the hand is rubbed with the fingers. Complications of pneumonia are more prevalent in elderly clients with underlying chronic disease. (Option 1) Clients with consolidative lung processes (pneumonia) may also have bronchial breath sounds due to over-transmission of sound over the chest wall. Breath sounds are diminished or absent over a pleural effusion or pneumothorax. (Option 2) Palpable vibration felt on the chest wall is known as fremitus. Sound travels faster in solids (consolidation) than in an aerated lung, resulting in increased fremitus in pneumonia. It is an expected finding in clients with pneumonia. It is concerning if decreased because the client may be developing pleural effusion. (Option 3) Low-pitched wheezing (rhonchi) is a continuous adventitious breath sound heard over the large airways, usually during expiration. It indicates the presence of secretions in the larger airways and is an expected finding as pneumonia resolves. Educational objective: Pleurisy is characterized by stabbing pleuritic chest pain that increases on inspiration. It is a complication of pneumonia caused by inflamed parietal and visceral pleurae rubbing together. Copyright © UWorld. All rights reserved.

Block Time Remaining: 00:38:50 TUTOR Test Id: 72490738 QId: 30763 (921666) 31 of 75 A A A The nurse conducts a program about strategies for preventing community-acquired pneumonia at a center for senior citizens. Which statement made by a participant indicates the need for further instruction? 1. "I got the flu vaccine, and it can help to prevent pneumonia." [23%] 2. "I got the one-time pneumonia shot, so I won't need it again." [67%] 3. "I stopped smoking a year ago, so that should help me a lot." [4%] 4. "I try to avoid going to the mall during the winter months." [3%] Omitted Correct answer 2 Answered correctly 67% Time: 1 seconds Updated: 04/03/2017 Explanation: A once-in-a-lifetime pneumococcal vaccination is ineffective for preventing community-acquired pneumonia (CAP). The current guidelines for pneumococcal vaccination state that all adults age ≥65 should receive 2 pneumococcal vaccinations: PCV13, or 13-valent pneumococcal conjugate vaccine (Prevnar 13) followed by PPSV23, or 23-valent pneumococcal polysaccharide vaccine (Pneumovax 23) ≥1 year apart. In addition, pneumococcal pneumonia revaccination (PPSV23) is recommended after 5 years for clients who are immunocompromised, those with a splenectomy, and those who are ≥65 years old if the first dose was given before this age. (Option 1) CAP often follows a viral illness; therefore, an annual influenza vaccination is an effective prevention strategy for CAP. (Option 3) Smoking cessation is an effective prevention strategy for CAP. Smoking or exposure to secondhand smoke is a significant risk factor associated with pneumococcal infections, especially in individuals age ≥65. (Option 4) Effective prevention strategies for CAP include using proper cough etiquette, practicing respiratory and hand hygiene, and avoiding crowds and contact with individuals with viral respiratory illnesses. Educational objective: Effective prevention strategies for CAP include smoking cessation, vaccination for influenza and pneumococcal pneumonia, avoidance of contact with individuals with viral respiratory illnesses, respiratory and hand hygiene practices, and use of proper cough etiquette. Copyright © UWorld. All rights reserved.

Block Time Remaining: 00:38:58 TUTOR Test Id: 72490738 QId: 30557 (921666) 32 of 75 A A A A client got sick shortly after a snowstorm knocked out electrical power to the house 3 days ago. The client has been weak and tired and unable to leave the house or go to work. The client reports a continual dull headache, nausea, dizziness, and shortness of breath. Which question would be most helpful for the nurse to ask while completing the history? 1. "Did you get influenza (flu) vaccine this year?" [21%] 2. "Have you ever experienced seasonal affective disorder in the winter?" [11%] 3. "How have you heated your house and stayed warm since the power went out?" [61%] 4. "What do you do for a living?" [4%] Omitted Correct answer 3 Answered correctly 61% Time: 8 seconds Updated: 01/19/2017 Explanation: Carbon monoxide (CO) is a tasteless, colorless, odorless gas. It is a byproduct of the incomplete burning of carbon-containing materials (eg, oil, kerosene, coal, wood). CO toxicity is most often associated with smoke inhalation during a structure fire. Other potential sources include gas and fuel oil heating systems, gas hot water heaters, coal or wood stoves, gas grills, and engine exhaust. Clients who are victims of a house fire or those found in a closed garage with the car running are easily identified because they have a known history. Most clients who come to the emergency department with CO toxicity have vague, nonspecific signs and symptoms and the diagnosis is often missed. It is essential to ask the right questions during the history to make an accurate diagnosis, initiate appropriate diagnostic testing and treatment, and prevent possible neurologic impairment associated with toxicity. To help identify the presence of elevated levels of CO in the home, the nurse can ask about the following: Similar symptoms in other family members that developed at the same time A sick indoor pet with sick family members (pets do not share viral illnesses or eat the same food as people) Heating and cooking sources (fuel-burning appliances); the risk of CO toxicity from malfunctioning heaters and inadequate ventilation increases during the fall and winter months (Option 1) Influenza can cause similar symptoms; however, the symptom onset in relation to the loss of power makes CO poisoning more likely. (Option 2) Sadness and depression are manifestations of seasonal affective disorder. (Option 4) This client became ill after the power in his house went out, not before, and probably started using space heaters; occupation is therefore not an issue. Educational objective: Most clients who come to the emergency department with CO toxicity have vague, nonspecific signs and symptoms (eg, headache, fatigue, dizziness, nausea, vomiting, dyspnea) and the diagnosis is often missed. Asking about family members and pets experiencing similar symptoms and about heating and cooking sources can help determine the diagnosis of CO toxicity. Copyright © UWorld. All rights reserved.

Block Time Remaining: 00:38:59 TUTOR Test Id: 72490738 QId: 30789 (921666) 33 of 75 A A A A nurse on the medical surgical unit has just received report. Which client should be seen first? 1. Client 1 day post femoral-popliteal bypass grafting who has an intravenous (IV) antibiotic due now [1%] 2. Client diagnosed with deep venous thrombosis (DVT) yesterday who reports some chest discomfort and cough [75%] 3. Client with hypertension and blood pressure of 180/92 mm Hg who reports a headache [8%] 4. Client on fall precautions who just called the nurses' station for assistance in using the bathroom immediately [14%] Omitted Correct answer 2 Answered correctly 75% Time: 1 seconds Updated: 05/11/2017 Explanation: The client with DVT who is experiencing chest discomfort and cough should be seen first. This client is exhibiting possible signs of pulmonary embolism (PE), which can be a life-threatening complication. Signs and symptoms of PE include dyspnea, hypoxemia, tachypnea, cough, chest pain, hemoptysis, tachycardia, syncope, and hemodynamic instability. The nurse should elevate the head of the bed, administer oxygen, and assess the client. The health care provider should be notified of these findings. (Option 1) The administration of an IV antibiotic is important but should be done after the nurse has assessed the client with DVT. (Option 3) This client is hypertensive and most likely has a headache due to the high blood pressure. The nurse should assess this client after the client with DVT and administer any antihypertensives needed. (Option 4) This client can be delegated to unlicensed assistive personnel who can go to the room immediately. Educational objective: The nurse should prioritize the assessment of any client with DVT who is experiencing respiratory signs and symptoms and/or chest pain due to potential development of PE. Copyright © UWorld. All rights reserved.

Block Time Remaining: 00:39:00 TUTOR Test Id: 72490738 QId: 32064 (921666) 34 of 75 A A A The nurse cares for a child newly diagnosed with cystic fibrosis. What should be included in the client's multidisciplinary plan of care to be discussed with the parents? Select all that apply. 1. Chest physiotherapy 2. Genetic counseling 3. Low-calorie diet 4. Oral fluid restriction 5. Spiritual support Omitted Correct answer 1,2,5 Answered correctly 44% Time: 1 seconds Updated: 12/24/2016 Explanation: Cystic fibrosis (CF) is a genetic disorder involving the cells that line the respiratory, gastrointestinal (GI), and reproductive tracts. A defective protein responsible for transporting sodium and chloride causes the secretions from exocrine glands in these areas to be thicker and stickier than normal. These abnormal secretions plug smaller airway passages and ducts in the GI tract. Secretions of impaired digestive enzymes in the GI tract result in ineffective absorption of essential nutrients. The sticky respiratory secretions lead to an inability to clear the airway and a chronic cough. The client eventually develops chronic lung disease (bronchiectasis). As a result of these changes, the client's lifespan is shortened; most affected individuals live only into their 30s. Chest physiotherapy helps remove sticky secretions that cause ineffective airway clearance. Clients and parents should receive genetic testing and counseling as CF is transmitted in an autosomal recessive inheritance pattern. Spiritual support should be offered as clients must deal with the impact of CF on lifespan and future pregnancies. (Option 3) Adequate nutrition cannot occur due to defective digestive enzymes and impaired nutrient absorption. A diet high in fat and calories is recommended. (Option 4) Oral fluids are not restricted, and liberal intake is recommended to assist with thinning the sticky respiratory secretions. Educational objective: Clients with cystic fibrosis should adhere to a diet high in fat and calories to combat nutrient malabsorption. Liberal fluid intake is encouraged to loosen the thick secretions. Spiritual support should be offered to deal with the shortened life span. Copyright © UWorld. All rights reserved.

Block Time Remaining: 00:39:01 TUTOR Test Id: 72490738 QId: 30688 (921666) 35 of 75 A A A The charge nurse of the emergency department (ED) is mentoring a new registered nurse (RN). They are caring for a client who has a chest tube connected to wall suction for a pneumothorax. The client is being transferred from the ED to the telemetry unit. Which action by the new RN would cause the charge nurse to intervene? 1. Clamping the chest tube at the insertion site during the transfer [69%] 2. Disconnecting the suction tubing from the wall suction unit [7%] 3. Hanging the chest tube collection unit to the underside of the stretcher [11%] 4. Taping connections between the chest tube and suction tubing [11%] Omitted Correct answer 1 Answered correctly 69% Time: 1 seconds Updated: 01/13/2017 Explanation: Clamping the chest tube during transport is contraindicated. Doing so can cause air to accumulate in the pleural cavity as it has no means of escape. This can lead to the development of a tension pneumothorax, a potentially life-threatening condition. A tension pneumothorax results in compression of the unaffected lung and pressure on the heart and great vessels. As the pressure increases, venous return is decreased and cardiac output falls. (Option 2) The wall suction needs to be temporarily disconnected during transport. It should be reconnected promptly at the destination. (Option 3) The chest tube collection unit should be hung below the level of the chest to promote drainage and keep fluids from re-entering the chest cavity. (Option 4) All connections should be secured with tape to prevent accidental disconnection or air to enter the system. Educational objective: Chest tubes should not be clamped during transport of a client. A clamped chest tube may cause a tension pneumothorax, a potentially life-threatening event. Copyright © UWorld. All rights reserved.

Block Time Remaining: 00:39:05 TUTOR Test Id: 72490738 QId: 30710 (921666) 38 of 75 A A A A client is admitted with a pulmonary embolus. The nurse assesses restlessness, one-word dyspnea and shortness of breath with activity, tachycardia, pleuritic chest pain, and severe anxiety. Arterial blood gases indicate respiratory alkalosis and hypoxemia. When initiating the care plan, the nurse should choose which nursing diagnosis as the highest priority? 1. Activity intolerance related to imbalance between oxygen supply and demand [6%] 2. Acute pain related to inspiration and inflammation of pleura [1%] 3. Anxiety related to fear of the unknown, chest pain, and dyspnea [5%] 4. Impaired gas exchange related to ventilation-perfusion imbalance [86%] Omitted Correct answer 4 Answered correctly 86% Time: 2 seconds Updated: 03/22/2017 Explanation: Pulmonary embolism (PE) is usually caused by a dislodged thrombus that travels through the pulmonary circulation, becomes lodged in a pulmonary vessel, and causes an obstruction to blood flow in the lung. The nursing diagnosis of impaired gas exchange involves an alteration in the normal exchange of oxygen and carbon dioxide at the alveolar-capillary membrane, resulting in inadequate oxygenation and hypoxemia (respiratory alkalosis, pO2 <80 mm Hg, restlessness, dyspnea, and tachycardia). Impaired gas exchange related to a ventilation-perfusion (V/Q) imbalance is the highest priority nursing diagnosis. It addresses the most basic physiologic need—oxygen. Clients will not survive without adequate oxygenation. (Options 1, 2, and 3) Activity intolerance, acute pain, and anxiety elicit autonomic responses (exertional discomfort, dyspnea, tachycardia) and are all appropriate nursing diagnoses. However, none are the highest priority or pose the greatest threat to survival. Educational objective: Activity intolerance, anxiety, acute pain, and impaired gas exchange are all appropriate nursing diagnoses to include in the plan of care for a client with PE. The highest priority nursing diagnosis is the one that poses the greatest threat to the client's survival. Copyright © UWorld. All rights reserved.

Block Time Remaining: 00:39:06 TUTOR Test Id: 72490738 QId: 30254 (921666) 39 of 75 A A A A client with community-acquired pneumonia is receiving 0.9% normal saline (NS) at 50 mL/hr. Pulse oximetry shows 95% on nasal O2 at 3 L/min. The nurse identifies a nursing diagnosis of ineffective airway clearance. Which prescription would the nurse expect to best facilitate secretion removal? 1. Incentive spirometer every 2 hours [45%] 2. Increase 0.9% NS to 125 mL/hr [20%] 3. Increase nasal oxygen to 4 L/min [1%] 4. Place the client in semi-Fowler's position [33%] Omitted Correct answer 2 Answered correctly 20% Time: 1 seconds Updated: 05/25/2017 Explanation: Airway clearance is impaired with inadequate hydration. The client is receiving NS at only 50 mL/hr (1200 mL/24 hr). In addition, fever and tachypnea increase insensible losses through the skin and respiration. A fluid intake of 2,500-3,000 mL/day is recommended in clients with pneumonia; additional fluids are needed to replace insensible losses. Low-pitched wheezing indicates the presence of secretions in the airways. Increasing hydration by increasing the infusion rate to 125 mL/hr (eg, 3,000 mL/day) will help thin secretions and facilitate expectoration of mucus, and is the best choice for this client. (Option 1) An incentive spirometer will increase ventilation, reverse atelectasis (crackles), and facilitate gas exchange. Although it encourages deep breathing and coughing, it is not the best choice for thick secretion removal. (Option 3) The saturation (95%) is adequate. Increasing the oxygen flow rate will not facilitate secretion removal and so is not the best choice for this client. (Option 4) Positioning in high-Fowler's (not semi-Fowler's) position would improve gas exchange. With the effects of gravity, it would be easier for the client to cough up thinner secretions. Semi-Fowler's position is not the best choice for this client. Educational objective: Airway clearance is impaired with inadequate hydration. A fluid intake of 2,500-3,000 mL/day is recommended in clients with pneumonia, and additional fluids are needed to replace insensible losses associated with fever and tachypnea. Increasing oral and intravenous hydration helps to thin secretions and facilitate expectoration of mucus. Copyright © UWorld. All rights reserved.

Block Time Remaining: 00:39:07 TUTOR Test Id: 72490738 QId: 30321 (921666) 40 of 75 A A A A client with an asthma exacerbation has been using her albuterol rescue inhaler 10-12 times a day because she cannot take a full breath. What possible side effects of albuterol does the nurse anticipate the client will report? Select all that apply. 1. Constipation 2. Difficulty sleeping 3. Hives with pruritus 4. Palpitations 5. Tremor Omitted Correct answer 2,4,5 Answered correctly 51% Time: 1 seconds Updated: 04/26/2017 Explanation: Albuterol is a short-term beta-adrenergic agonist used as a rescue inhaler to treat reversible airway obstruction associated with asthma. Dosing in an acute asthma exacerbation should not exceed 2-4 puffs every 20 minutes x 3. If albuterol is not effective, an inhaled corticosteroid is indicated to treat the inflammatory component of the disease. Albuterol is a sympathomimetic drug. Expected side effects mimic manifestations related to stimulation of the sympathetic nervous system, and commonly include insomnia, nausea and vomiting, palpitations (from tachycardia), and mild tremor. (Option 1) Constipation is not a common side effect of inhaled beta-agonist drugs. (Option 3) Hives can occur as a sign of an allergic reaction and are not a common anticipated side effect of an inhaled beta-agonist drug. Educational objective: Albuterol is a short-term beta-agonist rescue drug used to control symptoms of airway obstruction and promote bronchodilation. It is a sympathomimetic drug; common expected side effects include insomnia, nausea and vomiting, palpitations (tachycardia), and mild tremor. Copyright © UWorld. All rights reserved.

Block Time Remaining: 00:39:08 TUTOR Test Id: 72490738 QId: 33392 (921666) 41 of 75 A A A The nurse receives an obese client in the postanesthesia care unit who underwent a procedure under general anesthesia. The nurse notes an oxygen saturation of 88%. Which is the most appropriate initial intervention? 1. Assess pupillary response [6%] 2. Auscultate lung sounds [52%] 3. Inform anesthesia professional [3%] 4. Perform head tilt and chin lift [37%] Omitted Correct answer 4 Answered correctly 37% Time: 1 seconds Updated: 12/27/2016 Explanation: Head tilt and chin lift is a maneuver used to open the airway. The tongue may fall back and occlude the airway due to muscular flaccidity after general anesthesia. Manifestations associated with airway obstruction include snoring, use of accessory muscles, decreased oxygen saturations, and cyanosis. (Option 1) Constricted pupils can help identify opioid overdose. However, this should not be assessed before opening the airway. (Option 2) Auscultation of lung sounds should be done for every client as part of the postoperative assessment. However, the initial goal is to return the oxygen saturation level to normal (95%-100%). Hypoxia in an obese postoperative client who received general anesthesia is most likely due to airway obstruction. (Option 3) The anesthesia professional may need to be informed, but methods to restore the oxygen saturation level should be tried first. The anesthesia professional may then want to assess the sedation level of the client and prescribe a reversal agent. Educational objective: Postoperative client care after general anesthesia requires careful monitoring for hypoxia. One of the first nursing interventions is the head tilt and chin lift to open an occluded airway. Copyright © UWorld. All rights reserved.

Block Time Remaining: 00:39:09 TUTOR Test Id: 72490738 QId: 31093 (921666) 42 of 75 A A A The nurse is giving report at the end of a shift to the incoming nurse at 1900. A client was admitted with pneumonia that morning. Which information is most important for the nurse to communicate about the client during the change-of-shift report (hand-off)? 1. Chest x-ray showed left lobe infiltrate and white blood cell count of 14,000/mm3 [33%] 2. Client's spouse was rude to the nurse earlier [1%] 3. Current respirations are 24/min; pulse oximetry is 93% on 2 L/min [63%] 4. Intravenous (IV) line has been infusing without complications [1%] Omitted Correct answer 3 Answered correctly 63% Time: 1 seconds Updated: 12/09/2016 Explanation: Current respiratory status would be the most essential as it is current, relevant, objective information related to the reason the client is hospitalized. This information helps the incoming nurse prioritize and get a baseline measurement of the client's status today. It is not always documented in the chart as assessments are usually charted every 4, 8, or 12 hours. The abnormal findings are concerning enough for the nurse to recheck the client and possibly institute some additional nursing interventions. This information communicates the client's responses to the current treatment. Typical reports start off with the client, room number, age, gender, health care provider, and admitting and current diagnoses. They should include the client's location (if off the unit), key treatment plan goals/steps to be achieved or in progress, and client responses to treatment. Priority needs and any changes from previous days should be included. (Option 1) These are expected findings in a client with pneumonia and were part of the criteria used to make the medical diagnosis. The findings would be significant if there was a positive or negative trend to report, but an isolated, expected result is not helpful in planning care. The nurse can find these results in the client medical record. (Option 2) Personal opinion/value judgment not pertinent to providing care should be omitted. This would not be the case if the client made a comment about the hospitalization or treatment plan that the nurse needed to address. (Option 4) It would be assumed the IV site was patent without phlebitis or infiltration, or the nurse would have changed it. The nurse will need to make an individual assessment as to current patency. Educational objective: Key pertinent information related to the client's current condition is given during shift report/hand-off. This information would not be readily available to the incoming nurse. The nurse should not just recite facts from the medical record that any health care provider could access. Copyright © UWorld. All rights reserved.

Block Time Remaining: 00:39:10 TUTOR Test Id: 72490738 QId: 32719 (921666) 43 of 75 A A A Based on the lung assessment information included in the hand-off report, which client should the nurse assess first? 1. Client 1-day postoperative abdominal surgery who has fine inspiratory crackles at the lung bases [19%] 2. Client with chronic bronchitis who has rhonchi in the anterior and posterior chest [5%] 3. Client with right-sided pleural effusion who has decreased breath sounds at the right lung base [30%] 4. Client with severe acute pancreatitis who has inspiratory crackles at the lung bases [45%] Omitted Correct answer 4 Answered correctly 45% Time: 1 seconds Updated: 03/24/2017 Explanation: Clients with acute pancreatitis can develop respiratory complications including pleural effusions, atelectasis, and acute respiratory distress syndrome (ARDS). These complications are often due to activated pancreatic enzymes and cytokines that are released from the pancreas into the circulation and cause focal or systemic inflammation. ARDS is the most severe form of these complications and can rapidly progress to respiratory failure within a few hours. Therefore, the presence of inspiratory crackles in this client could indicate early ARDS and needs to be assessed further for progression. (Option 1) Fine crackles are a series of distinct, discontinuous, and high-pitched snapping sounds usually heard on inspiration. The sound originates as small atelectatic bronchioles quickly reinflate and can be expected in clients who have undergone abdominal surgery due to shallow breathing related to pain. Although the presence of fine crackles requires treatment (eg, ambulation, deep breathing), this is not the priority assessment. (Option 2) Rhonchi are continuous, low-pitched wheezes usually heard on expiration that sound like moaning or snoring. The sound originates from air moving through large airways (bronchi) filled with mucus secretions and are expected in clients with chronic bronchitis. Although they require treatment (eg, medication, mobilization of secretions), this is not the priority assessment. (Option 3) The lung under the pleural effusion is compressed, and the breath sounds are decreased/absent if auscultated over the area; this is an expected finding. Until the pleural effusion is treated with diuretics or thoracentesis, these findings will remain unchanged. Educational objective: Clients with acute pancreatitis are at high risk for developing acute respiratory distress syndrome. Copyright © UWorld. All rights reserved.

Block Time Remaining: 00:39:11 TUTOR Test Id: 72490738 QId: 30558 (921666) 44 of 75 A A A A self-employed auto mechanic is diagnosed with carbon monoxide poisoning. Admission vital signs are blood pressure 90/42 mm Hg, pulse 84/min, respirations 24/min, and oxygen saturation 94% on room air. What is the nurse's priority action? 1. Administer 5 mg inhaled albuterol nebulizer treatment to decrease inflammatory bronchoconstriction [12%] 2. Administer 100% oxygen using a nonrebreather mask with flow rate of 15 L/min [48%] 3. Administer methylprednisolone to decrease lung inflammation from toxic inhalant [21%] 4. Titrate oxygen to maintain pulse oximeter saturation of >95% [17%] Omitted Correct answer 2 Answered correctly 48% Time: 1 seconds Updated: 04/26/2017 Explanation: The purpose of hemoglobin (Hgb) is to pick up oxygen in the lungs and deliver it to the tissues. It must be able to pick up oxygen and release it in the right places. Carbon monoxide (CO) has a much stronger bond to Hgb than oxygen does. Consequently, CO displaces oxygen from Hgb, causing hypoxia that is not reflected by a pulse oximeter reading. The nurse's primary action is to administer highly concentrated (100%) oxygen using a nonrebreather mask at 15 L/min in order to reverse this displacement of oxygen. (Option 1) Albuterol is not a priority action as bronchoconstriction is not a consequence of CO poisoning. (Option 3) Administration of corticosteroids is not a priority/primary action as direct inflammation of the lungs is not an underlying cause for hypoxemia and hypoxia associated with CO poisoning. (Option 4) When all available Hgb binding sites are occupied (oxyhemoglobin or carboxyhemoglobin), saturation (SaO2) is 100%. The conventional pulse oximeter cannot differentiate carboxyhemoglobin from oxyhemoglobin as both absorb the oximeter's red and infrared light wavelengths. Consequently, the pulse oximeter reading may be adequate (>90%), but severe hypoxemia and hypoxia may be present. Alternate methods of CO saturation measurement (eg, multiple wavelength CO pulse oximeter, spectrographic blood gas analysis) are recommended. Educational objective: The conventional pulse oximeter is not effective in identifying hypoxia in CO poisoning; diagnosis requires co-oximetry of a blood gas sample. The priority action is to administer 100% oxygen using a nonrebreather mask to treat hypoxia and help eliminate CO. Copyright © UWorld. All rights reserved.

Block Time Remaining: 00:39:12 TUTOR Test Id: 72490738 QId: 30800 (921666) 45 of 75 A A A The nurse provides discharge instructions to a 67-year-old client with chronic bronchitis who was hospitalized for community-acquired pneumonia. Which instructions should be included in the discharge teaching plan? Select all that apply. 1. "Avoid the use of over-the-counter cough suppressant medicines." 2. "Oral antibiotics are not needed at home as you had intravenous (IV) therapy in the hospital." 3. "Pneumonia vaccination is not needed as you now have lifelong immunity." 4. "Schedule a follow-up with the health care provider (HCP) and chest x-ray." 5. "Use a cool mist humidifier in your bedroom at night." 6. "Use the incentive spirometer at home." Omitted Correct answer 1,4,5,6 Answered correctly 49% Time: 1 seconds Updated: 06/01/2017 Explanation: Clients should be taught to understand that symptoms of pneumonia (eg, cough, sputum production, shortness of breath, fatigue, and activity intolerance) remain after discharge even though the bacteria are no longer present and will dissipate over a 2-4 week period, depending on current health status and preexisting conditions. Discharge teaching includes the following instructions: Avoid the use of over-the-counter cough suppressant medicines. Unless prescribed by the HCP, cough suppressants are avoided as they impair secretion clearance, especially in clients with chronic bronchitis. Schedule a follow-up with the HCP and chest x-ray. Follow-up is needed at about 2 weeks after completion of antibiotic therapy. X-ray may be needed at a later time in certain high-risk clients to make sure the pneumonia is resolved with no underlying cancer. Use a cool mist humidifier in your bedroom at night. Humidifiers keep mucus membranes moist, maintain effectiveness of the mucociliary escalator, and facilitate expectoration of mucus. A warm bath also loosens the secretions. Continue using the incentive spirometer at home. Deep breathing and coughing promote lung expansion, ventilation, oxygenation, and airway clearance. Drink 1-2 liters of water a day, if not contraindicated, to help thin secretions and facilitate mobilization. Limit caffeine and alcohol as they can dry mucus membranes due to diuretic effects. Notify the HCP of any increase in symptoms (eg, shortness of breath, cough, sputum production, chest pain, fever, confusion). Avoid all tobacco products and second-hand smoke as these irritate the airways and impair mucociliary clearance and oxygenation. Eat a balanced diet, increase activity slowly over about 2 weeks, and take rest periods when needed to help maintain resistance to infection. (Option 2) After a client has IV antibiotic therapy, completing a full course of oral antibiotic therapy is necessary to prevent reoccurrence of disease and antibiotic resistance. (Option 3) Contracting pneumonia does not provide lifelong immunity to the disease. Yearly influenza vaccination and pneumonia vaccination as directed by the HCP are recommended. Educational objective: Discharge instructions for a client recovering from pneumonia focus on proper medication regimen, lung expansion and coughing techniques, activity level, hydration, nutrition, avoidance of tobacco products, reportable manifestations (eg, respiratory distress, chest pain, fever, cough, change in mucus), follow-up care, influenza and pneumonia vaccinations, and respiratory and hand hygiene. Copyright © UWorld. All rights reserved.

Block Time Remaining: 00:39:13 TUTOR Test Id: 72490738 QId: 30570 (921666) 46 of 75 A A A A nurse in the emergency department assesses 4 clients. Based on the laboratory results, which client is the highest priority for treatment? 1. Client with abdominal pain, respirations 28/min, and blood alcohol level 80 mg/dL (0.08 mg% [17.4 mmol/L]) [25%] 2. Client with chronic obstructive pulmonary disease, pH 7.34, pO2 86 mm Hg (11.5 kPa), pCO2 48 mm Hg (6.4 kPa), and HCO3 30 mEq/L (30 mmol/L) [22%] 3. Client with dull headache, pulse oximeter reading 95%, and serum carboxyhemoglobin level 20% [27%] 4. Client with emesis of 100 mL coffee-ground gastric contents and serum hemoglobin 15 g/dL (150 g/L) [24%] Omitted Correct answer 3 Answered correctly 27% Time: 1 seconds Updated: 06/03/2017 Explanation: Carbon monoxide (CO) is a toxic inhalant that enters the blood and binds more readily to hemoglobin than oxygen does. When hemoglobin is saturated with CO, the pulse oximeter reading is falsely normal as conventional devices detect saturated hemoglobin only and cannot differentiate between CO and oxygen. The diagnosis of CO poisoning is often missed in the emergency department because symptoms are nonspecific (eg, headache, dizziness, fatigue, nausea, dyspnea) and the pulse oximeter reading often appears within normal limits. A serum carboxyhemoglobin test is needed to confirm the diagnosis. Normal values are <5% in nonsmokers and slightly higher (<10%) in smokers. This client with CO poisoning is the highest priority for treatment and requires immediate administration of 100% oxygen to increase the rate at which CO dissipates from the blood to prevent tissue hypoxia and severe hypoxemia (Option 3). (Option 1) Normal blood alcohol content is 0 mg/dL (0 mmol/L), and the legal level for driving under the influence is 80 mg/dL (0.08 mg% [17.4 mmol/L]). The client's abdominal pain and increased respiratory rate require adequate assessment but are not the highest priority. (Option 2) The arterial blood gases indicate compensated respiratory acidosis, which is characteristic for a client with chronic obstructive pulmonary disease; this is not the highest priority. (Option 4) Emesis of 100 mL coffee-ground gastric contents would indicate an older, not fresh, gastrointestinal bleed; the hemoglobin level is normal (13.2-17.3 g/dL [132-173 g/L] for males, 11.7-15.5 g/dL [117-155 g/L] for females). The cause of the gastrointestinal bleed must be determined, but this is not the highest priority. Educational objective: Clients with carbon monoxide (CO) poisoning have elevated serum carboxyhemoglobin levels (normal <5% in nonsmokers) and false-normal pulse oximeter readings. They require immediate administration of 100% oxygen to correct hypoxemia and eliminate toxic CO from the blood. Copyright © UWorld. All rights reserved.

Block Time Remaining: 00:39:14 TUTOR Test Id: 72490738 QId: 30777 (921666) 47 of 75 A A A An elderly client with pneumonia has a temperature of 102.2 F (39 C), blood pressure 98/66 mm Hg, pulse 115/min, and respirations 30/min. Assessment reveals crackles in the right lower lobe, dusky nail beds, and dry mucus membranes. Arrange the nursing actions chronologically to prioritize care. All options must be used. Your Response/ Incorrect Response Correct Response Oxygen per nasal cannula at 4 L/min Normal saline (NS) solution at 125 mL/hr Blood cultures x 2 for temperature >102 F (38.9 C) Levofloxacin 750 mg intravenous (IV) every 24 hours Teaching incentive spirometer use Omitted Correct answer 4,2,1,3,5 Answered correctly 55% Time: 1 seconds Updated: 03/23/2017 Explanation: The nurse prioritizes nursing actions for the hospitalized client with pneumonia in the following order: Oxygen per nasal cannula at 4 L/min - This client is in respiratory distress (respirations 30/min, dusky nail beds). Oxygen administration is the priority action. NS at 125 mL/hr - Most elderly clients with pneumonia present with dehydration (dry mucus membranes, low blood pressure). Initiation of IV fluids is important to thin secretions, facilitate expectoration of mucus, and provide access for antibiotic therapy. Blood cultures x 2 for temperature >102 F (38.9 C) - Cultures should always be drawn before antibiotic administration, as these can be inaccurate if drawn afterward. Identifying the causative pathogen is necessary to ensure that the appropriate antibiotic is prescribed. Cultures are drawn x 2 (from 2 different venipuncture sites) to rule out contaminants. Levofloxacin 750 mg IV every 24 hours - Levofloxacin (Levaquin) is a fluoroquinolone antibiotic recommended for the treatment of pneumococcal pneumonia. Antibiotics should be administered as soon as possible after the pneumonia diagnosis is made and cultures have been drawn. Incentive spirometer every 2 hours - Deep breathing can be performed after the initiation of antibiotics. Incentive spirometry increases alveolar expansion, facilitates removal of secretions, and prevents atelectasis. Educational objective: The nurse must prioritize nursing actions appropriately by identifying the client's current health status and acuity level, threats to survival (airway, breathing, circulation), safety issues (infection), and desired outcomes. Copyright © UWorld. All rights reserved.

Block Time Remaining: 00:39:16 TUTOR Test Id: 72490738 QId: 31671 (921666) 48 of 75 A A A A client at 32 weeks gestation goes into cardiac arrest. What is the nurse's best action while performing cardiopulmonary resuscitation for this client? 1. Compress chest at second intercostal space, right sternal border [14%] 2. Perform chest compressions slightly higher on the sternum [53%] 3. Place hands just below the diaphragm to perform chest compressions [6%] 4. Position client in the supine position for optimal compressions [26%] Omitted Correct answer 2 Answered correctly 53% Time: 2 seconds Updated: 12/18/2016 Explanation: Common causes of sudden cardiac arrest in pregnant clients include embolism, eclampsia, magnesium overdoses, and uterine rupture. If cardiopulmonary resuscitation (CPR) is required, several modifications must be made to ensure efficacy of the rescue efforts. During pregnancy, the heart is displaced toward the left because the growing uterus pushes upward on the diaphragm, particularly in the third trimester. To accommodate this displacement, the hands should be placed on the sternum slightly higher than usual for chest compressions during CPR (Option 2). In addition, a gravid uterus can significantly compress the client's vena cava and aorta, thereby hindering effective blood flow during CPR. The uterus should be manually displaced to the client's left to reduce this pressure. The nurse can also place a rolled blanket or wedge under the right hip to displace the uterus. If return of spontaneous circulation (ROSC) does not occur after 4 minutes of CPR, emergency cesarean section is usually initiated. Delivery should occur within 5 minutes of initiating CPR. (Option 1) Compressions to the right sternal border will lack effectiveness as the heart is displaced to the left side during pregnancy. (Option 3) Chest compressions given below the diaphragm are not effective for ROSC and greatly increase the risk of injury to the client's uterus, spleen, or liver. (Option 4) In the supine position, the vena cava and aorta are compressed by the uterus, hindering effective blood flow during CPR. The uterus should be displaced to the left to reduce pressure. Educational objective: Two important modifications for cardiopulmonary resuscitation of a pregnant client include performing chest compressions slightly higher on the sternum and displacing the uterus to the client's left side. Copyright © UWorld. All rights reserved.

Block Time Remaining: 00:39:37 TUTOR Test Id: 72490738 QId: 30267 (921666) 49 of 75 A A A An elderly client is admitted with chronic obstructive pulmonary disease (COPD) exacerbation. Pulse oximetry is 84% on room air. The client is restless, has expiratory wheezing and a productive cough, and is using his accessory muscles to breathe. Which prescription should the nurse question? 1. Albuterol 2.5 mg by nebulizer [9%] 2. Intravenous (IV) methylprednisolone 125 mg now and every 6 hours [6%] 3. IV morphine 2 mg now and may repeat every 2 hours [69%] 4. Oxygen at 2 L/min by nasal cannula [15%] Omitted Correct answer 3 Answered correctly 69% Time: 21 seconds Updated: 04/26/2017 Explanation: This client has exacerbation (wheezing) of chronic obstructive pulmonary disease (COPD). Restlessness is an early, subtle sign of hypoxemia. Although morphine can be used to decrease restlessness and slow the respiratory rate to decrease oxygen demand, it can depress respirations. Morphine and other medications (eg, benzodiazepines) that can depress the respiratory center should not be used in clients with COPD exacerbation as they can further worsen CO2 retention. The oxygen and medications should increase saturation and decrease restlessness, pulse, and respiratory rate. (Option 1) Albuterol is a beta agonist and is appropriate for the immediate relief of bronchoconstriction because of its rapid, short action. (Option 2) Methylprednisolone (Solu-Medrol) is a corticosteroid and is appropriate for decreasing inflammation of the lungs during an acute exacerbation of COPD. (Option 4) The initiation of low flow, low concentration oxygen at 2 L/min by nasal cannula is appropriate in a client with COPD. It is best to start oxygen at a lower concentration and titrate upward if the saturation does not reach 90% within 20-30 minutes as many clients with COPD rely on their hypoxemic drive to breathe. Educational objective: Morphine and other medications (eg, benzodiazepines) that can depress the respiratory center should not be used in clients with COPD exacerbation as they can further worsen CO2 retention. Copyright © UWorld. All rights reserved.

Block Time Remaining: 00:39:39 TUTOR Test Id: 72490738 QId: 32165 (921666) 51 of 75 A A A The nurse is gathering data on a client with obstructive sleep apnea. Which findings are consistent with this client's diagnosis? Select all that apply. 1. Difficulty arousing from sleep 2. Excessive daytime sleepiness 3. Morning headaches 4. Postural collapse and falling 5. Snoring during sleep 6. Witnessed episodes of apnea Omitted Correct answer 2,3,5,6 Answered correctly 25% Time: 1 seconds Updated: 01/09/2017 Explanation: Obstructive sleep apnea (OSA) is the most common type of breathing disorder during sleep and is characterized by repeated periods of apnea (>10 seconds) and diminished airflow (hypopnea). A partial or complete obstruction occurs due to upper airway narrowing that results from relaxation of the pharyngeal muscles or from the tongue falling back on the posterior pharynx due to gravity. During periods of apnea, desaturation (hypoxemia) and hypercapnia occur; these stimulate the client to arouse and breathe momentarily to restore airflow. These cycles of apnea and restored airflow can occur several hundred times per night, resulting in restless and fragmented sleep. Partners of clients with OSA witness loud snoring, apnea episodes, and waking with gasping or a choking sensation (Options 5 and 6). During the day, clients experience morning headaches, irritability, and excessive sleepiness. Excessive daytime sleepiness can lead to poor work performance, motor vehicle crashes, and increased mortality (Options 2 and 3). (Option 1) Frequent (not difficult) arousal from sleep is associated with OSA. (Option 4) Cataplexy is a brief loss of skeletal muscle tone or weakness that can result in a client falling down. It is associated with narcolepsy, a chronic neurologic sleep disorder. Educational objective: At night, clients with obstructive sleep apnea experience repeated periods of apnea, loud snoring, and interrupted sleep. During the day, morning headaches, irritability, and excessive sleepiness are common. Copyright © UWorld. All rights reserved.

Block Time Remaining: 00:39:42 TUTOR Test Id: 72490738 QId: 30492 (921666) 52 of 75 A A A A client was medicated with intravenous morphine 2 mg 2 hours ago to relieve moderate abdominal pain after appendectomy. The client becomes lethargic but arouses easily to verbal and tactile stimuli, and is oriented to time, place, and person. The pulse oximeter reading has dropped from 99% to 89% on room air. Which oxygen delivery device is the most appropriate for the nurse to apply? 1. [48%] 2. [19%] 3. [16%] 4. [15%] Omitted Correct answer 1 Answered correctly 48% Time: 3 seconds Updated: 04/26/2017 Explanation: The nasal cannula is the most appropriate oxygen delivery device to apply at this time because it is comfortable, used for the short term, inexpensive, and permits the client to eat and drink fluids. It can supply adequate oxygen concentrations of up to 44%. This client is most likely hypoventilating as a result of the opioid medication. The client is alert and oriented and able to follow directions. Because pain relief is effective according to the pain scale, the client should be able to breathe deeply through the nose, and the hypoxemia should reverse rapidly. (Option 2) The non-rebreather mask is used in emergencies, delivers high concentrations of oxygen (up to 90%-95%), requires a tight face seal, and is restrictive and uncomfortable. (Option 3) The simple face mask delivers a higher concentration of oxygen (40%-60%), is more uncomfortable and restrictive, must be removed to eat or drink, and is not appropriate at this time. It can be used if hypoxemia does not resolve. (Option 4) The Venturi mask is a more expensive device used to deliver a guaranteed oxygen concentration to clients with unstable chronic obstructive pulmonary disease. These clients cannot tolerate changes in oxygen concentration. Educational objective: The nasal cannula is an inexpensive, comfortable, low-flow oxygen delivery device capable of delivering oxygen concentrations of up to 44%. It can be used in the short term in responsive postoperative clients to treat hypoventilation and reverse hypoxemia effectively. Copyright © UWorld. All rights reserved.

Block Time Remaining: 00:39:42 TUTOR Test Id: 72490738 QId: 30492 (921666) 52 of 75 A A A A client was medicated with intravenous morphine 2 mg 2 hours ago to relieve moderate abdominal pain after appendectomy. The client becomes lethargic but arouses easily to verbal and tactile stimuli, and is oriented to time, place, and person. The pulse oximeter reading has dropped from 99% to 89% on room air. Which oxygen delivery device is the most appropriate for the nurse to apply? 1. [48%] 2. [19%] 3. [16%] 4. [15%] Omitted Correct answer 1 Answered correctly 48% Time: 3 seconds Updated: 04/26/2017 Explanation: The nasal cannula is the most appropriate oxygen delivery device to apply at this time because it is comfortable, used for the short term, inexpensive, and permits the client to eat and drink fluids. It can supply adequate oxygen concentrations of up to 44%. This client is most likely hypoventilating as a result of the opioid medication. The client is alert and oriented and able to follow directions. Because pain relief is effective according to the pain scale, the client should be able to breathe deeply through the nose, and the hypoxemia should reverse rapidly. (Option 2) The non-rebreather mask is used in emergencies, delivers high concentrations of oxygen (up to 90%-95%), requires a tight face seal, and is restrictive and uncomfortable. (Option 3) The simple face mask delivers a higher concentration of oxygen (40%-60%), is more uncomfortable and restrictive, must be removed to eat or drink, and is not appropriate at this time. It can be used if hypoxemia does not resolve. (Option 4) The Venturi mask is a more expensive device used to deliver a guaranteed oxygen concentration to clients with unstable chronic obstructive pulmonary disease. These clients cannot tolerate changes in oxygen concentration. Educational objective: The nasal cannula is an inexpensive, comfortable, low-flow oxygen delivery device capable of delivering oxygen concentrations of up to 44%. It can be used in the short term in responsive postoperative clients to treat hypoventilation and reverse hypoxemia effectively. Copyright © UWorld. All rights reserved.

Block Time Remaining: 00:39:44 TUTOR Test Id: 72490738 QId: 34215 (921666) 54 of 75 A A A A client presents to the emergency department with a stab wound to the chest. The nurse assesses tachycardia, tachypnea, and a sucking sound coming from the wound. Which of the following actions is priority? 1. Administer prescribed IV fluids [2%] 2. Apply supplemental oxygen via nonrebreather mask [16%] 3. Assist the health care provider to prepare for chest tube insertion [12%] 4. Cover the wound with petroleum gauze taped on three sides [68%] Omitted Correct answer 4 Answered correctly 68% Time: 1 seconds Updated: 03/16/2017 Explanation: In a traumatic, or "open," pneumothorax, air rushes in through the wound with each inspiration, creating a sucking sound, and fills the pleural space. The lungs cannot expand, so the client develops respiratory distress and air hunger. Tachycardia and hypotension result from impaired venous return, as the heart and great vessels shift with each breath. A tension pneumothorax may also develop if air cannot escape the pleural space. The priority action in this medical emergency is to apply a sterile occlusive dressing (eg, petroleum gauze) taped on three sides, preventing inward air flow while allowing air to escape the pleural space. (Option 1) This client's tachycardia and hypotension are likely related to pneumothorax and should improve once the pneumothorax is resolved; administering fluids alone would not help if the pneumothorax continues to worsen. Fluids are given to treat blood loss hypotension, but this should not be the first step in this case. (Option 2) Supplemental oxygen should be applied as needed after covering the wound. If possible, correcting the underlying cause is always a priority over treating manifestations. (Option 3) After covering the wound, chest tube placement is usually performed to evacuate air and blood from the pleural cavity. The client may need more than one chest tube to evacuate both air (placed higher) and fluid or blood (placed lower). Educational objective: A sucking chest wound indicates a traumatic, or "open," pneumothorax and is a medical emergency. Respiratory distress results from inability to expand the lung. The priority action is to apply a sterile occlusive dressing (eg, petroleum gauze) taped on three sides. Copyright © UWorld. All rights reserved.

Block Time Remaining: 00:39:38 TUTOR Test Id: 72490738 QId: 30301 (921666) 50 of 75 A A A The nurse is evaluating how well a client with chronic obstructive pulmonary disease understands the discharge teaching. Which statements made by the client indicate an understanding of the pursed-lip breathing technique? Select all that apply. 1. "I exhale for 2 seconds through pursed lips." 2. "I exhale for 4 seconds through pursed lips." 3. "I inhale for 2 seconds through my mouth." 4. "I inhale for 2 seconds through my nose, keeping my mouth closed." 5. "I inhale for 4 seconds through my nose, keeping my mouth closed." Omitted Correct answer 2,4 Answered correctly 41% Time: 1 seconds Updated: 05/07/2017 Explanation: The pursed-lip breathing technique helps to decrease shortness of breath by preventing airway collapse, promoting carbon dioxide elimination, and reducing air trapping in clients with chronic obstructive pulmonary disease (COPD). Clients with COPD are taught to use this technique when experiencing dyspnea as it increases ventilation and decreases work of breathing. Regular practice (eg, 5-10 minutes 4 times daily) enables the client to do pursed lip breathing when short of breath, without conscious effect. Clients are taught the following steps: Relax the neck and shoulders Inhale for 2 seconds through the nose with the mouth closed (Option 4) Exhale for 4 seconds through pursed lips. If unable to exhale for this long, exhale twice as long as inhaling (Option 2). (Option 1) Exhalation through pursed lips is done for 4 seconds, not 2 seconds, or twice as long as inhalation. (Option 3) Inhalation is done through the nose, not the mouth. (Option 5) Inhalation is done for 2 seconds, not 4 seconds. Educational objective: The pursed-lip breathing technique helps to decrease shortness of breath by preventing airway collapse, promoting carbon dioxide elimination, and reducing air trapping in clients with chronic obstructive pulmonary disease. Clients are taught to relax the shoulders and neck, inhale through the nose for 2 seconds with the mouth closed, and exhale through pursed lips for 4 seconds (or twice as long as inhalation). Copyright © UWorld. All rights reserved.

Block Time Remaining: 00:39:46 TUTOR Test Id: 72490738 QId: 30680 (921666) 55 of 75 A A A In the intensive care unit, a client is on mechanical ventilation (MV) after having undergone a fresh tracheostomy with retention sutures placed yesterday. The nurse hears the MV alarm sound and enters the room. The client is coughing, respirations are 40/min, heart rate is 132/min, and the pulse oximeter reading is 80%. The nurse also sees the tracheostomy tube lying on the client's chest. What is the nurse's immediate action? 1. Apply a rebreathing mask with high concentration oxygen at 12 L/min [29%] 2. Attempt to reinsert the tube with the obturator in place [24%] 3. Insert a sterile catheter into the stoma and suction the airway [20%] 4. Pull the retention sutures apart to lift the trachea and hold the stoma open [25%] Omitted Correct answer 4 Answered correctly 25% Time: 2 seconds Updated: 02/28/2017 Explanation: Accidental dislodgement of the tube after a fresh (immature, <1 week) tracheostomy is a medical emergency as the tract is not yet healed (matured). Significant tracheal inflammation, edema, bleeding, and closure of the tract (resulting in airway loss) can occur. The goal is to keep the stoma open to maintain the airway and oxygenate the client. If accidental dislodgement occurs, immediate nursing actions should include pulling the retention sutures apart (if present) to lift the trachea and hold the stoma open or inserting a curved hemostat to hold the stoma open if sutures are not present. If desaturation progresses while awaiting the arrival of the emergency team, the nurse can apply a sterile occlusive dressing over the stoma and ventilate the client with a bag-valve mask over the nose and mouth (using gentle pressure). (Option 1) Application of the rebreathing mask will not be effective due to the severity of respiratory distress and because oxygen can escape from the stoma. (Option 2) Before inserting a new tube into an immature tracheostomy, the stoma is first held open with a curved hemostat or by pulling the retention sutures. (Option 3) Advancing a suction catheter into an immature tract can cause increased inflammation, swelling, and bleeding; the oxygen will also be removed when suction is applied. Educational objective: Accidental dislodgement of the tube in a fresh (immature, < 1 week) tracheostomy is a medical emergency. Immediate nursing actions include pulling the retention sutures apart if present, or inserting a curved hemostat to hold the stoma open if sutures are absent. If desaturation progresses, the nurse should apply a sterile occlusive dressing over the stoma and ventilate the client with a bag-valve mask over the nose and mouth. Copyright © UWorld. All rights reserved.

Block Time Remaining: 00:39:47 TUTOR Test Id: 72490738 QId: 31243 (921666) 56 of 75 A A A A nurse is reviewing the laboratory results of a client admitted for an asthma exacerbation. Elevation of which of these cells indicates that the client's asthma may have been triggered by an allergic response? 1. Eosinophils [69%] 2. Lymphocytes [9%] 3. Neutrophils [14%] 4. Reticulocytes [6%] Omitted Correct answer 1 Answered correctly 69% Time: 1 seconds Updated: 12/31/2016 Explanation: Normal eosinophil count is 1%-2%. Elevated eosinophils are seen in allergy. In a client with an asthma exacerbation, a high eosinophil count would indicate an allergic trigger for the asthmatic response. The nurse should explore the client's allergy history and ways to reduce the allergic exposure that may be contributing to the exacerbation. (Option 2) Lymphocytes form the major part of immune system. Elevated levels are seen with viral infections and hematologic malignancies. (Option 3) Normal neutrophils are 55%-70%. Elevated neutrophils indicate infection. (Option 4) Reticulocytes are immature red blood cells. Normal reticulocyte count is 0.5%-2.0%. Levels are elevated in hemolytic anemia or hemorrhage when the marrow is attempting to compensate for lost blood. Educational objective: An elevated eosinophil count in the complete blood count is associated with allergy. Allergies are frequently triggers of asthma exacerbation. Copyright © UWorld. All rights reserved.

Block Time Remaining: 00:39:48 TUTOR Test Id: 72490738 QId: 30636 (921666) 57 of 75 A A A The nurse in the outpatient procedure unit is caring for a client immediately post bronchoscopy. Which assessment data indicate that the nurse needs to contact the health care provider immediately? 1. Absence of gag reflex [14%] 2. Bright red blood mixed with sputum [31%] 3. Headache [4%] 4. Respirations 10/min and saturation of 92% [48%] Omitted Correct answer 2 Answered correctly 31% Time: 1 seconds Updated: 12/17/2016 Explanation: An endoscopic bronchoscopy is a procedure in which the bronchi are visualized with a flexible fiberoptic bronchoscope that is passed down through the nose (or through the mouth, or endotracheal or tracheostomy tube). The client receives mild sedation (eg, midazolam) to provide relaxation and promote comfort. A topical anesthetic (eg, lidocaine, benzocaine) is applied to the nares and throat to suppress the gag and cough reflexes, prevent laryngospasm, and facilitate passage of the scope. The procedure is done to diagnose, obtain tissue samples for biopsy, lavage, and to remove secretions (mucus plugs), foreign objects, or abnormal tissue with a laser. Blood-tinged sputum is common and can occur from inflammation of the airway, but hemoptysis of bright red blood can indicate hemorrhage, especially if a biopsy was performed. Other complications include hypoxemia, hypercarbia, hypotension, laryngospasm, bradycardia, pneumothorax (rare), and adverse effects from medications used before and during the procedure. (Option 1) Absence of the gag reflex for about 2 hours following the procedure is expected from the topical anesthetic. (Option 3) Headache is not a complication of bronchoscopy. (Option 4) Respirations of 10/min and saturation of 92% are expected after mild sedation before and/or or during the procedure. Educational objective: Immediately post bronchoscopy, the nurse monitors for associated potential complications, including hemoptysis, hypoxemia, hypercarbia, hypotension, laryngospasm, bradycardia, pneumothorax, and adverse effects from medications used before and during the procedure. Copyright © UWorld. All rights reserved.

Block Time Remaining: 00:39:57 TUTOR Test Id: 72490738 QId: 30519 (921666) 60 of 75 A A A The nurse is assisting a client with a diagnosis of asthma in use of a peak flow meter. Place the steps for measuring peak expiratory flow using a peak flow meter in the correct order. All options must be used. Your Response/ Incorrect Response Correct Response Slide the indicator on the numbered scale on the flow meter to the 0 or lowest number and instruct the client to stand or sit as upright as possible Instruct the client to breathe in deeply, place the mouthpiece in the mouth, and close the lips tightly around it to form a seal Instruct the client to exhale as quickly and forcibly as possible and note the reading on the numbered scale Repeat the procedure 2 more times with a 5-10-second rest period between exhalations Record the highest reading achieved (personal best) Omitted Correct answer 5,1,2,4,3 Answered correctly 75% Time: 1 seconds Updated: 04/26/2017 Explanation: A peak flow meter is a portable, inexpensive, hand-held device used to measure the client's ability to push air out of the lungs. Flow meters are available in low ranges for measuring smaller volumes and for small children, and in a standard range for older children, teenagers, and adults. To obtain the most accurate readings to help guide, maintain, and evaluate treatment in clients with moderate to severe asthma, the procedure is carried out in the following order: Before using, slide the indicator on the numbered scale on the flow meter to 0 or lowest number and instruct the client to stand or sit as upright as possible Instruct the client to breathe in deeply, place the mouthpiece in the mouth, and the close lips tightly around it to form a seal Instruct the client to exhale as quickly and forcibly as possible and note the reading on the numbered scale Repeat the procedure 2 more times with a 5-10-second rest period between exhalations Record the highest reading achieved (personal best) Educational objective: A peak flow meter is a hand-held device used to measure the client's ability to push air out of the lungs. It is used for assessment of moderate to severe asthma. Copyright © UWorld. All rights reserved.

Block Time Remaining: 00:39:59 TUTOR Test Id: 72490738 QId: 30243 (921666) 61 of 75 A A A The registered nurse (RN) is caring for an elderly client with chronic obstructive pulmonary disease (COPD) whose pulse oximeter shows 91% on room air. After physical therapy, the client reports feeling "short of breath and exhausted" to the student nurse and says he just wants to sleep. To provide comfort, the student nurse initiates the prn nasal oxygen to maintain a saturation ≥92%, as prescribed. When the RN conducts end-of-shift rounds 3 hours later, the client is still sleeping soundly and the pulse oximeter shows 91%. Which nursing action is most appropriate at this time? 1. Check a full set of vital signs [40%] 2. Continue to monitor [23%] 3. Increase the oxygen flow by 1 L/min [10%] 4. Remove the nasal oxygen and measure saturation [25%] Omitted Correct answer 4 Answered correctly 25% Time: 2 seconds Updated: 04/26/2017 Explanation: When oxygen is delivered via a nasal cannula, the concentration of inspired oxygen will vary with changes in ventilation and respiratory pattern. In a deep sleep, with smaller tidal volumes and decreased respiratory rate, the concentration of inspired oxygen increases. In some clients with COPD, too high a level of inspired oxygen can depress the respiratory drive to breathe, causing alveolar hypoventilation. Because the saturation has remained constant at 91% while the client sleeps soundly, the nurse should remove the nasal oxygen, recheck the saturation, and re-evaluate. Removing the supplemental oxygen may actually increase the reading. (Option 1) The findings from a full set of vital signs would not provide information about the presence of alveolar hypoventilation. (Option 2) The client continues to sleep soundly after the initiation of supplemental oxygen. Continuing to monitor, without assessing and re-evaluating oxygenation status, is not an appropriate intervention. (Option 3) If sleeping soundly is related to depressed respiratory drive and hypoventilation, increasing oxygen concentration will worsen the problem. Educational objective: Some clients with COPD breathe in response to low arterial oxygen levels (hypoxemia). If they receive more oxygen than they need to maintain an arterial saturation, the increased level can decrease the drive to breathe. Therefore, supplemental oxygen should be administered in the lowest concentration possible to maintain a pulse oxygen saturation of 90%-93% or a PaO2 of 60-70 mm Hg. Copyright © UWorld. All rights reserved.

Block Time Remaining: 00:40:02 TUTOR Test Id: 72490738 QId: 30819 (921666) 62 of 75 A A A A client with left-sided pneumonia is admitted to the medical unit. The nurse assesses intermittent cough productive of copious amounts of thick yellow sputum and identifies ineffective airway clearance as the priority nursing diagnosis. Which interventions are appropriate to facilitate secretion removal? Select all that apply. 1. Chest physiotherapy 2. Cough suppressant 3. Huff coughing technique 4. Left-side lying position 5. Pursed lip breathing Omitted Correct answer 1,3 Answered correctly 34% Time: 3 seconds Updated: 03/24/2017 Explanation: In pneumonia, the lung is filled with infectious debris and exudate. This increase in secretions and a simultaneous decrease in mucociliary clearance result in possible airway obstruction. Interventions to facilitate airway clearance include the following: Hydration - IV fluids, oral intake (2-3 L/day), and respiratory humidification help thin secretions, maintain moisture of mucous membranes, and promote mucociliary clearance. Huff coughing technique - the most effective way to raise secretions from the lower to the upper airway for expectoration. If pain limits deep breathing and coughing, analgesia can be prescribed (Option 3). Chest physiotherapy (percussion, vibration, and postural drainage) to open airways and break up thickened secretions (Option 1) Fowler's position - Sitting upright with the head of the bed at 45-60 degrees promotes lung expansion and facilitates coughing and secretion removal. (Option 2) Cough suppressants can be administered for treatment of a dry, hacking cough or persistent cough that interferes with sleep. They do not facilitate secretion removal. (Option 4) Positioning on the right side, with the good lung in the dependent position, increases ventilation to perfusion matching due to gravity and improves oxygenation. (Option 5) Pursed lip breathing prolongs exhalation, prevents airway collapse, and decreases air trapping to help alleviate dyspnea, but it does not facilitate secretion removal. This form of breathing is helpful in clients with chronic obstructive pulmonary disease. Educational objective: Ineffective airway clearance related to increased sputum production due to a respiratory infection is a priority nursing diagnosis in clients with pneumonia. Interventions to mobilize secretions, clear an airway obstruction, and maintain airway patency include coughing and deep breathing, chest physiotherapy, positioning (Fowler's or side lying with good lung down), hydration, and administration of medications. Copyright © UWorld. All rights reserved.

Block Time Remaining: 00:40:03 TUTOR Test Id: 72490738 QId: 30317 (921666) 63 of 75 A A A The nurse is teaching an overweight 54-year-old client about ways to decrease symptoms of obstructive sleep apnea. Which interventions would be most effective? Select all that apply. 1. Eating a high-protein snack at bedtime 2. Limiting alcohol intake 3. Losing weight 4. Taking a mild sedative at bedtime 5. Taking modafinil at bedtime 6. Taking a nap during the day Omitted Correct answer 2,3 Answered correctly 32% Time: 1 seconds Updated: 01/20/2017 Explanation: Obstructive sleep apnea (OSA) is characterized by partial or complete airway obstruction during sleep that occurs from relaxation of the pharyngeal muscles. The result is repeated episodes of apnea (≥10 seconds) and hypopnea (≤50% normal ventilation), which cause hypoxemia and hypercarbia. Common symptoms include frequent periods of sleep disturbance, snoring, morning headache, daytime sleepiness, difficulty concentrating, forgetfulness, mood changes, and depression. Interventions include: Continuous positive airway pressure device at night to keep the structures of the pharynx and tongue from collapsing backward Limiting alcohol intake at bedtime as it can cause muscles of the oral airway to relax and lead to airway obstruction (Option 2) Weight loss and exercise can reduce snoring and sleep apnea-associated airway obstruction. Obesity contributes to the development of OSA (Option 3). Avoiding sedating medications (eg, benzodiazepines, sedating antidepressants, antihistamines, opiates) as they may exacerbate OSA and worsen daytime sleepiness (Option 1) Eating before bedtime can interfere with sleep and contribute to excess weight. (Option 4) Sedatives at bedtime can relax the muscles of the oral airway and lead to airway obstruction. (Option 5) Stimulants such as modafinil may be prescribed for daytime sleepiness but should be avoided at bedtime as they can cause insomnia. (Option 6) Napping during the day can make it more difficult to sleep through the night. Educational objective: Obstructive sleep apnea is characterized by partial or complete airway obstruction during sleep. Interventions to relieve symptoms include a continuous positive airway pressure device during sleep and lifestyle changes (eg, weight loss; exercise; avoiding food, alcohol, and sedatives at bedtime). Copyright © UWorld. All rights reserved.

Block Time Remaining: 00:40:04 TUTOR Test Id: 72490738 QId: 33789 (921666) 64 of 75 A A A The nurse is assessing a client an hour after a left lung lobectomy. The client is awake, alert, and oriented, and reports pain of 6 on a 0-10 scale. Pulse oximetry is 92% on 4 L oxygen via nasal cannula. The chest tube is set to continuous water seal suction at -20 cm H2O. The collection chamber has accumulated 320 mL of frank red drainage in the last hour. What is the priority nursing action? 1. Clamp the chest tube immediately [4%] 2. Increase oxygen to 6 L via nasal cannula [3%] 3. Medicate client for pain and document the findings [22%] 4. Notify the health care provider immediately [69%] Omitted Correct answer 4 Answered correctly 69% Time: 1 seconds Updated: 01/22/2017 Explanation: Following lung surgery, a chest tube is inserted into the pleural space to create a negative vacuum to re-inflate the lung and prevent air from re-entering the space. A client with a chest tube should be assessed for signs of air/fluid in the chest (eg, diminished breath sounds), excessive drainage (>100 mL/hr), pain, and infection at the drainage site. The collection chamber should be inspected every hour for the first 8 hours following surgery, then every 8 hours until it is removed. Excess drainage of frank red blood is indicative of hemorrhage and must be managed immediately. The priority action is to contact the health care provider for further management. (Option 1) Clamping the chest tube prevents air or fluid from leaving the pleural space, which may cause a reciprocal tension pneumothorax. The chest tube is clamped only a few hours prior to removal, momentarily to check for an air leak, or if the drainage apparatus needs to be changed. (Option 2) Although a pulse oximetry of 92% is low, this is an expected finding following lung surgery. (Option 3) Pain following surgery is a concern and the client will require medication; however, hemorrhage is the priority. Educational objective: A client with a chest tube should be assessed for signs of air/fluid in the chest (eg, diminished breath sounds), excessive drainage (>100 mL/hr), pain, and infection at the drainage site. Excess drainage of frank red blood is indicative of hemorrhage and must be managed immediately. Copyright © UWorld. All rights reserved.

Block Time Remaining: 00:40:04 TUTOR Test Id: 72490738 QId: 34082 (921666) 65 of 75 A A A The nurse is caring for a client with a feeding tube that has become obstructed. Which intervention should the nurse implement first to unclog the tube? 1. Flush and aspirate the tube with warm water [60%] 2. Instill a digestive enzyme solution into the tube [3%] 3. Instill cola or cranberry juice into the tube [11%] 4. Use a small-barrel syringe to flush the tube [24%] Omitted Correct answer 1 Answered correctly 60% Time: 0 seconds Updated: 06/04/2017 Explanation: Enteral feeding tubes are more likely to become obstructed if the tube is not flushed frequently enough, medications are not adequately crushed or diluted before administration, a thick feeding formula is used, or a small-bore feeding tube is required. Interventions to unclog a feeding tube are more successful if they are initiated immediately. The nurse should first attempt to dislodge the clogged contents by using a large-barrel syringe to flush and aspirate warm water in a back-and-forth motion through the tube (Option 1). (Option 2) If a feeding tube cannot be unclogged with warm water, the nurse may then attempt to use a digestive enzyme solution. These commercial declogging kits contain prefilled syringes of enzymatic solution that must be added to the tube and dwell in it for a period of time (usually 30 minutes to 1 hour) before flushing and aspiration are attempted. (Option 3) Instilling a carbonated beverage (eg, dark cola) or cranberry juice into a clogged feeding tube is not appropriate. The acidity of either liquid can worsen an obstruction, and the dark color may mask gastrointestinal bleeding. (Option 4) Flushing a feeding tube with a small-barrel syringe can create too much pressure and rupture the tube. Educational objective: When a feeding tube becomes clogged, the nurse should first attempt to unclog the tube by using a large-barrel syringe to flush and aspirate warm water in a back-and-forth motion through the tube. A digestive enzyme solution may help if warm water flushing is not effective. Copyright © UWorld. All rights reserved.

Block Time Remaining: 00:40:05 TUTOR Test Id: 72490738 QId: 30259 (921666) 66 of 75 A A A The nurse performs the admission history for a 70-year-old client with newly diagnosed chronic obstructive pulmonary disease (COPD). Which statements made by the client does the nurse recognize as the most significant contributing factors to the development of COPD? Select all that apply. 1. "I have been drinking alcohol almost daily since age 20." 2. "I have been overweight for as long as I can remember." 3. "I have smoked about a pack of cigarettes a day since I was 16 years old but quit last year." 4. "I know I eat too much fast food." 5. "I was a car mechanic for about 40 years and had my own garage." Omitted Correct answer 3,5 Answered correctly 30% Time: 1 seconds Updated: 04/26/2017 Explanation: . Chronic obstructive pulmonary disease (COPD) generally refers to 2 conditions, emphysema and chronic bronchitis. A combination of the 2 is common. It affects about 12 million people and is the 3rd leading cause of death in the United States. It occurs most commonly in the seventh decade of life. COPD is categorized by slowly progressive, persistent airflow obstruction that is closely associated with chronic airway inflammation. The major risk factor is tobacco smoke (eg, cigarette, pipe, cigar). Although the client quit smoking cigarettes last year, he smoked a pack a day for 53 years. Working as a car mechanic for 40 years is a major risk factor because of prolonged exposure to carbon monoxide fumes. Exposure to irritating chemicals, fumes, or vapors in the presence of cigarette smoking increases the risk of developing COPD. (Option 1) Alcohol use is not associated with the development of COPD. (Options 2 and 4) Although poor nutrition can contribute negatively to overall health status, it is not one of the most significant risk factors for development of COPD. Educational objective: Chronic airway inflammation is closely associated with the development of COPD. Specific etiologic factors include tobacco smoke, prolonged occupational exposure to chemicals and dust, air pollution, and genetics (eg, alpha1-antitrypsin deficiency). Copyright © UWorld. All rights reserved.

Block Time Remaining: 00:40:06 TUTOR Test Id: 72490738 QId: 30579 (921666) 67 of 75 A A A The charge nurse evaluates the care provided by a new registered nurse (RN) for a client receiving mechanical ventilation (MV). Which action by the new RN indicates the need for further education? 1. Administers morphine to relieve anxiety and restlessness [7%] 2. Applies suction when inserting the catheter into the airway [80%] 3. Increases the oxygen concentration on the MV before suctioning [2%] 4. Suctions when MV high-pressure alarm continues to sound and rhonchi are present [9%] Omitted Correct answer 2 Answered correctly 80% Time: 1 seconds Updated: 04/26/2017 Explanation: Risks associated with suctioning include hypoxemia, microatelectasis, and cardiac dysrhythmias. Suctioning removes secretions and oxygen. To minimize both the amount of oxygen removed and mucosal trauma, suction is applied when removing, not inserting, the catheter into the artificial airway. If secretions are thick and difficult to remove, increasing hydration, not suctioning time, is indicated. Aerosols of sterile normal saline or mucolytics such as acetylcysteine (Mucomyst) administered by nebulizer can also be used to thin the thick secretions, but water should not be used. Aerosol therapy may induce bronchospasm in certain individuals and can be relieved by use of a bronchodilator (albuterol). (Option 1) Morphine is administered to promote breathing synchrony with the mechanical ventilator, reduce anxiety, and promote comfort in clients receiving MV. (Option 3) Preoxygenation with 100% oxygen for 30 seconds before suctioning, unless otherwise specified, is the recommended practice to reduce suctioning-associated risks for hypoxemia, microatelectasis, and cardiac dysrhythmias. (Option 4) It is appropriate to suction the client when the high-pressure alarm on the MV sounds, saturations drop, rhonchi are auscultated, and secretions are audible or visible. These manifestations can indicate excessive secretions impairing airway patency. Educational objective: To minimize removal of oxygen and mucosal trauma, suction should be applied only when removing the catheter, not when inserting it. Other interventions to reduce the risks associated with suctioning (eg, hypoxemia, microatelectasis, cardiac dysrhythmias) include assessment for the need to suction, preoxygenating with 100% oxygen, and limiting suction time to 10-15 seconds. Copyright © UWorld. All rights reserved.

Block Time Remaining: 00:40:07 TUTOR Test Id: 72490738 QId: 32848 (921666) 68 of 75 A A A Exhibit A client had a thoracotomy 2 days ago to remove a lung mass and has a right chest tube attached to negative suction. Immediately after turning the client to the left side to assess the lungs, the nurse observes a rush of approximately 125 mL of dark bloody drainage into the drainage tubing and collection chamber. What is the appropriate nursing action? Click on the exhibit button for additional information. 1. Document and continue to monitor chest drainage [46%] 2. Immediately clamp the chest tube [5%] 3. Notify the health care provider [38%] 4. Request repeat hematocrit and hemoglobin levels [8%] Omitted Correct answer 1 Answered correctly 46% Time: 1 seconds Updated: 03/20/2017 Explanation: Immediately following a thoracotomy, chest tube drainage (50-500 mL for the first 24 hours) is expected to be sanguineous (bright red) for several hours and then change to serosanguineous (pink) followed by serous (yellow) over a period of a few days. A rush of dark bloody drainage from the chest tube when the client was turned following a period of minimal drainage is most likely related to retained blood due to a partial blockage in the tube. Bright red drainage indicates active bleeding and would be of immediate concern. (Option 2) The chest tube should not be clamped because it is placed to drain the fluid leaking after surgery. (Option 3) The nurse would notify the health care provider immediately of bright red drainage or continued increased drainage (>100 mL/hr) and of changes in the client's vital signs and cardiovascular status that could indicate bleeding (eg, hypotension, tachycardia, tachypnea, decreased capillary refill, cool and pale skin). This is not the appropriate action. (Option 4) It would be appropriate to request repeat serum hematocrit and hemoglobin levels if active bleeding is suspected, but the postoperative levels are stable at this time. This is not the appropriate action. Educational objective: A client will usually have a chest tube in place for several days following a thoracotomy to drain blood from the pleural space. A rush of dark bloody drainage from the tube when the client coughs, turns, or is repositioned following a period of minimal drainage is most likely related to retained blood due to a partial blockage in the tube. Bright red chest drainage indicates active bleeding and would be of immediate concern. Copyright © UWorld. All rights reserved.

Block Time Remaining: 00:40:08 TUTOR Test Id: 72490738 QId: 32086 (921666) 69 of 75 A A A The nurse assesses a client with a history of cystic fibrosis who is being admitted with a pulmonary exacerbation. Which assessment finding would require immediate action? 1. Current pulse oximetry reading is 90% on room air [30%] 2. Expectorating blood-tinged sputum [49%] 3. Loss of appetite and recent 5-lb (2.3-kg) weight loss [8%] 4. No bowel movement in the past 48 hours [11%] Omitted Correct answer 1 Answered correctly 30% Time: 1 seconds Updated: 02/21/2017 Explanation: In cystic fibrosis (CF), a defective protein responsible for transporting sodium and chloride causes the secretions from the exocrine glands to be thicker and stickier than normal. The sticky respiratory secretions lead to the inability to clear the airway and a chronic cough. The client eventually develops chronic lung disease (bronchiectasis) and is at risk for recurrent lung infections. These clients are also at risk for rupture of the damaged alveoli, which results in sudden-onset pneumothorax. Findings of pneumothorax include sudden worsening of dyspnea, tachypnea, tachycardia, and a drop in oxygen saturation. Because many of these findings can be seen with lung infection, a sudden drop in oxygen saturation could be the only early clue. The client with CF will often have a decreased pulse oximetry (reflects oxygen saturation in the blood) reading due to the chronicity of the disease process and damage to the lungs; however, a reading of 90% requires urgent intervention. (Option 2) Clients with CF often cough up blood-streaked sputum (hemoptysis) as a result of damage to blood vessels in the airway walls secondary to infections. However, this usually resolves with treatment of the infection. Frank hemoptysis needs urgent assessment. (Option 3) Maintaining weight is a challenge in those with CF due to the malabsorption of carbohydrates, fats, and proteins caused by the impaired enzyme secretions in the gastrointestinal tract. In addition, weight and appetite loss may indicate an undiagnosed underlying lung infection. This will need to be addressed, but oxygenation is the priority. (Option 4) Fecal retention and impaction are common in CF due to decreased water and salt secretion into the intestines. This will need to be addressed, but oxygenation is the priority. Educational objective: When addressing the multiple needs of a client with cystic fibrosis, airway and oxygen saturation are the priorities. Pneumothorax can be a complication of cystic fibrosis. Copyright © UWorld. All rights reserved.

Block Time Remaining: 00:40:09 TUTOR Test Id: 72490738 QId: 31223 (921666) 70 of 75 A A A A client is brought to the emergency department due to loss of consciousness after binge drinking at a college party and then taking alprazolam. Pulse oximetry shows 87% on room air. Which findings would the nurse expect to assess on an arterial blood gas? 1. Metabolic acidosis and hyperventilation [9%] 2. Metabolic alkalosis and hypoventilation [17%] 3. Respiratory acidosis and hypoventilation [63%] 4. Respiratory alkalosis and hyperventilation [9%] Omitted Correct answer 3 Answered correctly 63% Time: 1 seconds Updated: 02/10/2017 Explanation: The combination of excessive alcohol ingestion and the benzodiazepine alprazolam (Xanax) causes respiratory depression, which leads to alveolar hypoventilation secondary to carbon dioxide retention, and respiratory acidosis. Therefore, clients should be advised not to take multiple substances that increase the risk of respiratory depression (eg, opioids, benzodiazepines, alcohol, sedating antihistamines). (Option 1) Diarrhea, ketoacidosis, lactic acidosis, and renal failure can cause metabolic acidosis due to loss of bicarbonate or retention of acids; the lungs would compensate by hyperventilating. (Option 2) Vomiting, gastrointestinal suction, and administration of alkali (ie, sodium bicarbonate) are common causes of metabolic alkalosis; the lungs would compensate by hypoventilating. (Option 4) Hypoxia, anxiety, and pain are common causes of respiratory alkalosis, which is due to alveolar hyperventilation (rapid breathing). Educational objective: Over-sedation, sleep apnea, anesthesia, drug overdose, progressive neuromuscular disease, and chronic obstructive pulmonary disease depress the respiratory center; this leads to alveolar hypoventilation, secondary to carbon dioxide retention, and respiratory acidosis. Copyright © UWorld. All rights reserved.

Block Time Remaining: 00:40:10 TUTOR Test Id: 72490738 QId: 30702 (921666) 71 of 75 A A A A client with obesity is diagnosed with pulmonary embolism (PE). Which assessment data would the nurse expect to find? Select all that apply. 1. Bradycardia 2. Chest pain 3. Chills and fever 4. Hypoxemia 5. Tachypnea 6. Tracheal deviation Omitted Correct answer 2,4,5 Answered correctly 53% Time: 1 seconds Updated: 03/25/2017 Explanation: A pulmonary embolus is a blood clot that usually originates from the deep veins of the legs (>90%), travels to the pulmonary circulation, and obstructs a pulmonary artery or one of its branches, resulting in decreased perfusion in relation to ventilation and impaired gas exchange (hypoxemia). Clients are at risk for formation of venous thromboembolism (VTE) when the conditions detailed in Virchow's Triad are present. Clients at risk for PE include those with prolonged immobilization (eg, during hospitalization if not ambulatory), obesity, recent surgery, varicose veins, smoking, heart failure, advanced age, or history of VTE. The assessment data most characteristic of PE include: Dyspnea (85%) Pleuritic chest pain (60%) Tachycardia Tachypnea Hypoxemia (impaired gas exchange, decreased perfusion with normal alveolar ventilation, shunting) Apprehension and anxiety A more atypical presentation can be associated with a larger sized PE, and may include manifestations of cardiopulmonary compromise and hemodynamic instability (eg, right ventricular dysfunction, pulmonary hypertension, systemic hypotension, syncope, loss of consciousness, distended neck veins). (Option 1) A classic manifestation of PE is tachycardia to compensate for hypoxemia (not bradycardia). (Option 3) Chills and fever can indicate the presence of an infection and are not characteristic of PE. However, a low-grade fever without chills can occur 1-2 weeks after PE due to inflammation. (Option 6) Tracheal deviation is a symptom of tension pneumothorax. The trachea deviates from midline toward the unaffected side, away from the collapsed lung. Educational objective: Classic clinical manifestations of PE include dyspnea, pleuritic chest pain, tachycardia, tachypnea, hypoxemia, and feelings of apprehension and anxiety. Risk factors for PE include those detailed in Virchow's Triad (eg, hypercoagulability, venous stasis, and endothelial damage). Massive PE can cause syncope and hemodynamic instability. Copyright © UWorld. All rights reserved.

Block Time Remaining: 00:40:11 TUTOR Test Id: 72490738 QId: 30268 (921666) 72 of 75 A A A A client has chronic obstructive pulmonary disease (COPD) exacerbation. The pulse oximeter shows a saturation of 86% on room air. The nurse assesses diminished lung sounds and low-pitched wheezing posteriorly, shallow respirations, respiratory rate of 32/min, and use of accessory muscles. What is the most appropriate oxygen delivery device for this client? 1. Nasal cannula [29%] 2. Non-rebreathing mask [30%] 3. Oxymizer [1%] 4. Venturi mask [39%] Omitted Correct answer 4 Answered correctly 39% Time: 1 seconds Updated: 04/26/2017 Explanation: The Venturi mask is a high-flow device that delivers a guaranteed oxygen concentration regardless of the client's respiratory rate, depth, or tidal volume (TV). The adaptor or barrel can be set to deliver 24%-50% (varies with manufacturer) oxygen concentration. In the presence of tachypnea, shallow breathing with decreased TV, hypercarbia, and hypoxemia, it is the most appropriate oxygen delivery device for this client as rapid changes in inspired oxygen concentration can blunt the hypoxemic drive to breathe in clients with COPD. (Option 1) The nasal cannula can deliver adequate oxygen concentrations and is best for clients with adequate TV and normal vital signs. It is not the best choice in an unstable COPD client with varying TVs because the inspired oxygen concentration is not guaranteed. (Option 2) The non-rebreathing reservoir mask can deliver 60%-95% oxygen concentrations and is usually used short term. It is often used for clients with low saturations resulting from asthma, pneumonia, trauma, and severe sepsis; it is not the most appropriate device for a COPD client in this situation. (Option 3) An oxymizer is a nasal reservoir cannula-type device that conserves on oxygen use. Clients can be sustained on a prescribed oxygen level using much less oxygen (eg, 3 L/min nasal cannula is equivalent to 1 L/min oxymizer device) to reach the same saturation. It is not the best choice in an unstable COPD client with varying TVs as the inspired oxygen concentration is not guaranteed. Educational objective: Low-flow oxygen delivery devices (eg, nasal cannula, simple face mask) deliver oxygen concentrations that vary with breathing patterns. They are appropriate for clients who can tolerate varying concentrations (eg, stable COPD, type I respiratory failure [hypoxemic]). High-flow oxygen delivery devices (eg, Venturi mask, mechanical ventilator) deliver oxygen concentrations that do not vary with breathing patterns. They are appropriate for clients who cannot tolerate varying concentrations (eg, exacerbation COPD, type II respiratory failure [hypercarbic]). Copyright © UWorld. All rights reserved.

Block Time Remaining: 00:40:12 TUTOR Test Id: 72490738 QId: 31359 (921666) 73 of 75 A A A The client has a chest tube for a pneumothorax. While repositioning the client for an x-ray, the technician steps on the tubing and accidently pulls the chest tube out. The client's oxygen saturation drops and the pulse is 132/min; the nurse hears air leaking from the insertion site. What is the nurse's immediate action? 1. Apply an occlusive sterile dressing secured on 3 sides [66%] 2. Apply an occlusive sterile dressing secured on 4 sides [27%] 3. Assess lung sounds [3%] 4. Notify the health care provider (HCP) [2%] Omitted Correct answer 1 Answered correctly 66% Time: 1 seconds Updated: 06/01/2017 Explanation: If the chest tube is dislodged from the client and the nurse hears air leaking from the site, the nurse's immediate action should be to apply a sterile occlusive dressing (eg, petroleum jelly dressing) taped on 3 sides. This action permits air to escape on exhalation and inhibits air intake on inspiration. The nurse would then notify the HCP and arrange for the reinsertion of another chest tube (Option 1). (Option 2) A tension pneumothorax develops when air enters the pleural space but cannot escape. Increased intrapleural pressure and excessive accumulation of air can apply pressure to the heart and great vessels and drastically decrease cardiac output. An occlusive dressing taped on 4 sides would prevent the air in the pleural space from escaping on exhalation and would increase the risk for a tension pneumothorax. (Option 3) The nurse would stay with the client, assess lung sounds, and monitor vital signs frequently; however, this is not the immediate action. (Option 4) The nurse would notify the HCP and prepare for reinsertion of a chest tube, but it is not the immediate action. Educational objective: If a chest tube is dislodged from the client and the nurse hears air leaking from the site, the immediate action should be to apply an occlusive sterile dressing taped on 3 sides. This action decreases the risk for a tension pneumothorax by inhibiting air intake on inspiration and allowing air to escape on expiration. Copyright © UWorld. All rights reserved.

Block Time Remaining: 01:05:32 TUTOR Test Id: 72321830 QId: 30449 (921666) 4 of 10 A A A The nurse is providing nutrition counseling during a preconception visit to a client who does not eat green vegetables. In addition to a daily prenatal vitamin, which foods can the client add to decrease the risk of neural tube defects? Select all that apply. 1. Black beans and rice 2. Breakfast cereal and milk 3. Medium baked sweet potato 4. Peanut butter on whole wheat toast 5. Raw carrots with dip Incorrect Correct answer 1,2,4 Answered correctly 25% Time: 42 seconds Updated: 01/23/2017 Explanation: Folic acid, or folate, is a water-soluble, B-complex vitamin necessary for red blood cell production. Pregnant women and those attempting pregnancy need a minimum of 400 mcg of folic acid per day to decrease the chance of fetal neural tube defects (eg, spina bifida, anencephaly). Most prenatal vitamins contain 400-800 mcg of folic acid; additional folic acid can come from the diet. Leafy green vegetables are the best dietary sources of folic acid. However, other appropriate food choices include cooked beans, rice, fortified cereals, and peanut butter, which provide at least 40 mcg folic acid per serving (Options 1, 2, and 4). (Option 3) Sweet potatoes provide vitamin A, vitamin C, and minerals to the diet but no folic acid. (Option 5) Raw carrots are a dietary source of vitamin A, vitamin C, minerals, and fiber but not folic acid. Educational objective: Folic acid is a B-complex vitamin that decreases the risk of fetal neural tube defects. Pregnant women require a minimum of 400 mcg of folic acid daily. Leafy green vegetables supply excellent dietary folic acid; alternate sources include beans, rice, peanut butter, and fortified cereals. Copyright © UWorld. All rights reserved.

Block Time Remaining: 01:05:32 TUTOR Test Id: 72321830 QId: 30401 (921666) 5 of 10 A A A A client at 38 weeks gestation is brought to the emergency department after a motor vehicle crash. She reports severe, continuous abdominal pain. The nurse notes frequent uterine contractions and mild, dark vaginal bleeding. What actions should the nurse take? Select all that apply. 1. Anticipate emergent cesarean birth 2. Apply continuous external fetal monitoring 3. Assess routine vital signs every 4 hours 4. Draw blood for type and crossmatch 5. Initiate IV access with a 22-gauge catheter Incorrect Correct answer 1,2,4 Answered correctly 35% Time: 553 seconds Updated: 06/01/2017 Explanation: Placental abruption occurs when the placenta separates prematurely from the uterine wall, causing hemorrhage beneath the placenta. Abruptions are classified as partial, complete, or marginal and may be overt (visible vaginal bleeding) or concealed (bleeding behind placenta). Risk factors include abdominal trauma, hypertension, cocaine use, history of previous abruption, and preterm premature rupture of membranes. Symptoms and their severity depend on extent of abruption and include abdominal and/or back pain, uterine contractions, uterine rigidity, and dark red vaginal bleeding. Tachysystole (ie, excessive uterine contractions), with or without fetal distress, is often present, and continuous fetal monitoring is necessary (Option 2). A type and crossmatch should be drawn as treatment may include blood transfusion (Option 4). In severe cases, emergent cesarean birth is indicated (Option 1). Although blood loss is maternal, the loss of functional placental surface area can result in decreased placental perfusion, impaired fetal oxygenation, and fetal death. (Option 3) Maternal vital signs should be assessed frequently for signs of shock (eg, tachycardia, hypotension) as client condition can decline rapidly. In this acute scenario, assessment of vital signs every 4 hours is not sufficient. (Option 5) Abruption may require rapid volume replacement with IV fluid and blood products, requiring large-bore IV access. Peripheral IV access with a 16- or 18-gauge catheter should be initiated. Educational objective: Placental abruption usually presents with abdominal pain and dark red vaginal bleeding. The main concerns are maternal blood loss resulting in hypotension and shock and fetal compromise. Maternal stabilization and expedited birth are indicated. Copyright © UWorld. All rights reserved.

Block Time Remaining: 01:05:32 TUTOR Test Id: 72321830 QId: 31516 (921666) 2 of 10 A A A A nurse is teaching a client about formula preparation for a newborn. Which statements by the client indicate proper understanding? Select all that apply. 1. "I can add water to the formula if my baby wants to eat more frequently." 2. "I must wash the top of the concentrated formula can before opening it." 3. "I shouldn't heat formula in the microwave for more than 1 minute." 4. "If my baby does not finish the bottle, the leftover milk should be refrigerated." 5. "Prepared formula should be kept in the refrigerator and discarded after 48 hours." Incorrect Correct answer 2,5 Answered correctly 22% Time: 23 seconds Updated: 03/31/2017 Explanation: Infant formula is readily available in 3 forms: ready-to-feed, concentrated, and powder. Parents who feed their infants commercial formula should closely follow the manufacturer's recommendations for preparation, particularly if the product requires dilution or reconstitution. Parents should also adhere to basic guidelines for safe storage and handling. Key teaching points include: Keep bottles, nipples, caps, and other parts as clean as possible, either by boiling or washing in the dishwasher Wash the tops of formula cans prior to opening to prevent contamination (Option 2) Prepared formula or opened cans of ready-to-feed or concentrated formula should be kept in the refrigerator and discarded after 48 hours if unused (Option 5). There is a risk of bacterial growth after this time. Prepared bottles can be warmed by placing in a pan of hot water for several minutes Test temperature on the inner wrist before serving to the infant; formula should feel lukewarm, but never hot Never microwave formula as it can cause mouth burns (Option 3) (Option 1) Formula should never be diluted or concentrated. Dilution of the formula does not allow the infant to receive the appropriate amount of calories, vitamins, and minerals needed for normal growth and development. Overconcentration of the formula can cause excessive proteins and minerals to be ingested that exceed the excretory ability of the infant's immature kidneys. (Option 4) Any formula left in a bottle after a feeding should be discarded immediately because the infant's saliva has mixed with it. This will encourage bacterial growth. Educational objective: Parents who feed their infants commercial formula should follow the manufacturer's recommendations for preparation and storage. Formula should never be diluted, concentrated, or microwaved for infant safety. After preparation, unused prepared formula can be stored in the refrigerator for up to 48 hours. Copyright © UWorld. All rights reserved.

Block Time Remaining: 01:05:32 TUTOR Test Id: 72321830 QId: 30644 (921666) 3 of 10 A A A A pregnant client comes in for a routine first prenatal examination. According to the last menstrual period, the estimated gestational age is 12 weeks. Where would the nurse expect to palpate the uterine fundus in this client? 1. 12 cm above the umbilicus [9%] 2. At the level of the umbilicus [7%] 3. Halfway between the symphysis pubis and the umbilicus [41%] 4. Just above the symphysis pubis [41%] Correct Answered correctly 41% Time: 41 seconds Updated: 01/24/2017 Explanation: The enlarging pregnant uterus should be just above the symphysis pubis at approximately 12 weeks gestation (Option 4). At 16 weeks gestation, the fundus is roughly halfway between the symphysis pubis and the umbilicus. It reaches the umbilicus at 20-22 weeks gestation and approaches the xiphoid process around 36 weeks gestation. At 38-40 weeks, the fetus engages into the maternal pelvis and the fundal height drops. After 20 weeks gestation, the fundal height, measured in centimeters from the symphysis pubis to the top of the fundus, correlates closely to the weeks of gestation. (Options 1, 2, and 3) At 12 weeks gestation, the uterine fundus should be just above the symphysis pubis. Educational objective: The fundus rises above the symphysis pubis at approximately 12 weeks gestation, reaches the umbilicus at 20-22 weeks gestation, and reaches the xyphoid process at 36 weeks gestation. After 20 weeks gestation, the fundal height in centimeters correlates closely to the weeks of gestation. Copyright © UWorld. All rights reserved.

Block Time Remaining: 01:05:32 TUTOR Test Id: 72321830 QId: 31666 (921666) 6 of 10 A A A The nurse is performing a postpartum assessment 12 hours after the prolonged vaginal delivery of a term infant. Which assessment findings should be reported to the health care provider? 1. Complaints of discomfort during fundal palpation [6%] 2. Foul-smelling lochia [56%] 3. Oral temperature 100.1 F (37.8 C) [3%] 4. White blood cell (WBC) count 24,000/mm3 (24.0 x 109/L) [33%] Incorrect Correct answer 2 Answered correctly 56% Time: 33 seconds Updated: 01/09/2017 Explanation: A foul odor of lochia suggests endometrial infection. This client has an increased risk of infection due to her prolonged labor, which involved multiple cervical examinations. The odor of lochia is usually described as "fleshy" or "musty." A foul smell warrants further evaluation. Other signs of endometrial infection are maternal fever, tachycardia, and uterine pain/tenderness. (Option 1) Palpation of the postpartum uterine fundus is commonly uncomfortable for the client. If the client complains of increasing pain, further evaluation is needed. (Option 3) Major signs and symptoms of endometrial infection include temperature above 100.4 F (38.0 C); chills; malaise; excessive uterine tenderness; and purulent, foul-smelling lochia. During the first 24 hours postpartum, the temperature is normally elevated; temperature above 100.4 F (38 C) requires further evaluation. (Option 4) The WBC count is normally elevated during the first 24 hours postpartum (up to 30,000/mm3 [30.0 x 109/L]). Leukocyte levels that are not decreasing require further evaluation. Educational objective: Signs of endometrial infection include elevated temperature, chills, malaise, excessive pain, and foul-smelling lochia. During the first 24 hours postpartum, temperature and WBC count are normally elevated. Fever and leukocyte counts that do not decrease require further evaluation. Copyright © UWorld. All rights reserved.

Block Time Remaining: 01:05:32 TUTOR Test Id: 72321830 QId: 31577 (921666) 7 of 10 A A A A client in labor has reached 8 cm dilation, is fully effaced, and feels an urge to push. The nurse observes thick, blood-tinged mucus during the vaginal examination. What is the nurse's best action? 1. Administer prescribed IV meperidine for pain relief [1%] 2. Encourage client to bear down with spontaneous urges to push [11%] 3. Place client in the lithotomy position in preparation for birth [35%] 4. Provide encouragement and coaching in breathing techniques [51%] Incorrect Correct answer 4 Answered correctly 51% Time: 47 seconds Updated: 05/22/2017 Explanation: Stages of labor Stage Definition 1 Latent 0-5 cm cervical dilation Active 6-10 cm cervical dilation 2 10 cm (complete) cervical dilation to birth 3 Birth of baby to expulsion of placenta 4 1-4 hours after birth, maternal physiologic readjustment The period of active labor from 8-10 cm dilation (ie, "transition") is often the most emotionally challenging phase of labor, marked by increased maternal anxiety. A mixture of mucus and pink/dark brown blood ("bloody show") is commonly observed during transition. Nursing priorities include providing emotional support and encouragement, and coaching the client in breathing techniques (Option 4). (Option 1) Meperidine (Demerol) is an opioid occasionally prescribed for analgesia during early labor. It has a rapid onset (5 minutes) when given IV, and a duration of 2-3 hours. However, it should be avoided within 1-4 hours of birth due to the potential for neonatal respiratory depression. (Option 2) The second stage of labor begins once complete dilation (10 cm) is achieved. Clients may feel the urge to push (Ferguson reflex) prior to complete dilation if the fetal head is low in the pelvis. However, pushing should be delayed until complete dilation is achieved to avoid cervical swelling and/or cervical lacerations. (Option 3) Although lithotomy positioning may be more convenient for the birth attendant, upright or lateral positions encourage fetal rotation and descent, increase client comfort, and decrease the risk of perineal trauma. This client has not yet reached complete dilation and should be allowed to move freely. Educational objective: The period of active labor from 8-10 cm dilation (ie, "transition") can be emotionally challenging for laboring clients. Signs of near-complete dilation include bloody show and the urge to push. Clients should be coached in breathing techniques and should avoid pushing until fully dilated to prevent cervical trauma. Copyright © UWorld. All rights reserved.

Block Time Remaining: 00:30:31 TUTOR Test Id: 72325665 QId: 30484 (921666) 9 of 10 A A A A pregnant client at 30 weeks gestation comes to the prenatal clinic. Which vaccines may be administered safely at this prenatal visit? Select all that apply. 1. Influenza injection 2. Influenza nasal spray 3. Measles, mumps, and rubella 4. Tetanus, diphtheria, and pertussis 5. Varicella Incorrect Correct answer 1,4 Answered correctly 33% Time: 53 seconds Updated: 03/26/2017 Explanation: Health promotion during pregnancy includes the administration or avoidance of certain vaccines to decrease risks to mother and fetus. Pregnant women have suppressed immune systems and are at increased risk for illness and subsequent complications. Some viruses (eg, rubella, varicella) can cause severe birth defects if contracted during pregnancy. Inactivated vaccines contain a "killed" version of the virus and pose no risk of causing illness from the vaccine. Some vaccines contain weakened (ie, attenuated) live virus and pose a slight theoretical risk of contracting the illness from the vaccine. For this reason, women should not receive live virus vaccines during pregnancy or become pregnant within 4 weeks of receiving such a vaccine. The tetanus, diphtheria, and pertussis (Tdap) vaccine is recommended for all pregnant women between the beginning of the 27th and the end of the 36th week of gestation as it provides the newborn with passive immunity against pertussis (whooping cough) (Option 4). During influenza season (October-March), it is safe and recommended for pregnant women to receive the injectable inactivated influenza vaccine regardless of trimester (Option 1). (Options 2, 3, and 5) The influenza nasal spray; measles, mumps, and rubella (MMR) vaccine; and varicella vaccine contain live viruses and are contraindicated in pregnancy. Educational objective: Inactivated vaccines (eg, inactivated influenza; tetanus, diphtheria, and pertussis) may be given during pregnancy to protect pregnant clients from illness and provide the fetus with passive immunity. Live virus vaccines are contraindicated in pregnancy. Copyright © UWorld. All rights reserved.

Block Time Remaining: 01:05:32 TUTOR Test Id: 72321830 QId: 31704 (921666) 1 of 10 A A A The nurse performs Leopold's maneuvers while preparing to apply an external fetal heart monitor. Identify the area where the fetal heart transducer should be applied if the fetal position is left occiput anterior. Hotspot Question Incorrect Correct answer Refer to Hotspot Answered correctly 26% Time: 9 seconds Updated: 01/02/2017 Explanation: The nurse applies the uterine activity transducer to the woman's upper abdomen, in the fundal area. Before applying an external fetal heart monitor, Leopold's maneuvers are performed to assess the position and presentation of the fetus. The Doppler transducer for sensing the fetal heart rate is usually placed on the lower abdomen when the fetus is in the cephalic presentation. The Doppler transducer should be placed at the approximate location of the fetal back. This fetus is in the left occiput anterior position; therefore, the fetal back is located halfway between the midline and anterior superior iliac spine on the mother's lower left anterior abdomen (versus lower lateral left abdomen for the left occiput posterior position). Educational objective: Leopold's maneuvers are performed to determine the position and presentation of the fetus. After determining the position and presentation, the nurse should place the Doppler transducer over the fetal back. Copyright © UWorld. All rights reserved.

Block Time Remaining: 01:05:32 TUTOR Test Id: 72321830 QId: 31569 (921666) 8 of 10 A A A A full-term newborn of a mother with gestational diabetes is slightly jittery with a blood glucose level of 45 mg/dL (2.2 mmol/L). What is the nurse's first action? 1. Administer oral glucose [25%] 2. Feed the newborn [62%] 3. Notify the pediatrician [5%] 4. Warm the room [6%] Incorrect Correct answer 2 Answered correctly 62% Time: 36 seconds Updated: 04/28/2017 Explanation: In women with poorly controlled diabetes mellitus during pregnancy, the fetus is subjected to high blood glucose levels. Fetal hyperglycemia leads to insulin hypersecretion by the fetus, which promotes abnormal growth and storage of excess calories as fat (macrosomia). After birth, the infant is no longer exposed to the mother's high blood glucose levels, but a transient hyperinsulinemic state will persist for several days, during which the infant is susceptible to developing hypoglycemia. The normal range for serum glucose in a newborn at day 1 is 40-60 mg/dL (2.2-3.3 mmol/L); however, no standard definition for newborn hypoglycemia currently exists. Treatment plans are based on clinical signs and overall status of the infant. The most common sign of low blood glucose is jitteriness or tremors. If the blood glucose is low, newborns should be fed immediately with formula or breast milk (Option 2). If the infant continues to exhibit signs of hypoglycemia and/or blood glucose levels are <40-45 mg/dL (2.2-2.5 mmol/L) after feeding, the pediatrician should be notified (Option 3). Further treatment, such as oral or IV glucose, may be required. (Option 1) A hypertrophied pancreas is very sensitive to blood glucose levels. Oral glucose administration would cause massive release of insulin and produce rebound hypoglycemia. Feeding the newborn with breast milk or formula would be sufficient in most situations. (Option 4) Although cold stress may exacerbate existing hypoglycemia, warming the room is not the best initial intervention. Feeding this infant is the priority as there are early clinical signs of hypoglycemia. Educational objective: Hypoglycemia can occur in infants born to mothers with gestational diabetes due to elevated insulin levels and consumption of stored glucose. The most common sign of hypoglycemia is jitteriness or tremors. The newborn should be fed breast milk or formula immediately. Copyright © UWorld. All rights reserved.

Block Time Remaining: 01:05:32 TUTOR Test Id: 72321830 QId: 33155 (921666) 9 of 10 A A A The nurse is caring for a baby born at 30 weeks gestation and diagnosed with necrotizing enterocolitis. Which nursing action should be implemented? 1. Encourage parents to increase skin-to-skin care [10%] 2. Measure abdominal girth daily [62%] 3. Measure rectal temperature every 3-4 hours [7%] 4. Position client on side and check diaper for stool [18%] Correct Answered correctly 62% Time: 37 seconds Updated: 12/17/2016 Explanation: Necrotizing enterocolitis occurs predominantly in preterm infants secondary to gastrointestinal and immunologic immaturity. On initiation of enteral feeding, bacteria can be introduced into the bowel, where they can proliferate excessively due to compromised immune clearance. This results in inflammation and ischemic necrosis of the intestine. As the disease progresses, the bowel becomes congested and gangrenous with gas collections forming inside the bowel wall. Measuring the client's abdominal girth daily is an important nursing intervention to note any worsening intestinal gas-associated swelling. Clients are made NPO and receive nasogastric suction to decompress the stomach and intestines. Parenteral hydration and nutrition and IV antibiotics are given. (Option 1) Skin-to-skin care (kangaroo care) promotes bonding with a healthy newborn. It is allowed in some instances for premature infants depending on the condition and week of gestation. Skin-to-skin care should be avoided in infants who are not stable as it may cause additional stress. (Option 3) Taking a client's temperature every 3-4 hours is important; however, rectal temperatures should be avoided due to the risk of perforation of the gangrenous, friable colon. (Option 4) To avoid pressure on the abdomen and facilitate observation for a distended abdomen, clients are placed supine and undiapered. Educational objective: Necrotizing enterocolitis is a life-threatening complication in premature infants due to underdeveloped intestine and gut immunity. Frequent abdominal girth measurements are essential to assess for worsening distension. Clients are placed supine and undiapered. Rectal temperatures should be avoided due to the risk of perforation. Copyright © UWorld. All rights reserved.

Block Time Remaining: 02:46:05 TUTOR Test Id: 72348754 QId: 30479 (921666) 16 of 25 A A A The nurse is teaching a client, gravida 1 para 0, at 8 weeks gestation about expected weight gain in pregnancy. The client's prepregnancy BMI is 21 kg/m2. Which statement made by the client indicates an understanding about weight gain? 1. "I should gain 10-15 lb (4.5-6.8 kg) during the first trimester." [14%] 2. "I should gain a total of about 30 lb (13.6 kg) during my pregnancy." [65%] 3. "I should gain no more than 0.5 lb (0.2 kg) per week during the third trimester." [17%] 4. "If I gain <20 lb (9.1 kg) during pregnancy, it will be easier to lose weight postpartum." [2%] Correct Answered correctly 65% Time: 40 seconds Updated: 03/28/2017 Explanation: BMI (kg/m2) Weight gain Total First trimester Second & third trimesters <18.5 (underweight) 28-40 lb (12.7-18.1 kg) 1.1-4.4 lb (0.5-2.0 kg) Approximately 1 lb/wk (0.5 kg/wk) 18.5-24.9 (appropriate weight) 25-35 lb (11.3-15.9 kg) 25-29.9 (overweight) 15-25 lb (6.8-11.3 kg) Approximately 0.6 lb/wk (0.3 kg/wk) ≥30 (obese) 11-20 lb (5.0-9.1 kg) Approximately 0.5 lb/wk (0.2 kg/wk) Weight gain during pregnancy should be determined by prepregnancy BMI. Underweight clients need to gain more weight (1 lb [0.5 kg]/wk) during the second and third trimesters of pregnancy than do obese clients (0.5 lb [0.2 kg]/wk). However, weight gain in the first trimester should be 1.1-4.4 lb (0.5-2.0 kg) regardless of BMI. With a prepregnancy BMI of 21 kg/m2, this client is classified as an appropriate weight and should gain 25-35 lb (11.3-15.9 kg) (Option 2). (Option 1) Weight gain during the first trimester should be approximately 1.1-4.4 lb (0.5-2.0 kg). A 10-lb (4.5-kg) weight gain during the first 3 months of pregnancy would be too much in any client. (Option 3) A client of normal weight should gain approximately 1 lb (0.5 kg)/wk during the second and third trimesters of pregnancy. A weight gain of only 0.5 lb (0.2 kg)/wk is recommended for obese clients. (Option 4) A weight gain of 20 lb (9.1 kg) during pregnancy is inadequate for a client of normal weight. Restricting weight gain places the fetus at risk for low birth weight (<5.5 lb [<2500 g]) and preterm birth. Educational objective: Appropriate total weight gain during pregnancy decreases risks to the client and fetus. Weight gain in the first trimester should be 1.1-4.4 lb (0.5-2.0 kg) regardless of BMI. The optimal total weight gain during pregnancy should be determined by the client's prepregnancy BMI. Copyright © UWorld. All rights reserved.

Block Time Remaining: 02:46:05 TUTOR Test Id: 72348754 QId: 30474 (921666) 17 of 25 A A A A nurse is admitting a client at 42 weeks gestation to the labor and delivery unit for induction of labor. What is a predictor of a successful induction? 1. Bishop score of 10 [54%] 2. Firm and posterior cervix [11%] 3. History of precipitous labor [5%] 4. Reactive nonstress test [28%] Omitted Correct answer 1 Answered correctly 54% Time: 19 seconds Updated: 01/17/2017 Explanation: Cervix Bishop score 0 1 2 3 Consistency Firm Medium Soft — Position Posterior Mid-position Anterior — Dilation 0 cm 1-2 cm 3-4 cm >5 cm Effacement 0%-30% 40%-50% 60%-70% >80% Station −3 −2 −1, 0 +1, +2 The Bishop score is a system for the assessment and rating of cervical favorability and readiness for induction of labor. The cervix is scored (0-3) on consistency, position, dilation, effacement, and station of the fetal presenting part. A higher Bishop score indicates an increased likelihood of successful induction that results in vaginal birth. For nulliparous women, a score ≥8 usually indicates that induction will be successful (Option 1). (Option 2) A cervix that is firm and posterior is associated with a low Bishop score, which reflects a low likelihood of successful labor induction. (Option 3) A history of precipitous labor (<3 hours from onset of contractions to birth) may indicate that the client will again experience precipitous labor once labor is established. However, such a history is not an independent predictor of successful induction. (Option 4) A reactive nonstress test indicates that the fetus is well oxygenated and establishes fetal well-being. It does not provide information about the likely success or failure of labor induction. Educational objective: The Bishop score is a system for the assessment and rating of cervical favorability and readiness for induction of labor. A score >8 in nulliparous women is associated with successful induction and subsequent vaginal birth. Copyright © UWorld. All rights reserved.

Block Time Remaining: 02:46:05 TUTOR Test Id: 72348754 QId: 31836 (921666) 18 of 25 A A A When triaging 4 pregnant clients in the obstetric clinic, the nurse should alert the health care provider to see which client first? 1. First-trimester client reporting frequent nausea and vomiting [0%] 2. Second-trimester client with dysuria and urinary frequency [6%] 3. Second-trimester client with obesity reporting decrease in fetal movement [47%] 4. Third-trimester client with right upper quadrant pain and nausea [45%] Correct Answered correctly 45% Time: 24 seconds Updated: 03/18/2017 Explanation: Right upper quadrant (RUQ) or epigastric pain can be an indicator of HELLP syndrome, a severe form of preeclampsia. HELLP syndrome (Hemolysis, Elevated Liver enzymes, and Low Platelet count) is often mistaken for viral gastroenteritis due to its variable and nonspecific presentation. Misdiagnosis may lead to severe complications (eg, placental abruption, liver failure, stroke) and/or maternal/fetal death. Clients may have RUQ pain, nausea, vomiting, and malaise. Headache, visual changes, proteinuria, and hypertension may or may not be present. (Option 1) Nausea and vomiting during the first trimester are normal, expected findings. Vomiting that continues past the first trimester or that is accompanied by fever, pain, or weight loss is considered abnormal and requires intervention. (Option 2) Although urinary frequency is common in pregnancy, dysuria could indicate a urinary tract infection. This client should be evaluated but does not take priority over a client with symptoms of HELLP. (Option 3) Maternal perception of fetal movement can be altered by obesity, maternal position, fetal sleep cycle, fetal position, anterior placenta, and amniotic fluid volume (increased or decreased). This client should be evaluated to determine the cause of decreased fetal movement; however, this is not the priority. Educational objective: HELLP syndrome (Hemolysis, Elevated Liver enzymes, and Low Platelet count) is a severe form of preeclampsia. Its clinical presentation can be quite variable and may include nonspecific symptoms such as right upper quadrant/epigastric pain, nausea, vomiting, and malaise. Complications including placental abruption, stroke, and death may occur if HELLP syndrome is not treated immediately. Copyright © UWorld. All rights reserved.

Block Time Remaining: 02:46:05 TUTOR Test Id: 72348754 QId: 30515 (921666) 19 of 25 A A A A 28-year-old client is admitted to the labor and delivery unit for severe preeclampsia. She is started on IV magnesium sulfate. Which signs indicate that the client has developed magnesium sulfate toxicity? Select all that apply. 1. 0/4 patellar reflex 2. Blood pressure is 156/84 mm Hg 3. Client voided 600 mL in 8 hours 4. Respirations are 10/min 5. Serum magnesium level is 5 mEq/L (2.5 mmol/L) Incorrect Correct answer 1,4 Answered correctly 13% Time: 34 seconds Updated: 05/22/2017 Explanation: Normal blood level of magnesium is 1.5-2.5 mEq/L (0.75-1.25 mmol/L). However, a therapeutic magnesium level of 4-7 mEq/L (2.0-3.5 mmol/L) is necessary to prevent seizures in a preeclamptic client (Option 5). Magnesium toxicity causes central nervous system depression because toxic magnesium levels (>7 mEq/L) [3.5 mmol/L]) block neuromuscular transmission. Absent or decreased deep-tendon reflexes (DTRs) are the earliest sign of magnesium toxicity. DTRs are scored on a scale of 0-4+ and should be assessed during magnesium infusion; normal findings are 2+ (Option 1). If toxicity is not recognized early (eg, decreasing DTRs), clients can progress to respiratory depression, followed by cardiac arrest (Option 4). Assessments (including vital signs) should be performed every 5-15 minutes during the loading dose, followed by 30- to 60-minute intervals until the client stabilizes, then every 2 hours. Treatment for magnesium toxicity is immediate discontinuation of the infusion. Administration of calcium gluconate (antidote) is recommended only in the event of cardiorespiratory compromise. (Option 2) Hypertension is a sign of preeclampsia, not of magnesium toxicity. Hydralazine (Apresoline), methyldopa (Aldomet), or labetalol (Trandate) is used to lower blood pressure (BP) if needed (typically when BP is >160/110 mm Hg). (Option 3) Urine output <30 mL/hr is a sign of magnesium toxicity. Educational objective: The therapeutic level of magnesium for preeclampsia-eclampsia treatment is 4-7 mEq/L (2.0-3.5 mmol/L). Signs of magnesium toxicity are decreased or absent deep-tendon reflexes, respiratory depression, decreased urine output (<30 mL/hr), and cardiac arrest. Calcium gluconate (antidote) should be readily available in the event of cardiorespiratory compromise. Copyright © UWorld. All rights reserved.

Block Time Remaining: 02:46:05 TUTOR Test Id: 72348754 QId: 30476 (921666) 22 of 25 A A A During the first prenatal assessment, the client reports the last normal menstrual period starting on March 1 and ending on March 5, but also slight spotting on March 23. The client had unprotected intercourse on March 15. Using Naegele's rule, what is the estimated date of birth? 1. December 8 [57%] 2. December 12 [22%] 3. December 22 [13%] 4. December 30 [6%] Correct Answered correctly 57% Time: 24 seconds Updated: 12/27/2016 Explanation: Various methods to determine the estimated date of birth (EDB) include use of Naegele's rule, ultrasound, uterine height measurement (McDonald's measurement), and auscultation of fetal heart rate with a Doppler device. The most accurate dating of pregnancy involves use of ultrasound around the 16th-18th week of pregnancy. However, Naegele's rule can be used to quickly determine an EDB early in the pregnancy. This calculation uses the date of the first day of the last normal menstrual period (LMP) for determination of EDB. EDB = (LMP minus 3 months) + 7 days This client's LMP is March 1, minus 3 months = December 1. Add 7 days to obtain EDB = December 8. Clients who conceive in January, February, and most of March will deliver in the current year. Those who conceive after March will deliver in the following year; as a result, a third step is adding 1 to the current year to determine the estimated date of birth. For example, LMP of May 10, 2014, would have an EDB of February 17, 2015. It is important to note that Naegele's rule is based on a client having a menstrual cycle of 28 days. It therefore may not be as accurate if the client has a shorter or longer menstrual cycle. (Option 2) Using the last day of the LMP to calculate EDB provides an inaccurate due date as clients may have varying lengths of menstrual bleeding. (Option 3) Conception occurs around the time of ovulation and is about 14 days from the beginning of the LMP. Eggs are fertile for about 12-24 hours after ovulation with sperm able to remain fertile for 24-72 hours. Implantation of the trophoblast occurs about 7-10 days after fertilization. Using the conception date calculates the gestational age of the embryo approximately 2 weeks later than the true gestational age. (Option 4) Spotting around the time the next menstrual period is due may be considered normal and is probably caused by implantation of the trophoblast into the uterine endometrial lining. This is not considered a problem, but using this occurrence to date the pregnancy erroneously delays the EDB by 4 weeks. It is important to calculate EDB from the beginning of the last normal menstrual period. Educational objective: Naegele's rule provides a quick determination of the estimated date of birth (EDB). EDB = (LMP minus 3 months) + 7 days. If the LMP occurs in January, February, or March, the EDB will be in the current year. If the LMP occurs after March, the EDB will be in the next year. Copyright © UWorld. All rights reserved.

Block Time Remaining: 02:46:05 TUTOR Test Id: 72348754 QId: 30566 (921666) 23 of 25 A A A A client is at 20 weeks gestation. The client reports having to "run to the bathroom all the time," "it hurts to pee," and my urine "smells bad." Which statement by the nurse is the most appropriate? 1. "Drink cranberry juice to relieve the symptoms." [1%] 2. "Make sure to wipe from the front to the back after voiding." [3%] 3. "Most women have urinary frequency at this stage and it is normal." [1%] 4. "You may need to be checked for a urinary tract infection." [93%] Correct Answered correctly 93% Time: 14 seconds Updated: 05/05/2017 Explanation: The most common bacterial infection during pregnancy is a urinary tract infection (UTI). Pregnant women are predisposed to UTIs due to physiological changes (urine stasis) in the renal system. Symptoms include frequency, dysuria, urgency, foul-smelling urine, sediment/pus/blood in the urine, or sensation of bladder fullness. Diagnosis is made on signs/symptoms and urinalysis. The prescribed antibiotic course must be completed to treat the infection appropriately. The priority is to deal with the current infection. If the UTI is untreated, the infection can lead to pyelonephritis or premature labor. (Option 1) Cranberry juice works by preventing the attachment of bacteria to the epithelial cells in the bladder wall. It is recommended that clients drink 8 ounces of unsweetened juice 3 times a day. Although this is reasonable teaching, it is not the priority at this time. (Option 2) This is good general advice to help prevent UTIs as the most common causative organism is Escherichia coli. However, completing a urinalysis and initiating antibiotic therapy should be implemented before instructions related to UTI prevention. Actual over potential is always the priority. (Option 3) Most women experience urinary symptoms of frequency (voiding >7 times/day), urgency, and nocturia (voiding >2 times/night) beginning early in pregnancy and continuing to some degree throughout pregnancy. These are primarily a result of hormonal changes and may be due to anatomical changes in the renal system. They are not indicative of UTI. However, the client has new, additional symptoms of a UTI (dysuria, foul-smelling urine). The client should be tested and treated with an antibiotic. Educational objective: Signs/symptoms of a UTI in pregnancy include new-onset dysuria and sediment/cloudy, foul-smelling urine. This differentiates it from the common urinary frequency throughout pregnancy. The priority for the nurse is to identify the need for a urinalysis and to provide instructions to finish the course of antibiotics to prevent the untreated infection from causing pyelonephritis or premature labor. Copyright © UWorld. All rights reserved.

Block Time Remaining: 02:46:05 TUTOR Test Id: 72348754 QId: 30514 (921666) 24 of 25 A A A A client at 39 weeks gestation with preeclampsia has a blood pressure of 170/100 mm Hg, 2+ proteinuria, and moderate peripheral edema. Immediately after hospital admission, she develops seizures and uterine contractions. Magnesium sulfate is prescribed. Which finding indicates that the drug has achieved the desired therapeutic effect? 1. Blood pressure <130/80 mm Hg [24%] 2. Seizure activity stops [55%] 3. Urine has 1+ protein [1%] 4. Uterine contractions stop [18%] Omitted Correct answer 2 Answered correctly 55% Time: 39 seconds Updated: 03/08/2017 Explanation: Preeclampsia is a systemic disease characterized by hypertension and proteinuria after the 20th gestational week with unknown etiology. Eclampsia is the onset of convulsions or seizures that cannot be attributed to other causes in a woman with preeclampsia. Delivery is the only cure for preeclampsia-eclampsia syndrome. Magnesium sulfate is a central nervous system depressant used to prevent/control seizure activity in preeclampsia/eclampsia clients. During administration, the nurse should assess vital signs, intake and output, and monitor for signs of magnesium toxicity (eg, decreased deep-tendon reflexes, respiratory depression, decreased urine output). A therapeutic magnesium level of 4-7 mEq/L (2.0-3.5 mmol/L) is necessary to prevent seizures in a preeclamptic client. (Option 1) Hypertension is a sign of preeclampsia. Hydralazine (Apresoline), methyldopa (Aldomet), or labetalol (Trandate) is used to lower blood pressure (BP) if needed (usually considered when BP is >160/110 mm Hg). (Option 3) Proteinuria is a symptom of preeclampsia. Control of hypertension and delivery will reduce the protein level. Magnesium sulfate is not prescribed to decrease proteinuria. (Option 4) Tocolytic drugs (eg, terbutaline, magnesium sulfate, indomethacin, nifedipine) are used to suppress uterine contractions in preterm labor, allowing pregnancy to be prolonged for 2-7 days so that corticosteroid administration can improve fetal lung maturity. This client is at term, and there is no need to delay delivery. Educational objective: Magnesium sulfate is prescribed for clients with preeclampsia to prevent seizure activity. A therapeutic magnesium level of 4-7 mEq/L (2.0-3.5 mmol/L) is necessary to prevent seizures in a preeclamptic client. Copyright © UWorld. All rights reserved.

Block Time Remaining: 02:46:05 TUTOR Test Id: 72348754 QId: 30643 (921666) 25 of 25 A A A The nurse is admitting a client at 41 weeks gestation for induction of labor due to oligohydramnios. Considering the client's indication for induction, what should the nurse anticipate? 1. Additional neonatal personnel present for birth [37%] 2. Intermittent fetal monitoring during labor [26%] 3. Need for forceps-assisted vaginal birth [13%] 4. Need for uterotonic drugs for postpartum hemorrhage [21%] Incorrect Correct answer 1 Answered correctly 37% Time: 85 seconds Updated: 04/24/2017 Explanation: Amniotic fluid is produced by the fetal kidney and serves 2 major purposes - to prevent cord compression and promote lung development. Oligohydramnios is a condition characterized by low amniotic fluid volume. This can occur due to fetal kidney anomalies (eg, renal agenesis or urine flow obstruction) or fluid leaking through the vagina (eg, undiagnosed ruptured membranes). Fluid volume also declines gradually after 41 weeks. Small uterine size for gestational age or a fetal outline that is easily palpated through the maternal abdomen should raise suspicion for oligohydramnios. Ultrasound confirms the diagnosis. Major complications of oligohydramnios are: Pulmonary hypoplasia - due to the lack of normal alveolar distension by aspirated amniotic fluid. Therefore, additional neonatal personnel should attend the birth in anticipation of possible resuscitation (Option 1). Umbilical cord compression - continuous (not intermittent) fetal monitoring should be applied to monitor for variable decelerations (Option 2). (Option 3) Operative vaginal birth (ie, use of forceps or vacuum) may be indicated due to prolonged second-stage labor or fetal distress. Oligohydramnios does not increase the likelihood of operative vaginal birth. (Option 4) Polyhydramnios (excessive amniotic fluid volume) is a risk factor for postpartum hemorrhage due to overdistension of the uterus. Oligohydramnios is not associated with postpartum hemorrhage. Educational objective: Oligohydramnios increases the risk for umbilical cord compression and pulmonary hypoplasia. Additional neonatal personnel should be present for possible resuscitation and/or evaluation of the newborn. The nurse should anticipate continuous fetal monitoring during labor to monitor for signs of cord compression. Copyright © UWorld. All rights reserved.

Block Time Remaining: 02:46:05 TUTOR Test Id: 72348754 QId: 34472 (921666) 20 of 25 A A A The nurse is counseling a pregnant client who is HIV positive. Which information is appropriate to discuss? 1. Infant should be exclusively breastfed for 6 months to receive maternal antibodies [3%] 2. Infant will not require treatment for HIV after birth [3%] 3. Prescribed antiretroviral therapy should be continued during pregnancy [79%] 4. Tetanus-diphtheria-pertussis vaccine should be avoided until after birth [12%] Correct Answered correctly 79% Time: 21 seconds Updated: 02/21/2017 Explanation: Perinatal transmission of HIV infection can occur from mother to baby anytime during the antepartum, intrapartum, or postpartum periods. Maternal antiretroviral therapy (ART) during pregnancy is imperative for decreasing viral load (amount of virus detectable in maternal serum) and decreasing risk of transmission to the fetus. (Option 1) HIV can be transmitted to the newborn via breast milk. Breastfeeding is contraindicated for HIV-positive mothers in developed countries, where safe alternatives (eg, commercial formula) are available. (Option 2) In addition to routine newborn care, infants born to HIV-positive clients should receive at least 4-6 weeks of ART after birth to reduce the chance of developing HIV infection. Infants are tested for HIV infection at birth and again at age 1 and 4 months. Identification of HIV-negative status requires 2 consecutive negative results at age ≥1 month and ≥4 months. (Option 4) Pregnant clients who are HIV positive are immunocompromised and at increased risk for other infections. They should receive all inactivated vaccines that are recommended for the general pregnant population, such as tetanus-diphtheria-pertussis and intramuscular influenza. Live vaccines (eg, measles-mumps-rubella) are not given during pregnancy. Educational objective: Transmission of HIV infection from mother to baby can occur during antepartum, intrapartum, or postpartum periods. Maternal antiretroviral therapy (ART) during pregnancy is imperative for decreasing the risk of perinatal transmission. Pregnant clients who are HIV positive should receive recommended inactivated vaccines. Newborns born to HIV-positive clients should not breastfeed and should receive 4-6 weeks of ART after birth. Copyright © UWorld. All rights reserved.

Block Time Remaining: 02:46:05 TUTOR Test Id: 72348754 QId: 31453 (921666) 21 of 25 A A A Which meal should the nurse recommend for a pregnant client at 13 weeks gestation? 1. Baked chicken, turnip greens, peanut butter cookie, and grape juice [62%] 2. Baked swordfish, fries, baked apples, and fat-free milk [4%] 3. Chilled ham and cheese sandwich, broccoli, orange slices, and water [18%] 4. Fried liver and onions, pasteurized cheese squares, fresh fruit cup, and water [14%] Correct Answered correctly 62% Time: 33 seconds Updated: 03/09/2017 Explanation: During pregnancy, it is important for the client to consume a balanced diet with appropriate nutrients, vitamins, and minerals. Foods containing folic acid, protein, whole grains, iron, and omega-3 fatty acids are especially important. Due to the risk for bacterial contamination (eg, Listeria, toxoplasmosis), pregnant clients should avoid consuming unpasteurized milk products, unwashed fruits and vegetables, deli meat and hot dogs (unless heated until steaming hot), and raw fish/meat. They should also avoid intake of fish high in mercury (eg, shark, swordfish, king mackerel, tilefish). (Option 2) This meal contains swordfish, which is high in mercury and should be avoided during pregnancy. (Option 3) This meal contains cold deli meat, which should be avoided during pregnancy due to the risk of listeriosis from Listeria monocytogenes. (Option 4) Liver should be avoided during pregnancy due to high amounts of vitamin A. Although liver is a good source of iron, the excessively high amounts of vitamin A can be teratogenic. Educational objective: An appropriate diet is essential to meet the needs of the pregnant client and growing fetus. Pregnant clients should avoid deli meats and hot dogs (unless steaming hot), liver, unpasteurized milk products, unwashed fruits and vegetables, raw fish, and fish high in mercury. Copyright © UWorld. All rights reserved.

Block Time Remaining: 02:46:05 TUTOR Test Id: 72348754 QId: 33999 (921666) 9 of 25 A A A The clinic nurse is collecting data on a pregnant client in the first trimester. Which finding is most concerning and warrants priority intervention? 1. Client has not been taking prenatal vitamins [13%] 2. Client is taking lisinopril to control hypertension [53%] 3. Client reports a whitish vaginal discharge [3%] 4. Client reports mild cramping pain in the lower abdomen [29%] Incorrect Correct answer 2 Answered correctly 53% Time: 20 seconds Updated: 03/05/2017 Explanation: Angiotensin-converting enzyme (ACE) inhibitors (eg, enalapril, lisinopril, ramipril) and angiotensin II receptor blockers (eg, losartan, valsartan, telmisartan) should be avoided in clients who are planning to become pregnant. These drugs are teratogenic, leading to fetal renal and cardiac abnormalities, and are contraindicated in all stages of pregnancy. (Option 1) Prenatal supplements, especially folic acid and iron, are recommended during pregnancy. Although important, this is not a priority over discontinuing ACE inhibitors. (Option 3) Leukorrhea, a whitish vaginal discharge, is common during the prenatal period. The client should be instructed to call the health care provider if the discharge is accompanied by other signs or symptoms, such as a foul odor, redness, or itching. (Option 4) As the uterus enlarges, cramping may occur in the lower abdomen and inguinal region. This common finding can be caused by stretching of the round ligaments, and is usually not concerning in the absence of vaginal bleeding. Educational objective: Angiotensin-converting enzyme inhibitors and angiotensin II receptor blockers are teratogenic and need to be discontinued when planning pregnancy. Copyright © UWorld. All rights reserved.

Block Time Remaining: 02:46:05 TUTOR Test Id: 72348754 QId: 31459 (921666) 10 of 25 A A A A nurse is caring for a pregnant client who has hyperemesis gravidarum. Which assessment findings should the nurse anticipate? Select all that apply. 1. Blood pressure 160/94 mm Hg 2. Large urine protein 3. Positive urine ketones (moderate) 4. Pulse 106/min 5. Urine specific gravity 1.010 Incorrect Correct answer 3,4 Answered correctly 12% Time: 19 seconds Updated: 01/02/2017 Explanation: Hyperemesis gravidarum is a disorder that causes pregnant clients to have severe nausea and vomiting. This leads to fluid and electrolyte imbalances, nutritional deficiencies, ketonuria, and weight loss. On assessment, the nurse should expect signs and symptoms of dehydration, which include dry mucous membranes, poor skin turgor, decreased urine output, tachycardia, and low blood pressure. Ketonuria indicates that the body is breaking down fat to use for energy due to the client's starvation state. (Options 1 and 5) The client with hyperemesis gravidarum is dehydrated. Blood pressure is expected to be low due to lack of blood volume. The urine specific gravity will be increased (>1.030) as the urine is in a concentrated state. Blood urea nitrogen is also elevated (>20 mg/dL [7.1 mmol/L]). (Option 2) Proteinuria is associated with kidney disease or preeclampsia. Hyperemesis does not cause proteinuria. Educational objective: Excessive vomiting (hyperemesis gravidarum) leads to fluid and electrolyte imbalances (hypokalemia, metabolic alkalosis), weight loss, nutritional deficiencies, and ketonuria. The signs and symptoms of dehydration include poor skin turgor, decreased urine output, tachycardia, low blood pressure, and dry mucous membranes. Copyright © UWorld. All rights reserved.

Block Time Remaining: 02:46:05 TUTOR Test Id: 72348754 QId: 31595 (921666) 14 of 25 A A A The obstetric nurse is reviewing phone messages. Which client should the nurse call first? 1. Client at 18 weeks gestation taking ceftriaxone and reporting mild diarrhea [10%] 2. Client at 22 weeks gestation with twins who is taking acetaminophen twice a day [23%] 3. Client at 28 weeks gestation taking metronidazole and reporting dark-colored urine [27%] 4. Client at 32 weeks gestation taking ibuprofen for moderate back pain [38%] Incorrect Correct answer 4 Answered correctly 38% Time: 46 seconds Updated: 01/03/2017 Explanation: Nonsteroidal anti-inflammatory drugs (NSAIDs) (eg, ibuprofen, indomethacin, naproxen) inhibit prostaglandin synthesis and can be taken to decrease pain and inflammation or to reduce fever. NSAIDs are pregnancy category C in the first and second trimesters and pregnancy category D in the third trimester. NSAIDs must be avoided during the third trimester due to the risk of causing premature closure of the ductus arteriosus in the fetus (Option 4). During the first and second trimesters, NSAIDs should be taken only if benefits outweigh risks and under the supervision of a health care provider (HCP). (Option 1) Beta lactam antibiotics (eg, amoxicillin, ceftriaxone [Rocephin]) are pregnancy category B. Diarrhea is a common side effect of beta lactams. Although diarrhea should be reported to the HCP, as it could indicate pseudomembranous (Clostridium difficile) colitis or lead to dehydration if prolonged, this client is not the priority. (Option 2) Acetaminophen (pregnancy category B) is a common pain reliever and/or fever reducer used during pregnancy. Acetaminophen intake should not exceed 4 g per day, including any over-the-counter or prescription combination medications that contain acetaminophen. (Option 3) Metronidazole (Flagyl) is an anti-infective and pregnancy category B. Dark-colored urine is an expected side effect of metronidazole and not cause for concern. Educational objective: Nonsteroidal anti-inflammatory drugs (NSAIDs) should be avoided in the third trimester due to risk of premature closure of the fetal ductus arteriosus. NSAIDs should be taken only under the direction and supervision of a health care provider during the first and second trimesters. Copyright © UWorld. All rights reserved.

Block Time Remaining: 02:46:05 TUTOR Test Id: 72348754 QId: 31535 (921666) 15 of 25 A A A A nurse is caring for a pregnant client at 33 weeks gestation. The diagnosis of placenta previa is made via ultrasound. What should the nurse include in the plan of care? Select all that apply. 1. Activity as tolerated 2. Biophysical profile 1 or 2 times a week 3. Prepare for cesarean birth at any time 4. Type and screen blood 5. Vaginal examinations twice weekly Incorrect Correct answer 2,3,4 Answered correctly 31% Time: 19 seconds Updated: 02/10/2017 Explanation: In placenta previa, the placenta is implanted over or very near the cervix. As a result, placental blood vessels may be disrupted during dilation and effacement. Because of the increased risk of hemorrhage, the client should have a type and screen on file at the selected hospital. A nonstress test or biophysical profile should be performed once or twice a week to ensure fetal well-being. With asymptomatic clients, a cesarean birth is planned after 36 weeks gestation and prior to the onset of labor to prevent blood loss of mother and fetus. However, if the client is bleeding profusely or constantly or goes into active labor, then a cesarean birth is typically performed immediately. (Option 1) The recommended activity for a client at less than 36 weeks gestation with diagnosed placenta previa is bed rest with bathroom privileges. A stable client may be released to continue bed rest at home, but the client must be closely monitored and return to the hospital immediately if bleeding occurs. (Option 5) Clients with placenta previa are placed on pelvic rest. Vaginal examinations, douching, and vaginal intercourse are contraindicated due to the risk of hemorrhage. Educational objective: Clients with placenta previa are at risk for hemorrhage. Bed rest with bathroom privileges is recommended for clients at less than 36 weeks gestation. A nonstress test or biophysical profile should be performed once or twice a week to evaluate fetal well-being. Pelvic rest is instituted to prevent disruption of the cervix. A cesarean birth is planned prior to onset of labor. Copyright © UWorld. All rights reserved.

Block Time Remaining: 02:46:05 TUTOR Test Id: 72348754 QId: 31587 (921666) 11 of 25 A A A A client who reports sudden-onset severe right lower abdominal pain and dizziness is being evaluated for suspected ectopic pregnancy. Which assessment findings should the nurse anticipate? Select all that apply. 1. Blood pressure 82/64 mm Hg 2. Crackles on auscultation 3. Distended jugular veins 4. Pulse 120/min 5. Shoulder pain Omitted Correct answer 1,4,5 Answered correctly 46% Time: 36 seconds Updated: 02/23/2017 Explanation: Ectopic pregnancy occurs when a fertilized ovum implants outside the uterine cavity. The majority of ectopic pregnancies occur in the fallopian tubes. Risk factors are sexually transmitted infections, tubal damage or infection, intrauterine devices, and tubal surgeries. Clinical manifestations are lower-quadrant abdominal pain on one side, mild to moderate vaginal bleeding, and missed or delayed menses. Signs of subsequent hypovolemic (hemorrhagic) shock from tubal rupture include dizziness, hypotension, tachycardia, and decreased urinary output to <30 mL/hr. Free intraperitoneal blood pooling under the diaphragm can cause referred shoulder pain. Peritoneal signs (eg, tenderness, rigidity, low-grade fever) develop subsequently. (Options 2 and 3) Distended jugular veins and lung crackles indicate volume overload conditions. The main risk with ectopic pregnancy is hypovolemic (internal hemorrhage) shock. Jugular veins would be flat in hypovolemic shock. Educational objective: The most common site for an ectopic pregnancy is the fallopian tubes. As the ovum grows and expands, tubal rupture may occur and result in active bleeding that progresses to life-threatening hypovolemic (hemorrhagic) shock. Copyright © UWorld. All rights reserved.

Block Time Remaining: 02:46:05 TUTOR Test Id: 72348754 QId: 31553 (921666) 12 of 25 A A A A client at 21 weeks gestation has intense heartburn (pyrosis). What should the nurse recommend? Select all that apply. 1. Avoid dairy products 2. High-protein, low-fat diet 3. Lie on the left side after meals 4. Six small meals a day 5. Sodium bicarbonate antacid Incorrect Correct answer 2,4 Answered correctly 15% Time: 13 seconds Updated: 02/12/2017 Explanation: Pyrosis, also known as heartburn, occurs during pregnancy from an increase in the hormone progesterone. Progesterone causes the esophageal sphincter to relax, leading to pyrosis. Interventions to reduce heartburn include: Upright position after meals to reduce gastroesophageal reflux Small, frequent meals rather than 3 large meals a day (Option 4) Keeping the head of the bed elevated using pillows Drinking smaller amounts of fluid while eating Eliminating dietary triggers, including fried and fatty foods, caffeine/chocolate, spicy foods, carbonated drinks, and peppermint (Option 2) (Option 1) Dairy products do not lead to heartburn and should not be avoided during pregnancy. Clients should avoid fried and fatty foods, coffee, and carbonated beverages. (Option 3) Clients should stay upright 2-3 hours after meals. Lying down immediately after eating will cause heartburn to increase and lead to more discomfort. (Option 5) Most antacids are safe during pregnancy. However, those containing sodium bicarbonate (Alka-Seltzer) or magnesium trisilicate should be avoided during pregnancy. Educational objective: Heartburn (pyrosis) is caused by the hormone progesterone, which causes the esophageal sphincter to relax. Methods the pregnant client can take to reduce pyrosis include maintaining an upright position after meals, eating small meals, taking approved antacids, and avoiding causative foods and beverages. Copyright © UWorld. All rights reserved.

Block Time Remaining: 02:46:05 TUTOR Test Id: 72348754 QId: 34306 (921666) 1 of 25 A A A A 14-year-old client confides to the school nurse that she is about 22 weeks pregnant and has not had prenatal care. Which topics are most important and priorities for the nurse to discuss with the client in anticipation of referral for prenatal care? Select all that apply. 1. Adoption planning 2. Family and social support 3. Future education plans 4. Nutrition and prenatal vitamins 5. Sexual abuse Correct Answered correctly 30% Time: 26 seconds Updated: 01/29/2017 Explanation: The nurse should ascertain potential risks to the client and her baby due to teenage pregnancy. Lack of family/social support or fear of social discrimination may prevent the client from obtaining prenatal care (Option 2). Poverty, dangerous living conditions, and exposure to teratogens (eg, tobacco, alcohol, illicit drugs) may place the client at risk for complications. Adolescents are at risk for poor nutritional status and poor pregnancy weight gain, which can have deleterious effects on the baby (eg, small for gestational age, low birth weight) (Option 4). They are also less likely to take prenatal vitamins with folic acid. Adolescents who are pregnant should be evaluated for sexual abuse; girls age 11-14 do not usually seek sexual relationships, and the pregnancy may be evidence of abuse (Option 5). (Option 1) Adoption planning is not necessary at this first assessment, although the topic may be addressed later in collaboration with a social worker. (Option 3) The younger adolescent (age <16) is emotionally and developmentally immature and may be resistant to planning for the future. Education planning may be approached at subsequent encounters, but it is not a priority for the client and the baby's health at this time. Educational objective: Adolescents are at increased risk for obstetrical complications that can contribute to neonatal morbidity and mortality. Factors such as lack of family support, sexual abuse, and poor nutritional status can negatively impact the pregnancy. Copyright © UWorld. All rights reserved.

Block Time Remaining: 02:46:05 TUTOR Test Id: 72348754 QId: 31554 (921666) 2 of 25 A A A A client at 34 weeks gestation reports constipation. The client has been taking 325 mg ferrous sulfate tid for anemia since the last appointment 4 weeks ago. Which recommendations should the nurse make for this client? Select all that apply. 1. Decreased daily dairy intake 2. Increased fruit and vegetable intake 3. Moderate-intensity regular exercise 4. One laxative twice daily for a week 5. Two cups of hot coffee each morning Incorrect Correct answer 2,3 Answered correctly 40% Time: 58 seconds Updated: 02/27/2017 Explanation: Constipation is a common discomfort of pregnancy and is due to an increase in the hormone progesterone, which causes decreased gastric motility. Ferrous sulfate (iron) supplementation may also cause constipation. Interventions to prevent or treat constipation include: High-fiber diet: High amounts of fruits, vegetables, breakfast cereals, whole-grain bread, prunes High fluid intake: 10-12 cups of fluid daily Regular exercise: Moderate-intensity exercise (eg, walking, swimming, aerobics) Bulk-forming fiber supplements: Psyllium, methylcellulose, wheat dextrin (Option 1) Dairy is a great source of calcium, which is essential for fetal bone development. However, dairy products should be consumed at least 2 hours before or 1 hour after iron supplements as they bind to iron and decrease absorption. (Option 4) Laxatives are not recommended during pregnancy due to the risk of dehydration and electrolyte imbalance, which can lead to uterine cramping and contractions. The client should consult with the health care provider before using any over-the-counter stool softeners or laxatives. (Option 5) Caffeine consumption in pregnancy should be limited to 200-300 mg/day. Coffee may contain 100-200 mg caffeine per cup and should therefore be consumed in moderation during pregnancy. Educational objective: Constipation in pregnancy may be caused by increased progesterone levels and iron supplementation. It is best treated with 10-12 cups of fluid daily, a high-fiber diet/supplementation, and regular exercise. Clients should not take laxatives without first discussing this with the health care provider. Copyright © UWorld. All rights reserved.

Block Time Remaining: 02:46:05 TUTOR Test Id: 72348754 QId: 30453 (921666) 5 of 25 A A A A pregnant client provides the following obstetric history to the nurse at the first prenatal visit: Elective abortion at age 17; a 5-year-old daughter born at 40 weeks gestation; and 3-year-old twin boys born at 34 weeks gestation. Using the GTPAL system, which option is correct? 1. G3, T1, P1, A1, L3 [19%] 2. G3, T1, P2, A1, L3 [28%] 3. G4, T1, P1, A1, L3 [22%] 4. G4, T1, P2, A1, L3 [29%] Incorrect Correct answer 3 Answered correctly 22% Time: 90 seconds Updated: 01/04/2017 Explanation: The GTPAL system is a shorthand system of documenting a client's obstetric history. Under this system, twins, triplets, or other multiple births count as one in the term (T) or preterm (P) category but are counted separately (as 2, 3, or more) in the living child (L) category. A current pregnancy (not yet delivered), as in this client, counts in the gravida (G) category as this category includes all pregnancies, past and present. In this scenario, the client is a G4 T1 P1 A1 L3. She is gravida (G) 4 as she has a history of 4 pregnancies (which includes the present pregnancy) (Option 3). The client delivered a child at 40 weeks gestation (counts 1 in the term [T] category). She delivered twins at 34 weeks gestation, reflected as a single birth (1 pregnancy) in the preterm (P) category and as 2 living children in the living child (L) category. She had an elective abortion, reflected as 1 in the abortion (A) category. She has a total of 3 living children (1 term and 2 preterm children), reflected in the living child (L) category. Educational objective: Under the GTPAL system, G - gravida indicates the number of pregnancies, delivered or undelivered; T - term deliveries are from 37 wk 0 days and beyond; P - preterm deliveries are from 20 wk 0 days to 36 wk 6 days gestation; A - abortions (spontaneous or elective) occur prior to 20 wk 0 days gestation; and L - living children are counted individually regardless of multiple birth status. Copyright © UWorld. All rights reserved.

Block Time Remaining: 02:46:05 TUTOR Test Id: 72348754 QId: 31691 (921666) 6 of 25 A A A Exhibit The nurse reviews laboratory test results for a pregnant client at 32 weeks gestation. What is the nurse's best action based on these results? Click on the exhibit button for additional information. 1. Complete the client assessment and documentation [56%] 2. Draw another sample for repeat complete blood count [7%] 3. Prepare for transfusion of packed red blood cells [9%] 4. Request a prescription for iron supplementation [26%] Incorrect Correct answer 1 Answered correctly 56% Time: 34 seconds Updated: 03/08/2017 Explanation: Pregnant women experience a 40%-45% increase in total blood volume during pregnancy to meet the increased oxygen demand and nutritional needs of the growing fetus and maternal tissues. Because the increase in plasma volume is greater than the increase in red blood cells, a hemodiluted state called physiologic anemia of pregnancy occurs, and is reflected in lower hemoglobin and hematocrit values. It is also normal for the white blood cell count to increase during pregnancy; counts can be as high as 15,000/mm3 (15.0 x 109/L). These laboratory results are within the normal ranges for a pregnant client in the third trimester, and no intervention is required (Options 1 and 2). (Option 3) A blood transfusion should not be considered in pregnancy unless severe anemia (hemoglobin <7.0 g/dL [<70 g/L]) is suspected. (Option 4) Iron is frequently prescribed for pregnant women to prevent or treat iron deficiency anemia (hemoglobin ≤11 g/dL [110 g/L] and hematocrit ≤33% [0.33]). However, this pregnant client's laboratory results are within normal ranges, and iron supplementation is not necessary. Educational objective: Pregnant women experience an increase in total blood volume to meet the increased oxygen demand and nutritional needs of the growing fetus and maternal tissues. The increase in plasma volume is greater than the increase in red blood cells, creating a hemodiluted state termed physiologic anemia of pregnancy, which is reflected in decreased hemoglobin (>11.0 g/dL [>110 g/L]) and hematocrit (>33% [0.33]) values. Copyright © UWorld. All rights reserved.

Block Time Remaining: 02:46:05 TUTOR Test Id: 72348754 QId: 31543 (921666) 3 of 25 A A A A pregnant client arrives in the labor and delivery unit with mild contractions and brisk, painless vaginal bleeding. The client received no prenatal care and reports being "about 7-8 months." Which actions should the nurse anticipate? Select all that apply. 1. Blood draw for type and screen 2. Electronic fetal monitoring 3. Initiation of 2 large-bore IV catheters 4. Pad counts to assess bleeding 5. Vaginal examination for cervical dilation Incorrect Correct answer 1,2,3,4 Answered correctly 32% Time: 29 seconds Updated: 05/19/2017 Explanation: Placenta previa is an abnormal implantation of the placenta resulting in partial or complete covering of the cervical os (opening). The condition is diagnosed by ultrasound. In clients reporting painless vaginal bleeding after 20 weeks gestation, placenta previa should be suspected. Placenta previa found early in pregnancy may resolve by the third trimester, but women with persistent placenta previa or hemorrhage require cesarean birth. A type and screen to determine blood type and Rh status is appropriate due to the potential for excessive blood loss and need for blood transfusion (Option 1). Fetal well-being is assessed via continuous electronic fetal monitoring to help determine appropriate timing for birth (Option 2). Large-bore IV access is established in anticipation of fluid resuscitation and administration of blood products (Option 3). The client should also be monitored frequently for any changes in bleeding via pad counts (Option 4). (Option 5) Digital vaginal examinations are contraindicated in the presence of vaginal bleeding of unknown origin. When placenta previa is present, manual manipulation of the cervix can damage placental blood vessels, causing subsequent bleeding that can progress to hemorrhage. Clients with placenta previa are on pelvic rest (ie, no intercourse, nothing per vagina). Educational objective: Placenta previa is suspected in clients reporting painless vaginal bleeding after 20 weeks gestation. Clients with placenta previa are at high risk for hemorrhage. The nurse should initiate electronic fetal monitoring and pad counts, draw a type and screen, and initiate large-bore IV access. Digital vaginal examinations are contraindicated. Copyright © UWorld. All rights reserved.

Block Time Remaining: 02:46:05 TUTOR Test Id: 72348754 QId: 33131 (921666) 4 of 25 A A A The nurse is providing education to several first-trimester pregnant clients. Which client requires priority anticipatory teaching? 1. Client who gardens and eats homegrown vegetables [52%] 2. Client who has gained 4 lb (1.8 kg) from prepregnancy weight [22%] 3. Client who has noticed thin, milky white vaginal discharge [18%] 4. Client who practices yoga and swims in a pool 3 times a week [5%] Incorrect Correct answer 1 Answered correctly 52% Time: 54 seconds Updated: 05/18/2017 Explanation: Toxoplasmosis is a parasitic infection caused by Toxoplasma gondii, which may be acquired from exposure to infected cat feces or ingestion of undercooked meat or soil-contaminated fruits/vegetables. Pregnant clients who contract toxoplasmosis can transfer the infection to the fetus and potentially cause serious fetal harm (eg, stillbirth, malformations, blindness, mental disability). Pregnant clients should be advised to take precautions when gardening and thoroughly wash all produce to decrease exposure risk. (Option 2) Weight gain recommendations vary by prepregnancy BMI. A 1.1- to 4.4-lb (0.5- to 2.0-kg) weight gain in the first trimester and approximately 1 lb (0.5 kg) per week thereafter is normal and expected for women with a healthy BMI. (Option 3) Leukorrhea is a thin, milky white vaginal discharge that is normal during pregnancy and is due to increased levels of progesterone and estrogen. If discharge changes color, becomes malodorous, or causes itching/burning, further investigation is needed. (Option 4) Exercise, particularly low-impact activities such as walking, swimming, and yoga, is recommended during pregnancy. Contact sports or activities with a risk for falls (eg, soccer, downhill skiing) should be avoided to prevent abdominal injuries. Educational objective: Toxoplasmosis is a parasitic infection acquired by exposure to infected cat feces or ingestion of undercooked meat or soil-contaminated fruits/vegetables. Pregnant clients who contract toxoplasmosis may transfer the infection to the fetus and potentially cause serious fetal harm. Pregnant clients should take precautions when gardening and thoroughly wash all produce to decrease exposure risk. Copyright © UWorld. All rights reserved.

Block Time Remaining: 02:46:05 TUTOR Test Id: 72348754 QId: 30448 (921666) 7 of 25 A A A A client visits the prenatal clinic for the first time. Which of the following are presumptive signs of pregnancy that the client may report during the intake assessment by the nurse? Select all that apply. 1. Breast fullness and tenderness 2. Last menstrual period 8 weeks ago 3. Nausea and vomiting that are worsening every day 4. Painless contractions at irregular intervals 5. Positive home pregnancy test 3 weeks ago Incorrect Correct answer 1,2,3 Answered correctly 27% Time: 31 seconds Updated: 01/17/2017 Explanation: Subjective (presumptive) signs of pregnancy are those that are self-reported by a pregnant client. These signs may have pathologic medical causes and therefore cannot be considered diagnostic for pregnancy. Subjective signs include: Breast fullness and tenderness (may occur just prior to menstrual periods or with use of birth control hormones) Amenorrhea (may be seen with early menopause, endocrine dysfunction, acute or chronic diseases, or psychological stress) Nausea and vomiting that begin around the sixth week after the last menstrual period (may be result of many other conditions, such as gastroenteritis) Objective (probable) signs of pregnancy are those that can be observed by the health care provider during assessment and examination. Combined with subjective signs, these may be more indicative of pregnancy but may still have some pathologic causes. Positive signs of pregnancy represent conclusive proof of pregnancy and cannot be confused with any pathologic state as they are all related to the fetus. (Option 4) Braxton Hicks contractions are painless and occur at irregular intervals. They can occur as early as 6 weeks gestation and are considered a probable sign of pregnancy. (Option 5) A positive pregnancy test is considered an objective sign of pregnancy. Hormonal disturbances, endocrine disease, and gestational trophoblastic disease can cause false positive pregnancy test results. Educational objective: Subjective (self-reported) signs of pregnancy include amenorrhea, nausea and vomiting, breast tenderness, and fatigue. Objective signs include uterine and cervical changes, palpable Braxton Hicks contractions, skin changes, and positive pregnancy tests. Diagnostic signs include a distinguishable fetal heartbeat, fetal movement palpated by the health care provider, and ultrasound visualization of the fetus. Copyright © UWorld. All rights reserved.

Block Time Remaining: 02:46:05 TUTOR Test Id: 72348754 QId: 33149 (921666) 8 of 25 A A A A client at 30 weeks gestation is hospitalized for preeclampsia. Which assessment finding requires priority intervention? 1. Elevated liver enzymes [17%] 2. Lower abdominal pain and vaginal bleeding [45%] 3. Swelling of the hands, feet, and face [19%] 4. Urine output of 25 mL/hr [17%] Incorrect Correct answer 2 Answered correctly 45% Time: 22 seconds Updated: 04/08/2017 Explanation: Placental abruption is a possible complication of preeclampsia that can be life-threatening to mother and baby. It occurs when the placenta tears away from the wall of the uterus due to stress, causing significant bleeding to the mother and depriving the baby of oxygen. Bleeding can be concealed inside the uterus. This may require immediate delivery of the baby. (Option 1) Elevated liver enzymes can indicate the start of a more serious condition called HELLP (Hemolysis, Elevated Liver enzymes, Low Platelets), another complication of preeclampsia. This client will need additional laboratory work. If HELLP is diagnosed, the only treatment is delivery. (Option 3) Swelling of the hands, feet, and face are expected findings of preeclampsia and should be assessed frequently. (Option 4) Normal urine output is 30 mL/hr; anything less represents a potential problem. Oliguria and elevated creatinine levels indicate kidney failure and require intervention. This client's urine output of 25 mL/hr does not represent an immediate, life-threatening condition. Educational objective: Preeclampsia in pregnancy manifests with high blood pressure and protein in the urine. Edema is expected, although it is not part of the criteria. Complications of preeclampsia include eclampsia, placental abruption, and HELLP (Hemolysis, Elevated Liver enzymes, Low Platelets) syndrome. Copyright © UWorld. All rights reserved.


Kaugnay na mga set ng pag-aaral

The General and Standard Form of Conics

View Set

Medical Terminology: -tripsy to uter(o)-

View Set

HEALTH ASSESSMENT TTL's Day 6-10

View Set